Sei sulla pagina 1di 262

Compendio de Geometría Euclidiana y

Geometría Analítica
Para Docentes de Educación Secundaria

g
h

r
INTRODUCCIÓN

La palabra geometría tiene dos raíces griegas: geo = tierra y metrón = medida; significa
"medida de la tierra", esto nos indica que la Geometría se originó en las mediciones de la
tierra. Sus inicios, unos tres mil años antes de Cristo, se remonta al Medio Oriente, en
particular al Antiguo Egipto, donde se necesitaba medir predios agrarios y en la
construcción de monumentos y pirámides.

Es razonable pensar que los primeros orígenes de la Geometría se encuentran en los


mismos orígenes de la humanidad, pues seguramente el hombre primitivo clasificaba -aun
de manera inconsciente- los objetos que le rodeaban según su forma. En la abstracción de
estas formas comienza el primer acercamiento -informal e intuitivo- a la Geometría.

Las primeras civilizaciones mediterráneas adquieren poco a poco ciertos conocimientos


geométricos de carácter muy práctico. Esencialmente algunas fórmulas algoritmos
expresados en forma de recetas para calcular áreas y longitudes. La finalidad era práctica,
pues se pretendía con ello calcular la producción proporcional de las parcelas de tierra para
determinar los impuestos, o reconstruir las parcelas de tierra después de las inundaciones
del río Nilo.

Siempre se ha dicho que los egipcios tenían una alta formación matemática, y se ha llegado
a insinuar que tuvieran un acervo de conocimientos secretos o que se hubieran perdido con
el paso de los tiempos. Estas hipótesis nunca han sido confirmadas, y los documentos
existentes tienden a echarlas por tierra. La Historia nos hace pensar que el conocimiento
que esta civilización -así como los de las culturas mesopotámicas- tuviera sobre Geometría
pasó íntegramente a la cultura griega a través de Thales de Mileto, los pitagóricos, y
esencialmente de Euclides.

Estos conocimientos pasaron a los griegos y fue Thales, quien unos seis siglos antes de
Cristo inició la geometría demostrativa. Las propiedades se demuestran por medio de
razonamientos y no porque resulten en la práctica. Las demostraciones pasan a ser
fundamentales y son la base de la Lógica como leyes del razonamiento.

Euclides fue otro gran matemático griego, del siglo III antes de Cristo, quien en su famosa
obra titulada "Los Elementos", recopila, ordena y sistematiza todos los conocimientos de
Geometría hasta su época y, salvo algunas pequeñas variaciones, son los mismos
conocimientos que se siguen enseñando en nuestros días. Euclides, usando un
razonamiento deductivo parte de conceptos básicos primarios no demostrables tales como
punto, recta, plano y espacio, que son el punto de partida de sus definiciones, axiomas y
postulados. Demuestra teoremas y a su vez, éstos servirán para demostrar otros teoremas.
Crea nuevos conocimientos a partir de otros ya existentes por medio de cadenas deductivas
de razonamiento lógico. Esta Geometría, llamada Geometría Euclidiana, se basa en lo que
históricamente se conoce como 5º postulado de Euclides: "por un punto situado fuera de
una recta se puede trazar una y sólo una paralela a ella".
 Ofrece grandes posibilidades dentro de la Matemática: ya que estimula el
razonamiento y posibilita el desarrollo de ciertas habilidades como clasificar,
definir, particularizar, generalizar, entre otras, partiendo de diferentes situaciones
mediante la experimentación con materiales adecuados.

 Es un medio para desarrollar la percepción espacial y la visualización: ya que al


enfrentarse a una nueva situación, el sujeto va percibiendo a través del sentido de la
vista diversos elementos que luego va integrando a una estructura más compleja y
así de forma gradual va obteniendo una imagen visual que puede reconocer
posteriormente. El desarrollo del proceso visual es esencial para lograr una
adecuada percepción espacial y así obtener un conocimiento de las estructuras
espaciales que lo rodean.

 Forma parte del lenguaje cotidiano: el lenguaje verbal diario posee muchos
términos geométricos, por ejemplo, punto, recta, plano, superficie, curva, paralelo,
etc. Para lograr la comunicación acerca de la ubicación, el tamaño o la forma de un
objeto, esta terminología específica es fundamental.

 Tiene importantes aplicaciones en la vida real: muchas tareas cotidianas involucran,


en mayor o menor medida, una base geométrica. Incluso la estructura del universo
y de la naturaleza puede ser descripta en términos geométricos.

 Se usa en las distintas ramas de la Matemática y también sirve de base para la


Comprensión de conceptos de otras disciplinas: se constituye en un eje unificador
de la Matemática, como recurso para la visualización de variados conceptos -
aritméticos, algebraicos, de cálculo y de estadística- y en un lenguaje necesario para
el estudio de diferentes disciplinas, tales como: Física, Química, Astronomía,
Biología, Cartografía, Fotografía, Cinematografía, Tecnología y Plástica, etc.

El Ministerio de Educación a través de esta capacitación a los docentes de matemática


impulsara una nueva visión hacia la geometría en la que cada estudiante adquiera una
cultura geométrica, una cultura que requiere desarrollar unas habilidades específicas, tener
un vocabulario adecuado y poseer una visión global de las aplicaciones actuales y una
sensibilidad por el buen razonar, por la belleza y por la utilidad.

Esta rama de la Matemática ha sido durante siglos uno de las base fundamentales de la
formación de las personas, por ser históricamente considerada como resultado de
producciones del hombre y una posible representación de la realidad.
Geometría
Geometria Euclidiana

Arista
Eje de Base básica
Base Vértice
Giro

Altura
Cara
Arista lateral
Base lateral

Base Eje
Vértice
Base

Vértice
Eje de
giro

Cara Altura
Radio lateral

Apotema

Centro
Base

Para Nicaragua, con la VIII Cumbre de Petrocaribe con sede en Managua, se ratificaron
los acuerdos sobre cooperación energética, programas sociales y productivos; en
especial, la construcción de la Refinería en Nicaragua; la inyección financiera para
impulsar la agricultura, mejorar la producción de arroz y café; y el desarrollo de mataderos
industriales y plantas procesadoras de leche y maíz.

Fuente: 19 digital.
02 de Julio 2013.
3.1. CONCEPTOS BÁSICOS

3.1.1. TÉRMINOS PRIMITIVOS

En la Geometría Moderna se asumen como términos primitivos o indefinidos, es decir como punto de
partida, los conceptos de Punto, Recta y Plano.

• ↔
P m π
La Geometría toma estos términos como punto de partida, evitando el tratar de dar una definición,
precisa, para no caer en un círculo vicioso en el que se tenga que usar conceptos cuyas definiciones
sean más complejas que el término que se quiere definir.

NOTACIÓN:

◊ Representaremos los puntos con letras mayúsculas: A, B,.., P, Q,..etc.


◊ Las líneas rectas las representaremos con letras minúsculas, colocándole encima una flecha con
↔ ↔
doble punta: m, l , ..., etc. ; también las representaremos usando dos letras mayúsculas, que

representen puntos contenidos en dicha recta, así PQ representa la recta que pasa por los puntos P y Q.
◊ Los planos los representaremos usando letras griegas o bien usando tres letras mayúsculas que
representen puntos contenidos en el plano, pero que no estén sobre la misma recta: α, β, π, σ,..., plano
ABC,..

3.1.2. AXIOMAS Y DEFINICIONES BÁSICAS

AXIOMA 1. Todas las rectas y planos son conjuntos de puntos.

DEFINICIÓN 1. El conjunto de todos los puntos se llama Espacio.

◊ Hemos de tener presente estos enunciados al momento de representar simbólicamente alguna relación
entre puntos, rectas y planos. El espacio, que podemos representarlo por S, representará el Conjunto
Universo; las rectas y planos son conjuntos, mientras que los puntos son los elementos.
↔ ↔
Así para indicar que el punto P está contenido en la recta m o bien que la recta m pasa por el punto P,
↔ ↔ ↔
escribimos P ∈ m . Para indicar que la recta m está contenida en el plano π, escribimos m ⊂ π.
↔ ↔ ↔ ↔
Para indicar que las rectas m y n se cortan en el punto P, escribimos m ∩ n = { P }
POSTULADO DE LA RECTA

AXIOMA 2. Dados dos puntos distintos cualesquiera, hay exactamente una recta que los contiene.
↔ ↔
Simbólicamente: P, Q ∈ S, P ≠ Q ⇒ ∃! m ⊂ S tal que {P, Q} ⊂ m

1
Elementos Fundamentales de la Geometría

Notemos que el axioma asegura la existencia y unicidad de la recta que pasa por dos puntos
cualesquiera. Podemos afirmar entonces que dos puntos determinan una recta.

Notación: La recta que contiene a los puntos P y Q se denota por PQ

DEFINICIÓN 2. Los puntos de un conjunto decimos que están alineados o que son colineales, si hay
una recta que los contiene.

D•
• En la figura: A, B y C son colineales.

• B C A, B y D no son colineales
A

DEFINICIÓN 3. Los puntos de un conjunto son coplanares si hay un plano que los contiene.

C• En la figura A, B y C representan puntos coplanares,


lo que indicamos por: {A, B, C} ⊂ π
• •
π A B

AXIOMA 3. Tres puntos cualesquiera están al menos en un plano y tres puntos no colineales están
exactamente en un plano.

Es equivalente a afirmar:
i) tres puntos cualesquiera son coplanares
ii) tres puntos cualesquiera no colineales determinan un plano.


Simbólicamente: P, Q, R ∈ S, P ∉ QR ⇒ ∃ ! π ⊂ S tal que {P, Q, R} ⊂ π

AXIOMA 4. i) cada recta contiene al menos dos puntos


ii) cada plano contiene al menos tres puntos no colineales
iii) el espacio contiene al menos cuatro puntos no coplanares.

AXIOMA 5. Si dos puntos de una recta están en un plano, entonces la recta está contenida en el
mismo plano.
↔ π
m •
↔ ↔ • B
A,B ∈ m y A, B ∈ π ⇒ m ⊂ π A


AXIOMA 6. Si dos planos diferentes se intersecan, su m
intersección es una recta. π1

π1 ≠ π2, π1 ∩ π2 ≠ φ ⇒ π1 ∩ π2 = m π1

π2

2
Elementos Fundamentales de la Geometría

3.1.3. ALGUNOS TEOREMAS BÁSICOS

TEOREMA 1. Si dos rectas diferentes se intersecan, su intersección


contiene un punto solamente. ↔
↔ ↔ ↔ ↔ ↔ ↔ P m1
m1 ≠ m 2 , m1 ∩ m 2 ≠ φ ⇒ m1 ∩ m 2 = {P } •

TEOREMA 2. Si una recta interseca a un plano que no la contiene, m2
entonces la intersección contiene un solo punto.

↔ ↔ ↔ ↔
m ⊄ π , m ∩ π ≠ φ ⇒ m ∩ π = {P} m
P•
π

TEOREMA 3. Dada una recta y un punto fuera de ella, hay exactamente


P ↔
un plano que las contiene. • m
↔ ↔
P∉ m ⇒ ∃ ! π ⊂ S, tal que { P } ∪ m ⊂ π
π

R
TEOREMA 4. Dadas dos rectas que se intersecan, hay • Q P•
exactamente un plano que las contiene. ↔ •
↔ ↔ ↔ ↔ ↔ ↔ m2
m1 ≠ m 2 , m1 ∩ m 2 ≠ φ ⇒ ∃ ! π tal que m1 ∪ m 2 ⊂ π ↔
π m1

3.1.4. DISTANCIA. SEGMENTOS Y RAYOS.

DEFINICIÓN 4. A cada par de puntos P, Q ∈ S podemos asociar un número real no negativo llamado la
distancia de P a Q, denotado por d (P, Q) o bien por PQ, que satisface las siguientes propiedades:
i) d : S x S → R
ii) d ( P, Q) ≥ 0, ∀ P, Q ∈ S
iii) d (P, Q) = d (Q, P)
iv) Si P = Q entonces d (P, Q) = 0
Las propiedades (i) y (ii) nos indican que para toda pareja de puntos podemos encontrar un número que
llamamos distancia y que este número es no negativo.
La propiedad (iii) nos indica que no importa el orden en que tomemos los puntos para encontrar la
distancia entre ellos.

En algunas aplicaciones es conveniente considerar distancias “negativas” y “positivas”; en tales casos


hablamos de distancia dirigida, y la distancia de P a Q es el negativo de la distancia de Q a P. En
nuestro caso consideraremos la distancia en el sentido dado por esta definición, es decir como distancia
“no dirigida”.

VALORES INTERMEDIOS

DEFINICIÓN 6. Decimos que B está “entre” A y C, y escribimos A – B – C si


i) A, B y C son puntos distintos de una misma recta y
ii) AB + BC = AC • • •
A B C
Pueden probarse los siguientes teoremas sobre valores intermedios:

TEOREMA 5. Si A, B, C ∈ m ∧ A–B–C ⇒ C–B–A

3
Elementos Fundamentales de la Geometría


TEOREMA 6. Si A, B, C ∈ m ⇒ A – B – C ∨ A – C – B ∨ B – A – C
Es decir dados tres puntos (distintos) colineales, necesariamente uno de ellos está entre los otros dos.

TEOREMA 7. Dados A ≠ B, entonces existen puntos C, D y E tales que A – C – D, A – B – D y


E – A – B.

SEGMENTOS
__
DEFINICIÓN 7. Para dos puntos cualesquiera A y B el segmento, denotado por AB , es el conjunto de
los puntos A y B, y de todos los puntos que están entre A y B.
__
AB = {A, B} U {x / A – X – B}
__ __ __
Los puntos A y B se llaman extremos de AB . Se tiene también que AB = BA
A la distancia entre los puntos A y B, AB, se le llama la longitud o medida del segmento.

• • __
A B d (A, B) = m ( AB ) = AB

CONGRUENCIA DE SEGMENTOS

DEFINICIÓN 8. Dos segmentos con la misma medida se llaman congruentes.


La relación de congruencia se denota con el símbolo ≅
___ ___
AB ≅ CD ⇔ AB = CD.
◊ El concepto de congruencia está muy cercano al concepto de igualdad, sin embargo son conceptos
diferentes. La igualdad es estrictamente la relación que conecta dos “nombres” para el mismo “objeto”, en
cambio la congruencia relaciona dos “objetos” que tienen en común la forma y el tamaño.

◊ En las figuras los segmentos congruentes se indican colocando la misma marca sobre ellos. Así por
ejemplo

D C
__ __ __ __
En la figura se indica que AB ≅ DC y AD ≅ BC

A B

TEOREMA 8. Para segmentos, la relación de congruencia es una relación de equivalencia, es decir una
relación reflexiva, simétrica y transitiva:
___ ___ ___ ___ ___ ___
i) AB ≅ AB (Reflexividad) ii) Si AB ≅ CD ⇒ CD ≅ AB (Simetría)
___ ___ ___ ___ ___ ___
iii) AB ≅ CD ∧ CD ≅ EF ⇒ AB ≅ EF (Transitividad)

DEFINICIÓN 9. RAYO
↔ →
Sean A y B dos puntos de una recta m . El rayo, denotado por AB , es el conjunto de los puntos del
segmento AB y el conjunto de todos los puntos X tales que B está entre A y X.

AB = AB U {X / A – B – X}

El punto A se llama extremo del rayo AB . Notemos que el

rayo es un conjunto infinito. A B C

4
Elementos Fundamentales de la Geometría

→ → →
Si C es un punto cualquiera que pertenece al rayo AB , C ≠ A, se tiene AB = AC .
Esto es, para representar un rayo, nombramos el punto inicial y otro punto cualquiera que pertenezca al
rayo.


A B
→ → → → __ → → ↔
En general AB ≠ BA . Además AB ∩ BA = AB y AB ∪ BA = AB .

→ →
DEFINICIÓN 10. Si A está entre B y C, entonces AB y AC se llaman rayos opuestos.


C A B

DEFINICIÓN 11. PUNTO MEDIO


___
Un punto B se llama punto medio de un segmento AC , si B está entre A y C (A – B – C) y AB = BC.

A B C

◊ Decimos que el punto medio de un segmento biseca al segmento.

◊ En una recta con un sistema de coordenadas, los segmentos representan intervalos cerrados y los
rayos intervalos infinitos, cerrados por el extremo.


TEOREMA 9. Sea AB un rayo y sea x un número positivo, entonces existe exactamente un punto P de

AB tal que AP = x.
__ → → __ __
TEOREMA 10: Dado un segmento AB y un rayo CD existe un único punto E ∈ CD , tal que AB ≅ CE .
__ __ __ ___ __ ___ __ __ __
TEOREMA 11: Si A – B – C, A’ – B’ – C’, AB ≅ A' B', BC ≅ B' C' entonces AC ≅ A' C' .O sea AB+ BC = AC

∗ Estos teoremas nos posibilitan poder construir cualquier segmento sin importar su longitud y considerar
la suma y resta de segmentos. Notemos que ésta es una suma “especial”, por cuanto los segmentos son
conjuntos de puntos, no son números.

TEOREMA 12. Todo segmento tiene exactamente un punto medio.

3. 1. 5. CURVAS CERRADAS SIMPLES Y REGIONES. CONVEXIDAD.

Una curva en el plano es un conjunto continuo, infinito de puntos. Esta puede ser abierta o cerrada. Si la
asociamos al trazado en una hoja de papel con un lápiz, de manera informal podemos decir que una
curva en el plano, es un conjunto de puntos que puede ser trazado sin levantar el lápiz.
Si la curva puede ser recorrida iniciando en cualquier punto y regresar al mismo, decimos que es una
curva cerrada, en caso contrario decimos que es abierta.

Una curva cerrada simple es aquella que comienza y termina en el mismo punto sin que se cruce o se
vuelva a “tocar”. Toda curva cerrada simple en un plano, separa a éste en tres conjuntos disjuntos: la
curva misma, su interior y el exterior. Todo segmento que una un punto interior con uno exterior, corta a
la curva.

5
Elementos Fundamentales de la Geometría

La unión de los puntos que están sobre la curva o en su interior recibe el nombre de región y decimos
que la curva es la frontera de la región.
Una característica importante de las regiones y en general de los conjuntos de puntos es la convexidad.

DEFINICIÓN 12. Un conjunto A se llama convexo, si para cada par de puntos P y Q del conjunto,
todo el segmento PQ está en A.

A B C D
Figuras Convexas

D E F G
Figuras no Convexas

SEPARACIÓN EN UNA RECTA: SEMIRRECTAS



Un punto P separa a una recta AB en tres conjuntos disjuntos: {P} y dos conjuntos llamados
o→ o→
semirrectas, denotadas por PA y PB
o→ →
Debe notarse que PA = PA – {P}, es decir, la semirrecta es el rayo sin el punto inicial.

POSTULADO DE SEPARACIÓN DEL PLANO: SEMIPLANOS

Dada una recta y un plano que la contiene. Los puntos del plano que
no están en la recta forman dos conjuntos tales que • ↔
m
i) cada uno de los conjuntos es convexo P •
ii) si P está en uno de los conjuntos y Q en el otro, entonces el Q
segmento PQ interseca a la recta.
π
SEMIPLANOS

DEFINICIÓN 14. Dada una recta m y un plano π que la contiene. Los dos conjuntos determinados por el
↔ ↔
postulado de separación del plano se llaman semiplanos o lados de m . A m se le llama la arista o
borde de cada semiplano.
◊ Si P está en uno de los semiplanos y Q está en el otro, entonces decimos que P y Q están en lados
↔ ↔
opuestos de m y escribimos P | m | Q.

◊ Una forma de denotar un semiplano es indicando la arista y un punto contenido en él.



Así tenemos: σ ↔ indica el semiplano con arista AB y que contiene al punto C.
AB , C

◊ Si P y T están en el mismo semiplano, decimos que están al mismo lado de m y escribimos

P, T | m .

6
Elementos Fundamentales de la Geometría

↔ ___ ↔ ↔ ___ ↔
DEFINICIÓN 15. Si A, B, C, D ∈π i) C, D | AB si CD ∩ AB = φ ii) C | AB | D si CD ∩ AB = {P}
D •
C •
C • B B
P
A A
↔ ___ ↔ ↔ •
D ___ ↔
C, D ⎢ AB ⇒ CD ∩ AB = φ C ⎢ AB ⎢D ⇒ CD ∩ AB ≠ φ

Pueden probarse los siguientes teoremas sobre separación en el plano:

TEOREMA 10. ↔
↔ ↔ ↔ ↔ m
i) Si P | m | Q y Q | m | T entonces P, T | m
T •
m
↔ ↔ ↔ P • P •
ii) Si P | m | Q y Q, T | m entonces P | m | T. Q • T •
Q •

POSTULADO DE SEPARACIÓN DEL ESPACIO

Los puntos del espacio que no están en un plano dado forman dos conjuntos tales que:

i) Cada uno de los conjuntos es convexo


ii) Si P está en uno de los conjuntos y Q está en el otro, entonces el segmento PQ interseca al plano.

DEFINICIÓN 16. Los dos conjuntos determinados por el postulado de separación del espacio se llaman
semiespacios, y el plano dado se llama la cara de cada uno de ellos.

3. 1. 6. ÁNGULOS

DEFINICIÓN 17. Un ángulo es la unión de dos rayos no colineales que tienen el origen en común.

◊ Los dos rayos se llaman los lados del ángulo y el extremo común B
se llama vértice. A • C
→ →
• → •→
◊ Si los rayos se denotan por AB y AC , el ángulo se denota por
∠ BAC = AB ∪ AC
∠ BAC o ∠ CAB o ∠ A.
Cuando se utilizan tres puntos, el vértice siempre es el segundo.

◊ La notación ∠ A, es decir nombrando únicamente el vértice la usamos siempre que no se genere


confusión acerca del ángulo al cual nos estamos refiriendo, es decir que A no sea al mismo tiempo vértice
de otro ángulo.

TRIÁNGULOS

DEFINICIÓN 18. Si A, B y C son tres puntos cualesquiera no colineales, entonces la unión de los
segmentos AB , BC y AC se llama un triángulo y se denota por Δ ABC.

◊ Los puntos A, B y C se llaman vértices y los segmentos AB , BC y AC los lados del triángulo.

◊ Todo triángulo determina tres ángulos ∠ A, ∠ B y ∠ C, llamados ángulos internos del triángulo.

◊ La suma de las longitudes de los lados se llama Perímetro del triángulo.

7
Elementos Fundamentales de la Geometría

DEFINICIÓN 19. Sea ∠ BAC un ángulo en un plano π. Un punto P está en el interior del ∠BAC si

i) P y B están del mismo lado de la recta AC y

ii) P y C están del mismo lado de la recta AB
El exterior del ∠ BAC es el conjunto de todos los puntos del plano π que no están en el ángulo ni en el
interior.
Si denotamos el interior del ∠BAC por i ∠BAC y su exterior por e∠BAC, tenemos: P∈ i ∠BAC si
↔ ↔
P, B | AC ∧ P, C | AB , lo cual equivale a
i ∠BAC = σ ↔ ∩σ↔ y
AB , C AC , B
exterior
e ∠ BAC = π – ( ∠BAC ∪ i∠BAC )
interior

INTERIOR DE UN TRIÁNGULO

DEFINICIÓN 20. Un punto está en el interior de un triángulo, si está


en el interior de cada uno de los ángulos del triángulo. Un punto exterior
está en el exterior de un triángulo, si está en el plano del triángulo,
interior
pero no está en el triángulo ni en su interior. Luego

i ΔABC = i ∠A ∩ i ∠B ∩ i ∠C

Por tanto, un triángulo divide a un plano en tres conjuntos disjuntos: el triángulo mismo, su interior y el
exterior.
MEDIDA ANGULAR

POSTULADO: EL POSTULADO DE LA MEDIDA DE ÁNGULOS


A cada ángulo ∠ BAC le corresponde un número real entre 0 y 180.

NOTA: En Geometría, el concepto de ángulo se refiere a la figura geométrica formada por la unión de dos
rayos que tienen el extremo en común. Por eso “no existe” ángulo con medidas 0º ni 180º, ya que la
figura que se formaría con 0º es un rayo y con 180º, una recta. Tampoco se consideran ángulos
negativos.
En otras áreas de la Matemática difiere el concepto de ángulo usado; en Trigonometría, por ejemplo está
asociado a las rotaciones, y la medida de un ángulo puede tomar cualquier valor real, positivo o negativo.

DEFINICIÓN 21. El número dado por el postulado de la medida de ángulos se llama la medida en grados
(sexagesimales) del ∠ BAC. Se denota por m∠ BAC. Si r es el número escribimos m∠ BAC = r°

∗ Hemos de tener presente que existen otros sistemas de medidas angulares tales como el sistema
centesimal y el sistema cíclico, pero el más usado en Geometría es el sistema sexagesimal.

POSTULADO: El POSTULADO DE LA CONSTRUCCIÓN DE ÁNGULOS



Sea AB un rayo de la arista de un semiplano H. Para cada número r,

0 < r < 180, hay exactamente un rayo AC , con C en H, tal que m∠ BAC = r°.

POSTULADO: EL POSTULADO DE LA SUMA DE ÁNGULOS


D
Si D está en el interior del ∠ BAC entonces m∠ BAC = m∠ BAD + m∠ DAC C

• •
A B

8
Elementos Fundamentales de la Geometría


DEFINICION 22: Si D ∈ i ∠BAC y m∠BAD = m ∠DAC, entonces decimos que AD biseca al ∠ BAC.

Al rayo AD se le llama rayo bisector o bisectriz del ∠BAC.
DEFINICIÓN 23: PAR LINEAL
→ → →
Si AB y AD son rayos opuestos y AC es otro rayo cualquiera, C
entonces ∠ BAC y ∠CAD forman un par lineal

D A B
DEFINICIÓN 24 : Si la suma de las medidas de dos ángulos es 180º,
entonces decimos que los ángulos son suplementarios y que cada uno es el suplemento del otro.

r° s° r° + s° = 180°

POSTULADO Si dos ángulos forman un par lineal, entonces son suplementarios.


DEFINICIÓN 25. Si los ángulos de un par lineal tienen la misma medida, entonces cada uno de ellos se
llama ángulo recto.

DEFINICIÓN 26. Un ángulo recto es un ángulo cuya medida es 90°.


Un ángulo con medida menor que 90° se llama agudo.
Un ángulo con medida mayor que 90° se llama obtuso.

Agudo Recto Obtuso

→ →
DEFINICIÓN 27. Si AB y AC forman un ángulo recto, entonces se llaman perpendiculares y
→ →
escribimos AB ⊥ AC .

Empleamos el mismo término y la misma notación para rayos y segmentos:


↔ ↔ →
Si ∠ BAC es un ángulo recto, entonces AB ⊥ AC , AB ⊥ AC , AB ⊥ AC , etc.

◊ En general decimos que dos segmentos, dos rayos o un segmento y un rayo son perpendiculares si
están contenidos en rectas perpendiculares.

En los dibujos la perpendicularidad se B A B


representa dibujando un pequeño cuadrado o
un paralelogramo en el vértice. C D
A ___ ___ ___ ___ C
AB ⊥ AC AD ⊥ DC

DEFINICIÓN 28. Si la suma de las medidas de dos ángulos es 90°, entonces los ángulos se llaman
complementarios y cada uno de ellos es el complemento del otro.
DEFINICIÓN 29. Dos ángulos con la misma medida se llaman congruentes.

m ∠BAC = m ∠ DEF ⇔ ∠ BAC ≅ ∠ DEF

9
Elementos Fundamentales de la Geometría

TEOREMAS SOBRE CONGRUENCIA DE ÁNGULOS

1. Para ángulos la relación de congruencia es una relación de equivalencia (reflexiva, simétrica y


transitiva)

2. Si dos ángulos son complementarios, entonces ambos son agudos

3. Dos ángulos rectos cualesquiera son congruentes.


4. Si dos ángulos son a la vez congruentes y suplementarios, entonces cada uno de ellos es un ángulo
recto.

5. Los complementos de ángulos congruentes son congruentes.

6. Los suplementos de ángulos congruentes son congruentes.

3
4
2 4
1 3 2 1
∠1 ≅ ∠3 ⇔ ∠ 2 ≅ ∠4 ∠1 ≅ ∠3 ⇔ ∠ 2 ≅ ∠4

DEFINICIÓN 30. Decimos que dos ángulos son opuestos por el vértice o que forman un par vertical,
si sus lados forman dos pares de rayos opuestos.

1
2 4
3
∠1 ≅ ∠3 y ∠ 2 ≅ ∠4

TEOREMA 17. Los ángulos opuestos por el vértice son congruentes.

TEOREMA 18: Si dos rectas que se cortan forman un ángulo recto, entonces forman cuatro ángulos
rectos.

TEOREMA 19: En un plano dado y por un punto dado de una recta pasa una y solamente una recta
perpendicular a la recta dada.

DEFINICIÓN 31. En un plano dado, la mediatriz de un segmento es la recta perpendicular al segmento


en su punto medio.

A ↔
m

B
TEOREMA 20: La mediatriz de un segmento en un plano, es el conjunto de todos los puntos del plano
que equidistan de los extremos del segmento.

Nota: Se necesitan los teoremas sobre congruencia triángulos para la demostración de este teorema.
↔ ↔
COROLARIO: Dados un segmento AB y una recta m en el mismo plano, si dos puntos de m

equidistan de A y de B, entonces m es la mediatriz de AB .

10
Elementos Fundamentales de la Geometría

↔ ↔
TEOREMA 21: Sea m una recta y P un punto fuera de m . Entonces hay una recta que es

perpendicular a m y contiene a P.

COROLARIO: Ningún triángulo tiene dos ángulos rectos.

3. 1. 7. PARALELISMO EN EL PLANO

RECTAS PARALELAS

DEFINICION 32. Dos rectas diferentes son paralelas si m2
i) están en un mismo plano ↔
ii) no se intersecan. m1

↔ ↔ ↔ ↔
Notación: Escribimos m1 ⎜⎜ m2 para indicar que m1 y m2 son paralelas.
∗ Convencionalmente se considera que toda recta es paralela consigo misma.

DEFINICIÓN: Dos segmentos, dos rayos o un segmento y un rayo son paralelos, si están contenidos en
rectas paralelas.

DEFINICIÓN 33: Dos rectas que no están en un mismo plano se llaman


rectas alabeadas.

TEOREMA 22: Dos rectas paralelas están exactamente en un plano.

TEOREMA 23: Dos rectas en un plano son paralelas, si ambas son perpendiculares a la misma recta.

s ↔
m1
↔ ↔ ↔ ↔ ↔ ↔
m1 ⊥ s ∧ m 2 ⊥ s ⇒ m1 || m 2

m2

P
V POSTULADO DE EUCLIDES •

Por un punto externo a una recta pasa exactamente una recta paralela m
a la recta dada.

Este postulado es la base de la geometría Euclidiana. La negación del mismo da lugar a las llamadas
Geometrías No Euclidianas.

DEFINICIÓN 34: Una secante a dos rectas coplanarias es una recta que las interseca en puntos
diferentes.

En los puntos de intersección de la secante con las rectas se forman ocho ángulos, y las parejas que se
forman reciben nombres particulares.

11
Elementos Fundamentales de la Geometría


s

2 1
m1
3 4

6
5
7 8 ↔
m2

ANGULOS CORRESPONDIENTES: Las parejas ∠1 y ∠5; ∠2 y ∠6;


∠3 y ∠7; ∠4 y ∠8

ANGULOS INTERNOS: Los ángulos ∠3, ∠4, ∠5 y ∠6.

ANGULOS EXTERNOS: Los ángulos ∠1, ∠2, ∠7 y ∠8.

ANGULOS ALTERNOS INTERNOS: Las parejas ∠3 y ∠5; ∠4 y ∠6.

ANGULOS ALTERNOS EXTERNOS: Las parejas ∠1 y ∠7; ∠2 y ∠8.

ANGULOS INTERNOS A UN MISMO LADO: Las parejas ∠3 y ∠6, ∠4 y ∠5.

ANGULOS EXTERNOS A UN MISMO LADO: Las parejas ∠1 y ∠8, ∠2 y ∠7.

TEOREMAS SOBRE PARALELISMO

Si dos rectas paralelas son cortadas por una secante, entonces se


cumplen los siguientes Teoremas:
1. Los ángulos correspondientes son congruentes.

2. Los ángulos alternos internos son congruentes.

3. Los ángulos alternos externos son congruentes

4. Los ángulos internos a un mismo lado de la secante son suplementarios.

5. Los ángulos externos a un mismo lado de la secante son suplementarios.

Al asumir como válido el V Postulado de Euclides, también son válidos los recíprocos de los teoremas
anteriores, es decir si se da la congruencia señalada en los teoremas 1, 2 y 3 o la condición de ángulos
suplementarios señalados en el 4 y 5, podemos concluir que las rectas son paralelas.
a a a a
b b b
b a b b a a
b

Ángulos Ángulos Ángulos


Ángulos Ángulos
Alternos-externos Externos al mismo
Correspondientes Internos al mismo Alternos – internos
lado lado
∠a ≅ ∠b ∠a ≅ ∠b m∠a+ m∠b = 180º
∠a ≅ ∠b m∠a + m∠b = 180º
TEOREMA 29: La suma de los ángulos internos de un triángulo es 180°.

12
Elementos Fundamentales de la Geometría

EJEMPLO 1

En la figura, a partir de la información dada determine el valor de x:

De la figura deducimos que los ángulos indicados forman un


3xº par lineal, y por tanto son suplementarios, luego:
45º
3x + 45 = 180 ∴3x = 180 – 45 = 135,
luego x = 135 / 3 = 45

EJEMPLO 2

F
θº
A
B
C En la figura, A – B – C – D, m ∠ ECD = 100º,
40º m ∠ BEC = 40º. Determine el valor de θ.
D
100º
E

SOLUCIÓN:
Se tiene que ∠BCE y ∠ECD forman un par lineal, luego son suplementarios:
m∠BCE = 180 – m∠ECD = 180º – 100º = 80º (1)

∠ABF y ∠EBC son opuestos por el vértice, luego son congruentes:


m∠EBC = m∠ABF = θº (2)

∠EBC, ∠BCE y ∠BEC son los ángulos internos del triángulo EBC, luego sus medidas suman 180º:
De (1), (2) y el dato sobre la medida del ángulo BEC, se tiene:
θ + 80 + 40 = 180, de donde obtenemos θº = 60º.

EJEMPLO 3

A continuación se presentan dos rectas paralelas y una secante que determinan ángulos cuyas medidas
se indican en la figura. Determine los valores de x y y.

2x De la figura se deduce que los ángulos cuyas medidas son 2x y


3x – 40, forman una pareja de ángulos alternos – internos entre
3x – 40 paralelas y por tanto son congruentes. Por otro lado los ángulos
cuyas medidas son 2x y y + 18 son ángulos correspondientes
y + 18 entre paralelas y por tanto también son congruentes, luego:

3x – 40 = 2x ⇒ x = 40, y + 18 = 2x = 2 (40) = 80 ∴ y = 80 – 18 = 62.

EJEMPLO 4

↔ ↔
En la figura, m1 || m 2 . A partir de la información dada, determine el valor de x.

13
Elementos Fundamentales de la Geometría


60º m1
110º SOLUCIÓN:
↔ Si trazamos una línea auxiliar paralela a las rectas dadas que
xº pase por el vértice del ángulo que mide 110º, se forman ángulos
m2
alternos – internos entre paralelas con los ángulos que miden 60º
y xº, luego se tiene:
60º
60º
x + 60 = 110 ∴ x = 110 – 60 = 50

EJERCICIOS SOBRE CONCEPTOS BÁSICOS.

__
1. En la figura A – B – C – D – E. B es punto medio de AE , C es
__ __
punto medio de BE y D es punto medio de CE . A B C D E
a) si AE = 40, encuentre AB, AC, AD, BC, BD y CD
b) si BD = 9, encuentre AB, BE y CD.

2. En la figura A – B – C – D – E. AB = DE = 2 CD. BC = AB + 1
A B C D E AE = 50. Encuentre AB, BC, CD y DE.

3. ¿Cuántas rectas pasan por un punto? ¿Cuántos planos pasan por un punto?

4. ¿Cuántas rectas pasan por dos puntos? ¿Cuántos planos pasan por dos puntos?

5. Dados tres puntos no colineales A, B y C. ¿Cuántas rectas determinan? ¿Cuántos planos?

6. Dados cuatro puntos coplanares, A, B, C y D, tales que tres cualesquiera de ellos no son colineales,
¿cuántas rectas determinan?

7. Dados cinco puntos coplanares A, B, C, D y F, tales que tres cualesquiera de ellos no son colineales,
¿cuántas rectas determinan?

8. Dados n puntos coplanares A1, A2, A3,... An, tales que tres cualesquiera de ellos no son colineales,
¿cuántas rectas determinan?
↔ ↔ → →
9. i) ¿Es AB = BA ? ¿Por qué? ii) ¿Es AB = BA ? ¿Por qué?
___ ___ ___
iii) ¿Es AB = BA ? ¿Por qué? iv) ¿Es AB = AB? ¿Por qué?
→ ___ ↔ ___
10. ¿Cuál es la intersección de CD y DC ? ¿ y la de CD y DC ?

11. Si A, B y C son tres puntos colineales tales que AC + BC = AB,


→ → → → → →
¿cuál es la intersección de CB y BA ? ¿de AC y AB ? ¿de CA y CB ?

14
Elementos Fundamentales de la Geometría

12. Si A, B, C y D son puntos colineales, tales que A – B – C – D, determine


__ __ → → → → ___ ___ → →
i) CA ∩ BD ii) BA ∩ BD iii) AB ∩ CD iv) AB ∩ BC v) BC ∩ CA
→ ___ → → ___ ___
vi) BC ∩ AC vii) AC ∩ DC viii) BA ∩ BD
E
13. Para la figura plana siguiente, determine:

___ ___ ___ ___ ___ F


i) FB ∩ AD ii) AE ∩ Δ FGA iii) FE ∪ AF
G D
___ ___ ___ ___ A
iv) AG ∪ AD v) AB ∩ EF
B C

14. La siguiente figura representa una pirámide cuadrada.


Nombre los planos que determinan los vértices (siete planos).
D

15. Si A, B, C y D son puntos distintos tales que AC contiene a B
A C

y BD contiene a C, ¿Cuáles de los siguientes enunciados tienen que ser
ciertos? B
↔ ↔ ↔ → →
i) B está entre A y C ii) BC contiene a A iii) AC = BD iv) AC es opuesto a DB
↔ ↔ ↔ ↔
v) AC y BD se intersecan en B y C solamente vi) AD y BC no se intersecan.

16. Responda cada una de las siguientes preguntas, explicando su respuesta


i) ¿Es una recta un conjunto convexo?
ii) ¿Es convexo un conjunto que consiste solamente en dos puntos?
iii) Si le quitamos un punto a una recta, ¿formarán los puntos restantes un conjunto convexo?
iv) ¿Es un segmento un conjunto convexo?
v) ¿Es un rayo un conjunto convexo?
vi) ¿Es una circunferencia un conjunto convexo?
vii) ¿Es un círculo un conjunto convexo? C
viii) ¿Es todo plano un conjunto convexo?
A B
17. En la siguiente figura, ¿cuáles de las regiones marcadas con letras
D
mayúsculas son conjuntos convexos?


18. Dos planos α y β se intersecan en AB . Cada uno de los puntos P y Q está en los planos α y β , ¿se

puede concluir que P y Q están en AB ? Explique.

SOLUCIONES:
1. a) AB = 20, AC = 30, AD = 35, BC = 10, BD = 15, CD = 5 b) AB = 12, BE = 12, CD = 3,
2. AB = 14, BC = 15, CD = 7 y DE = 14 3. Infinitas rectas e infinitos planos.
4. Sólo una recta; infinitos planos. 5. Tres rectas; un solo plano
n(n− 1)
6. 6 rectas 7. 10 rectas 8. 9. i) Si, representan la misma recta
2
ii) No, representan distintos rayos iii) Si, representan el mismo segmento

15
Elementos Fundamentales de la Geometría

___
iv) No, AB es un segmento, es decir un conjunto de puntos, mientras AB es la longitud del segmento
___ → ___ ___ ↔ ___ ___
AB , es decir un número. 10. CD ∩ DC = DC , CD ∩ DC = DC
→ → __ → → → → → __ __ __ → →
11. CB ∩ BA = BC , AC ∩ AB = AB , CA ∩ CB = {C} 12. i) CA ∩ BD = BC ii) BA ∩ BD = {B}
→ → → ___ ___ → → __ → ___ __ → → __
iii) AB ∩ CD = CD iv) AB ∩ BC = {B} v) BC ∩ CA = BC vi) BC ∩ AC = BC vii) AC ∩ DC = AD
___ ___ ___ ___ ___ __ ___ ___ __
viii) BA ∩ BD = {B} 13. i) FB ∩ AD = {G} ii) AE ∩ Δ FGA = AF iii) FE ∪ AF = AE
___ ___ ___ ___ ___
iv) AG ∪ AD = AD v) AB ∩ EF = Φ 14. ABC, ABE, ADE, ACE, BCE, BDE, CDE.

15. (ii) y (iii) 16. i) si ii) no iii) no iv) si v) si vi) no vii) si viii) si. 17. A y D 18. P, Q ∈ AB .

EJERCICIOS SOBRE ÁNGULOS


D
C
1. En la figura, los puntos A, B y C son colineales. B
Nombre los cinco ángulos que se forman. A E

2.
D
i) ¿Cuántos ángulos están determinados en la figura
de la izquierda? Nómbrelos.
ii) ¿Cuántos de ellos es posible nombrarlos utilizando
A C solamente la letra del vértice?
B
A
3. En la figura A, B y C son colineales. D E
Si m ∠ DBA = m ∠ EBA
i) Nombre las parejas de ángulos que forman un par lineal
ii) Nombre las parejas de ángulos suplementarios. B
C

4. E
En la figura plana de la izquierda
i) m ∠CAB + m ∠DAC = m ∠____
A D ii) m ∠EAD + m ∠DAC = m ∠____
iii) m ∠EAD + m ∠DAB = m ∠____
iv) m ∠EAC – m ∠DAC = m ∠____
B C

S
5. En la siguiente figura
i) m ∠SPR + m ∠QPO = m ∠____ R
ii) m ∠RSQ + m ∠___ = m ∠ RSP O
iii) m ∠POQ + m ∠POS = ____
P
iv) m ∠SRQ – m ∠SRO = m ∠____
v) m ∠ROQ = 180° – m ∠____ Q

6. Si la medida de un ángulo es tres veces la medida de su suplemento, ¿cuál es la medida de dicho


ángulo?

16
Elementos Fundamentales de la Geometría

7. Dos veces la medida de un ángulo es 30° menos que cinco veces la medida de su complemento,
¿cuál es la medida de dicho ángulo?

8. Encuentre la medida del ángulo cuya medida es 50° mayor que


a) su complemento b) su suplemento.
9. Dos ángulos son complementarios. Tres veces la medida de uno de ellos es 30° más que el doble de
la del otro. Encuentre las medidas de los ángulos.

F E 10. A partir de la figura conteste las siguientes


G preguntas
D a) ¿ Es m ∠ BFC = m ∠ BFD ?
b) ¿ Es ∠ BFC = ∠ BFD ?
c) ¿ Es ∠ FDB ≅ ∠ EDC ?
A d) Nombre el ángulo suplementario del ∠ ABF.
B C

11. Se da la figura con el vértice B del ángulo recto ∠ DBE en AC y m ∠EBC = 45°. A
D
i) Nombre un par de rayos perpendiculares, si hay alguno
ii) Nombre un par de ángulos complementarios, si hay alguno. B
iii) Nombre un par de ángulos suplementarios, si hay alguno
iv) Nombre un par de ángulos congruentes, si hay alguno. E
C

12. En la figura, tres rectas coplanares se cortan en el mismo punto. Se tiene


a a = 75° y e = 45°.
f b
Hallar b, c, d y f.
e c
d

13.
A partir de la información dada en la figura,
(2x + 18)° (x + 15)° encuentre el valor de x.
K
→ → G
14. En la figura plana que se muestra a continuación, BA y BE son D
rayos opuestos, ∠ ABG ≅ ∠ KBG
y ∠KBD ≅ ∠DBE. Hallar m ∠ GBD.
A B E
15.
D
2x° → ↔
(x + 30)° Si A, B y C son colineales, BD ⊥ AC , hallar el valor de x
A B C
16.

P S
A partir de la información dada en la figura,
R Q 61° T halle m ∠ PQS

17
Elementos Fundamentales de la Geometría

17.
A __ __
Si AD ⊥ CD y m ∠ ADE = 120°,
B D E ¿Cuánto mide el ∠ BDC?
¿Cuánto mide el ∠ CDE?
C

18. T Q
↔ ↔ ↔
PQ , RS y TV se cortan en O.
R O S
a) Si m ∠ ROQ = 142° y m∠ TOS = 129°,
encuentre m ∠ROT y m ∠ POV
b) Si m ∠ ROV = 120° y m ∠ QOT = 47°,
P V
determine m ∠ SOQ.
19. P• → ↔
Q, R y S son colineales. RP ⊥ QS .
3
2 4 m∠ 1 = 45°, m∠ 3 = 15°,
1 5 m∠ 2 = m ∠4 . Encuentre m∠ 5.
• • •
Q R S

20. m1 m3
60°
m2 m4


↔ ↔ ↔ ↔
c° En la figura m1 || m2 y m3 || m 4 , determine

las medidas de los ángulos indicados

21. En las siguientes figuras, los rayos, segmentos y rectas paralelas se representan colocándoles
puntas de flecha en el mismo sentido. En cada uno de ellos determine el valor del ángulo indicado

a) b) c)

x° x° 75° 45°
40°
30° 88°

d) e) f)
x° 35° 2x °

50° (3x – 20)°


y° x° (y + 8)°

18
Elementos Fundamentales de la Geometría

g) h) i)
84°
(5x + 30)° 135°
(x – 6)° x°
(3x + 18)° (3x + 10)° 150°

j) k) l)

150° 55° x° 118°

(x – 2y)° (x + y)° x°
65° (2x + 10)°

22. A w° B
y° z°

x° → →
Si AB || CD y x = 62, encuentre los valores de y, z y w.
C D


↔ ↔
23 1 3 m1 Si m1 || m2 , m ∠ 1 = 60° y m ∠ 6 = 40°.
2
Encuentre las medidas de los ángulos 2, 3, 4, 5 y 7.
4 5 6 7 ↔
m2
.
24. ↔ ↔ ↔ ↔
x° m1 Si m1 || m2 || m3 , determine x + y

m2


m3

25. 26. C
110° F
y° ___ ___ ___ ___
A B Si AB || DE y CD || FE ,

¿cuál es el valor de x + y?
¿Cuál es el valor de x?
D x° 108° E

27.
A E
__ __
D Si AE || BC , cuánto vale m∠E + m∠D + m∠C?

B C

19
Elementos Fundamentales de la Geometría

28. ↔ ↔ ↔ ↔
m1 m2 m3 m4 ↔ ↔ ↔ ↔
En la figura m1 ||m2 y m3 || m 4 .
Determine el valor de x y y.

40° y°
55° x°

29.
En la figura, m∠ 1 = (2x – 7)°,
3 m∠ 2 = (5x + 3)° y m∠3 = (6x + 2)° .
4 1 2 Halle la medida del ángulo 4.

30 31.
A
b° Exprese x en
términos de b. Si AB = AC y m ∠B = a°,
determine m∠A en término de a.

B C

A El Δ ABC es isósceles con AB = AC.


32. → →
70° BD y CD son bisectores de los ángulos de la
base. Si m∠ A = 70°, encuentre la medida del
D ∠ BDC.
B C

D
33
. BD biseca al ∠ADC.
Si m ∠ DBC = 105° y m ∠ DCB = 55°, halle
α° β° los valores de α y β.
A B C

SOLUCIONES:
1. ∠ABD, ∠ABE, ∠DBC, ∠CBE, ∠DBE 2. i) ∠DAB, ∠DBC, ∠DBA, ∠ADC, ∠ADB, ∠BDC, ∠BCD ii)
∠A y ∠C. 3. i) ABD y DBC; ABE y EBC ii) ABD y DBC; ABE y EBC; ABD y EBC; ABE y DBC
4. i) BAD ii) EAC iii) EAB iv) EAD 5. i) SPQ ii) QSP iii) 180º iv) ORQ v) ROS ó POQ
→ →
6. 135º 7. 60º 8. a) 70º b) 115º 9. 42º y 48º 10. a) si b) si c) si d) ∠FBC 11. i) BD y BE
ii) ∠ABD y ∠EBC iii) ∠ABD y ∠DBC; ∠EBC y ∠EBA; ∠ABD y ∠EBC 12. b = 45º, c = 60º, d = 75º,
f = 60º 13. 49º 14. 90º 15. 20º 16. 29º 17. a) 30º b) 150º 18. a) 51º y 91º b) 73º 19. 30º
20. a = c = 60º b = d = 120º 21. a) 140º b) 45º c) 43º d) x = 50º, y = 130º e) 55º f) x = 20º, y = 32º
g) x = 16.5º h) x = 20º i) x = 75º j) x = 110º, y = 40º k) x = 120º l) x = 36º
22. y = w = 62º, z = 118º 23. m∠2 = 80º, m∠3 = 40º, m∠4 = 120º, m∠5 = 60º, m∠7 = 140º
24. 270º 25. x = 70º 26. x + y = 180º 27. 360º 28. x = 85º, y = 95º 29. m∠4 = 159º
30. x = 90 – b/2 31. m∠A = 180º – 2aº 32. 125º 33. α = 85º, β = 125º

20
Elementos Fundamentales de la Geometría

3.2. TRIÁNGULOS Y CUADRILÁTEROS


INTRODUCCIÓN:

Anteriormente definimos un triángulo, denotado por Δ ABC, como la unión de los segmentos
AB , BC y AC , siendo A, B y C puntos no colineales. Además tenemos que los puntos A, B y C se
llaman vértices, los segmentos AB , BC y AC los lados del triángulo y los ángulos determinados en
los vértices, ∠ A, ∠ B y ∠ C, son llamados ángulos internos del triángulo.

De manera similar si A, B, C y D son cuatro puntos coplanares, tales que no hay tres colineales y los
___ ___ ___ ___
segmentos AB , BC , CD y DA únicamente se intersecan en los extremos, entonces la unión de estos
segmentos forman el cuadrilátero ABCD.

Los triángulos y cuadriláteros constituyen dos de las figuras más importantes, por cuanto las
encontramos en muchos de los objetos que nos rodean y por otro lado sirven de base para el estudio de
otras figuras más complejas.

3. 2. 1. CLASIFICACIÓN DE TRIÁNGULOS:

◊ De acuerdo a las medidas de sus lados los triángulos se clasifican en isósceles, equiláteros y
escalenos.

◊ De acuerdo a las medidas de sus ángulos tenemos triángulos rectángulos, equiángulos, acutángulos y
obtusángulos.

DEFINICIÓN 1: Un triángulo con dos lados congruentes se llama isósceles.


Al otro lado generalmente se le llama base y a los ángulos asociados con la base se les llaman ángulos
en la base. Al ángulo opuesto a la base se le llama ángulo en el vértice.

Vértice
A

__ __
B C AB ≅ AC y ∠B ≅ ∠C
Base

Nota: En general, cualquiera de los lados de un triángulo es una base y cualquiera de los
puntos extremos es un vértice, pero en el caso particular de los triángulos isósceles se
acostumbra referirse al “vértice” y a la “base”, tal como se indica en la definición anterior.

TEOREMA 1: Los ángulos en la base en un triángulo isósceles son congruentes.


(El recíproco también es válido)

TEOREMA 2: Si dos ángulos de un triángulo son congruentes entonces


es un triángulo isósceles.

21
Elementos Fundamentales de la Geometría

DEFINICIÓN 2: Un triángulo con sus tres lados congruentes se llama equilátero.

DEFINICIÓN 3: Un triángulo con sus tres ángulos congruentes A


se llama equiángulo.

TEOREMA 3: Todo triángulo equilátero es equiángulo y


viceversa. Cada ángulo de un triángulo equilátero mide 60°.
B C

AB ≅ AC ≅ BC , ∠A ≅ ∠ B ≅ ∠C
DEFINICIÓN 4: Un triángulo para el cual dos lados
cualesquiera no son congruentes, se llama escaleno.

DEFINICIÓN 5: Un triángulo rectángulo, es un triángulo que tiene un ángulo recto. El lado opuesto al
ángulo recto se llama hipotenusa y a los otros dos lados se les llama catetos.

B C B

C C
A B A A
En los triángulos dibujados, tenemos:
___ __ __
AB : hipotenusa; AC y BC : catetos

DEFINICIÓN 6: Un triángulo se le llama acutángulo, si sus tres ángulos son agudos, y se le llama
obtusángulo si tiene un ángulo obtuso.
A los triángulos acutángulos y a los obtusángulos se les llama oblicuángulos.

3. 2. 2. RECTAS Y PUNTOS NOTABLES EN UN TRIÁNGULO

DEFINICIÓN 7: Una mediana de un triángulo es un segmento cuyos extremos son un vértice del
triángulo y el punto medio del lado opuesto.

◊ Todo triángulo tiene tres medianas, una para cada vértice A


(lo que equivale a decir una para cada lado).
Puede probarse que las medianas se cortan en un punto
llamado baricentro.
P Baricentro
◊ También puede probarse que la distancia del baricentro a
un vértice es el doble de la distancia del baricentro al punto
medio correspondiente. Es decir la distancia del punto M
medio correspondiente al baricentro es 1/3 la medida de la AP = 2⋅ PM, AM = 3⋅PM
mediana.

DEFINICIÓN 8: Si D ∈ i ∠BAC y ∠BAD ≅ ∠DAC, entonces decimos que AD biseca al ∠ BAC.

Al rayo AD se le llama rayo bisector o bisectriz del ∠BAC.

TEOREMA 4: Todo ángulo tiene exactamente una bisectriz. B

◊ Una propiedad de la bisectriz es que todo punto de ella equidista de D


los lados del ángulo.
A C

22
Elementos Fundamentales de la Geometría

DEFINICIÓN 9: Un segmento es una bisectriz de un


ángulo de un triángulo, si: i) está en el rayo bisector del ángulo y ii) sus extremos son el vértice del
ángulo y un punto del lado opuesto.

◊ Todo triángulo tiene tres bisectrices, una para cada ángulo. Puede probarse que se cortan en un punto
llamado incentro, y que el incentro equidista (igual distancia) de cada lado del triángulo. El incentro
siempre pertenece al interior del triángulo.

◊ El incentro es el centro de la circunferencia que puede inscribirse en el triángulo es decir tangente a los
tres lados.

Incentro

DEFINICIÓN 10: Una recta es una mediatriz de un lado de un triángulo, si está en el plano que contiene
al triángulo y es mediatriz de dicho lado (es decir, una recta perpendicular al lado, que pasa por su punto
medio).

◊ Una propiedad de la mediatriz es que todo punto de ella equidista de los extremos del segmento.

◊ Todo triángulo tiene tres mediatrices, una para cada lado. Puede probarse que se cortan en un punto
llamado circuncentro y que el circuncentro equidista de cada vértice.

◊ En los triángulos acutángulos, el circuncentro se halla en el interior del triángulo, mientras que en los
obtusángulos se halla en el exterior del triángulo. Para los triángulos rectángulos el circuncentro coincide
con el punto medio de la hipotenusa.

◊ El circuncentro es el centro de la circunferencia que puede circunscribirse a un triángulo, es decir que


pasa por los vértices del triángulo.

Circuncentro
P
B C
AP = BP = CP

DEFINICIÓN 11:
i) La altura correspondiente a un lado de un triángulo es un segmento perpendicular a dicho lado, cuyos
extremos son el vértice del ángulo opuesto y un punto de la recta que contiene a dicho lado.

ii) La altura correspondiente a un lado de un triángulo también es la distancia desde el vértice del ángulo
opuesto a dicho lado a la recta que contiene a dicho lado.

◊ Puede probarse que las rectas que contienen a las alturas se cortan en un punto llamado ortocentro.
◊ En los triángulos acutángulos, el ortocentro se halla en el interior del triángulo, mientras que en los
obtusángulos se halla en el exterior del triángulo. Para los triángulos rectángulos el ortocentro coincide
con el vértice del ángulo recto.

23
Elementos Fundamentales de la Geometría

A A

• hA
hB
B C
B C
hC Ortocentro

• Ortocentro

TEOREMA 5: En un triángulo isósceles, la mediana, la bisectriz, la mediatriz y la altura correspondientes


a la base son coincidentes.

TEOREMA 6: En un triángulo equilátero, el baricentro, el incentro, el circuncentro y el ortocentro


coinciden.
◊ En un triángulo no equilátero, el baricentro, el circuncentro y el ortocentro son colineales. La recta que
los contiene se llama recta de Euler.

3. 2. 3. CONGRUENCIA DE TRIÁNGULOS

En general dos figuras geométricas son congruentes si tienen exactamente la misma forma y tamaño.
Anteriormente hemos visto la congruencia de segmentos y de ángulos, estableciendo la misma a partir de
la igualdad de sus medidas.
Recordemos que el concepto de congruencia está muy cercano al concepto de igualdad, sin embargo
son conceptos diferentes aunque frecuentemente se confunden.
La igualdad es estrictamente la relación entre dos nombres o dos expresiones para el mismo “objeto”, en
cambio la congruencia en geometría, relaciona dos “objetos” que tienen la misma forma y tamaño.

Si dotamos de movimiento rígido a las figuras geométricas, es decir si trasladamos, rotamos o reflejamos,
sin que se modifique su forma y tamaño, podemos afirmar que dos figuras geométricas son congruentes
cuando al superponerlas se ajustan “exactamente”.

Veamos como describir la relación de congruencia entre triángulos:


C F H

A B D E I G
Consideremos el triángulo ABC. Si lo desplazamos hacia la derecha y lo superponemos al ΔDEF,
veremos que se ajustan exactamente. En este caso el punto A quedará sobre el punto D, el punto B
sobre el E y el punto C sobre el F. Se presenta una correspondencia A ↔ D, B ↔ E y C ↔ F.
Observamos que coinciden los lados correspondientes y los ángulos internos, lo cual podemos indicar
usando la notación Δ ABC ≅ Δ DEF.

Si intentamos superponer el Δ ABC sobre el Δ GHI, vemos que no basta desplazar el triángulo ABC hacia
la derecha, sino que hemos de “invertir” el ΔABC, es decir realizar una reflexión sobre el lado AC , de
manera que el vértice A corresponda al vértice G, el vértice B corresponda al vértice I y el vértice C al H.

24
Elementos Fundamentales de la Geometría

Se presenta ahora la correspondencia A ↔ G, B ↔ I y C ↔ H. Además se observa que también se


ajustan los lados y ángulos correspondientes. Esto lo denotamos por Δ ABC ≅ Δ GIH.
Recordemos que los dibujos de las figuras geométricas son auxiliares, no siempre se trazan con
exactitud, de manera que es necesario ser precisos con la simbología utilizada.

DEFINICIÓN 12: Sea ABC ↔ DEF una correspondencia entre los vértices de dos triángulos. Si los pares
de lados correspondientes son congruentes y los pares de ángulos correspondientes son congruentes,
entonces la correspondencia ABC ↔ DEF se llama una congruencia entre los dos triángulos y decimos
que los triángulos son congruentes. Esta relación se denota por Δ ABC ≅ Δ DEF.
Se tiene entonces:
⎧ __ __
⎪ AB ≅ DE ∠ A ≅ ∠D
⎪⎪ __ __
Δ ABC ≅ ΔDEF ⇔ ⎨ BC ≅ EF ∠B≅ ∠E
⎪ __ __
⎪ CA ≅ FD ∠C≅ ∠F
⎪⎩
Notemos que el orden en que se escriben los vértices es muy importante: deben reflejar la
correspondencia entre los lados y los ángulos respectivos:

Δ ABC ≅ Δ DEF

Se acostumbra nombrar las longitudes de los lados de un triángulo usando la respectiva letra minúscula
__ __ __
usada en el vértice opuesto. Así tenemos en el ΔABC, m ( AB ) = c, m ( AC ) = b y m ( BC ) = a.

DEFINICIÓN 13: Un lado de un triángulo se dice estar comprendido por los ángulos cuyos vértices son
los extremos del segmento. De manera similar, un ángulo de un triángulo se dice estar comprendido por
los lados del triángulo que están en los lados del ángulo.

∗ Recordemos que un ángulo es la unión de dos rayos y que un triángulo es la unión de tres segmentos.
Ejemplo.
C
__
En el Δ ABC, AB está comprendido por los ángulos A y B.
__ __
El ∠ C está comprendido por los lados AC y BC
A B
TEOREMAS SOBRE CONGRUENCIA DE TRIÁNGULOS

TEOREMA 7: L.A.L ( Lado – Angulo – Lado)


Toda correspondencia L.A.L. es una congruencia. Es decir, si dos lados de un triángulo y el
ángulo comprendido por dichos lados, son congruentes respectivamente a dos lados y el ángulo
comprendido por dichos lados de un segundo triángulo, entonces los triángulos son congruentes.

C F

Si AC ≅ DF , AB ≅ DE y ∠ A ≅ ∠ D
entonces Δ ABC ≅ Δ DEF
A B D E

25
Elementos Fundamentales de la Geometría

TEOREMA 8: A.L.A. (Angulo – Lado – Angulo)


Toda correspondencia A.L.A. es una congruencia. Es decir, si dos ángulos de un triángulo y el lado
comprendido por dichos ángulos son congruentes respectivamente a dos ángulos y al lado
comprendido por dichos ángulos de un segundo triángulo, entonces los triángulos son congruentes
C F

Si ∠ A ≅ ∠ D , ∠ B ≅ ∠ E y AB ≅ DE
entonces Δ ABC ≅ Δ DEF
A B D E
TEOREMA 9: L.L.L. ( Lado – Lado – Lado)
Toda correspondencia L.L.L. es una congruencia. Es decir si los tres lados de un triángulo son
congruentes respectivamente a los tres lados de un segundo triángulo, entonces los triángulos son
congruentes
C F

Si AB ≅ DE , AC ≅ DF y BC ≅ EF
entonces Δ ABC ≅ Δ DEF

A B D E

EJEMPLOS

1. Demostrar que si dos segmentos se bisecan, entonces los segmentos que unen los extremos de los
segmentos dados son congruentes.

SOLUCIÓN:
1ro: Dibujamos una figura que refleje la situación planteada y rotulamos los puntos, indicando los datos
dados. En el ejemplo se nos plantea que los segmentos se bisecan, es decir se cortan en el punto medio.
No hay información sobre las longitudes de los segmentos, por tanto podemos tomarlos de longitud
arbitraria.

2do: Enunciamos la hipótesis y tesis simbólicamente.


A D
Hipótesis: AC y DE se bisecan.
Tesis: AE ≅ CD !
!
3ro: A partir de los conocimientos previos: teoremas, definiciones, etcétera, B
buscamos una estrategia de demostración.
En este caso vemos que una forma de llegar a la conclusión buscada es a partir de la E C
congruencia de triángulos.
Si logramos probar que los triángulos ABE y CBD son congruentes, los segmentos en cuestión resultan
ser partes correspondientes. Esta vía surge al observar que tenemos dos pares de lados congruentes por
dato, y en la figura se forman dos ángulos opuestos por el vértice.

4to: Escribimos las afirmaciones que vamos deduciendo y su respectiva justificación.

26
Elementos Fundamentales de la Geometría

Afirmación Justificación
__ __
1 AC y DE se bisecan en B Dato
__ __
2 AB ≅ BC Definición de bisecar
__ __
3 EB ≅ BD Definición de bisecar
4 ∠ ABE ≅ ∠CBD Ángulos opuestos por el vértice
5 Δ ABE ≅ Δ CBD Teorema LAL (pasos 2,3 y 4)
__ __ Partes correspondientes de triángulos congruentes
6 AE ≅ CD

2. Demostrar que si en el Δ GHK, GK = HK y G – M – H , tal que ∠ GKM ≅ ∠ HKM, entonces M es el punto


medio de GH

Datos: GK = HK
G–M–H
∠ GKM ≅ ∠ HKM
Tesis : M es punto medio de GH

G ! M ! H

Afirmación Justificación
1 __ __
GK ≅ HK Por dato y definición de congruencia
__ __
2 KM ≅ KM Por reflexividad
3 ∠ GKM ≅ ∠ HKM Dato
4 Δ GKM ≅ Δ HKM Teorema LAL ( pasos 1,2 y 3)
__ __
5 GM ≅ HM Partes correspondientes en triángulos
congruentes
6 G–M–H Dato

__
7 ∴ M es punto medio de GH Pasos 5, 6 y definición de punto medio

EJERCICIOS PROPUESTOS SOBRE CONGRUENCIA


1. a) Los triángulos ΔABC y ΔDEF no se intersecan y M es un punto entre B y C. ¿Cuál de los dos
símbolos = o ≅ corresponde colocar en cada uno de los espacios en blanco para completar un
enunciado que tenga sentido y posiblemente sea cierto?
i) ΔABC __ ΔDEF ii) m∠B__m∠E iii) BC __ EF iv) AB __ DE

v) ∠E__∠F vi) ∠ABM__∠ABC vii) ∠ABM__∠DEF viii) AB__ DE

27
Elementos Fundamentales de la Geometría

b) ¿Qué espacio en blanco se pudo llenar con cualquiera de los dos símbolos?
c) Si AB hubiera sido el mismo segmento que DE pero C fuera un punto diferente de F ¿En que
caso se cambiaría ≅ por = ?

2. Se da el triángulo ABC. Si Δ ABC ≅ Δ BAC y Δ ABC ≅ Δ ACB ¿Qué conclusión se puede obtener
acerca del Δ ABC? ¿Cómo se demostraría que la conclusión es válida?
→ → ↔
3. Se da PC ⊥ KM con K-P-M. Los puntos A y B están del mismo lado de KM que C, pero A y B

están en lados opuestos de PC . A está del mismo lado de PC que K. Si Δ ACP ≅ Δ BCP, demostrar
que ∠ KPA ≅ ∠ MPB.

4. En un plano los puntos C y D están en lados opuestos de AB de tal modo que el Δ ABC es un
triángulo equilátero y el Δ ABD es equiángulo. Demostrar que ∠ C ≅ ∠ D.

5. Se nos da el Δ ABC, con AC = BC. Las bisectrices de los ángulos en la base ∠ A y ∠ B se cortan en
un punto F ¿Qué nos permite afirmar que CF es perpendicular a AB ?

6. Si Δ BCA ≅ Δ PQR, indique las congruencias respectivas relativas a los lados y ángulos de dichos
triángulos.
7. En el Δ ABC ¿Qué ángulo es determinado por los lados BC y AB ? ¿Qué lado es determinado por
los ∠ A y ∠ B ?

8. En algunos problemas, el hecho de que dos segmentos sean congruentes o que dos ángulos sean
congruentes, se deduce de las definiciones de los conceptos involucrados. Para cada una de las
siguientes situaciones, haga un dibujo indicando que segmentos o ángulos son congruentes a partir de la
información dada:
a) AB es perpendicular a CD en el punto E. b) C es el punto medio de AB
→ ↔
c) AT es el bisector de ∠ BAC d) m es la mediatriz de AB
e) El Δ ABC es isósceles con base BC f) CD es una altura del triángulo acutángulo Δ ABC.
g) CD es una mediana del triángulo acutángulo Δ ABC. h) El Δ ABC es equilátero.

9. Para cada par de triángulos dibujados a continuación, si las marcas semejantes indican partes
congruentes, citar que teorema de congruencia (LAL, ALA, LLL) es aplicable para demostrar la
congruencia de los triángulos o indique en que caso falta información para establecer la congruencia.
A B C
a) b) C
E
A
F E D D
B

c) P Q d)
P Q R

T U R S S

28
Elementos Fundamentales de la Geometría

B C
e) D
f)
A C
A D E

B C
F E
M N
g) h)
D

A B
Q P O
10. Si AE = BC, AD = BD y DE = DC,
Demostrar que ∠ E ≅ ∠ C. A B

1
E D C Q
11. Si PQ = PS, ∠1 ≅ ∠2, ∠3 ≅ ∠4,
demostrar que ∠5 ≅ ∠6. 5
3
P R
4 6

E D S 2
12.
Si AB = BC, ∠ MAE ≅ ∠ NCD,
AE = CD. Demostrar que
Δ ABE ≅ Δ CBD
R
M A B C N S
P
13. Si RS = QT, PS = PT, ∠ RTP ≅ ∠ QSP, T
demostrar que Δ RTP ≅ Δ QSP. Q

14.

B Δ ABC es isósceles con AB = BC


D
D es el punto medio de AB
E es el punto medio de BC
A E Demuestre que Δ ACE ≅ Δ CAD

C C

E B
A
15. En la figura, CD biseca a AB
y ∠ C ≅ ∠ D. Demostrar que
AB biseca a CD
D

29
Elementos Fundamentales de la Geometría

16. T
Datos: PT ⊥ RT , SV ⊥ QV ,
RT = QV, PQ = SR
Demostrar que PT = SV
P Q R S

17 En la figura de la izquierda se tiene V


J K m∠ K = m∠ J y MR = NR.
R Demostrar que MK = NJ

M N

18. Los puntos K y M trisecan a GH y G-K-M. Los puntos J e I, al mismo lado de GH , están en las
perpendiculares a GH en G y H, respectivamente, de manera que JM = IK. JM e IK se intersecan
en P. Demostrar que el Δ PKM es isósceles. B

19.
Datos AB = PQ y BP = AQ
Demostrar que a) ∠ A ≅ ∠ P A M Q
b) Δ ABM ≅ ΔPQM

20
Y Z P
a) Demostrar que en la figura, si X es el punto medio
de MN , MZ = NY y XZ = XY entonces ∠ Y ≅ ∠ Z
b) Si M, N, X, Y y Z son coplanares y X es el punto
medio de MN , ∠ M ≅ ∠ N ,
∠ MXY ≅ ∠ NXZ , demostrar que ∠ Y ≅ ∠ Z

M X N

21.
A partir de la información en la figura,
__ __
D E Demuestre que AB ≅ BC

A B C
A
22.
__ __
Si ∠BAD ≅ ∠ CAE, AB ≅ BC
__ __ __ __ F
FB ≅ FC pruebe que AD ≅ AE
D E
B C

30
Elementos Fundamentales de la Geometría

23. En la figura, ABC es un triángulo rectángulo en B. Los puntos E y F A


están sobre la hipotenusa de manera que AE = AB y CF = CB. ¿Cuánto F
mide el ángulo EBF?
E

B C

SOLUCIONES
1. a) i) ≅ ii) = iii) = iv) ≅ v) ≅ vi) = ó ≅ vii) ≅ viii) = b) el (vi) c) el (iv)
2. El triángulo es equilátero. Aplicando la definición de congruencia de triángulos, se deduce que los tres
lados son congruentes y por tanto es equilátero.
3. Indicación: Demostrar primero la congruencia de los triángulos formados y luego hacer ver que ∠KPA y
∠MPB son complementos de ángulos congruentes.
4. Indicación: establecer que m∠C = m∠D = 60º y por tanto son congruentes.

5. El punto F es el incentro y por tanto CF es la bisectriz del ∠C. Por ser un triángulo isósceles, las rectas
__ __ __
notables de la base coinciden, luego CF está sobre la altura correspondiente a la base y CF ⊥ AB .
__ __ __ __ __ __ __
6. ∠A ≅ ∠R, ∠B ≅ ∠P, ∠C ≅ ∠Q, AB ≅ RP, AC ≅ RQ, BC ≅ PQ 7. ∠B; AB

8. ↔
A B m
a) b) c) d)
• T
C E D A C B A B
B A C
A C C C
e) f) g) h)

B C
A D B A D B A B
9. a) ALA; ΔABF ≅ ΔDEC b) Falta información c) LLL; ΔTPU ≅ ΔSQR d) Falta información
e) ALA; ΔAFB ≅ ΔDEC f) LAL; ΔABC ≅ ΔEDC g) Falta información h) Falta información.
10. Indicación: establecer que ∠E y ∠C son ángulos correspondientes en triángulos congruentes. Use LLL.
11. Indicación: establecer que ∠5 y ∠6 son ángulos correspondientes en triángulos congruentes. Use ALA.
__ __
12. Use LAL 13. Indicación: establezca primero que RT ≅ QS y luego aplique el teorema LAL.
14. Utilice las propiedades de un triángulo isósceles y luego aplique el teorema LAL
__ __ __ __
15. Primero establezca que PR ≅ SQ y luego que PT ≅ SV por ser partes correspondientes en triángulos
congruentes.
16. Indicación: complete los pasos para aplicar el teorema ALA y establecer la congruencia de los
triángulos AEC y BED
17. Establezca la congruencia de los triángulos KRM y JNR
18. Establezca la congruencia de los triángulos GMJ y HKI, de ahí resulta ∠IKM ≅ ∠JMK
__
19. Trace BQ , establezca la congruencia entre los triángulos ABQ y PQB. Luego complete los pasos
para aplicar el teorema ALA. 23. 45º

31
Elementos Fundamentales de la Geometría

3. 2. 4. SEMEJANZA DE TRIÁNGULOS

En forma general decimos que dos figuras son semejantes cuando tienen la misma forma, pero no
necesariamente el mismo tamaño. Así por ejemplos, todos los triángulos equiláteros son semejantes entre
si, de igual manera todos los cuadrados, todas las circunferencias, etc. Cuando las figuras semejantes
tienen distintos tamaños podemos considerar una de ellas como una copia de la otra, a una escala
determinada, y el factor de escala le llamamos razón de semejanza. Al considerar áreas o volúmenes de
figuras semejantes, el factor de escala facilita los cálculos.

DEFINICIÓN 14: Dada una correspondencia ABC ↔DEF, entre los vértices de los triángulos Δ ABC y
Δ DEF, si los ángulos correspondientes son congruentes y los lados correspondientes son proporcionales,
decimos que la correspondencia es una semejanza y escribimos Δ ABC ∼ Δ DEF.

⎧ ∠ A ≅ ∠D
⎪ ∠B ≅ ∠E

Δ ABC ∼ Δ DEF ⇔ ⎨ ∠ C≅ ∠F
⎪ AB BC AC
⎪ = =
⎩ DE EF DF

RAZONES Y PROPORCIONES

DEFINICIÓN 15: Una RAZÓN, es la relación que se establece entre dos cantidades de la misma naturaleza,
al compararlas, considerando que múltiplo o parte es una cantidad de la otra.
Notación: x : y ( se lee “ x es a y”).

A las cantidades x y y se les llama términos de la razón. Al primer término se le llama antecedente y al
segundo consecuente.

Para hallar la razón entre dos cantidades, simplemente dividimos el antecedente entre el consecuente, por
tal motivo la razón, x : y, también se representa como una fracción
x
x:y ≡ .
y

DEFINICIÓN 16: Una PROPORCIÓN es la igualdad entre dos o más razones.


a c e
= = = .... = k .
b d f
Decimos que {a, c, e, ..} y {b, d, f, ..} son conjuntos proporcionales y que el valor k es la razón de
proporcionalidad.

a c
Se usa la notación a:b::c:d ( se lee “ a es a b como c es a d” ), esto equivale a la expresión = .
b d
A las cantidades a y d se les llama extremos y a las cantidades b y c se les llama medios.
a b
Cuando tres cantidades están relacionadas por la proporción = decimos que
b c
b es la media proporcional o media geométrica entre a y c, y c es la tercera proporcional a a y b.
a c
Si a, b, c y x son números positivos y = , se dice que x es cuarta proporcional de a, b y c.
b x

◊ ALGUNAS PROPIEDADES:
a c
A partir de = y las propiedades de la igualdad se obtienen los siguientes resultados:
b d

32
Elementos Fundamentales de la Geometría

a b b d d c a+ b c+ d
1. a d = b c 2. = 3. = 4. = 5. =
c d a c b a b d
a− b c− d a c a c a+ b c+ d a c a+ c
6. = 7. = 8. = 9. = 10. = =
b d a + b d+ c b − a d− c a− b c− d b d b+ d

a b c a + b+ c + ...
En general si = = = .... = =k
p q r p + q+ r + ...

EJEMPLOS
1. Encuentre la media proporcional entre 5 y 45.
SOLUCIÓN:
5 x
De acuerdo a la definición, se pide el valor x, tal que = , luego x2 = (5) (45) = 225 ∴ x = 225 = 15
x 45
2. Encuentre la tercera proporcional entre 4 y 16
SOLUCIÓN:
4 16 16 ⋅ 16
De acuerdo a la definición, se pide el valor x, tal que = ∴ x= = 64
16 x 4
3. Encuentre la cuarta proporcional de los números 4, 12 y 15.
SOLUCIÓN:
4 15 12 ⋅ 15
De acuerdo a la definición, se pide el valor x, tal que = ∴ x= = 45
12 x 4
4. Si un triángulo tiene un perímetro de 84 cm. y sus lados son proporcionales a los números 5, 7 y 9,
encuentre las longitudes de dichos lados.
SOLUCIÓN:
a b c
Sean a, b y c las longitudes de los lados del triángulo, luego = =
5 7 9
a b c a+ b+ c
Por las propiedades de las proporciones se tiene = = =
5 7 9 5+7+9
a b c 84
Pero el perímetro es P = a + b + c = 84, luego = = = = 4,
5 7 9 21
Por tanto a = (5) (4) = 20 cm., b = (7) (4) = 28 cm. y c = (9) (4) = 36 cm.

RAZÓN ENTRE DOS SEGMENTOS:

La razón entre dos segmentos se define como la razón entre sus longitudes.
___ ___ ___ ___
Así por ejemplo si m ( AB ) = 4 y m ( CD ) = 2, decimos que AB y CD están en una razón 2:1, lo que
___ ___
significa que la longitud de AB es el doble que la longitud de CD .

PROYECCIONES
↔ ↔ ___ ↔
◊ Se llama proyección de un punto P sobre una recta m al punto P’ sobre la recta m , tal que PP' ⊥ m .
__
El segmento PP' recibe el nombre de proyectante.

◊ Si un punto Q ∈ m , su proyección Q’, es el mismo punto Q.

33
Elementos Fundamentales de la Geometría

___
◊ La proyección de un segmento AB sobre una recta B C

m , es el segmento cuyos extremos son las proyecciones P •
de sus puntos extremos. A D
___ ↔ ___ ___ ↔
Q
◊ Si AB || m , entonces A ' B' ≅ AB . m
___ ↔ • • •
◊ Si AB ⊥ m , entonces A’ = B’ P’ Q’ A’ B’ C’ = D’

TEOREMAS SOBRE PROPORCIONALIDAD Y SEMEJANZA DE TRIÁNGULOS

10. Si una recta es paralela a uno de los lados de un A


triángulo, entonces los otros dos lados quedan divididos
en segmentos proporcionales.
x u
D E x u
y v DE || BC ⇒ =
11. Si una recta divide dos lados de un triángulo en y v
segmentos proporcionales entonces es paralela al
B C
tercer lado.
x u
= ⇒ DE || BC
y v

12. Teorema de Thales: Dos transversales cualesquiera cortadas por tres o más paralelas quedan divididas
en segmentos proporcionales.
A A’ A A’
B B’ B B’ AB AC BC
= =
A' B' A' C' B' C'
C C’ C’ C

13. TEOREMA DE LA BISECTRIZ INTERIOR


__
En el triángulo ABC, sea D un punto del lado BC. AD es bisectriz del ángulo A si y solo si
AB AC
= . Es decir la bisectriz de un ángulo interno de un triángulo divide al lado opuesto en dos
BD CD
segmentos proporcionales a los lados que determinan dicho ángulo.
A

c b
AB AC c b
= =
BD DC m n
B m D n C

Dado que en este caso se tiene m + n = a, al sumar los numeradores y los denominadores en la expresión
c b b+ c
anterior se obtiene: = = .
m n a
ac ab
Despejando m y n, resultan m = y n=
b+ c b+ c

34
Elementos Fundamentales de la Geometría

14. TEOREMA DE LA BISECTRIZ EXTERIOR


__
En el triángulo ABC, sea D un punto de la prolongación del lado BC. AD es bisectriz exterior del ángulo A
AB AC
si y solo si = .
BD CD

c
AB AC b c b
= =
BD DC q p
a C p D
B
q

ac ab
Los segmentos en que la bisectriz divide al lado opuesto están dados por: q = y p=
c− b c− b

15. TEOREMA DE LA BASE MEDIA A

En el triángulo ABC, sean M y N los puntos medios de los lados AB y AC M N


__ __ 1
respectivamente, entonces MN || BC y MN = BC .
2 B C

16. TEOREMA DE SEMEJANZA A.A.


Dos triángulos son semejantes si dos ángulos de uno de ellos son congruentes a sus correspondientes en
el otro.
A D

∠ B ≅ ∠E ∧ ∠ C ≅ ∠ F ⇒ Δ ABC ∼ Δ DEF
B C E F
17. TEOREMA DE SEMEJANZA L.A.L.
Dos triángulos son semejantes si un ángulo de uno de ellos es congruente a un ángulo del otro y si los lados
que comprenden al primero son proporcionales a los lados correspondientes del segundo.

A D
c b f e
a c
∠B≅∠E ∧ = ⇒ Δ ABC ∼Δ DEF
B a C E d F d f

18. TEOREMA DE SEMEJANZA L.L.L.


Dos triángulos son semejantes si sus lados correspondientes son proporcionales.

A D

AB AC BC
= = ⇒ Δ ABC ∼ Δ DEF
B C E F DE DF EF

35
Elementos Fundamentales de la Geometría

TEOREMAS DE SEMEJANZA DE TRIÁNGULOS RECTÁNGULOS

19. Dos triángulos rectángulos son semejantes si un ángulo agudo de uno de ellos es congruente a un
ángulo agudo del otro.

20. Dos triángulos rectángulos son semejantes si tienen los catetos proporcionales.

21. Dos triángulos rectángulos son semejantes si tienen proporcionales uno de los catetos y la hipotenusa.

B B
F F
c
a e d

C A D E C b A D f E

a b a c
m∠C = m∠F = 90º ∧ ∠A ≅ ∠D m∠C = m∠F = 90º ∧ ( = ∨ = )
⇒ Δ ABC ∼ Δ DEF d e d f
⇒ Δ ABC ∼ Δ DEF

22. En un triángulo rectángulo la altura correspondiente a la hipotenusa divide al triángulo en otros dos
semejantes a éste y semejantes entre si.
C

Δ ABC ∼ Δ ACD ∼ Δ CBD

A D B

23. RELACIONES MÉTRICAS EN UN TRIÁNGULO RECTÁNGULO:

Dado un triángulo rectángulo y la altura correspondiente a la hipotenusa

i) la altura es media geométrica de los segmentos en los


C cuales dicha altura divide a la hipotenusa.
AD CD n h
= o = ∴ h = nm
a CD DB h m
b
h
ii) cada cateto es la media geométrica de la hipotenusa y
n m el segmento de ésta adyacente al cateto.
A D B AD AC n b
= ó = ∴ b = nc
c AC AB b c

DB CB m a
= ó = ∴ a= mc
CB AB a c
24. TEOREMA DE PITÁGORAS

En todo triángulo rectángulo, la suma de los cuadrados de los catetos es igual al cuadrado de la hipotenusa.
Si a y b representan las longitudes de los catetos y c la longitud de la hipotenusa, se tiene:

a2 + b2 = c2

36
Elementos Fundamentales de la Geometría

25. GENERALIZACION DEL TEOREMA DE PITÁGORAS

i) En todo triángulo, el cuadrado de la longitud del lado opuesto a un ángulo agudo es igual a la suma de
los cuadrados de las longitudes de los otros dos lados menos el doble producto de uno de estos lados por la
proyección del otro sobre él.
ii) En un triángulo obtusángulo el cuadrado de la longitud del lado opuesto al ángulo obtuso es igual a la
suma de los cuadrados de las longitudes de los otros dos lados más el doble producto de uno de estos lados
por la proyección del otro sobre él.

C C
a a
b b

A D B D A c B
c

a 2 = b 2 + c 2 − 2 c ⋅ AD a 2 = b 2 + c 2 + 2 c ⋅ AD

Como una consecuencia, el tipo de triángulo puede obtenerse al conocer la relación entre las longitudes de
sus lados: si a representa la longitud del lado mayor, se tiene

a TRIÁNGULO RECTÁNGULO
c a2 = b2 + c2

TRIÁNGULO ACUTÁNGULO
c a a2 < b2 + c2

a TRIÁNGULO OBTUSÁNGULO
c a2 > b2 + c2

b
OTRAS PROPIEDADES DE LOS TRIÁNGULOS

1. Las líneas homólogas (medianas, alturas, etc.) de triángulos semejantes son proporcionales, con la
misma razón de semejanza de dichos triángulos.

2. Si dos triángulos son semejantes, entonces la razón entre sus áreas es el cuadrado de la razón entre dos
lados correspondientes cualesquiera.

3. Dado un ΔABC, si mA, es la mediana correspondiente al lado BC, entonces su longitud está dada por:
2 b2 + 2 c2 − a2
mA = .
2
4. Si bA, es la bisectriz correspondiente al vértice A, su longitud está dada por
2
bA = b c s (s− a) donde s es el semiperímetro
b+ c
5. Si hA es la altura correspondiente al vértice A o sea al lado BC, entonces su longitud está dada por
2
hA = s(s− a) (s− b) (s− c)
a

37
Elementos Fundamentales de la Geometría

EJEMPLO 1 C ___ ___


En la figura FG || BA . Si AG = 3, BF = 5 y CF = 12,
Encuentre CG.
F G
B A
SOLUCIÓN:
CG CF GA ⋅ CF
Por el teorema 1, tenemos que = , luego CG = .
GA FB FB
Notemos que GA = AG y FB = BF, por tanto al sustituir los respectivos valores, resulta
( 3) (12)
CG = = 7 .2
5
EJEMPLO 2
A

8 x

B G En la figura, con las dimensiones indicadas, se tiene


v __ __ __
4 y
C F BG || CF || DE . Si x + y + z = 27, encuentre los valores
6 u correspondientes de x, y, z, u y v.
z
D E
36
SOLUCIÓN:

Por el Teorema fundamental de semejanza, el Teorema de Thales, se tiene


x y z
= =
8 4 6
Por las propiedades de las proporciones

x y z x + y+ z 27 3
= = = = =
8 4 6 8 + 4 + 6 18 2
8⋅3 4⋅3 6⋅3
∴ x= = 12, y = = 6, z = =9
2 2 2

Se tiene también que Δ ABG ∼ Δ ACF ∼ Δ ADE, por el Teorema de semejanza A:A.
__ __ __
Dado que BG || CF || DE , las parejas de ángulos correspondientes que se forman son congruentes

A A A
Se puede establecer la relación
v u 36
8 = = =2
12 8 12 18
18 ∴ v = 8 ⋅ 2 = 16, u = 12 ⋅ 2 = 24
B G
v F
C
u

D E
36

38
Elementos Fundamentales de la Geometría

EJEMPLO 3

B E C

F
En la figura, AD || BC , AC = 25, AF = 15, AD = 12.
Determine el valor de EC.
A D

Tenemos que ∠DAC ≅ ∠ECF ∧ ∠ ADF ≅ ∠ CEF por ser ambos ángulos alternos internos entre paralelas,
luego Δ ADF ∼ Δ CEF, por el teorema AA de semejanza.

B E C
EC CF
= Tenemos CF = AC – AF = 25 – 15 = 10
F DA AF
EC 10 12 ⋅ 10
= ∴ EC = =8
12 15 15
A D
EJEMPLO 4
D E
En la figura AB = 12, CD = 8 y DE = 15. Encuentre el valor de AE.
C

A B SOLUCIÓN:

Observamos que en el punto C se forma una pareja de ángulos opuestos por el vértice, y que ∠B ≅ ∠D, ya
que ambos son ángulos rectos; luego por el Teorema de Semejanza AA, se tiene ΔABC ∼ ΔEDC.
BC AB DC⋅ AB 8 ⋅ 12
Por tanto = ∴ BC = = = 6.4
DC ED ED 15

Aplicando el Teorema de Pitágoras en cada triángulo podemos encontrar AC y CE, cuya suma forma AE,
pero también podemos prolongar el segmento AB y trasladar el segmento BD para formar un solo triángulo
rectángulo AEF y calcular directamente AE.

Se tiene AF = AB + DE = 12 + 15 = 27 y EF = BD = BC + CD = 6.4 + 8 = 14.4, luego

E
Aplicando el Teorema de Pitágoras resulta
14.4 AE = 27 2 + 14.4 2 = 30.6

A 27 F

EJEMPLO 5

Un asta de metal se rompió en cierto punto quedando con la parte de arriba doblada a manera de bisagra y
la punta en el piso en un punto localizado a 3 m de la base. Se reparó, pero se rompió de nuevo, esta vez en
un punto 75 cm. más abajo que la vez anterior y la punta tocando el piso a 4.5 m. de la base ¿qué longitud
tiene el asta?

39
Elementos Fundamentales de la Geometría

SOLUCIÓN:

Sea h la altura del asta y sea x la distancia desde el punto donde se rompió por primera vez, al piso.
El asta forma un triángulo rectángulo con el piso siendo su hipotenusa h – x.

Cuando se rompe otra vez, 0.75 m más abajo, el cateto vertical se reduce a x – 0.75 y la hipotenusa
aumenta en 0.75, quedando h – x + 0.75. Aplicando el teorema de Pitágoras en cada triángulo formado,
obtenemos un sistema de ecuaciones que nos permite hallar el valor de h.

(1) (h – x)2 = x2 + 32 ⇒ (h – x)2 = x2 + 9


(2) (h – x + 0.75) = (x – 0.75)2 + 4.52 ⇒
2

h (h – x)2 + 1.5 (h – x) + 0.5625 = x2 – 1.5x +


0.5625 + 20.25
h–x h – x + 0.75 Sustituyendo (1) en (2) y simplificando se obtiene:
x
x – 0.75
x2 + 9 + 1.5h – 1.5x + 0.5625 = x2 – 1.5x +
3m 0.5625 + 20.25 ⇒ 1.5 h = 11.25
4.5 m
∴ h = 7.5 m.
EJEMPLO 6

C
10
15 A partir de la información dada en la figura, encuentre el valor de x.
x

A 18 B

SOLUCIÓN:
Se aprecia que la línea trazada dentro del triángulo corresponde a la bisectriz del ángulo A, luego los
segmentos que determina en el lado BC son proporcionales a los lados AB y AC. Por tanto por el Teorema
de la Bisectriz se tiene:

x 10 18 ⋅ 10
= ∴ x= = 12
18 15 15
EJEMPLO 7

A
En la figura, AC = 7, AB = 25.
Halle el valor de BC.
C B

SOLUCIÓN:
Dado que es un triángulo rectángulo, por el Teorema de Pitágoras se tiene AB2 = AC2 + BC2 ,
luego

BC = AB 2 − AC 2 = 25 2 − 7 2 = 625 − 49 = 576 = 24

40
Elementos Fundamentales de la Geometría

EJEMPLO 8

La altura respecto a la hipotenusa de un triángulo rectángulo mide 10 C


cm. y los segmentos que determina sobre la hipotenusa son entre sí
como 7 es a 14. Determine la longitud del cateto menor.
b a
h
SOLUCIÓN:
n m
A D B
Sea b la longitud del cateto buscado, h la altura relativa a la c
hipotenusa y c la longitud de la hipotenusa como se muestra en la
figura.
De las relaciones métricas en un triángulo equilátero se tiene (1) b2 = n⋅c , (2) c = n + m y (3) h2 = n⋅m.
n 7
Los datos indican que h = 10 y = ∴ m = 2n. Sustituyendo en (3) se obtiene:
m 14
102 = 100 = n (2n) ⇒ n2 = 50 ∴ n = 5 2 y m = 2n = 10 2
Luego c = m + n = 5 2 + 10 2 = 15 2.
Sustituyendo en (1) resulta b2 = 5 2 ⋅ 15 2 = 150 ∴ b = 150 = 5 6

. EJEMPLO 9

Dado un Δ ABC, demuestre que la longitud de sus medianas están dadas por
2 b2 + 2 c2 − a2 2 a2 + 2 c2 − b2 2 a2 + 2b2 − c2
mA = , mB = . mC = .
2 2 2
SOLUCIÓN:

Vamos a probar la primera fórmula, puesto que las otras dos se obtienen de manera similar o bien
intercambiando las literales utilizadas.
Consideremos los diferentes casos que pueden presentarse:

A A A

c h m b

B D M C B M C B M C D
x y a/2
Caso 1: El pie de la altura correspondiente al lado considerado queda entre un vértice y el pie de la mediana
como se muestra en la figura.

Sean D y M los pies de la altura y la mediana respectivamente. Sea x = BD, y = DM.


__
Por ser M punto medio de BC se tiene:
a a a a
x + y = , MC = y DC = y + =a–x ∴x= –y.
2 2 2 2
Al aplicar el teorema de Pitágoras a los triángulos rectángulos que se forman se obtiene:
a
i) h2 + x2 = c2 o sea h2 + ( – y)2 = c2
2
ii) h2 + y2 = m2

41
Elementos Fundamentales de la Geometría

a 2
iii) h2 + (y + ) = b2
2
Al desarrollar (i) y (iii) y sumar se obtiene:
a2
h2 + − ay+ y 2 = c 2
4
a2
h2 + + ay+ y 2 = b 2
4
a2
2 h2 + + 2 y2 = b2 + c2
2
a2 2 b2 + 2 c2 − a2 2 b2 + 2 c2 − a2
2 (h 2 + y 2 ) = b 2 + c 2 − = ∴ h2 + y 2 =
2 2 4

De (ii) h2 + y2 = m2A , sustituyendo en la expresión anterior, se obtiene

2 b2 + 2 c2 − a2
mA =
2

Caso 2. Si la altura coincide con la mediana, al aplicar el teorema de Pitágoras


a2 a2
i) m2 = c2 – ii) m2 = b2 –
4 4
Sumando (i) y (ii) se obtiene el resultado buscado.

Caso 3. Si un vértice queda entre el pie de la mediana y el de la altura (ver figura), aplicando también el
Teorema de Pitágoras mediante un procedimiento es muy similar al caso (1) se llega al mismo resultado

EJEMPLO 10 C
Si BD = 9, BC = 15, CD = 12, determine el
perímetro del Δ ABC.

A D B
SOLUCIÓN:
El perímetro es P = AC + BC + AB. Como ya se conoce BC = 15, falta hallar AC y AB.

BC AC BC ⋅ CD 15 ⋅ 12
Tenemos Δ ABC ∼ Δ CBD, luego = , por tanto AC = = = 20
BD CD BD 9
Aplicamos el teorema de Pitágoras para hallar AB. Se tiene
AB = AC 2 + BC 2 = 20 2 + 15 2 = 400 + 225 = 625 = 25
El perímetro buscado es P = AC + BC + AB = 20 + 15 + 25 = 60

42
Elementos Fundamentales de la Geometría

EJEMPLO 11
C
En el triángulo de la figura, b = 8 y c – a = 2.
b h a Determine los valores de a, c, m, n y h.
m n
A D B
c
SOLUCIÓN:

Por el teorema de Pitágoras tenemos c2 = a2 + b2 , al sustituir el valor de b, se obtiene


c2 – a2 = 82 = 64 (1)
Además por dato tenemos c–a=2 (2).

Resolviendo el sistema de ecuaciones formado por (1) y (2), obtenemos c = 17 y a = 15


Por las relaciones métricas en un triángulo rectángulo tenemos
a ⋅ b 15 ⋅ 8
h= = = 7.06
c 17
b2 82
b2 = m ⋅ c ∴ m = = = 3.765
c 17
a 2 15 2
a2 = n ⋅ c ∴ n =
= = 13.235
c 17
Para comprobar verificamos que m + n = c, lo cual se cumple.

EJEMPLO 12

Para calcular la altura de un edificio, una persona coloca verticalmente una vara de 3 m a una distancia de
25 metros del edificio. Retrocede hasta mirar alineados el extremo de la vara y la parte superior del edificio.
Si la persona ha retrocedido 2.55 m y sus ojos están a 1.65 m del piso, determine la altura del edificio.
E

D
A B C h
3
1.65

E
Al trazar una línea paralela al piso, a la
D h – 1.65 altura de los ojos de la persona, y a partir
1.35 de los datos dados obtenemos los
triángulos de la figura.
A 2.55 B 25 C
27.55 Tenemos Δ ABD ∼ Δ ACE, luego
h− 1.65 1.35
= ∴ h = 16.235 m
27.55 2.55

43
Elementos Fundamentales de la Geometría

EJEMPLO 13
Demostrar que las bisectrices de un triángulo son concurrentes.

A __ __ __
Sean AP, BQ y CR las bisectrices del triángulo ABC.
Q __ __
R AP y BQ se cortan en un punto ; dado que A y B son ángulos
I internos del ΔABC, m∠A + m∠B < 180º, luego sus mitades
__ __
B P C suman menos de 90º, por tanto AP y BQ no son paralelas y
deben cortarse en algún punto
↔ ↔ ↔ ↔
Tenemos d(I, AC) = d(I, AB) (1) y d(I, AB) = d(I, BC) (2), ya que I está en las bisectrices de A y B, y
por tanto equidista de los lados.
↔ ↔
De (1) y (2), por transitividad, d(I, AC) = d(I, BC) por tanto I está también en la bisectriz del ángulo C.

EJEMPLO 14
Demostrar que las mediatrices de un triángulo son concurrentes.
A
m1 Sean m1 , m 2 y m 3 las mediatrices del triángulo ABC.
__ __
Como AC y BC son lados de un triángulo, no son paralelos y por tanto
O las líneas perpendiculares a ellas tampoco son paralelas, luego se
m2
cortan en algún punto. Sea O el punto de intersección de m1 y m 2 .
B C Luego OB = OC y OA = OC (propiedad de las mediatrices).
m3
Por transitividad se tiene también que OA = OB, por tanto O está en la
__
mediatriz de AB , es decir O pertenece a las tres mediatrices

EJEMPLO 15
Demostrar que las tres alturas de un triángulo son concurrentes.

F A A través de cada vértice trazamos rectas paralelas al respectivo


E
lado opuesto, formando el triángulo DEF. Se tiene AE = BC y AF
=BC por ser segmentos de paralelas comprendidos entre
N
P H paralelas, luego A es punto medio de EF.
__ __
De manera similar B y C son puntos medios de FD y DE
B M C respectivamente.
___ ___ ___
Sean AM , BN y CP las alturas del ΔABC.
___ __
La altura AM por ser perpendicular a BC también lo es a su
D
__
paralela EF ; de manera similar el resto de alturas son
perpendiculares a los respectivos lados del triángulo DEF.

Luego las alturas del ΔABC son mediatrices del ΔDEF y por tanto son concurrentes.

44
Elementos Fundamentales de la Geometría

3. 2.5. DESIGUALDADES GEOMETRICAS

Es útil recordar las propiedades de las desigualdades para los números reales, ya que las desigualdades
entre segmentos y ángulos se establecen a partir de sus medidas, las cuales son números reales.

Entre las principales propiedades tenemos:

1. Si x, y ∈ R, entonces una y solo una de las siguientes relaciones se cumple:


i) x = y ii) x > y iii) x < y (Tricotomía)
2. Si x > y ∧ y > z ⇒ x > z (Transitividad) 3. Si x ≤ y ∧ a < b ⇒ a + x < y + b
4. Si x < y ∧ a > 0 ⇒ ax < ay 5. Si x < y ∧ a < 0 ⇒ ax > ay
6. Si a = b + c, c > 0 ⇒ a > b
__ __
DEFINICIÓN 17: Decimos que AB > CD (el segmento AB es mayor que el segmento CD) si AB > CD. De
manera similar ∠ ABC > ∠ DEF si m∠ ABC > m∠DEF.
__ __ __ __
Si AB > CD , ∃ E tal que A – E – B ∧ AE ≅ CD .
De manera similar si ∠ ABC > ∠ DEF, ∃ G ∈ i ∠ABC tal que ∠ GBC ≅ ∠DEF
A
C D
G D

A __ __ E B
AB > CD B C E F
∠ABC > ∠DEF
DEFINICIÓN 18: Si C está entre A y D, entonces el ∠ BCD es un ángulo externo del
Δ ABC
B

A C D
E
En todo triángulo se forman seis ángulos externos, formando tres pares de ángulos opuestos por el vértice.

DEFINICIÓN 19: Si B – C – E y A – C – D, los ángulos ∠ A y ∠ B se llaman ángulos internos no contiguos


de los ángulos externos ∠ BCD y ∠ACE.

TEOREMA 26: Un ángulo externo de un triángulo es mayor que cada uno de los ángulos internos no
contiguos.

TEOREMA 27: La medida de un ángulo externo de un triángulo es igual a la suma de las medidas de los
ángulos internos no contiguos.
En la figura anterior m∠ BCD = m∠ A + m∠ B
COROLARIO: Si un triángulo tiene un ángulo recto, entonces los otros dos son agudos y complementarios.
TEOREMA 28: Si dos lados de un triángulo no son congruentes, entonces los ángulos opuestos no son
congruentes y el ángulo mayor es el opuesto al lado mayor.
El recíproco también es válido, es decir el lado mayor se opone al ángulo mayor.

TEOREMA 29: El segmento más corto que une un punto a una recta es el
P
segmento perpendicular a la recta.
PB < PC

C B

45
Elementos Fundamentales de la Geometría

DEFINICIÓN 20: La distancia entre una recta y un punto fuera de ella, es la longitud del segmento
perpendicular desde el punto a la recta.
La distancia entre una recta y un punto de la misma recta se define como cero.

TEOREMA 30: La suma de las longitudes de dos lados cualesquiera de un triángulo es mayor que la
longitud del tercer lado.
A

c b a+b>c
a+c>b
B C b+c>a
a
TEOREMA 31: Si dos lados de un triángulo son congruentes, respectivamente, con dos lados de un
segundo triángulo, y el ángulo comprendido en el primer triángulo, es mayor que el ángulo comprendido en
el segundo, entonces el tercer lado del primer triángulo es mayor que el tercer lado del segundo.
El recíproco también es válido.

B E
AC = DF ∧ AB = DE
∠A > ∠D ⇔ BC >EF

A C D F

EJEMPLOS
1. En el ΔABC, AB = 9, BC = 12 y AC = 15. Nombre el ángulo mayor y el ángulo menor.

El Teorema # 25 establece que el ángulo mayor es el opuesto al lado mayor y en consecuencia el ángulo
__
menor es el opuesto al lado menor. De acuerdo a los datos el lado mayor es AC , por tanto el ángulo
__
mayor es el ángulo B. El lado menor es AB , luego el ángulo menor es el ángulo C.

2. En el Δ DEF, m∠D = 37° y m∠E = 71°. Nombre el lado mayor y el lado menor.

El recíproco del teorema # 25 también es válido, es decir el lado mayor se opone al ángulo mayor y el
lado menor se opone al ángulo menor.
Como datos se da la medida sólo de dos ángulos, luego hemos de encontrar primero la medida del
ángulo que falta antes de decidir el orden de mayor a menor.

Dado que las medidas de los ángulos de un triángulo suman 180º, tenemos
m∠F = 180º – m∠D – m∠E = 180º – 37º – 71° = 72º, luego el ángulo mayor es F y el menor es D.
__ __
Resulta entonces que ED es el lado mayor y EF el lado menor.
EJERCICIOS
1.
K
En el triángulo KMN, nombre los lados de
84° mayor a menor

M 42° 54° N

46
Elementos Fundamentales de la Geometría

2 P
En la figura de la derecha, Q 60°
nombre el segmento más largo. 45°
60°
R 30° 75° S

A
3. 75° En la figura de la izquierda
50° ¿cuál es el segmento más corto?
B 52° D
70°
C

4. En las siguientes figuras compare los valores de a, b y c.

(a) (b) 50º (c) cº (d)


a b a c 12 8
b
30º 50º 70º aº bº aº bº cº
c 15

5.
D
Si AD es el lado más corto y A
BC el lado más largo en el
cuadrilátero ABCD, ¿ quien es
mayor, el ∠B o el ∠D?
B C

SOLUCIONES:
___ ___ ___ ___ ___
1. MN, MK, NK 2. PQ 3. CD 4. a) c > a > b b) a > c > b
c) c > b > a d) c > b > a 5. D

47
Elementos Fundamentales de la Geometría

3.3. CUADRILÁTEROS

Un cuadrilátero es una figura plana cerrada, de cuatro lados que se cortan únicamente en sus extremos.

DEFINICIÓN 21: Sean A, B, C y D cuatro puntos coplanares, tales que no hay tres colineales y los
___ ___ ___ ___
segmentos AB , BC , CD y DA únicamente se intersecan en los A
extremos entonces el cuadrilátero ABCD, denotado por ABCD se define
por B
___ ___ ___ ___
ABCD = AB ∪ BC ∪ CD ∪ DA
D
___ ___
◊ Los segmentos AC y BD se llaman diagonales del ABCD.
C
◊ Decimos que dos lados son adyacentes si tienen un extremo en común,
___ ___
ejemplo AB y BC .
___ ___
◊ Dos lados no adyacentes se llaman lados opuestos. En el ABCD, AB y DC son lados opuestos.

◊ Ángulos contiguos son los que contienen un lado y ángulos opuestos son los no contiguos.

◊ Decimos que el ABCD es convexo si cada uno de sus lados está en uno de los semiplanos
determinado por el lado opuesto. Una de sus propiedades es que sus diagonales siempre se cortan.

Cuadrilátero Cuadrilátero
Convexo No Convexo

CLASIFICACIÓN DE LOS CUADRILÁTEROS

Una forma de clasificar los cuadriláteros es a partir del paralelismo de sus lados:
Paralelogramos: son los cuadriláteros que tienen los dos pares de lados opuestos paralelos.
Trapecios: son los cuadriláteros con una pareja de lados paralelos
Trapezoides: son los cuadriláteros que no tienen lados paralelos.

PARALELOGRAMOS

Un paralelogramo ABCD, es un cuadrilátero que se caracteriza por A B


__ __ __ __
i) Sus lados opuestos son paralelos: AB || DC y AD || BC E
__ __ __ __
ii) Sus lados opuestos son congruentes: AB ≅ DC y AD ≅ BC
iii) Los ángulos opuestos son congruentes: ∠A ≅ ∠C y ∠B ≅ ∠D.
iv) Los ángulos consecutivos son suplementarios: D C
m∠A + m∠B = m∠B + m ∠C = m∠C + m∠D = m∠D + m∠A = 180°
v) Las diagonales se bisecan AE = EC y BE = ED

Entre los paralelogramos tenemos:

48
Elementos Fundamentales de la Geometría

RECTÁNGULO: Es un paralelogramo cuyos ángulos internos son congruentes y por tanto son ángulos
rectos.
A B

m∠A = m∠B = m∠C = m∠D = 90°

AB = DC y AD = BC
D C
CUADRADO: Es un rectángulo con sus lados congruentes.

ROMBO: Es un paralelogramo con sus lados congruentes.


En un rombo las diagonales son perpendiculares entre si.
Nota: Los rectángulos, cuadrados y los rombos son paralelogramos y por tanto tienen las características
señaladas anteriormente para los paralelogramos en general, además de sus propiedades específicas.
Todo cuadrado es un rombo, pero el recíproco no es cierto.

TRAPECIO: Es un cuadrilátero con una pareja de lados paralelos. La pareja de lados paralelos reciben
el nombre de bases y los lados no paralelos, soportes.
A su vez se subdividen en:
◊ Trapecios rectángulos, cuando tienen dos ángulos rectos.
◊ Trapecios isósceles, cuando los lados no paralelos y los ángulos en las bases son congruentes
◊ Trapecios escalenos, solamente tienen un par de lados opuestos paralelos.

Trapecio
Rectángulo Cuadrado Rombo

3.4. PERÍMETROS Y ÁREAS DE TRIÁNGULOS Y CUADRILÁTEROS

Región Triangular: Es la unión de un triángulo y su interior.


En el lenguaje corriente, las regiones triangulares las llamamos, simplemente triángulos, aunque en
sentido estricto son dos conceptos diferentes. Sin embargo por ser una costumbre tan arraigada, la
diferencia la establecemos según el contexto en el cual estemos usando dichos términos.

Región Poligonal: Es la unión de varias regiones triangulares, en un plano, tales que sus intersecciones
dos a dos, si no son vacías, son un punto o un segmento.
Las regiones poligonales y los polígonos se diferencian de manera similar a la región triangular y el
triángulo, pero también en el lenguaje corriente los confundimos y la diferencia se establece según el
contexto en el cual estemos.

POSTULADOS DE ÁREAS
1. A toda región poligonal le podemos asociar un número real no negativo, llamado su área.
Si R representa la región, una forma de representar su área es α(R) o colocar entre corchetes los
puntos que representan los vértices de la región. Así por ejemplo [ABC] representa el área del
triángulo con vértices A, B y C.

2. Dos regiones poligonales congruentes, tienen la misma área.

49
Elementos Fundamentales de la Geometría

3. Si una región es subdividida en subregiones de manera que dos de ellas no se traslapen, es decir si
se interceptan, su intersección es un punto o un segmento, entonces el área de la región es la suma de
las áreas de las subregiones.

4. El área de una región cuadrada es el cuadrado de la longitud de su lado.

A partir de los Postulados de área y las propiedades de los triángulos y cuadriláteros, se obtienen los
siguientes resultados, donde A es el área, P el perímetro y d la diagonal:
PARALELOGRAMO A=b⋅h
a h
P = 2 (a + b)

b
RECTÁNGULO
A=a⋅b
a d P = 2 ( a+ b)
d= a2 + b2
b

CUADRADO
A = a2
a d P = 4a
d= 2a

ROMBO
a a
D
D⋅d
A=
d 2
P=4a
a a

TRIÁNGULO
1
altura A= base x altura
altura 2

base
base

De manera particular, en un triángulo rectángulo, el área es la mitad del producto de sus catetos.

a 1
A= ab
2

50
Elementos Fundamentales de la Geometría

◊ Si dos triángulos tienen igual base (o altura), sus áreas son proporcionales a sus alturas (o sus bases)

C C

C’
h [ABC] AB
[ABC] = h [A' B' C]
=
A' B'
h’
[ABC'] h'
A B A A’ B’ B

C C’ C’’ C’’’

m1
h

m2
A B A’ B’ A’’ B’’ A’’’ B’’’
↔ ↔
Si m1 || m 2 , AB = A’B’ = A’’B’’ = A’’’B’’’ = …entonces
[ABC] = [A’B’C’] = [A’’B’’C’’] = [A’’’B’’’C’’’]

En un triángulo equilátero de lado x, al trazar una altura, se forman dos


triángulos rectángulos cuyos ángulos agudos miden 30° y 60°
respectivamente, razón por la cual se conocen como triángulos 30 – 60.
Al aplicar el Teorema de Pitágoras obtenemos que su altura está dada
3
por h = x.
2
Por tanto en todo triángulo 30 – 60, el cateto menor mide la mitad de la

x 3
30° 3 hipotenusa y el cateto mayor la hipotenusa.
x 2
2 Luego el área de todo triángulo equilátero de lado x está dada por:
60°
x 3 2
A= x
2 4

FÓRMULA DE HERÓN

Puede demostrarse que el área de un triángulo, en


función de sus lados está dada por
a b A= s ( s− a) (s− b) (s− c)
a + b+ c
donde s es el semiperímetro, es decir s =
2
c Esta fórmula se atribuye a Herón de Alejandría,
matemático del siglo II a.c.

51
Elementos Fundamentales de la Geometría

FÓRMULA TRIGONOMÉTRICA
C Recordemos que en la trigonometría, en un triángulo rectángulo,
el seno de un ángulo agudo está dado por la razón entre el
b a cateto opuesto y la hipotenusa.
h Al trazar la altura CD, vemos que se forma el triángulo rectángulo
ADC, luego h = b sen A. Por tanto
1 1 1
A D B [ABC] = c⋅ h = c b sen A = bc sen A
2 2 2
c
Al considerar los otros ángulos se obtiene en forma similar:
1 1
[ABC] = A = ab sen C = ac sen B
2 2

TRAPECIO b B+ b
A= ⋅ h = B’ h
2
h B : base mayor
B’
b: base menor
B’: base media
B

__ __
D C
En un trapecio ABCD, con AB || CD , si E es el punto donde
se cortan las diagonales, entonces [AED] = [BEC].
E
Esto surge al considerar que [ABD] = [ABC] ya que tienen
A B la misma base y la misma altura y [ABD] = [ABE] + [AED]
al igual que [ABC] = [ABE] + [BEC]

En general para un cuadrilátero convexo se tienen las siguientes relaciones:


D C En los triángulos AED y ECD, al tomar como bases los
y segmentos AE y EC, podemos apreciar que tienen la misma
E altura. De manera similar, los triángulos AEB y ECB, tomando
x w como bases los segmentos AE y EC comparten la misma altura.
Luego se cumple:
z [AED] = AE y [AEB] = AE , por tanto [AED] = [AEB] (1)
A B [ECD] EC [ECB] EC [ECD] [ECB]
Si hacemos [AED] = x, [ECD] = y, [AEB] = z y [ECB] = w, se tiene [ABCD] = x + y + z + w. (2)
x z
La relación (1) podemos escribirla como = o sea x w = z y. (3)
y w
Al multiplicar cada miembro de (2) por z, se obtiene:
z⋅[ABCD] = x z + z y + z2 + w z
Al sustituir la expresión (3) se tiene z⋅[ABCD] = x z + x w + z2 + w z = x (z + w) + z (z + w)
z⋅[ABCD] = x (z + w) + z (z + w) = (x + z) (z + w) (4)

Pero x + z = [ABD] y z + w) = [ABC]


Luego al despejar z de la expresión (4) se tiene z = [AEB] =
[ABD] ⋅ [ABC]
[ABCD]

52
Elementos Fundamentales de la Geometría

EJEMPLO 1
___ ___
C 12 D
La figura representa un trapecio isósceles con AB || CD
Si AC = BD = 13, CD = 12 y AB = 22, determine su área. 13 13
SOLUCIÓN:
A E F B
Al trazar perpendiculares desde C y D al lado AB, se tiene
22
EF = CD = 12. Por ser un trapecio isósceles,
C
AB− EF 22 − 12
AE = FB = = =5
2 2 13 h
Aplicando el Teorema de Pitágoras obtenemos
h= 13 2 − 5 2 = 169 − 25 = 144 = 12 A 5 E
( 22 + 12)
Por tanto el área buscada es A = ⋅ 12 = 204 u2
2

EJEMPLO 2
La altura de un triángulo divide a la base en dos partes que miden 36 y 14. Una recta perpendicular a la
base, divide al triángulo en dos partes de áreas iguales. Determine las medidas de los segmentos en que
ésta divide a la base.

SOLUCIÓN:
__
Sea ADE el triángulo dado y EB la altura indicada, luego AB = 14 y BD = 36 y por tanto AD = 50.
__
Sea CF la perpendicular que divide al triángulo en dos regiones de igual área. A su vez divide al triángulo
BDE en dos regiones. Sean A2 y A3 las respectivas áreas.
Sean a y b las longitudes buscadas.

Los triángulos ABE y BDE tienen la misma altura, luego sus áreas son proporcionales a sus bases

A1 14 7
E Tenemos (1) A3 = A1 + A2 y (2) = =
A2 + A3 36 18
F
Sustituyendo (1) en (2)
A1 7 14
A1 A2 A3 = ⇒ 18 A 1 = 7 A 1 + 14 A 2 ∴ A 1 = A (3)
A + ( A + A ) 18 11 2
2 1 2

A B C D 14 25
14 36 Sustituyendo (3) en (1): A 3 = A2 + A2 = A 2 , por tanto [BDE] = A3
11 11
a b
36
50 + A2 = A3
25
36
25
A3 36 2
Por otro lado ΔBDE ∼ ΔCDF, luego A3
= , resultando b = 30.
b2
El otro segmento mide a = 50 – b = 50 – 30 = 20.

EJEMPLO 3

53
Elementos Fundamentales de la Geometría

Demuestre que si wA, es la bisectriz correspondiente al vértice A, su longitud está dada por
2
wA = b c s (s− a) donde s es el semiperímetro
b+ c

A De la Trigonometría tenemos que el área de un triángulo está


1
dada por A = bc sen A.
2
c b De las identidades del ángulo medio se tiene
A A
sen A = 2 sen cos .
2 2
Por otro lado al combinar la Ley de los cosenos y la identidad
B m D n C correspondiente al coseno del ángulo medio se obtienen las
fórmulas de Briggs:

b2 + c2 − a2 A A s (s− a) A (s− b) (s− c)


cos A = = 2 cos 2 − 1 . Resultando cos = y sen = donde s
2 bc 2 2 bc 2 bc
es el semiperímetro del triángulo.

Al considerar los triángulos en que la bisectriz AD divide al triángulo se tiene: [ABD] + [ADC] = [ABC]

Aplicando la fórmula trigonométrica para el área de un triángulo


1 A 1 A 1
c w sen + b w sen = b c sen A
2 2 2 2 2
Simplificando y usando la identidad trigonométrica:
A A A
w (b+ c) sen = b c 2 sen cos
2 2 2
A
w (b+ c) = 2 b c cos
2
Usando la fórmula de Briggs para el coseno del ángulo medio:
s (s− a)
w (b+ c) = 2 b c
bc
2
∴ w= b c s (s− a)
EJEMPLO 4 b+ c

__ __
En un cuadrado ABCD de lado 1, E es punto medio de la diagonal BD y F es punto medio de EB .
¿Cuál es el área del triángulo BCF?

D C La diagonal divide al cuadrado en dos triángulos de igual área, luego [BCD] = 1


2
__ __ 1
E Como E es punto medio de BD y F es punto medio de EB , se tiene BF = BD y
F 4
como los triángulos BCD y BCF tienen la misma altura, sus áreas son
A B proporcionales a las bases BD y BF, luego
1 1 1
[BCF] = ⋅ =
4 2 8

54
Elementos Fundamentales de la Geometría

EJEMPLO 5
Los lados paralelos de un trapecio tienen medidas iguales a “a” y a “b”. Hallar la longitud del segmento
paralelo a ellos y que divide al área del trapecio en dos áreas iguales.

a
h1 Sean h1 y h 2 las alturas de los trapecios formados. Se tiene
c
1 ⎡1 ⎤ a+ b 2 h1
h2 (h1 + h 2 )(a+ b) = 2 ⎢ h1(a + c)⎥ ∴ = (1)
2 ⎣2 ⎦ a + c h1 + h 2
b

Al trazar una paralela a uno de los lados no paralelos se forma la figura de la izquierda donde se aprecian
dos triángulos semejantes
a
h1 c− a
h1 De aquí resulta = . Sustituyendo en (1), se obtiene:
h1 + h 2 b− a
c–a a
h2 a+ b 2 h1 2(c − a)
= = . Al simplificar y despejar c, resulta
b–a a a + c h1 + h 2 (b− a

a2 + b2
c=
2

EJEMPLO 6
D G C
Hallar el área sombreada en cada figura
H
i) Si ABCD es un cuadrilátero convexo de área S. A
E, F, G y H son los puntos medios de los lados. F
E
B

Al trazar la diagonal BD se forman dos triángulos. Sean [ABD] = A 1 y [CBD] = A 2 , luego A 1 + A 2 = S


1
Como H y E son puntos medios en los lados del ΔABD, se tiene [AEH] = A 1 , de manera similar
4
1 1 1 1 1
[CFG] = A 2 , luego [AEH] + [CFG] = A 1 + A 2 = ( A 1 + A 2 ) = S
4 4 4 4 4

D F C
ii) El área del rectángulo mide 1. E y F son puntos
medios de BC y CD
E Se tiene [AEF] = [ABCD] – [ABE] – [ADF] – [FCE]
1
Por ser E y F puntos medios [ABE] = [ADF] =
4
1 1 1 1 3
A B y [FCE] = . Luego [AEF] = 1 – – – =
8 4 4 8 8

55
Elementos Fundamentales de la Geometría

C
iii) ABC es un triángulo de área S. Los lados
P AC, CB y AB se dividen en 2, 3 y 4 partes
iguales respectivamente
Q
Tenemos [MNPQ] = [ABC] – [AMQ] – [QPC] – [NBP]
1 1 1 1 1 2 3
[MNPQ] = (1 – ⋅ – ⋅ – ⋅ )⋅S = S
4 2 2 3 2 3 8
A M N B

D C
iv) El área del rectángulo mide 1. E es punto medio de AB.
Hallar [FEC]
1 1
F Se tiene [EBC] = [AEC] = , [DCE] =
4 2
3
[AECD] = [ABCD] – [EBC] =
4
A E B
[FEC] = [AEC] ⋅ [DEC] (1 / 4) ⋅ (1 / 2) 1
= =
[AECD] 3/4 6

H M
v) ABC es un triángulo de área S. Los
lados AC, AB y AB se dividen en 3
G
partes iguales

A B
1 2 2
Al considerar el cuadrilátero ABGH, se tiene [ABGH] = [ABC] – [HGC], pero [HGC] = ⋅ S = S, luego
3 3 9
2 7
[ABGH] = S – S= S
9 9
⎛1 ⎞ ⎛2 ⎞
⎜ S⎟ ⋅ ⎜ S⎟
Se tiene [ABE] =
[ABG] ⋅ [ABH] = ⎝ 3 ⎠ ⎝ 3 ⎠ = 2 S
[ABGH] 7
S
7
9
Con el mismo procedimiento al considerar los cuadriláteros BCHJ y ACGJ se obtiene
2 2 1
[BCF] = [ACD] = S, por tanto [DEF] = [ABC] – [ABE] – [BCF] – [ACD] = (1 – 3⋅ ) S = S
7 7 7

56
Elementos Fundamentales de la Geometría

vi) ABC es un triángulo de área S. Los lados AC, BC y AB se dividen en


C
2, 3 y 4 partes iguales respectivamente

1 2 1
H
AD = DE = AB, HB = BC, GH = GB = BC
4 3 3
Se tiene [DMH] = [DEH] – [DEM]
[DHM] =
[DHE] ⋅ [DHG]
M
G
[DEGH]
1 2 1
[DHE] = ⋅ [ABC] = [ABC]
4 3 6
A D E B 1 3 1
[DHG] = ⋅ [ABC] = [ABC]
3 4 4
⎛ 3 2 2 1⎞ 1
[DEGH] = ⎜ ⋅ − ⋅ ⎟ [ABC] = [ABC]
⎝ 4 3 4 3⎠ 3

∴ [DHM] =
[DHE] ⋅ [DHG] (1 / 6) ⋅ (1 / 4)
=
1
[ABC] = S
[DEGH] 1/ 3 8

EJERCICIOS SOBRE SEMEJANZA Y ÁREAS.

1. Una recta paralela a un lado de un triángulo, determina sobre un segundo lado dos segmentos de
18 m. y 7 m. ¿Cuáles son los segmentos determinados en el otro lado, cuya longitud es 30 m.?

2. Dos lados de un triángulo miden 158 y 176 m. A partir del vértice común se marca un punto a 120 m.
del vértice en el primer lado ¿A qué distancia del vértice se debe marcar un segundo punto en el otro
lado, para que la recta que une los puntos obtenidos sea paralelo al tercer lado?

3.
C

__ __
En la figura KH || BA . Si AH = 3 , BK = 5 y CK = 12
K H
Hallar CH
D
B A
E
4.
__ __ __ __
En la figura, CD ⊥ AC , EB ⊥ AC ,
AB = 12, EB = 8 y CD = 120
Calcular BC A B C

5. A B
__ __
E En la figura, AB || CD , DE = 15, AB = 8, DC = 20.
Determine EB.

D C

57
Elementos Fundamentales de la Geometría

6. A En el triángulo ABC se traza MN paralela a BC .


Si AB = 8 y AC = 4 y BC = 6, determine el valor
x = AM para que el perímetro del triángulo AMN
M N sea igual al perímetro del trapecio MNCB

7. C
B C
i) Hallar BD si AD = 2 y CD = 5

ii) Hallar AC si BD = 9, BC = 15 y CD = 12

A D B iii) Hallar BC si AB = 20 y AD = 15

8. F
y 15
x __ __ __ __
E G En la figura, FG || EH y EG || DH .
45 Encuentre el valor de x y y.
30

D 40 + x H

9. La hipotenusa de un triángulo rectángulo mide 30 m. Uno de los segmentos determinados por la altura
en la hipotenusa es 20 m. Calcular las longitudes de las alturas y los catetos.

10. Los tres lados de un triángulo miden 25, 50 y 51 m. Calcular las proyecciones de los dos primeros
sobre el tercero. Calcular también la altura relativa al tercer lado.

11. Si Δ ABC es equilátero con AB = 6 cm. ¿Cuál es el área del Δ ABC?

12. Si Δ ABC ∼ Δ PQR y la razón entre sus lados correspondientes es 4 / 3 ¿Cuál es la razón entre sus
áreas?

13. Si Δ ABC ∼ Δ PQR, el área del Δ ABC es 64 y el área del Δ PQR es 16 ¿Cuál es la razón entre
los lados correspondientes?

14. ¿Cuál es el perímetro de un triángulo rectángulo cuyos catetos miden 16 y 30?


A
15. A 16.
13 Halle el área del
Halle
5 Δ ABC
a) x 25 24 25
b) área del Δ ABC
B x C
B D C
17. El Δ ABC es un triángulo rectángulo con el ángulo recto en B si m ∠ C = 30° y BC = 6, calcule el
área del Δ ABC.

18. En el Δ PQR, m ∠ QPR = 120° , PQ = PR = 10. Encuentre el área del Δ PQR.

19. En un triángulo rectángulo, la altura relativa a la hipotenusa mide 12 cm. La diferencia entre las
proyecciones de los catetos sobre la hipotenusa es 7 cm. Determine el perímetro de dicho triángulo.

58
Elementos Fundamentales de la Geometría

20. El perímetro de un triángulo rectángulo es 36 m. y su área es 54 m2. Calcular la longitud de los


catetos.

21. La hipotenusa de un triángulo rectángulo es 25 m.; la suma de los catetos con la altura es 47 m.
Calcular la longitud de los catetos.

22. Determine las medidas de los catetos y la hipotenusa de un triángulo rectángulo, sabiendo que su
área mide 54 m2 y la altura relativa a la hipotenusa mide 7.2 m.

23. El perímetro de un triángulo mide 50 cm. y sus lados son proporcionales a los números 6, 9 y 10.
¿Cuánto mide cada lado?

24. Los lados de un triángulo rectángulo son proporcionales a los números 3, 4 y 5 y tiene un área de
24 m2 ¿Cuánto mide cada lado?

25. En un triángulo rectángulo las longitudes de los catetos y la hipotenusa forman tres números
consecutivos. Determine dichas longitudes.

26. Las proyecciones de los catetos sobre la hipotenusa en un triángulo rectángulo miden 10cm. y 20 cm.
Determine la longitud de los catetos y la altura relativa a la hipotenusa.
C
27.
R Si Δ ABC ∼ Δ PQR, determine el
75 perímetro del Δ ABC
b
36

A c B P 48 Q
28. Determine el radio de la circunferencia circunscrita a un triángulo isósceles cuya base mide 12 cm. y
sus lados congruentes 10 cm.

29.

En un rombo, la suma de las diagonales es 70 m.


y el radio del círculo inscrito es 12 m. Calcular las
longitudes de las diagonales y la longitud del lado

C
30. En el Δ ABC, b = 15 , a – c = 9. Determine el perímetro y el a
b
área de dicho triángulo.
A c B
2
31. El área de un rectángulo es 5,886.2 m y su perímetro es 365 m. Calcular sus dimensiones.

32. Calcular las dimensiones de un rectángulo, si su diagonal mide 17 m. Y su superficie mide 120 m2.

33. Calcular los lados de un rectángulo, sabiendo que si se agregan 3 m. a su base y se quita otro tanto
a la altura, el área no se altera; pero si se agregan 5 m. a su base y se quitan 3 m a su altura, el área
aumenta en 16 m2.

59
Elementos Fundamentales de la Geometría

34. Los 2 / 5 del área de un rectángulo equivalen al área de un cuadrado de 2.4 m. de lado. La base del
rectángulo es 4.5 m. Calcular su altura
A
35.
La base de un triángulo acutángulo es 63 m. y su altura
54 m. Se inscribe en éste un rectángulo cuyo
D E perímetro es 116 m. Encuentre las dimensiones del
rectángulo
B C
N M
36. A B

ABCD es un trapecio. AE ⊥ CD , AB = 8,
AE = 6, CD = 12.
Encuentre el área del trapecio

D E C
37.
A B
ABCD es un trapecio. AB || CD , AD = 6,
AB = 8, m∠ D = 90° y m∠ C = 30° .
Encuentre el área de ABCD

D C
38. La base menor de un trapecio es los 3 / 7 de la mayor. La altura es los 15 / 16 de la diferencia de las
bases. La superficie mide 28.83 m2. Calcular las bases y la altura
39.
C D

En el trapecio de la figura, AB || CD
AC = BD = 13, CD = 12 y AB = 22.
Determine el área.
A E F B
40. A B

ABCD es un rectángulo. CE es 1 / 4 de CD
El área del Δ ECB es 16 ¿Cuál es el área del
trapecio ABED?
D E C
41. ¿Cuál es la altura del triángulo que resulta al prolongar los lados no paralelos de un trapecio cuyas
bases son de 27 m y 38.5 m y su altura es de 15 m?
42. Una cruz es formada al cortar cuadrados iguales en las esquinas de un pedazo de cartulina con lado
de 9 pulgadas. Si cada uno de los cuadrados cortados tiene un área de 9 pulg.2 ¿Cuál es el perímetro
de la cruz? ¿Cuál es el área?
A B
43. ABCD y BFED son cuadrados.
Si AB = 4, encuentre el área del BFED C
D F

60
Elementos Fundamentales de la Geometría

44. D C

En un rectángulo ABCD, AB = 20 y BC = 15.


Calcular la distancia del vértice A a la diagonal BD.

A B

45. Por un punto M de un triángulo ABC se trazan dos paralelas a los otros dos lados , MN = z y MP = y,
como se muestra en la figura. Determinar x en función de los lados del triángulo ABC para que
z+y=m+n
A
n m
N M
z
x
y
B P C

46. Demostrar que en un triángulo rectángulo, el inverso del cuadrado de la altura relativa a la hipotenusa
es igual a la suma de los inversos de los cuadrados de los catetos.

47. Dos lados consecutivos de un paralelogramo tienen 17 y 32 m; una diagonal tiene 43 m. Calcular la
otra diagonal.

48. Dos lados de un triángulo tienen 17 y 21m; la proyección del tercero sobre el segundo es 11 m.
Calcular el tercer lado y la altura relativa del segundo.

49. Las diagonales de un paralelogramo tienen 25 y 40 m. y uno de los lados mide 18 m. Calcular el
perímetro.

50. Encontrar las dimensiones de un rectángulo cuya diagonal mide 75 m, sabiendo que es semejante a
un segundo rectángulo cuyos lados son 36 y 48 m.
51 A F B

G En la figura el área del cuadrado ABCD es 36 cm2


y el área del cuadrado EFGH es 25 cm2. Calcular
el perímetro del triángulo AFE.
E

D H C
52. El baricentro de un triángulo ABC dista 3 cm. del vértice A, 4.8 cm. del vértice B y
5 cm. del vértice C.
a) determine las longitudes de las medianas b) determine las longitudes de los lados del triángulo
c) calcule el área del triángulo.

61
Elementos Fundamentales de la Geometría

53. Calcule el área de la región sombreada en cada figura


D

C
E
i) El área del cuadrilátero ABCD mide 1. E es punto
medio de la diagonal AC.
A B
C
ii) ABC es un triángulo de área 1. Los lados AC,
CB y AB se dividen en 2, 3 y 4 partes iguales
Q respectivamente.
P

A M N B

C
Q iii) ABC es un triángulo de área 1. Los lados AC,
P CB y AB se dividen en 2, 3 y 4 partes iguales
respectivamente.

A M B

D H C

E G iv) ABCD es un cuadrado de lado 1. E, F, G y H son


puntos medios en sus lados respectivos. Calcule el
área de la región sombreada.

A F B

SOLUCIONES
1. 21.6 m. y 8.4 m. 2. 133.67 m. 3. 7.2 4. 168 5. 6 6. 6
7. i) 12.5 ii) 20 iii) 10 8. 20 y 10 9. 14.14, 24.49, 17.32 m. 10. 7.11, 43.89, 23.96 m. 11. 9 3
12. 16 / 9 13. 2 14. 80 15. 12 y 30 16. 168 17. 6 3 18. 25 3 19. 60
20. 9 y 12 21. 15 y 20 22. 9 , 12 y 15 23. 12 , 18 y 20 cm. 24. 6 , 8 y 10 m.
25. 3 , 4 y 5 26. 10 3 ,10 6 y 10 2 27. 180 28. 6.25 29. 40 , 30 y 25 30. 40 y 60
31. 140.65 y 41.85 m. 32. 8 y 15 m. 33. 8 y 11 34. 3.2 , 35. 28 y 30 m. 36. 60 37. 79.18
38. 8.68 m, 3.72 m y 4.65 m. 39. 204 40. 112 41. 50.217 42. P = 36 pulg., A = 45 pulg2
43. 32 44. 12 45. x = [b(b+c-a)] / (b+2c-a) 47. 27.875 48. 17.607 y 13.748 49. 92.16
50. 45 y 60 51. 11 52. mA = 4.5 , mB = 7.2 , mC = 7.5 ; 9.33 , 6.25 , 6.7 cm ; 20.9 cm2
54. i) 1/2 ii) 3/8 iii) 5/24 iv) 1/6

62
Elementos Fundamentales de la Geometría

3.5. CIRCUNFERENCIAS Y POLÍGONOS


3. 5. 1. CONCEPTOS GENERALES

CIRCUNFERENCIA: Es el conjunto de todos los puntos de un plano que equidistan de otro punto fijo
llamado centro de la circunferencia.

RADIO: Se llama radio a todo segmento que une el centro con un punto de la circunferencia. También se
le llama radio a la longitud de dicho segmento.

Una circunferencia con centro en Q y radio r, divide a un plano π en tres conjuntos disjuntos:
- la circunferencia misma: { P∈ π / PQ = r }
- el interior de la circunferencia: { P∈ π / PQ < r }
- el exterior de la circunferencia: { P∈ π / PQ > r }

CÍRCULO: Es el conjunto de todos los puntos que están sobre una circunferencia o en su interior:
{ P∈ π / PQ ≤ r }

exterior tangente secante


A
P

interior E Q D
diámetro
G radio B

círculo F
C
ángulo central
cuerda

CUERDA: Es todo segmento determinado por dos puntos de la circunferencia.

DIÁMETRO: Es toda cuerda que pasa por el centro. Su longitud es el doble del radio: D = 2r.

SECANTE: Cualquier recta que corta a la circunferencia en dos puntos.

TANGENTE: Toda recta en el plano de la circunferencia que tiene uno y sólo un punto P en común con la
circunferencia. Al punto P se le llama punto de tangencia.

ÁNGULO CENTRAL: Es todo ángulo determinado por dos radios, es decir un ángulo en el plano de la
circunferencia con vértice en el centro.

ARCO: es una “porción” continua de la circunferencia. Se clasifican en arco menor y arco mayor.
Si A, B y C están en la circunferencia, con C en el exterior del ángulo ∠ AOB, el arco menor, denotado
⁀ es la unión de A, B y todos los puntos de la circunferencia que están en el interior del ∠ AOB.
por AB,
El arco mayor es la unión de A, B y todos los puntos de la circunferencia que están en el exterior del
∠ AOB.
Los puntos A y B son los extremos del arco.
A menudo para evitar ambigüedades, es necesario indicar otro punto que pertenezca al arco. Así por
⁀ denota el arco cuyos extremos son P y R, y contiene a Q.
ejemplo PQR,
MEDIDA DE UN ARCO: La medida de un arco menor se define como la medida del ángulo central que lo
⁀ = m∠ AOB. La medida del arco mayor correspondiente es m ACB
determina: m AB ⁀ = 360° – m ⁀ AB

191

63
Elementos Fundamentales de la Geometría

C
O

B
SEMICIRCUNFERENCIA: es un arco cuya medida es 180°.

CIRCUNFERENCIAS CONGRUENTES: son circunferencias con radios de igual longitud.

CIRCUNFERENCIAS CONCÉNTRICAS: son circunferencias, en el mismo plano, que tienen el mismo


centro.

CIRCUNFERENCIAS TANGENTES: son circunferencias tangentes a la misma recta, en el mismo punto


de tangencia. Es decir dos circunferencias que tienen exactamente un punto en común.

ALGUNAS PROPIEDADES:

1. i) Una tangente a una circunferencia es perpendicular al radio que une el centro con el punto de
tangencia.
___ ↔
ii) Si el radio OP de una circunferencia de centro O, es P t
↔ ↔
perpendicular a una recta t que pasa por P, entonces la recta t __ ↔
es tangente a la circunferencia en el punto P. O OP ⊥ t

2. En una circunferencia o en circunferencias congruentes, a


cuerdas congruentes le corresponden arcos congruentes y viceversa.
B C

A ⁀= m CD
AB = CD ⇔ m AB ⁀
D A

3. Todo diámetro o radio perpendicular a una cuerda, divide a ésta y a los O


•C D
arcos correspondientes en partes congruentes.

4. En el plano de una circunferencia, la mediatriz de una cuerda pasa por el B


centro de la circunferencia.

5. En la misma circunferencia o en circunferencias congruentes, las cuerdas equidistantes del centro son
congruentes y viceversa, si las cuerdas son congruentes, equidistan del centro. Además los arcos
determinados por las cuerdas son congruentes.

B C AB = CD ⇔ OE = OF

A E
F ⁀
⁀≅ CD
AB = CD ⇔ AB
O
D

64
Elementos Fundamentales de la Geometría

3. 5. 2. POSICIONES RELATIVAS DE DOS CIRCUNFERENCIAS EN EL PLANO

Al considerar las posiciones relativas entre dos circunferencias en un plano se presentan los siguientes
casos:

1. Las dos circunferencias no tienen ningún punto en común

◊ Si una circunferencia es exterior a la otra, la distancia entre los centros es mayor que la suma de los
radios.

◊ Si una de las circunferencias es interior a la otra y los centros de las circunferencias son diferentes, la
distancia entre los centros es menor que la diferencia de los radios.

◊ Si las circunferencias son concéntricas, es decir tienen el mismo centro, obviamente la distancia entre
los centros es cero.

O1 O2 r2 r2

O2 ••
r1
•r r1 O1= O2 •
r1
2 O1

O1O2 > r1+ r2 O1O2 < r1 – r2 O1= O2

2. Las circunferencias tienen exactamente un punto en común.

En este caso decimos que las circunferencias son tangentes entre sí.
◊ Si las circunferencias se “tocan” exteriormente, la distancia entre los centros es la suma de los radios.
◊ Si las circunferencias se “tocan” interiormente, la distancia entre los centros es la diferencia de los
radios y la recta que une los centros es perpendicular a la tangente común.

O2 r2 O2 r2
O2 r2 r1 O1 O1 r1 r1 O1

O1O2 = r1 + r2 O1O2 = r2 – r1
r2 – r1 < O1O2 < r1 + r2

3. Las dos circunferencias tienen dos puntos comunes

En este caso la distancia entre los centros es mayor que la diferencia de los radios pero menor que la
suma de los mismos. El segmento que une los centros es perpendicular a la recta que pasa por los
puntos de intersección y biseca a la cuerda común.

4. Si las circunferencias tienen tres puntos comunes, tendríamos la misma circunferencia, ya que por tres
puntos no colineales pasa exactamente una circunferencia.

65
Elementos Fundamentales de la Geometría

3. 5. 3. RELACIONES ENTRE ÁNGULOS Y ARCOS EN UNA CIRCUNFERENCIA

En dependencia del punto donde se localice el vértice de un ángulo y la relación entre sus lados y una
circunferencia se tienen diversos tipos de ángulos, entre ellos se tienen:

ANGULO CENTRAL: Es todo ángulo con su vértice B


en el centro de la circunferencia. Su medida es igual
a la de su arco correspondiente
O A

A
ÁNGULO INSCRITO: Es el ángulo
que tiene su vértice en la circunferencia
y sus lados son rayos secantes
P

TEOREMA 1. La medida de un ángulo inscrito es igual a la mitad del arco comprendido entre sus lados.
m⁀
1
En la figura, m ∠ APB = AB
2

P
ANGULO SEMI-INSCRITO: Es el ángulo que tiene su vértice en la
circunferencia y sus lados lo forman un rayo tangente y un rayo
secante
B

TEOREMA 2. La medida de un ángulo sem.-inscrito es igual a la mitad del arco comprendido entre sus
m⁀
1
lados. En la figura, m ∠ APB = PB
2

C
ÁNGULO INTERIOR: Es el ángulo cuyo vértice A
es un punto interior de la circunferencia . Sus
lados y sus prolongaciones forman dos secantes P
que se cortan en el interior. B D

TEOREMA 3. La medida de un ángulo interior es igual a la semisuma de las medidas de los arcos
comprendidos por sus lados y sus prolongaciones.
AB + m ⁀
1
En la figura , m ∠ APB = [m ⁀ CD ] y
2

⁀+ m ⁀
1
m ∠ APC = [ m AC BD ]
2

ÁNGULO EXTERIOR: Es el ángulo cuyo vértice es un punto exterior de la circunferencia y sus lados son
dos secantes, dos tangentes o uno secante y otro tangente.

66
Elementos Fundamentales de la Geometría

TEOREMA 4. La medida de un ángulo exterior es igual a la semi-diferencia de las medidas de los arcos
comprendidos por sus lados y sus prolongaciones.

⁀– m⁀
1
En la figura, m∠P= [ m AB CD ].
2
Si el ángulo exterior es formado por una tangente y una secante o bien por dos tangentes, la fórmula
anterior es válida, aunque la forma de denotar los arcos varíe.

A A A
C
P P
D B P
B C

ÁNGULO EX – INSCRITO: Es el ángulo


adyacente a un ángulo inscrito que forma
par lineal con dicho ángulo.

ALGUNAS PROPIEDADES

1. Dos ángulos cualesquiera inscritos en el mismo arco son congruentes.


P1 A

P2 1 ⁀
m∠P1 = m∠P2 = m∠P3 = m AB
2
B

P3

2. Un ángulo cualquiera inscrito en una semicircunferencia es un ángulo recto.

3. Los ángulos opuestos de un cuadrilátero inscrito en una circunferencia (cuadrilátero cíclico) son
suplementarios.

C
D m∠ A + m∠ C = 180°

m∠ B + m∠ D = 180°
A
B

67
Elementos Fundamentales de la Geometría

EJEMPLOS 1. En la figura O representa el centro de la circunferencia. Si m∠AOC = 120°, determine


los valores de α, β y x.

C x SOLUCIÓN:
⁀ = m ∠AOC = 120°
i) x = m AC
B β α A ii) β = 180° – m ∠ AOC = 180° – 120° = 60°
O
1
ii) α = ⁀ pero m BC
m BC, ⁀ = β = 60°
2
1
∴ α= (60°) = 30°
2


2. Datos: m CD = 120°; EF es tangente a la circunferencia en B.
D Halle los valores de α, β y x.
A i) x = m BC = 180° – m CD = 180° – 120° = 60°
O ya que BC ∪ CD forman una semicircunferencia.
β 1 1
ii) β = m BC = (60°) = 30° , por ser un ángulo inscrito.
C 2 2
α x
1
iii) α = m AB ⁀ por ser un ángulo semi-inscrito, pero
E B F 2
m AB ⁀ = m∠AOB = m∠DOC = 120° (por definición de medida de
arco y por ser opuestos por el vértice)
1
∴α= (120°) = 60°
3. 2
En la figura de la izquierda, determine el valor de
C A α si m AB⁀ = 30° y m CD ⁀ = 90°.
90° α 30° SOLUCIÓN:
Por ser α un ángulo interno tenemos:
B 90° + 30°
D α= = 60°
2

4. 65°
A
C ⁀ = 130°, m DB
En la figura, mAB ⁀ = 115°, m AC
⁀ = 65°.
s° β Determine los valores de s, α, β y m ∠P.
P α 130°
D B
115°

SOLUCIÓN:
⁀ + m DB
Tenemos m AB ⁀ + m DC⁀ + m CA ⁀ = 360° y m DC
⁀ = s°, luego
s° = 360° – 130° – 115° – 65° = 50°.

1
∠ADB es un ángulo inscrito que determina el arco AB, luego α = m∠ ADB = ⁀ = 1 ( 130°) = 65°
m AB
2 2

β es la medida de un ángulo interno que determinan los arcos AB y CD, luego


1
β= ( m AB ⁀ = 1 ( 130° + 50°) = 90°
⁀ + m CD)
2 2

68
Elementos Fundamentales de la Geometría

∠P es un ángulo externo que determinan los arcos AB y CD, por tanto


1 1
m∠ P = ( m AB ⁀ = (130° – 50°) = 40°
⁀– m CD)
2 2
C
5. En el triángulo ABC, si I es el incentro, entonces m∠AIB = 90º +
2
C
Se tiene A + B + C = 180 ∴ A + B = 180 – C
m∠AIB = 180 – m∠IAB – m∠IBA
A B ⎛ A +B⎞
I = 180 – − = 180 – ⎜ ⎟
2 2 ⎝ 2 ⎠
⎛ 180 − C ⎞ C
A B = 180 – ⎜ ⎟ = 90º +
⎝ 2 ⎠ 2

A
6. Si O es el circuncentro del ΔABC, entonces m∠BOC = 2 m ∠A

O Dado que O es el circuncentro, ∠BOC es un ángulo central, por tanto


m∠BOC = m BC ⁀ (1)
1 ⁀ por ser un ángulo inscrito (2)
B C m∠A = m∠BAC = m BC
2
de (1) y (2) m∠BOC = 2 m ∠A

7.
A En la figura, la medida del arco AB es 80º.
B Las circunferencias son tangentes en D. La recta AB
C es tangente a la circunferencia pequeña en C.
Encuentre la medida del ∠BDC.
D

SOLUCIÓN:
Trazamos una línea que una los centros de las
circunferencias, por ser tangentes, esta línea pasa
por el punto de tangencia; prolongamos el
α/2 β/2 segmento AC de manera que corte a OO’ en P.
Sean α la medida del ángulo central determinado
por el arco BD y β la medida del ángulo central
B •
A F
80º • determinado por el arco DC.
C
100 – α α β 180 – β
Q
O D O’ P

Se tiene m∠BDF = α/2 y m∠FDC = β/2, por ser


ángulos semi-inscritos, luego el ángulo buscado
mide: m∠BDC = (α+β)/2.
El ángulo central AOB mide 80º, luego m∠AOQ = 180º – 80 – α = 100 – α, de manera similar
m∠CO’P = 180 – β , ya que en cada caso se determina una semicircunferencia.
El ángulo P es exterior a cada circunferencia, por tanto su medida es la semidiferencia de los arcos que
determina, luego
(100 − α) − α β − (180 − β)
m∠P = = ∴ α + β = 140º. Por tanto m∠BDC = 70º
2 2

69
Elementos Fundamentales de la Geometría

3. 5. 4. RELACIONES MÉTRICAS EN UNA CIRCUNFERENCIA



DEFINICIÓN: Si PA es tangente a la circunferencia en A, entonces PA se llama segmento tangente
desde P a la circunferencia.

TEOREMA: Los dos segmentos tangentes a una circunferencia desde un punto exterior son congruentes
y determinan ángulos congruentes con el segmento que une el punto exterior con el centro.

TEOREMA: Sean una circunferencia C y un punto Q de su exterior. Sea L1 una secante que pasa por Q e
interseca a C en los puntos R y S; y sea L2 otra secante que pasa por Q e interseca a C en los puntos U y
T, entonces QR ⋅ QS = QU ⋅ QT

TEOREMA: Sean QT un segmento tangente a una circunferencia en T, y una recta secante que pasa
por Q e interseca a la circunferencia en los puntos R y S, entonces QT2 = QR ⋅ QS
Al valor de QT2 se le conoce como la potencia de Q respecto a la circunferencia C. Este valor depende de
la distancia desde P a la circunferencia y el radio de la misma.

TEOREMA: Si RS y TU son cuerdas de la misma circunferencia que se cortan en Q, entonces


QR ⋅ QS = QU ⋅ QT

T U S
A S
R
O Q Q
P R Q
U
T R
S T
B
PA = PB QR ⋅ QS = QU ⋅ QT QT2 = QR ⋅ QS QR ⋅ QS = QU ⋅ QT
∠APO ≅ ∠BPO

EJEMPLOS
9. En la siguiente figura, AB = 25, AE = 18 y DC = 27. Determine el valor de EC
B
D SOLUCIÓN:
Sea EC = x. Como DC = 27, entonces ED = 27 – x
E EB = AB – AE = 25 – 18 = 7. Tenemos AE⋅EB = CE⋅ED.
A
Al sustituir los valores dados:
C 18 (7) = x (27 – x) ⇒ x2 – 27x + 126 = 0 ∴ x = 6 o x = 21
Hay dos valores posibles para EC.

10.
En la figura PA = AD, PB = 4 y BC = 7, hallar PD.
D
A SOLUCIÓN:
Se cumple PA⋅PD = PB⋅PC. Además PD = PA + AD y
P PC = PB + BC = 4 + 7 = 11.
Si hacemos PA = x entonces AD = PA = x y PD = 2x.
B Luego x (2x) = (4) (11) = 44
C
∴ x2 = 22 x = 22 . Resulta PD = 2 22

70
Elementos Fundamentales de la Geometría

EJERCICIOS PROPUESTOS

1.
→ S
En la figura, de la derecha, KS es T
tangente a la circunferencia en T; K
→ N M
KR es secante a la circunferencia
cortándola en Q y R .
Si m∠K = 30° y m∠TRQ = 20°, determine Q
las medidas de los arcos QNT y TMR
R

P
2. Q

Si m∠P = 60° y m ∠PQR = 128°


Determine m∠R y m∠S
R
B
S A
3.
Determine la medida del ángulo α si
⁀ = 120° y DC
a) AB ⁀ = 150° α
D C
⁀ = 140°
⁀ = 80° y BC
b) AD

C
4.
A Determine las medidas de los ángulos α y β si:
⁀ = 50o y CD
a) AB ⁀ = 80o
P α β ⁀ = 90 , BD
b) AC o
⁀ = 120o y AB ⁀ = 40°
⁀ = 100º
c) m∠ ADB = 20º y m CD

B
D
5.
A β
Halle la medida de α , β y θ si
⁀ = 50°, AD
BC ⁀ = 100°, m∠ ABC = 85° P α B

D θ
C
6.
T P
PT es tangente a la circunferencia en T.
⁀ = 60° y m∠ TPQ = 30° determine las
Si TQ
medidas de los arcos TR y RQ.
Q

71
Elementos Fundamentales de la Geometría

7. En cada una de las siguientes figuras, O representa el centro de la circunferencia. Con la información
dada en cada caso, determine el valor de α , β y x indicados:
D
C x
A
O
β α α
B O A C
x
β
Dato : m ∠ AOC = 120° E B F
︵ ↔
Datos: m CD =150°, EF tangente en B

8.
B A ︵
En la figura m BD = 70 ° , m ∠DMB = 4 m ∠K.
Determine m AC⁀ y m∠K
M K

D C

9. Un triángulo ABC está inscrito en una circunferencia; m∠A = 50° y m∠B = 70°. Se trazan tangentes a la
circunferencia por A, B y C de modo que forman el triángulo circunscrito A’ B’ C’. Determine las medidas de los
ángulos del triángulo A’ B’ C’.

10. A partir de la figura y la información dada, encuentre el valor indicado


A
B a) AC = 16, PB = 6, PD = 8. Hallar AP y PC
P b) AP = 3, PC = 5, PD = 4. Hallar PB
c) PC = 2⋅ PA, PD = 4 y BD = 12. Hallar AC
D C d) BD = 15, PB = 6 y PB = 3 PA. Hallar PC

11. D
A
a) PA = AD, PB = 4, BC = 7. Hallar PD
P
b) PD = 6, PA = 2 y PC = 5. Hallar PB
c) PD = 7, AD = 4 y BC = 5. Hallar PB
d) PA = 8, AD = 12 y BC = 10. Hallar PC B
e) PD = 12, PA = 4, PC = 10. Hallar PB
C

12. B a) PB = 18, AB = 4, hallar PT


b) PT = 8 , PB = 20, hallar PA
c) PT = 14, PA = 8, hallar AB
A d) PT = 20, AB = 9, hallar PB

T P

72
Elementos Fundamentales de la Geometría

A

13. Se tiene PA tangente a la circunferencia en A.
P
AP = PX = XB. Si PQ = 1 y QR = 8, determine AX.
x
Q
R
B
B


14.
A C Si m ABC = 120° y O es el centro de la circunferencia, encuentre
m ∠ OAC
O

15. En la figura, m∠QOR = 60º y ∠ PQO ≅ ∠ PRO.


O es el centro de la circunferencia. Q
Encuentre la medida del ∠ PQO.
P O
R

16 __ __
C En la figura AP y CP son tangentes a la
P O •B circunferencia de centro O y radio r = 17 3 .
Si la medida del arco ABC es 300º, determine la
A longitud de AP.

17. El diámetro de una circunferencia mide 50 cm. i) ¿cuánto habrá que prolongarlo en un extremo para
que la tangente a la circunferencia trazada desde el extremo de la prolongación mida 24?
ii) Si el diámetro se prolonga 10 cm. ¿cuánto mide la tangente trazada desde ese extremo?

C
18. Dos circunferencias con centros O y O’ se cortan en C y D. A
Los radios de las circunferencias son 5 cm. y 10 cm. respectivamente. • E
___ ___

O O’
Si AB = CD = EF y AB está a 4 cm. de O, calcule la distancia de EF a O’.
B D
F
SOLUCIONES

1.
︵ ︵
QNT = 40º, TMR = 100º 2. m∠R = 120º, m∠S = 52º
4. a)α = 15º, β = 65º b) α = 35º, β = 75º c) α = 30º, β = 70º
3.
5.
a) 135º, b) 70º
︵ ︵
α = 45º, β = 70º, θ = 60º 6. m TR = 120º, m RQ = 180º
7. i) α︵= 120º, β = 60º, x = 120º ii) α = 15º, β = 75º, x = 30º
8. m AC = 42º, m∠K = 14º 9. 40°, 60° y 80º 10. a) 4 y 12, b) 3.75, c) 12 d) 27
11.a) 2 22 , b) 2.4, c) 2.72, d) 18.6 e) 4.8 12. a) 15.87, b) 3.2, c) 16.5, d) 25
13. 4 14. 30º 15. 15º 16. 17 17. i) 9.65 ii) 24.49 18. 9.54 cm.

73
Elementos Fundamentales de la Geometría

3. 6. POLÍGONOS

3.6.1. CONCEPTOS GENERALES

Un polígono es una figura cerrada formada por la unión de varios segmentos, de manera que no se cruzan.
◊ Los segmentos se llaman lados del polígono.
◊ Los extremos de los segmentos se llaman vértices.
◊ La suma de las longitudes de los lados se llama perímetro.
◊ Los ángulos del polígono, son los que contienen dos lados consecutivos

CLASIFICACIÓN
1. De acuerdo con el número de lados los polígonos se clasifican en:

triángulo 3 lados decágono 10 lados


cuadrilátero 4 lados endecágono 11 lados
pentágono 5 lados dodecágono 12 lados
hexágono 6 lados pentadecágono 15 lados
heptágono 7 lados icoságono 20 lados
octógono 8 lados ...
eneágono 9 lados ....

2. De acuerdo a otras características se clasifican en:

Equiángulo: polígono que tiene sus ángulos internos congruentes.


Equilátero: polígono que tiene sus lados congruentes.

Convexo: si ningún par de sus lados están en lados opuestos de una recta que contenga un lado del
polígono. Se reconocen gráficamente por que no tiene ángulos internos con “medida” mayor que 180°.

Regular: los polígonos que son a la vez equiláteros y equiángulos.


◊ Todo polígono regular es convexo.
POLÍGONOS REGULARES

Una de las propiedades de los polígonos regulares es que pueden inscribirse y circunscribirse en una
circunferencia.

- El hexágono está circunscrito - El octógono está inscrito en la


a la circunferencia. circunferencia.
- La circunferencia está inscrita - La circunferencia está
en el hexágono circunscrita al octógono.

Se llama centro de un polígono regular al centro común de sus circunferencias inscrita y circunscrita.

El radio de un polígono regular es el


segmento que une el centro con un vértice,
es decir el de la circunferencia circunscrita. Ángulo interno
α O r Ángulo central
Se llama ángulo central de un polígono al radio
que contiene dos radios que pasan por dos θ
vértices consecutivos. apotema ap Ángulo externo
β

74
Elementos Fundamentales de la Geometría

Se llama apotema de un polígono regular al segmento perpendicular trazado desde el centro a uno de
sus lados. La apotema es igual al radio de la circunferencia inscrita en el polígono. En general, si R es el
radio de un polígono regular, ap la apotema y l la longitud de los lados,
l2 1
R 2 = a p2 +
ap = 4 R 2 − l2
4 2
Los ángulos internos son los que contienen dos lados consecutivos. Los ángulos externos, son los
ángulos adyacentes a los interiores, obtenidos al prolongar uno de los lados.

El número de lados n de un polígono es igual al número de vértices, ángulos centrales, ángulos internos
y ángulos externos.

Las diagonales de un polígono son los segmentos que unen cada vértice con otro no consecutivo.

Si llamamos θ: medida del ángulo central


α: medida de un ángulo interno
β: medida de un ángulo externo

Para un polígono regular de n lados se cumple:

360º ( n− 2 ) 360º
θ= α = 180º − θ = ⋅ 180º β= θ=
n n n

La suma de los ángulos centrales es 360°


La suma de los ángulos internos es (n – 2)⋅180°
La suma de los ángulos externos es 360°
Si d es el número de diagonales que pueden trazarse desde un vértice de un polígono convexo y D es el
número total de diagonales, se tiene:
n⋅ (n− 3)
d=n–3 , D=
2

Usando las propiedades de los triángulos equiláteros y los cuadrados, se tienen los siguientes resultados:

Si a es la longitud de los lados, ap la apotema, r el radio de la circunferencia inscrita y R el radio de la


circunferencia circunscrita

Para un triángulo equilátero

R 3 3
r R= ⋅ a , r = ap = ⋅ a , R = 2r
3 6
a

En general, puede probarse que el radio de la circunferencia circunscrita a un triángulo cualquiera está
a bc
dado por R = , donde a, b y c son las longitudes de los lados del triángulo y A su área.
4A
A
También puede probarse que el radio de la circunferencia inscrita está dado por r = , donde s es el
s
semiperímetro del triángulo.

75
Elementos Fundamentales de la Geometría

Para un cuadrado

2 1
R R= ⋅ a , r = ap = ⋅a
r 2 2

Para un hexágono regular:


3
R = a, r= a , ap = r
R 2
r

a
Para un pentágono regular:

R 1
R a5 = 10 − 2 5 , a p = 4 R 2 − a2
ap 2 2

◊ Para un polígono de 2n lados se cumple l 2n = 2R2 − R 4 R 2 − ln 2 , donde ln es la longitud del


lado del polígono regular que tiene la mitad de lados.

EJEMPLO

El lado de un cuadrado inscrito en una circunferencia de radio 1, es 2 . El lado de un octógono regular


inscrito en la misma circunferencia será:

L8 = 2 (1) 2 − 1 ⋅ 4 (1) 2 − ( 2 ) 2 = 2 − 2
De manera similar el lado de un polígono de 16 lados inscrito en la misma circunferencia será:

L16 = 2 − 4 − ( 2 − 2 )2 = 2 − 2 + 2

◊ Para el resto de polígonos la relación entre los lados y los radios se obtienen utilizando las funciones
trigonométricas.

3.6.2. ÁREAS Y PERÍMETROS DE POLÍGONOS REGULARES

Si n es el número de lados, a la longitud de cada lado y ap el apotema, se tiene:

- Perímetro: P=n⋅a
1 1
- Área: A= n⋅ a ⋅ ap = ⋅ P⋅ap
2 2

76
Elementos Fundamentales de la Geometría

Para un polígono irregular se requiere conocer información adicional tales como ángulos internos,
longitudes de los lados, longitudes de las diagonales, etc. Para encontrar el área generalmente
“triangulamos” el polígono de la manera más adecuada, de acuerdo a la información que se tenga. En
muchas ocasiones hay que hacer uso de la trigonometría.

EJEMPLO

1. El perímetro de un decágono regular es 24.72 y su apotema 3.8. Cuál es el radio de la circunferencia


circunscrita. Determine el área de la región poligonal correspondiente (redondee su respuesta).

SOLUCIÓN:
P 24.72
Datos: P = 24.72, ap = 3.8, n = 10 ∴ a = = = 2.472.
n 10
2 2
⎛l⎞ ⎛ 2.472 ⎞
Se tiene R = a p2 + ⎜ ⎟ = 3.8 2 + ⎜ ⎟ ≈4
⎝2⎠ ⎝ 2 ⎠
1 1
A= P ap = (24.72) (3.8) ≈ 47.
2 2

3. 6. 3. LONGITUD DE LA CIRCUNFERENCIA. ÁREAS DE UN CÍRCULO Y REGIONES CIRCULARES.

TEOREMA: La razón de la longitud de una circunferencia C a su diámetro es la misma para todas las
circunferencias y se denota por π , π ≈ 3.1416..:
C
= cons tan te = π ∴ C = 2πr
2r
El número π no es racional . Podemos afirmar que la longitud de la circunferencia es el “límite” de los
perímetros de los polígonos regulares inscritos.

Tenemos l2n = 2R2 − R 4 R 2 − ln2 , P = 2n⋅l2n → 2πR para n “grande”


n ⋅ l2 n n
Luego π ≈ ≈ ⋅ 2R2 − R 4 R 2 − ln2 para n “grande”.
R R

ÁREA DE UN CÍRCULO

El área de un círculo es el “límite” de las áreas de los polígonos regulares inscritos en la circunferencia
correspondiente. Puede probarse que el área de un círculo de radio r o diámetro d está dada por:

π ⋅ d2
A = π r2 =
4

REGIONES CIRCULARES

1. Sector circular: Es la sección de un círculo limitada


r
por dos radios y el arco correspondiente. O θ

77
Elementos Fundamentales de la Geometría

2. Segmento Circular: Es la sección del círculo limitada


por una cuerda y el arco correspondiente.
r
θ

3. Corona Circular: es la sección de un plano


limitada por dos circunferencias concéntricas. r
R

4. Trapecio Circular: Es la sección de una


corona circular, limitada por dos radios.

LONGITUDES DE ARCOS Y ÁREAS DE REGIONES CIRCULARES

TEOREMA: Si dos arcos tienen radios iguales, entonces sus longitudes son
proporcionales a sus medidas.
r
r s1 θ 1
θ1 s1 =
θ2 s2 s2 θ2

⁀ que mida θ°, su


◊ A partir de este teorema y usando la “Regla de tres” se obtiene que para un arco AB
πr θ °
longitud está dada por: s =
180°
◊ Si el arco se mide en radianes, resulta s = r θ

◊ El área de un sector circular también se obtiene usando la proporcionalidad anterior:


π r 2θ ° 1
A= = rs
360° 2
1 2
◊ Si el ángulo se mide en radianes, resulta la fórmula equivalente A = r θ
2
◊ El área de un segmento circular puede calcularse mediante
la diferencia entre el área del sector circular correspondiente y el triángulo s
formado por los radios y la cuerda que une los extremos del h
arco. Para calcular el área del triángulo generalmente hay que hacer
uso de la trigonometría, salvo los casos de triángulos con ángulos de 30°, x
45°, 60°, 90° o múltiplos de ellos. y
θ r

Tenemos
h=r–y
θ 1 θ 1 θ θ
y = r cos = x cot = 4r2 − x2 , x = 2 r sen = 2 y tan
2 2 2 2 2 2

◊ Si θ se mide en radianes, el área del segmento circular está dada por:


1 2 1
A= r (θ – sen θ) = (r s – x y) r
2 2
◊ El área de una corona circular puede obtenerse como la diferencia entre las R
áreas del círculo exterior y el círculo interior, o sea A = π (R2 – r2)

78
Elementos Fundamentales de la Geometría

◊ El área de un trapecio circular es la diferencia entre las áreas de los sectores


circulares que los determinan
R
πθ° 1 θ
A= (R 2 − r 2 ) ó A = θ (R2 – r2), θ en radianes r
360° 2
Si hacemos h = R – r y s1 y s2 son las longitudes de los arcos exterior e interior
respectivamente, se tiene:

EJEMPLOS 1 1
A= h θ (R + r) = h (s1 + s2)
2 2

1. ¿Cuál es el radio de una circunferencia cuya longitud es π?

1
SOLUCIÓN: Como C = 2πr = π , se obtiene r = .
2
2. La longitud de la circunferencia correspondiente de un círculo y el perímetro de un cuadrado son 20
cm. cada uno ¿cuál tendrá mayor área?

SOLUCIÓN:
20 10 10 2
Tenemos C = 2πr = 20 ∴ r = = ,A o = πr2 = π ( ) = 31.83 cm2
2π π π
P = 4 l = 20 ∴ l = 5 y A = l2 = 52 = 25 cm2. Luego el círculo tiene mayor área.

3. Si O es el centro de la circunferencia de la figura, r = 6 y m∠AOB = 60°, determine el área de la


región sombreada y la longitud del arco AB.

SOLUCIÓN:
A Tenemos que para un sector circular
r
πr2 θ 1 π r θº
O A= = rs ys=
θ 360º 2 180º
π (6) 2 (60º ) π (6) (60°)
B A= = 6π y s = = 2π
360º 180°

4. Si O es el centro de la circunferencia de la figura, r = 6 y m∠AOB = 120°, determine el área de la


región sombreada

A La región sombreada corresponde a un segmento circular, cuya área


es la diferencia entre el área del sector circular y el triángulo isósceles
r que se forma.
O θ
π r 2 θº π (6) 2 (120º )
Tenemos: A sc = = = 12π
360º 360º
B
El área del triángulo AOB, dado que m∠AOB = 120°, equivale a la de
A
3
un triángulo equilátero de lado l = r = 6, luego A Δ = (6)2 = 9 3
4
El área buscada es A = 12π – 9 3 = 22.11 u2
O B’

79
Elementos Fundamentales de la Geometría

5. La longitud del lado de un cuadrado es 8 cm. ¿cuál es la longitud de la circunferencia circunscrita?


¿Cuál es la longitud de la circunferencia inscrita?

Para la circunferencia inscrita se tiene que su diámetro


corresponde al lado del cuadrado, luego
d = 8 cm. y su longitud es C = π d = 8π cm.

R Para la circunferencia circunscrita se tiene que el diámetro es


r igual a la diagonal del cuadrado o sea
d = 8 2 , luego C = π d = 8 2 π cm.

6. Hallar las longitudes de las circunferencias inscrita y circunscrita en


a) un triángulo equilátero de lado igual a 12
b) un hexágono regular de lado igual a 6
Solución:
3 3
Tenemos l = 12, R= l y r= l.
3 6
3 1
Luego R = (12) = 4 3 y r = R = 2 3
3 2
R Por tanto la longitud de la circunferencia inscrita es C
r
= 2π(2 3 ) = 4 3 π
12 La longitud de la circunferencia circunscrita es C = 2π
(4 3 ) = 8 3 π

b)

Dado que es un hexágono regular se tiene:


3
R= l, r= l y l = 6.
2
R
r Luego R = 6 y r = 3 3 . Las longitudes buscadas son:
Circunferencia inscrita C = 2π(3 3 ) = 6 3 π
Circunferencia circunscrita C = 2π(6) = 12π

7. Determine si el perímetro del PQRS es igual, menor, o mayor


A
que la longitud de la circunferencia. ¿Quién tendrá mayor área,
P
el cuadrado PQRS o el círculo?
X Q
El ABCD es un cuadrado circunscrito a la circunferencia.
D Y B El XYZW es un cuadrado inscrito a la circunferencia.
W ↔ ↔
Z AC y BD contienen las diagonales de ambos cuadrados.
S
El PQRS es un cuadrado cuyos vértices son los puntos
R
C medios de AX , BY , CZ , DW respectivamente.

80
Elementos Fundamentales de la Geometría

SOLUCIÓN:
O Y Q B
__
Sea E el punto de tangencia sobre BC .
Sean OY = OE = r , por ser radios del círculo.
Tenemos ΔBOC ∼ ΔBEO, ya que ambos son triángulos
E
rectángulos isósceles.
Luego OB = 2 OE = 2r

1+ 2
Siendo Q el punto medio de BY tenemos OQ = r y
2
2+ 2
PQ = 2 OQ = r. El perímetro del PQRS es P = 4⋅PQ = 6.828 r
2
La longitud de la circunferencia C = 2πr = 6.283 r ∴ P > C, es decir el perímetro del cuadrilátero PQRS
es mayor que la longitud de la circunferencia.

2+ 2 2
El área de PQRS es A = ( r) = 2.914r2 u2
2
El área del círculo es A = π r2 = 3.1416 r2 u2. Luego el área del círculo es mayor que el área del
cuadrilátero PQRS.

8. Demostrar que en todo triángulo rectángulo la suma de los catetos es igual a la suma de los
diámetros de las circunferencias inscrita y circunscrita.

SOLUCIÓN:
Primera forma:
A abc 1
En general se tiene r = yR= , y en particular para un triángulo rectángulo A = ab y
s 4A 2
c2 = a2 + b2, donde a y b son los catetos, c la hipotenusa, s el semiperímetro, r el radio de la
circunferencia inscrita y R el radio de la circunferencia circunscrita.
Luego
2 A abc 2 ab abc 2 ab + ac + bc + c 2
2(r + R) = + = + = =
s 2A a + b+ c ab a + b+ c
2 ab + c(a + b) + a 2 + b 2 (a + b) 2 + c(a + b) (a + b)(a+ b + c)
= = = = a+ b
a + b+ c a + b+ c a+ b+ c

Segunda forma:
Para un triángulo rectángulo con hipotenusa de longitud c y catetos, a y b, en particular también se tiene
a+ b− c c a + b− c c a + b
r= y R = r+ R = + =
2
,
2 luego 2 2 2 ∴ 2 (r + R) = a + b

9. El ABCD es un cuadrilátero en el que cada uno de sus lados es tangente a una circunferencia de
radio 9. Si el perímetro del ABCD es 56 ¿cuál es su área?

81
Elementos Fundamentales de la Geometría

B
SOLUCIÓN:
Al descomponer el cuadrilátero en triángulos y dado que
A cada lado es tangente a la circunferencia, se tiene: h = r = 9
para cada triángulo. Luego
C αA = αΔAOB + αΔBOC + αΔCOD + αΔDOA
1 1
= ( AB + BC + CD + DA) h = (56) (9) = 252 u2
2 2

10. El perímetro del ΔABC es 36 y AB = 15. Calcular BT.

SOLUCIÓN:
Se tiene BT = BE y CE = CD, además AT = AD, ya que se
T
• trata de segmentos tangentes a la circunferencia trazadas
B
desde B, C y A respectivamente. Por tanto AT + AD = 36, de
donde se obtiene AT = 18.
•E Como AT = AB + BT, al sustituir el valor de AB se obtiene
BT = 18 – 15 = 3

A C D

D C
11. ABCD es un cuadrado de perímetro k. El semicírculo de la
figura tiene diámetro AB. Hallar el perímetro del ΔCDE,
__
F sabiendo que EC es tangente al semicírculo en el punto F.

E
SOLUCIÓN:
k
A B Sea a = el lado del cuadrado y sea EA = x.
4
Se tiene CB = CF = a, ya que ambos segmentos son
tangentes al semicírculo desde el mismo punto.

De manera similar EA = EF, luego DE = DA – EA = a – x y CE = CF + EF = a + x.

3
Luego el perímetro del ΔCDE resulta P = CD + DE + CE = a + (a – x) + (a + x) = 3a o sea P = k
4

12. Sea C una semicircunferencia de centro O y diámetro AB. Sea D el punto medio del arco AB. Sobre la
__
semirrecta OD se toma E tal que OE = BD. BE corta a la semicircunferencia en F y la proyección
___ 1
ortogonal de F sobre AB es P. Demuestre que BP = AB .
3

82
Elementos Fundamentales de la Geometría

D F SOLUCIÓN:
Sea r el radio de la semicircunferencia. Como OBD es un triángulo
rectángulo isósceles, BD = OE = 2 r.
Sea BP = x. Se tiene ΔOBE ∼ ΔPBF por el teorema de
OB OE r 2r
semejanza AA. Luego = o sea = ∴ PF = 2 r.
A O P B PB PF x PF

En el ΔOPF se tiene OP = OB – PB = r – x, OF = r, PF =
2 r. Aplicando el Teorema de Pitágoras
2
( )
r = 2 x + (r − x ) = 2 x + r − 2 r x + x
2 2 2 2 2 2 2
∴ 3 x − 2 r x = 0 ⇒ x (3 x − 2 r ) = 0 luego x = r
3
1 2⎛1 ⎞ 1
x = BP y r = AB, por tanto BP = ⎜ AB ⎟ = AB
2 3⎝2 ⎠ 3

13. En la siguiente figura ABCD es un cuadrado de lado a. El arco BD está D M C


trazado con centro en A y radio AB; M y N son puntos medios de BC y CD.
Calcular la longitud del radio de la circunferencia F.
F
N

D M C
SOLUCIÓN: A B
___ ___
Desde F trazamos perpendiculares a AB y AN . Sean P y S los pies de
F ___ ___
N estas perpendiculares. Sea Q el punto donde FP corta a AN .
Sea FS = r, el radio buscado.
S
Q AP PQ 1
Se tiene ΔAPQ ∼ ΔABN, luego = . Como BN = AB, se sigue
AB BN 2
A P B 1
que PQ = AP
2

1 1
También ΔAPQ ∼ ΔFSQ (Por A.A.), luego QS = FS = r
2 2
Sea a el lado del cuadrado y por tanto también el radio el arco mide a. Se tiene AF = a – r.
2 2 1 2
Como ΔAPF es triángulo rectángulo isósceles, AP = AF = ( a – r), PQ = AP =( a – r)
2 2 2 4
1 2
Además como AP = PF y PQ = AP, resulta que Q es punto medio de PF y por tanto FQ = PQ = ( a – r)
2 4
2
r2 ⎡ 2 ⎤
Aplicando el Teorema de Pitágoras al ΔFSQ se tiene: r 2 + =⎢ (a − r)⎥
4 ⎣⎢ 4 ⎦⎥
5 2 a2 − 2 a r+ r 2 − 2a ± 4 a 2 + 36 a 2 ⎛ 10 − 1 ⎞
r = ⇒ 9 r 2 + 2 a r− a2 = 0 ⇒ r= = ⎜ ⎟a
4 8 18 ⎜ 9 ⎟
⎝ ⎠
(Se descarta la otra raíz por resultar negativa).

83
Elementos Fundamentales de la Geometría

D C
15. ABCD es un cuadrado de lado 1 y contiene a una circunferencia
E inscrita. La diagonal BD corta a la circunferencia en el punto E que se
muestra en la figura. Calcule la longitud AE.

SOLUCIÓN:
Al observar la figura vemos que DE es la mitad de la diferencia entre la
diagonal del cuadrado y el diámetro de la circunferencia, ya que por la
simetría de la figura, la diagonal pasa por el centro.
A B
D Al trazar por E una perpendicular al lado AD, tenemos que AE es la hipotenusa del
F E triángulo rectángulo AFE, luego necesitamos encontrar la longitud de AF y FE.
Por otro lado ya que el ángulo ADB mide 45º, resulta que el triángulo EFD es rectángulo
isósceles, con DF = FE y DE = 2 DF.
Por ser ABCD un cuadrado de longitud 1, su diagonal mide 2 .
Como la circunferencia está inscrita en el cuadrado, su diámetro es igual al lado del
A cuadrado o sea 1.

2 −1 2 ⎛⎜ 2 − 1 ⎞⎟ 2 − 2
Se tiene entonces DE = , DF = FE = =
2 2 ⎜⎝ 2 ⎟⎠ 4
2− 2 2+ 2
AF = AD – DF = 1 – =
4 4
2 2
⎛2− 2 ⎞ ⎛ ⎞
Al aplicar el Teorema de Pitágoras al triángulo AFE, se tiene AE = ⎜ ⎟ +⎜2+ 2 ⎟ =
3
⎜ 4 ⎟ ⎜ 4 ⎟ 2
⎝ ⎠ ⎝ ⎠

EJERCICIOS

1. ¿Cuál es el radio de una circunferencia cuya longitud es 120 cm.?

2. ¿Cuál es el diámetro de una circunferencia cuya longitud es π?

3. La longitud del lado de un cuadrado es 8 cm. ¿Cuál es la longitud de la circunferencia inscrita? ¿Cuál
es la longitud de la circunferencia circunscrita?
.
4. Hallar las longitudes de las circunferencias inscrita y circunscrita en a) un triángulo equilátero de lado
igual a 12 y b) un hexágono regular de lado igual a 6.

5. El diámetro de una circunferencia mide 32.5 m.; se le prolonga 4.5m. Calcular la longitud del segmento
tangente trazado desde el punto obtenido.

6. El diámetro de una circunferencia mide 25.4 m. Calcular la longitud que se necesita prolongarlo para
que el segmento tangente trazado del punto obtenido mida 12 m.

7. En una circunferencia se trazan dos cuerdas paralelas a un mismo lado del centro, de longitudes 15 y
25. Si las cuerdas distan entre sí 8 ¿cuánto mide el radio de la circunferencia?

8. La longitud de la circunferencia de un círculo y el perímetro de un cuadrado son 20 cm. cada uno, ¿qué
figura tiene mayor área? .

84
Elementos Fundamentales de la Geometría

9. Las diagonales de un rombo miden 5 y 12 m. Respectivamente. Calcule el área del círculo inscrito en
el rombo.

10. Los segmentos AB, AC y BC son tangentes a la circunferencia en los puntos D, F y E


respectivamente. Si BD = 4, AE = 3 y EC = 5, a) ¿Cuánto mide el perímetro del Δ ABC? b) ¿Cuánto
mide el radio de la circunferencia? C

F
E

D B
A

11. Determine las medidas de los ángulos i) centrales ii) internos y iii) externos de un polígono regular
de a) 5 lados b) 9 lados c) 15 lados d) 24 lados.

12. Determine el número de lados que tiene un polígono regular si la medida de uno de sus ángulos
internos es a) 140° b) 144° c) 160°

13. Determine el número de lados que tiene un polígono regular si la medida de un ángulo externo es
a) 45° b) 72° c) 24°

14. El perímetro de un polígono regular es 48 y su apotema es 6 ¿Cuál es el área de la región poligonal


correspondiente?

15. La longitud de un lado de un hexágono regular inscrito en una circunferencia es 4. Determínese el


radio de la circunferencia y la apotema del hexágono.

16 A

En una circunferencia cuyo radio es 10 cm., se trazan dos


diámetros perpendiculares AC y BD , y la cuerda BC . O
B D
Si M es el punto medio de esta cuerda, calcular AM
M

C
A P B
17.
En un trapecio isósceles circunscrito a una
circunferencia, sus bases miden 6 y 8 cm.
M N Calcular
a) el radio de la circunferencia inscrita en el trapecio
b) la longitud de la cuerda MN que une los puntos de
contacto.
D E R C

18. Demuestre que si un polígono regular de 10 lados (decágono) se inscribe en una circunferencia con
r s
radio igual a r y s es la longitud de un lado del polígono, entonces =
s r− s

85
Elementos Fundamentales de la Geometría

19. Dado un octógono regular determine a) el lado del octógono y b) el área del octógono en función
del radio.

20.

En la figura A, B, C y D son centros de los cuatro círculos


tangentes en el punto E. Si estos cinco puntos son
A B C D E colineales y el área del círculo con centro en D es π u2,
encuentre el radio y el área del círculo con centro en A.

21. Las figuras representan una correa en torno a


dos poleas colocada en dos formas distintas. (a)
Para la figura (a) los radios de las poleas son 15
cm. y 3 cm. respectivamente, y la distancia O’
O
entre los centros OO’ = 24 cm.. Para la figura (b)
los radios de las poleas son 9 cm. Y 3 cm. Y la
distancia entre los centros OO’ = 24 cm., calcule
para ambos casos la longitud de la correa. (b)

O’ O

22. A B
Las circunferencias con centros en O y P
tienen radios de 2 cm. Si x = 1 cm.,
entonces ¿cuál es el perímetro del
rectángulo ABCD? O x P

D C
23. B
Encuentre el área de la corona circular si
a) OA = 6 y AB = 2
A b) OB = 6 y AB = 2
O c) OB = 6 y OA = 2

B
24
__
Si AC es un diámetro de la circunferencia y su longitud es
A C 60 , encuentre el área del Δ ABC.

86
Elementos Fundamentales de la Geometría

25. Calcule el área de la corona circular determinada por las circunferencias inscrita y circunscrita a un
hexágono regular de 6 cm. de lado.

26
A
Determine el área del segmento circular (área sombreada) si:
a) m ∠ AOB = 60º y OA = 6
O
b) m ∠ AOB = 90º y OA = 6
c) m ∠ AOB = 120º y OA = 6
B
A
27
Si el área del sector circular OAB
mide 31.5 π cm2 y la longitud del
O B
arco AB es 3π cm., encuentre el
radio del círculo.

28.

Siendo 1 m. el lado de un cuadrado, calcular el área de la cruz de


Malta, que se obtiene trazando desde los vértices arcos tangentes,
dos a dos, en el punto medio de las diagonales.

29.
r
En el interior de una circunferencia de radio R se
trazan cuatro circunferencias tangentes como se
muestran en la figura. R
Determine el valor del radio r de las circunferencias
pequeñas en función de R

30. Determine el área de la corona circular formada por las circunferencias inscrita y circunscrita a un
triángulo equilátero de 4 cm. de lado

31.
En la figura, las circunferencias son tangentes

entre sí y tangentes a la recta m .Si R = 10 y
r = 6 determine la distancia del punto de P

tangencia P a la recta m ( y) y la distancia r R y
entre los puntos de tangencia de las

circunferencias con la recta m ( x) . x

87
Elementos Fundamentales de la Geometría

32.

En la figura de la izquierda, el cuadrado está inscrito en el


círculo mayor y circunscrito en el círculo menor. Si el
cuadrado tiene lado l = 2’’, hallar la razón entre las áreas
del círculo mayor y el menor.

33.

La región de la figura es formada al trazar desde cada vértice


de un triángulo equilátero, arcos de radio igual al lado del
triángulo. Determine el área de la región si el lado del triángulo
mide 0.45 cm.

34.

En la figura, cada pétalo es formado al trazar arcos cuyo centro está


en la circunferencia y que pasan por el centro de la misma. Si el radio
de la circunferencia es 1’’, encuentre el área de los seis pétalos.

35.
Si el área del hexágono regular de la figura es 8,
determine el área del triángulo sombreado.

36. Un terreno tiene forma pentagonal como se muestra en la figura. Halle el área del terreno si
AF = 10 m., FG = 40 m. GH = 15 m. , HC = 20 m. , DG = 30 m. , EF = 20 m., HB = 35 m.
D

A F G H C

37
1 2 1 1 2 1 1

2
Determine el área cubierta por las
tres letras de la figura
2

88
Elementos Fundamentales de la Geometría

38.
A’ B’ E’ E A B
• •
O’ O
D’ C’ F’ F D C A

B
Datos: FF’ = 10 cm. , EF = E’F’ = 1 cm. C
A’, B’, E’, E, A y B son colineales. D’, C’, F’, F, D y C son
colineales F D
O y O’ son centros de las circunferencias.
A’B’C’D’ es un cuadrado. O, A, B, C y D son cinco vértices de un
hexágono regular.
E, B, C y F están sobre la circunferencia de la derecha.
A’, E’, F’ y D’ están sobre la circunferencia de la izquierda. E
Encuentre el área de la región sombreada.

39. En la figura, ADEF es un cuadrado inscrito en un círculo


de radio r = 1. Desde cada vértice del cuadrado se trazan
arcos de radio igual a un tercio de la longitud del lado del cuadrado. Calcule el área de la región
sombreada.

40 D C
ABCD es un cuadrado de lado 4. Cada arco es una semicircunferencia
con centro en el punto medio del respectivo lado. Encuentre el área de la
región sombreada.

A B

D C
41. En el trapecio isósceles ABCD, m∠DAB = 45º. El trapecio
contiene un círculo inscrito. Si CD = 1 cm. determine AB y BC.

A B
SOLUCIONES
1. 19.1 2. 1 3. 8π; 8 2 π 4. a) 4 3 π y 8 3 π b) 6 3 π y 12π 5. 12.9
6. 4.77 7. 12.7 8. El círculo 9. 16.73 10. a) 24 b) 5 11. a) θ = β = 72º,
β = 108º b) θ = β = 40º, β = 140º c) θ = β = 24º, β = 156º d) θ = β = 15º, β = 165º
12. a) 9 b) 10 c) 18 13. a) 8 b) 5 c) 15 14. 144 15. 4 y 3.464
16. 15.81 17. a) 2 3 b) 48/7 19. 2 − 2 r , 2 2 r 2 20. 64π
21. a) 22π + 24 3 b) 16π + 24 3 22. 22 23. a) 28π b) 20π c) 32π
24. 15 25. 9π 26. a) 3.2361 b) 12.274 c) 34.31 27. 21 28. 0.2576
29. r = R/3 30. 4π 31. x = 15.49; y = 7.5 32. 2 33. 0.1427 34. 1.087 35. 1
36. 3,112.5 37. 24.71 38. 14.285 39. 1.3018 40. 5.717

89
Elementos Fundamentales de la Geometría

3.7. CUERPOS SÓLIDOS

Los sólidos son regiones del espacio tridimensional cerradas, limitadas por superficies. Es de interés en
ellos, calcular su volumen, áreas de las superficies laterales y área de la superficie total o bien determinar
sus dimensiones a partir de información relacionada con los mismos. La Geometría Euclideana nos
ayuda a caracterizar las principales figuras sólidas que presentan algunas regularidades. Para el estudio
de sólidos irregulares se requieren conocimientos de Cálculo Diferencial e Integral y los conocimientos
que nos brinda la Geometría Euclideana.

3. 7.1. PRISMAS
Un prisma se caracteriza por tener dos bases paralelas, poligonales y congruentes; sus caras laterales
son paralelogramos.

E2
E2

h h

E1 E1
PRISMA RECTO PRISMA OBLICUO

◊ Los lados de las caras y las bases se llaman aristas laterales y de las bases respectivamente.
◊ Si las aristas laterales son perpendiculares a los planos que contienen a las bases se le llama Prisma
Recto, en caso contrario le llamamos Prisma Oblicuo.

◊ La distancia h entre los planos que contienen las bases se le llama altura del Prisma.

◊ Una sección transversal de un prisma es la intersección (no vacía) con un plano paralelo a los planos
de las bases.

Se puede probar que las secciones transversales de un prisma son congruentes y por tanto tienen igual
área.

◊ La unión de las caras laterales de un prisma se llama superficie lateral y la unión de las caras
laterales y las dos bases se llama superficie total.

90
Elementos Fundamentales de la Geometría

ALGUNOS TIPOS DE PRISMAS

◊ Un Paralelepípedo es un prisma cuyas bases son paralelogramos.

◊ Un paralelepípedo rectangular es un prisma cuyas bases y caras laterales son rectángulos. Una caja
común es un ejemplo de paralelepípedo rectangular.

◊ Un cubo es un paralelepípedo rectangular cuyas aristas son todas congruentes y por tanto sus bases y
caras laterales son cuadrados congruentes

Paralelepípedo
Paralelepípedo Cubo Rectangular

VOLÚMENES Y ÁREAS DE PRISMAS

En general el volumen de un Prisma está dado por:


V = Ab ⋅ h

donde Ab es el área de la base y h es la altura del prisma.

Las áreas laterales y totales dependen del tipo de prisma. No hay una fórmula general, sin embargo no
hay que olvidar que las bases son regiones poligonales y las caras laterales son regiones
paralelográmicas.

Para un prisma recto se tiene: AL = P⋅ a = P ⋅ h, AT = P⋅ a + 2 Ab

Donde AL: área lateral, AT: área total, P : perímetro de la base,


a = h: longitud de la arista lateral o altura

Para un paralelepípedo rectangular, si a, b y c representan el largo, ancho y alto, tenemos:


Volumen: V=abc
Area total: AT = 2⋅(a b + b c + a c) c
d
La diagonal está dada por d = a2 + b2 + c2
b
En particular, para un cubo de lado a, se tiene: a
Volumen: V = a3
Area total: AT = 6 a2
Diagonal: d= 3a

91
Elementos Fundamentales de la Geometría

EJEMPLO 1
Dado un cubo cuya diagonal mide 16 3 , determine su volumen y su área total.

SOLUCIÓN:
d 16 3
Tenemos que para un cubo d = 3 a, luego la longitud del lado es a = = = 16
3 3
Su volumen está dado por V = a3 ∴ V = 163 = 4,096 u3

Su área total está dada por AT = 6 a2 ∴ AT = 6 (16)2 = 1,536 u2.

EJEMPLO 2
Las dimensiones de un paralelepípedo rectangular son 12, 4 y 7. Determine la longitud de la diagonal, su
área total y su volumen.

SOLUCIÓN:
La diagonal está dada por d = a2 + b2 + c2 ∴ d = 12 2 + 4 2 + 72 = 209 ≈ 14.46 u
El área total es AT = 2⋅(a b + b c + a c) ∴ AT = 2⋅[(12)(4) + (4)(7) + (12)(7)] = 320 u2
Su volumen es V = a b c, luego V = (12) (4) (7) = 336 u3.

EJEMPLO 3
Las bases de un prisma recto son hexágonos regulares de 2 cm. de lado. Si la arista lateral mide 3cm.
¿cuál es el área lateral del prisma? ¿Cuál es su área total? ¿Cuál es su volumen?

SOLUCIÓN:

Dado que es un prisma recto se tiene AL = P⋅a, AT = AL + 2 AB, donde P es el perímetro de la base y
“a” es la arista lateral.

Como la base es un hexágono regular de 2 cm. de lado, P = 6 (2cm) = 12 cm.


Luego AL = (12 cm.)(3 cm.) = 36 cm2.

El área de la base, por ser un hexágono regular es


3 2 3
A B= 6 ( l ) = 6[ (2)2 ] = 6 3 = 10.39 cm2
4 4
luego el área total es A T = AL + 2 AB = 36 + 2 (10.39) = 56.78 cm2

El volumen está dado por V = AB⋅ h , h = a por ser un prisma recto, luego
V = (10.39 cm2) (3 cm.) = 31.17 cm3
EJEMPLO 4

¿Cuál es la altura de un prisma recto cuya área lateral es de 200 cm2, si la base es un octógono regular
de 4 cm. de lado?
SOLUCIÓN
A
Se tiene AL = P⋅h ∴ h = L
P
200 cm2
Como la base es un octógono regular P = 8⋅l = 8 ⋅ 4 = 32 cm., luego h = = 6.25 cm.
32 cm

92
Elementos Fundamentales de la Geometría

EJEMPLO 5

Calcular la superficie total y el volumen de un prisma cuya base es un pentágono regular inscrito en una
circunferencia de 4 cm. de radio y cuya altura es el doble del lado de la base.

SOLUCIÓN:
1
Tenemos que AT = 2 AB + AL , AB = P⋅ ap, AL = P⋅h y V = AB⋅ h. Por dato, R = 4 y h = 2⋅l
2
Como la base es un pentágono regular, la longitud de su lado está dada por
R 1
l5 = 10 − 2 5 y ap = 4 R 2 − l2 .
2 2
Sustituyendo el valor de R = 4, se obtiene l5 = 4.7022 y ap = 3.236.
Además h = 2 (4.7022) = 9.4044
1
Luego AB = (5) (4.7022) (3.236) = 38.04 cm2.
2
AL = P⋅h = (5) (4.7022) (9.4044) = 221.11 cm2
AT = 2 (38.04) + 221.11 = 297.19 cm2
V = (38.04) (9.4044) = 357.74 cm3

3. 7. 2. PIRÁMIDES

Una pirámide es un sólido que se caracteriza por tener una base poligonal y sus caras son regiones
triangulares que tienen un vértice en común V.
La altura de la pirámide es la distancia desde el vértice V al plano de la base.

Las pirámides se clasifican de acuerdo al tipo de base. Así tenemos: Pirámide Triangular, si su base es
un triángulo; pirámide cuadrada, si su base es un cuadrado; etc. También se clasifican en Regulares y no
Regulares.

PIRÁMIDE REGULAR

Es una pirámide cuya base es un polígono regular y el pie de la perpendicular trazada desde el vértice al
plano de la base es el centro de la base.
Las caras laterales son triángulos isósceles congruentes. A las alturas de dichos triángulos se les llama
apotema de la pirámide.

SECCIONES TRANSVERSALES: Similar que en los prismas, una sección transversal de una pirámide
es la intersección no vacía con un plano paralelo al plano que contiene a la base.
Toda sección transversal de una pirámide es semejante a la base.

93
Elementos Fundamentales de la Geometría

Si h es la altura de la pirámide y k es la
distancia del vértice a la sección transversal,
k y si A es el área de la base y A’ el área de la
sección transversal, entonces:
h

2
A' ⎛ k ⎞
=⎜ ⎟
A ⎝h⎠

Además, si la base de la pirámide es un polígono regular y l y l’ son las longitudes de los lados de la
l' k
base y la sección transversal respectivamente, entonces = .
l h

PRINCIPIO DE CAVALIERI

Dados dos cuerpos sólidos y un plano. Supongamos que todo plano paralelo al plano dado que interseca
a uno de los dos cuerpos, interseca también al otro y las secciones transversales tienen igual área,
entonces los cuerpos tienen el mismo volumen.

Basados en este principio se obtienen las fórmulas para el volumen de diversos cuerpos sólidos:
pirámides, conos, esfera, etc. Los resultados son los siguientes:

VOLUMEN DE UNA PIRÁMIDE


1
V= ⋅ Ab ⋅ h
3
Ab: área de la base

h: altura

ÁREAS: Únicamente hay fórmulas para las pirámides regulares.

1 1
AL = ⋅P⋅a AT = ⋅P⋅a + Ab
2 2
donde P: perímetro de la base
a: apotema de las caras
laterales

PIRÁMIDE TRUNCADA O TRONCO DE PIRÁMIDE

Es el sólido que resulta cuando una pirámide es cortada por un plano


paralelo a la base. Si AB es el área de la base de la pirámide original,
Ab el área de la sección transversal que forma la otra base, h la a h
altura del tronco de pirámide (distancia entre los planos que
contienen las bases) y a es la altura de los trapecios que forman
las caras laterales, se tiene:

94
Elementos Fundamentales de la Geometría

1
Volumen: V= h [AB + Ab + AB Ab ]
3
1
Area Lateral: AL = (P + P’) ⋅ a
2
1
Area Total: AT = (P + P’) ⋅ a + AB + Ab
2
Donde P y P’ son los perímetros de las bases.

Si consideramos la pirámide “original” y la pirámide que “quitamos” para formar un tronco de cono, se
obtiene la siguiente relación:

Si H es la altura de la pirámide original, k es


la distancia del vértice a la sección
k transversal y si h es la altura del tronco de
cono, entonces si V es el volumen de la
H pirámide original y V’ el volumen de la
pirámide que se quita:
h 3
V' ⎛ k ⎞
=⎜ ⎟
V ⎝ H⎠
El volumen del cono truncado es la
diferencia V – V’

EJEMPLO 6

Determine el volumen de una pirámide de base triangular regular si su altura es 20 cm. y la arista de la
base mide 15 cm.

1
Tenemos que V = ⋅ Ab ⋅ h
3
h
Por ser la base un triángulo equilátero se tiene
3 2 3
Ab = l = (15) 2 = 97.43 cm2
4 4
1
a Luego V = (97.43) (20) = 649.52 cm3
3

95
Elementos Fundamentales de la Geometría

EJEMPLO 7

En una pirámide cuadrada, en la que el lado de la base mide 8 cm. y la altura mide 20 cm., se traza una
sección paralela a la base a 14 cm. de ésta. Determine el área de la sección transversal y el volumen de
la pirámide truncada resultante.

Sea A’ el área de la sección transversal y A el área de la


base de la pirámide. Tenemos A = 82 = 64 cm2
6
20 Dado que la sección transversal se encuentra a 14 cm. de
la base, su distancia al vértice será
14
k = 20 – 14 = 6 cm.
2 2
A' ⎛ k ⎞ ⎛ 6 ⎞
Se tiene = ⎜ ⎟ ∴ A’ = ⎜ ⎟ ⋅ 64 = 5.76 cm
2
8 A ⎝h⎠ ⎝ 20 ⎠
El volumen del tronco de pirámide está dado por
1 1
V = h’ [AB + Ab + A B A b ] = (14) [64 + 5.76 + (64) (5.76) ] = 415.15 cm3
3 3
También puede hallarse como la diferencia entre el volumen de la pirámide original menos el volumen de
la pirámide pequeña que se forma.
1 1
V = (64) (20) – (5.76) (6) = 415.15 cm3.
3 3

EJEMPLO 8
Se da un tetraedro regular (sólido con cuatro caras triangulares equiláteras). Si su área total es 64 cm2
encuentre la longitud de su arista, la altura y el volumen.

A
Por ser un sólido de cuatro caras triangulares equiláteras
tenemos:
AT = 4 AΔ = 64 ∴ AΔ = 16 cm2
C
3 2 16 ⋅ 4 8
D AΔ = l = 16 ∴ l2 = ⇒l=
4 4
O 3 3
En este caso l es la arista.
B

A
Consideremos la cara formada por el triángulo ABC. Sea E
__
el punto medio de BC . Por ser un triángulo equilátero AE es
una altura, luego
3 3 8
AE = l = ⋅ = 443
B C 2 2 43
E

96
Elementos Fundamentales de la Geometría

A
Consideramos el triángulo AOE. Al aplicar el teorema de
Pitágoras, resulta
h
4 4 3 2 8 4 12
h= AE2 − OE 2 = (4 4 3 ) 2 − ( ) =
3 3
O E

D Al considerar el triángulo BDC, vemos que el pie de


la perpendicular de la altura del tetraedro AO
coincide con el baricentro del ΔBDC, luego

O 1 1 44 3
OE = DE = AE =
3 3 3

B E C

1 1 ⎛ 8 4 12 ⎞
Por ser una pirámide V = AB h = ⋅ (16) ⎜ ⎟ = 26.47 cm3
3 3 ⎜ 3 ⎟
⎝ ⎠
Nota: El volumen de un tetraedro regular también puede hallarse directamente mediante la fórmula:
V = 0.1178 a3, donde a es la arista, que se obtiene al realizar estos cálculos para un tetraedro regular
cualquiera de lado a.
EJEMPLO 9
Determine el área lateral de una pirámide regular de base hexagonal, si su altura es 12 cm. y la arista de
su base mide 8 cm.
SOLUCIÓN

V Al considerar el triángulo formado por el vértice de la


pirámide, el centro y un vértice de la base, tenemos:
V
12 cm.
OV = h = 12 cm. (Dato)

O OA = 8 cm. (Por ser la base de la


A B pirámide un hexágono regular)
A O
8 cm.

La arista de la pirámide AV, la obtenemos al aplicar el teorema de Pitágoras al triángulo AOV:


AV = 12 2 + 8 2 = 208

Las caras laterales son triángulos isósceles, luego la altura de las mismas, o sea la apotema de la
pirámide, la encontramos aplicando también el Teorema de Pitágoras:
V

208
h h= 208 − 4 2 = 192
A B
4 cm.
Dado que esta pirámide tiene seis caras, el área lateral buscada es:
⎡1 ⎤
AL = 6 ⎢ ⋅ 8 ⋅ 192 ⎥ ≈ 332.55 cm 3
⎣ 2 ⎦

97
Elementos Fundamentales de la Geometría

3. 8. CILINDROS Y CONOS CIRCULARES


DEFINICIÓN: CILINDRO CIRCULAR
Sean E1 y E2 dos planos paralelos, R una región circular en E1 y L una recta que interseque a E1 y E2.
Por cada punto P de R sea PP' un segmento paralelo a L y que una el punto P con un punto P’ de E2. El
sólido formado por la unión de todos los segmentos PP' se llama cilindro circular. Si L ⊥ E1, entonces
el cilindro se llama cilindro circular recto.

Los conceptos de altura y sección transversal son similares a los correspondientes para prismas.

E1
E1

h h

r r
E2

DEFINICIÓN: CONO CIRCULAR


Sean R una región circular en un plano E y V unEpunto
2 que no está en E. El cono circular con base en E
y vértice V es el sólido formado por la unión de todos los segmentos VQ para los cuales Q pertenece a
R. La altura h del cono es la distancia de V a E. Si el pie de la altura coincide con el centro de la base, se
le llama cono circular recto.

En un cono circular recto el segmento que une el vértice con un punto de la circunferencia de la base se
llama generatriz. Generalmente se denota por g. Por el teorema de Pitágoras se tiene g2 = h2 + r2,
siendo h la altura y r el radio del cono.

h h g

r
r

VOLÚMENES Y ÁREAS DE CONOS

◊ Para un cilindro circular de radio r y altura h, tenemos


.
Volumen: V = Ab⋅ h = π r2 h
Su área lateral será un rectángulo en el caso del cilindro circular recto o bien un paralelogramo en el caso
de un cilindro circular oblicuo, de base la longitud de la circunferencia y altura h. En ambos casos se
tiene:

Área Lateral: A L = 2 π r h. Área total: A T = AL + 2Ab = 2 π r h + 2 π r2

◊ Para los conos en general, su volumen está dado por: 1 1


V= Ab ⋅ h = π r2 h.
3 3

98
Elementos Fundamentales de la Geometría

◊ Para un cono circular recto, su área lateral está formada por un sector circular, cuyo radio es la
generatriz del cono y la longitud del arco corresponde a la circunferencia de su base. Luego

g
2 2
AL = π r g , donde g = h +r AT = AL + AB

2πr

CONO TRUNCADO: Es el sólido que resulta cuando un cono es cortado por un plano paralelo a la base.

1
V= π h ( R2 + r2 + Rr)
r 3
g 2πr g AL = π g (R + r)
h AT = AL + π (R2 + r2)

R g= h 2 + (R − r) 2
2πR

k k
H V’
H
h V 3
V' ⎛ k ⎞
=⎜ ⎟
V ⎝ H⎠
El volumen del cono truncado también puede hallarse como la diferencia de volumen entre el cono
original y el que se quita para formar el tronco de cono VTC = V – V’

3. 9. ESFERA

3.9.1. CONCEPTOS GENERALES

SUPERFICIE ESFÉRICA: Sea P un punto en el espacio tridimensional y sea r un número positivo. La


superficie esférica con centro P y radio r es el conjunto de todos los puntos del espacio que están a la
distancia r del punto P.
También se le llama radio a todo segmento que une P con cualquier punto de la
superficie esférica.

ESFERA: Es el cuerpo sólido formado por una superficie esférica y los puntos de
su interior. Es decir el conjunto de todos los puntos del espacio que están a una
distancia menor o igual a r del punto P.

99
Elementos Fundamentales de la Geometría

PLANOS TANGENTES Y PLANOS SECANTES A UNA ESFERA

P π Un plano π es tangente a la esfera E, si π ∩ E = {P}.


El punto P es el punto de tangencia.
___
C Tenemos que PC = R, PC ⊥ π , donde C es el centro de la
E Esfera.

Si d (π, C) > R, entonces π ∩ E = φ, pero si d (π, C) < R, el plano π es un plano secante a la esfera y la
intersección es un círculo. Si C es el centro de la esfera y O es el centro del círculo de intersección, se
___
tiene OC ⊥ π.
π
O

C
E

ALGUNAS PROPIEDADES:
◊ Un plano perpendicular a un radio en su extremo es tangente a la superficie esférica y viceversa, todo plano
tangente es perpendicular al radio trazado por el punto de tangencia.

◊ Si un plano interseca al interior de una superficie esférica (o esfera) entonces la intersección es una circunferencia
(o círculo). En este caso decimos que el plano es secante.

◊ El centro de la circunferencia de intersección es el pie del segmento perpendicular desde el centro de la esfera al
plano.

◊ Si el plano pasa por el centro, se obtiene la circunferencia máxima de la circunferencia esférica, la cual tiene el
mismo radio y el mismo centro de la superficie esférica.

ÁREA Y VOLUMEN DE UNA ESFERA.


Se puede probar a partir del principio de Cavalieri que para una esfera de radio R, su volumen V y área
de la superficie esférica S están dada por:

4
V= πR3 y S = 4 π R2
3

Veamos como se obtiene la fórmula del volumen:

t Consideremos una sección transversal a la distancia s del


centro, de radio t. Su área es As = π t2 (1)
s r Por el Teorema de Pitágoras se tiene t2 = r2 – s2 (2)
Luego sustituyendo (2) en (1) As = π (r2 – s2) = π r2 – π s2 (3)

100
Elementos Fundamentales de la Geometría

Pero (3) corresponde al área de una corona circular con radio


s exterior r y radio interior s
Si tomamos un cilindro circular recto de radio r y altura 2r, al
r extraer dos conos como se muestra en la figura, tenemos que las
generatrices de dichos conos tienen una inclinación de 45º, luego
el radio interno de la sección transversal que se forma a una
distancia s del centro, es igual a s.

Por tanto el área de la sección transversal a una distancia s del


r
centro sólido resultante será A = π r2 – π s2 lo que es igual al
s área de la sección transversal correspondiente de la esfera.

r Luego por el principio de Cavalieri, el Volumen de la esfera es


igual al volumen del sólido resultante:
1 2 4
V = π r2 (2r) – 2[ π r2 (r)] = 2π r3 – π r3 = π r3
3 3 3

3.9.2. REGIONES ESFERICAS


◊ Cuando un plano secante corta a una esfera se forman dos sólidos llamados segmentos esféricos de una base.
Si consideremos la superficie esférica en lugar del sólido, cada parte recibe el nombre se casquete esférico o zona
de una base.
◊ Si la esfera es cortada por dos planos secantes paralelos, la parte de la esfera limitada por dichos planos recibe el
nombre de segmentos esféricos de dos bases. De manera similar si consideremos la superficie esférica en lugar
del sólido, la superficie limitada por los planos recibe el nombre se zona de dos bases.
◊ Si consideramos dos semicírculos máximos con el diámetro común, se forma el sólido llamado cuña esférica. La
superficie esférica correspondiente se llama huso esférico.
◊ Si consideramos tres puntos sobre la superficie esférica, unidos por tres arcos de circunferencia máxima cuyas
longitudes sean menores que una semicircunferencia, se forma un triángulo esférico.
◊ Una cápsula esférica es un cuerpo sólido limitado por dos superficies esféricas concéntricas.
◊ La distancia esférica entre dos puntos, se define como la longitud del menor arco de círculo máximo que une
dichos puntos.
◊ Un ángulo esférico es el ángulo diedro determinado por dos semicírculos máximos.

p
h
a b
θ
O h
R a

ZONA Y SEGMENTO ZONA Y SEGMENTO DE HUSO ESFERICO


DE UNA BASE DOS BASES Y CUÑA ESFERICA
S = 2πRh
S = 2πRh = π p2 πR 2 θº
1 S=
1 V = πh(3a2 + 3b2 + h2) 90º
V = πh2(3R – h)
3 6
πR 3 θº
1 V=
V= πh(3a2 + h2) 270º
6

101
Elementos Fundamentales de la Geometría

EJEMPLO 10

Determine el diámetro de una esfera si su volumen y superficie tienen igual valor.


4
Solución: Tenemos d = 2 r, V = π r3 y S = 4π r2. Igualando las expresiones para V y S, resulta:
3
4
π r = 4π r2 → r3 = 3 r2 ∴ r = 3. Luego d = 2 r = 2(3) = 6
3
3
EJEMPLO 11

Se tiene un cilindro circular recto de 30 cm. de altura y 12 cm. de diámetro. Se perfora un agujero a lo
largo de su eje con diámetro 9 cm. Determine el volumen del sólido resultante.

El volumen buscado es la diferencia del volumen del cilindro original menos el volumen
del cilindro que representa el agujero.
Tenemos:
Vcilindro= π r2 h; h1 = h2 d1 = 12 ∴ r1 = 6, d2 = 9 ∴ r2 = 4.5

Luego V = π(6)2 (30) – π(4.5)2 (30) = 1484.4 cm3

EJEMPLO 12

Dado un cono recto de radio r = 3 m y generatriz g = 5 m. Determine su volumen y su área lateral.

g 1
h Tenemos V = π r2 h y h = g2 − r 2 .
3
1
Luego h = 52 − 32 = 4 y V = ⋅π⋅(3)2 (4) = 12π u3
3
r AL = π r g = π (3) (5) = 15π u2

EJEMPLO 13

Un cilindro recto está inscrito en una esfera de radio 4 cm. La altura del cilindro es igual al diámetro de la
base. Hallar el volumen del cilindro.

Al realizar un corte a través de un diámetro del


A B A B cilindro, encontramos que se forma un triángulo
rectángulo BCD con el diámetro de la esfera
como hipotenusa. Como d = CD, h = BD y d = h,
el triángulo BCD es también isósceles,
2
d=h= BC = 4 2 y r = 2 2 , luego
D C D 2
C
V = π r2 h = π (2 2 )2(4 2 ) = 142.17 cm3

EJEMPLO 14

En un recipiente esférico de 10 cm. de radio se vierte un líquido que alcanza una altura de 4.311 cm.
Encuentre el volumen del líquido.

102
Elementos Fundamentales de la Geometría

SOLUCIÓN
Dado que los líquidos por sus propiedades toman la forma del recipiente que
los contiene, se forma un segmento esférico de una base, cuyo volumen está
R 1 2
dado por V = π h ( 3R − h) .Luego al introducir los datos dados, se obtiene
3
h 1
V = π (4.311)2 (3⋅10 – 4.311) ≈ 500 cm3
3
EJERCICIOS PROPUESTOS

1. Las dimensiones de un paralelepípedo rectangular son 12, 4 y 7. Determine su volumen y su área total.

2. La diagonal de un paralelepípedo rectangular es 14.5 m; las dimensiones del paralelepípedo son proporcionales a
3, 6 y 7. Calcular el volumen.

3. El volumen de una caja es 6.48 decímetros cúbicos. Su longitud es de 30 cm.; su altura es igual a los 2/3 de su
anchura. Calcular la superficie total.

4. La densidad del plomo es 11.35 Kg. / dm3. Calcular la arista de un cubo de plomo cuyo peso es 1 kg.

5. Dado un cubo cuya diagonal mide 16 3 , determine su volumen y su área total.

6. Calcular el volumen, área lateral y el área total de un tronco de cono que se forma cuando se corta un cono recto
de 12 cm. de radio y 16 cm. de altura, por medio de un plano paralelo a la base del cono y que lo corta a una altura
de 9 cm. de la base.

7. Hallar el volumen de un prisma recto de altura 10 cm. si sus bases son triángulos equiláteros con área 9 3 cm2.
Determine la arista de la base y el área lateral.

8. Hallar el volumen de un prisma recto de altura 4 cm. si sus bases son hexágonos regulares y el área lateral es 144
cm2.

9. La altura de un prisma rectangular es 1/3 de su longitud y el ancho es la mitad de su longitud. Si la diagonal del
prisma mide 60 cm. Hallar su volumen.

10. Hallar el volumen de un prisma recto cuyas bases son regiones trapezoidales, si las aristas paralelas
de las bases miden 4 y 9 y las no paralelas 5 y 6. La altura del prisma es 12.

11. Un prisma recto hexagonal regular tiene una altura de 1 decímetro y una superficie total de 3 dm2.
Este prisma es de estaño, cuya densidad es 7.29 Kg./ dm3. Calcular el volumen y el peso del sólido.

11
12. Las bases del prisma de la figura son triángulos
equiláteros y sus caras son regiones rectangulares. Si
la longitud de una arista de la base es 6 y la altura del
prisma es 10, calcule el volumen del prisma y la
superficie total.

13. La base de un prisma recto es un hexágono regular. Una arista de la base mide 20 cm. de largo y una
arista lateral del prisma mide 50 cm. ¿Cuál es el área lateral del prisma? ¿Cuál es su área total? ¿Cuál
es su volumen?

14. La densidad del hierro es 7.788 Kg. / dm3. Un cilindro macizo de hierro pesa 120 kg. y su longitud es
de 3 m. Calcular su diámetro.

103
Elementos Fundamentales de la Geometría

15. Un cono tiene 87 mm. de diámetro y 1 dm. de altura. Calcular a) su volumen y b) su superficie
total.

16. La altura de un cono es de 5 cm. Un plano paralelo a la base lo interseca a 2 cm. de la misma
formando un cono pequeño en la parte superior. Si el volumen del cono pequeño es 24 cm3, halle el
volumen del cono original.

17. Un cono circular recto tiene 13 cm. de generatriz y el radio de la base es 5 cm. Se corta por un plano
paralelo a la base que corta a la generatriz en un punto distante 5.2 cm. del vértice. Hallar el volumen y el
área lateral del cono pequeño formado.

18. Determine el volumen y la superficie de una esfera de radio 4 cm.

19. Determine el diámetro de una esfera si su volumen y superficie tienen igual valor.

20. Calcular el volumen de una esfera a) inscrita en un cubo de 1 m. de lado, b) circunscrita en el mismo
cubo.

21. Se tiene un cilindro circular recto de 30 cm. de altura y 12 cm. de diámetro. Se perfora un agujero a lo
largo de su eje con diámetro 9 cm. Determine el volumen del sólido resultante.

22. Si la superficie de una esfera y un cubo tienen igual área y cada una mide
1 m2 ¿cuál de los dos cuerpos tiene mayor volumen?

23. Determine el volumen de una gota esférica, cuyo radio mide 0.2 mm
P
24. Si la diagonal de un cubo es 10 3 ¿cuál es su volumen? ¿Cuál es el área total?

25. La figura representa un cubo. M y N representan los puntos medios de dos


aristas adyacentes, Si la longitud de cada arista es x, ¿Cuál es el volumen de N
la pirámide PMNQ?
M Q
26. Calcular el volumen de la gran pirámide de Keops en Egipto, cuya base es un cuadrado de 230 m. de
largo y cuyas caras son triángulos equiláteros.

27. El techo de la torre de una iglesia tiene que ser renovado. El techo tiene la forma de una pirámide
regular con base cuadrada de 49m2 y una altura de 14.5m. Calcular los costos para renovar el techo, si
la reposición de 1 m2 cuesta $25. (materiales y mano de obra)

28. El agua de lluvia es recogida en un pluviómetro que tiene forma de pirámide cuadrangular regular
invertida. El agua recogida un día de lluvia alcanzó una altura de 9 cm., formando una pequeña pirámide
de 15 cm. de arista lateral. ¿Cuál es la altura alcanzada por el agua al verterla en un depósito cúbico de
50cm de arista?

29. Calcular el volumen de una pirámide truncada que se forma cuando se corta una pirámide
cuadrangular regular de altura 8 m. y arista de la base 4 m. por medio de un plano paralelo a la base de
la pirámide y que la corta a 2 m. del vértice.

30. Una pirámide cuadrada se inscribe en un cono circular de manera que tengan el mismo vértice y la
base de la pirámide queda inscrita en la base del cono. La altura común es 18 cm. y la longitud de un
lado del cuadrado es 15 cm. Determine el volumen del cono y la pirámide.

31. Las dos bases de una pirámide truncada miden 8 m2 y 2 m2. Calcular el lado de la base de un prisma
equivalente que tenga la misma altura y una base cuadrada.

104
Elementos Fundamentales de la Geometría

32. Se desea excavar un pozo cilíndrico de diámetro 2 m. a) ¿cuántos metros cúbicos de tierra se
deberán sacar, si el pozo debe tener una profundidad de 12 m.? b) ¿cuántas veces será necesario
cargar un camión, si en el mismo caben 3 m3 de tierra?

33. Una crayola debe tener 8 cm. de largo, 1 cm. de diámetro. Su


forma debe ser la de un cilindro circular con una pequeña punta
cónica. Encuentre la longitud de la parte cilíndrica y la altura del cono,
si la crayola contiene 5 cm3 de cera.
34. Un depósito tiene la forma de un cono truncado cuya altura es 1.5 m. El radio en el fondo es 0.6 m. y
en la parte superior es 1.5 m. Calcular la altura a la cual se eleva el agua, si se vierten 2 m3 de agua.

35.
C

En la figura, la esfera queda inscrita en un cono circular recto. AB es


un diámetro de la base y C es el vértice del cono. El triángulo ABC
que se forma es equilátero. Determine el volumen del cono, si el radio
de la esfera es 10 cm.

A B
36.
La altura de un cono circular recto es 16 y el radio
de la base es 7. Se le taladro un agujero de
diámetro 3 a lo largo de su eje, resultando un
cuerpo sólido como el que se muestra en la figura.
¿Cuál es el volumen de ese cuerpo sólido?

37. Se desea construir un tubo en forma de cilindro hueco de 2 cm. de espesor. Si el radio interior ( de la
parte hueca) es 0.5 cm. y el tubo tiene 1.2 m de longitud, calcular la cantidad de material necesario.

38. Se quiere construir un barril cilíndrico cerrado, con una altura de 4 pies y un área de superficie total de
10π pie2. Determine el diámetro del barril.

39. Una bobina está hecha con tres cilindros con bases concéntricas como se
muestra en la figura. Los cilindros de los extremos tienen un radio de 10 cm. y
altura 2 cm. cada uno. El cilindro interior tiene por radio la mitad del radio de los anteriores y altura 20
cm. Calcular el volumen de la bobina.

40. Se corta un cubo pequeño en una de las esquinas de un cubo


4 más grande, obteniéndose el sólido que se muestra en la figura,
cuyo volumen es 208 m3 . ¿Cuánto mide la arista del cubo
x
grande?

105
Elementos Fundamentales de la Geometría

SOLUCIONES
1. V = 336 AT = 320 2. 421.33 m3 3. 22.32 dm2 4. 0.444 dm 5. V = 4096 AT = 1536
6. 2,210.7cm , 609.66 cm y 1,148.64 cm . 7. V = 90 3 cm , l = 6 cm. , AL = 180 cm2
3 2 2 3

8. 374. 12 cm3 9. 22,670.55 cm3. 10. 374.4 u3. 11. V = 0.37 dm3. P = 2.697 Kg.
12. V = 155.88, A T = 211.18 13. AL = 6000 cm2 AT = 8078.46 cm2 V = 51,961.52 cm3
14. 0.8086 dm 15. V = 198.16 cm3 AL = 149.03 cm2 AT = 208.47 cm2
16. V = 111.11 cm3 17. V = 20.1 cm3 18. V = 268.08 cm3 S = 201.06 cm2 19. d = 6
3 3
20. a) 0.5235 m b) 2.72 m 21. V = 1484.4 cm3 22. La esfera 23. 0.03351 mm3
24. V = 1000, AT = 600 25. x /24. 26. 2, 867,789.4 m3
3
27. $5,220.75 28. 0.3456 cm.
29. 42 m3 30. 2,120.6 cm3 y 1,350 cm3. 31. 2.16 m. 32. a) 37.7 m3 b) 13 veces
33. 5.55 cm. y 2.45 cm. 34. 0.847 35. 3000π cm3
36. 724.06 u3 37. 720 π cm3 38. 2 pies. 39. 900 π cm3 40. 6

106
Geometría Analítica

Y Parábola:
-3
f(x) = ax² + bx + c
-2 1

-1
Vértice: (h,k)
-b
___
h= k = f(h)
2a

1
-1
2
X
3
-2

Pascual Rigoberto López Pérez, más conocido por Rigoberto López Pérez (1929 – 1956),
poeta nicaragüense e importante símbolo de la revolución, marcó el inicio del fin de la
tiranía, pasó a la inmortalidad el 21 de Septiembre de 1956. En septiembre de 1981,
Rigoberto López Pérez entró a la lista de héroes nacionales por la “gesta heroica llevada
a cabo al ajusticiar al tirano”. El Decreto fue aprobado el día en que se cumplieron 25 años
del asesinato de López Pérez.

Fuente: 19 digital.
20 de Septiembre 2013.
6.1. CONCEPTOS GENERALES
La Geometría Analítica, se apoya en el Álgebra para resolver con relativa facilidad una gran cantidad
de problemas geométricos.
Se considera al filósofo y matemático francés, René Descartes (1596-1650) como el precursor de la
Geometría Analítica. Uno de los recursos poderosos que usa la Geometría Analítica, son los sistemas
de coordenadas; los más usuales son los sistemas de coordenadas rectangulares, llamados Sistemas
Cartesianos, en honor de su creador, René Descartes.
Los puntos se describen mediante su posición en un sistema de coordenadas. Las figuras
geométricas se describen mediante ecuaciones que involucran las variables representadas en cada
eje. Las propiedades de cada figura se deducen a partir del “análisis” de la ecuación general que
describa al tipo de figura y las posiciones relativas entre puntos, rectas, planos, curvas, superficies,
etc. se determinan a partir de relaciones o fórmulas deducidas de propiedades generales o del tipo de
las ecuaciones involucradas.
La geometría Analítica contribuyó significativamente al desarrollo de la Matemática y es una
herramienta básica para el abordaje de muchas aplicaciones y el estudio de otros temas avanzados
de la Matemática.

6.1.1. RECTA NUMÉRICA REAL R1

Podemos establecer una correspondencia biunívoca entre el conjunto de los números reales R y los
puntos de una recta, de manera que a cada número le corresponda un único punto de la recta y
viceversa. Esta recta se conoce como la recta numérica real.

Usualmente la recta se dispone de manera horizontal. Al punto que le corresponde el número cero le
llamamos el origen; los números positivos se colocan a la derecha del origen y los negativos a la
izquierda. Tal disposición se le conoce como posición estándar de la recta numérica. A menudo es
necesario disponer esta recta en forma vertical o inclinada o bien con los números positivos hacia la
izquierda.
• • • • • • • • • • • • • •
–6 –5 –4 –3 –2 –1 0 1 2 3 4 5 6 7

Al número asociado a un punto P se le llama la coordenada de P.

6.1.2. SISTEMA COORDENADO RECTANGULAR O SISTEMA CARTESIANO R2

Es posible asociar de diferentes maneras, pares de números reales con puntos de un plano,
obteniendo así un sistema de coordenadas bidimensional.

Uno de los más importantes es el sistema de coordenadas rectangulares o cartesianas. Este sistema
lo formamos con dos rectas numéricas perpendiculares entre si, coincidiendo en el origen.
Generalmente una se dispone horizontalmente con la dirección positiva hacia la derecha y le
llamamos eje X o eje de las abscisas y la otra, en forma vertical con la dirección positiva hacia arriba
y le llamamos eje Y o eje de las ordenadas. Al plano formado le llamamos Plano Coordenado o
plano XY
A cada punto P del plano XY se le asigna un único par ordenado (a, b) y decimos que P tiene las
coordenadas (a, b). Recíprocamente a cada par (a, b) le corresponde un único punto P del plano
XY. La primera componente del par ordenado se le llama abscisa, mientras a la segunda componente
se le llama la ordenada.

1
GEOMETRÍA ANALÍTICA

6.1.3. DISTANCIA ENTRE DOS PUNTOS

Sobre una recta coordenada, o recta numérica, la distancia entre dos puntos cualesquiera P1 (x1) y
P2 (x2) es el valor absoluto de la diferencia de sus coordenadas.

d (P1, P2) = | x1 – x2 |

Si los puntos están sobre una recta paralela al eje X, la distancia entre dichos puntos es el valor
absoluto de la diferencia de sus abscisas, y de manera similar, si están sobre una recta paralela al eje
Y, su distancia es el valor absoluto de la diferencia de sus ordenadas.
Y Y P2 (x, y2)

P1 (x1, y) P2 (x2, y)
P1 (x, y1)
X d = |y2 – y1| X
d = |x2 – x1|

EJEMPLO 1

La distancia entre A (0, – 2) y B (0, 5) es d (A, B) = AB = | – 2 – 5 | = | – 7| = 7, por ser puntos que


están sobre el eje Y.

EJEMPLO 2

La distancia entre C (4, 3) y D (8, 3), dado que tienen la misma ordenada, deducimos que están
sobre una recta paralela al eje X, luego d (C, D) = CD = | 8 – 4 | = 4

En el plano, si P1 (x1, y1) y P2 (x2, y2) son dos puntos cualesquiera, sobre una recta no paralela a
los ejes coordenados, construimos un triángulo rectángulo con el segmento que une los puntos y los
segmentos paralelos a los ejes coordenados, como se muestra en la figura; al aplicar el resultado
anterior y el teorema de Pitágoras obtenemos una fórmula que nos permite calcular su distancia
P1(x1,y1)
Y P2(x2,y2) Y

| y2 – y1 | | y2 – y1 |

| x2 –x1| | x2 –x1| P2(x2,y2)


P1(x1,y1)

X X

P1P2 = (x 2 − x 1 ) 2 + (y 2 − y 1 ) 2

EJEMPLO 3

Si P1 (2, –1) y P2 (5, 3) entonces P1P2 = (5 − 2)2 + [3 − (−1)]2 = 32 + 4 2 = 5

EJEMPLO 4

Verifique que el triángulo con vértices A (1, 1), B (4, 4) y C (9, – 1) es un triángulo rectángulo y
encuentre su área.

2
GEOMETRÍA ANALÍTICA

SOLUCIÓN

Recordemos que las longitudes de los lados de un triángulo


rectángulo satisfacen el teorema de Pitágoras, de manera que una B (4, 4)
forma de verificar que el triángulo dado es rectángulo, es calcular
A (1, 1)
las longitudes de los lados y comprobar que se satisface el
teorema señalado. Aplicando la fórmula anterior, tenemos
C (9, – 1)

AB = (1 − 4) 2 + (1 − 4) 2 = 18 = 3 2 , AC = (1 − 9) 2 + (1 + 1) 2 = 68

BC = (9 − 4) 2 + (−1 − 4) 2 = 50 = 5 2
__ __
Vemos que el lado AC tiene la mayor longitud, luego es la hipotenusa del triángulo, y los lados AB y
__
BC son los catetos:.
Verificamos que AC2 = AB2 + BC2 ( 68 )2 = (3 2 )2 + (5 2 )2 ⇒ 68 = 18 + 50 se cumple.
Luego concluimos que el triángulo es rectángulo.

Por ser un triángulo rectángulo su área es la mitad del producto de sus catetos, en este caso se tiene:
AB⋅ BC 3 2 ⋅ 5 2
α (Δ ABC) = = = 15 u2
2 2

6.1.4. COORDENADAS DEL PUNTO MEDIO


El punto medio del segmento que une P1 (x1, y1) y P2 (x2, y2), es el punto denotado por ( x, y )
donde
x1+ x 2 y1 + y 2
x= , y=
2 2
Veamos como se obtienen estas fórmulas:
_ _
Sea P ( x, y ) el punto medio de P1P2. Tracemos P1A y
Y
P2(x2,y2) AP2 paralelas a los ejes como se muestra en la figura.
Sean B y C los puntos medios de P1A y AP2
respectivamente. Luego P1B = PC o sea
P ( x,y ) _ _
C | x – x1 | = | x – x2 |
X _
Tomando x1 < x < x2
_ _ _ _ x1+ x 2
P1(x1,y1) B A x – x1 = x2 – x ⇒ 2 x = x1 + x2 luego x =
2

_ y1+ y2
De manera similar al considerar que BP = CP2, se obtiene y =
2
EJEMPLO 4

Determine el punto medio del segmento que une los puntos P1 (– 1, 3) y P2 (5, – 7).

SOLUCIÓN:
x1+ x2 −1 + 5 y + y2 3 − 7
Aplicando las fórmulas anteriores, tenemos x= = = 2, y = 1 = =–2
2 2 2 2
Luego ( x, y ) = (2, – 2).

3
GEOMETRÍA ANALÍTICA

EJEMPLO 5

Dado el triángulo con vértices A (2, 3), B (3, – 3) y C (–1, –1), encontrar la longitud de la mediana
trazada desde el vértice A.

SOLUCIÓN:
Y
Recordemos que las medianas en un triángulo son las A (2, 3)
líneas que unen los vértices con los puntos medios de los
lados opuestos. En este caso el lado opuesto al vértice A
es el lado BC.
Sea M el punto medio del lado BC. C (– 1,– 1) X
3 + (−1) −3 + (−1) M
xM = = 1 , yM = = −2 B (3,– 3)
2 2

La longitud de la mediana AM la obtenemos al aplicar la fórmula de la distancia entre dos puntos:


AM = (2 − 1)2 + [3 − ( −2)]2 = 12 + 52 = 26

6.1.5. COORDENADAS DEL PUNTO DE DIVISIÓN DE UN SEGMENTO SEGÚN UNA RAZON


DADA
DIVISIÓN INTERNA
Si A – C – B, es decir si C se encuentra entre A y B, decimos que C divide internamente al
a AC a
segmento AB en la razón r = , si = =r
b CB b
a b
A C B

Para evitar ambigüedades, aunque AB = BA , no es lo mismo decir que C divide al segmento AB


en la razón r, que afirmar que C divide al segmento BA en la misma razón.

En el numerador de la proporción anterior, consideramos el primer punto nombrado en el segmento.


BC
De manera que en el caso que C divida al segmento BA tendremos =r.
CA
Debemos notar que las distancias se miden desde los extremos del segmento al punto divisor.

DIVISIÓN EXTERNA

Si A – B – C , es decir si B se encuentra entre A y C , decimos que C divide externamente al


AC a
segmento AB en la razón r ( r < 0) si − =− =r
CB b
b
A B C
a
Puede probarse que las coordenadas del punto de división (interna o externa) en una razón r, del
segmento P1P2 , donde P1 (x1 , y1) y P2 (x2 , y2) está dado por

x1+ r x 2 y1 + r y 2
x= y=
1+ r 1+ r

4
GEOMETRÍA ANALÍTICA

Veamos como se obtiene para el caso de la división interna:

P2 (x2, y2)
Y Sea P (x, y) el punto de división. Al trazar paralelas a los ejes
P (x, y) en los puntos extremos del segmento y en el punto de
división, se forman los triángulos que se muestran en la
PP PR
figura. Se tiene ΔP1QP ∼ Δ P1RP2. Luego 1 2
= 2 (1)
PP
1 PQ
P1 (x1, y1) Q (x, y1) R (x2, y1)
PP PP
1
= r ∴ PP2 = 1 .
PP2 r
PP ⎛ 1+ r ⎞
X Además P1P2 = P1P + PP2 = P1P + 1
= ⎜ ⎟ PP
1
r ⎝ r ⎠
PP 1+ r
1 2
= (2).
PP
1 r
P2R = |y2 – y1| y PQ = |y – y1|, por estar sobre segmentos verticales
.
P2R y 2 − y1
Si asumimos que y1 < y < y2, se tiene P2R = y2 – y1 y PQ = y – y1, = (3)
PQ y − y1
Si asumimos y2 < y < y1, el resultado es el mismo.

1 + r y 2 − y1 y + r y2
Sustituyendo (2) y (3) en (1) se obtiene = . Al despejar y, resulta y = 1 , r≠–1
r y − y1 1+ r
De manera similar se obtiene la respectiva fórmula para x.
Cuando la división es externa, la fórmula también se obtiene usando semejanza de triángulos.

◊ Siempre debe tenerse presente que si r > 0, se tiene una división interna y el punto divisor se
encuentra “entre” los extremos del segmento, mientras si r < 0, el punto divisor se encuentra en la
prolongación del segmento.

EJEMPLO 6
Encuentre el punto P(x, y) que divide al segmento que une los puntos A (2,3) y B (17,18) en la
1
razón r =
2
SOLUCIÓN:
Aplicamos las fórmulas obtenidas anteriormente
⎛ 1⎞ ⎛ 1⎞
2 + ⎜ ⎟ (17) 3 + ⎜ ⎟ (18)
x + r x2 ⎝ 2⎠ 21/ 2 y + r y2 ⎝ 2⎠ 24 / 2
x= 1 = = =7 y= 1 = = =8
1+ r 1+
1 3/2 1+ r 1+
1 3/2
2 2
Por tanto el punto buscado es P (7, 8)

EJEMPLO 7
Dado el triángulo con vértices A (– 2, 1), B (4, 7) y C (6, – 3), determine las coordenadas del
baricentro
Recordemos que el baricentro es el punto donde concurren las medianas de un triángulo y que las
medianas son los segmentos que unen un vértice con el punto medio del lado opuesto. Una de las
propiedades del baricentro es que la distancia desde cada vértice al baricentro es el doble de la
distancia del baricentro al punto medio correspondiente. Es decir si la mediana correspondiente al
___ ___
AQ
vértice A es AM y Q es el baricentro, se tiene = 2 , siendo M el punto medio del lado BC .
QM

5
GEOMETRÍA ANALÍTICA

___
4+6 7−3
Luego: Sea M el punto medio de BC : xM = =5 yM = =2
2 2
x + 2 xM −2 + 2(5) 8 y + 2 yM 1 + 2(2) 5
Coordenadas del baricentro x Q = A = = yQ = A = =
1+ 2 3 3 1+ 2 3 3

6.1.6. PENDIENTE DE UN SEGMENTO

La pendiente de un segmento mide su “inclinación”. Se define como la razón entre la distancia vertical
Δy y la distancia horizontal, Δx, entre dos puntos. Generalmente se denota por m.
A mayor valor de m , la inclinación del segmento se acerca a la vertical. La pendiente m indica el
incremento o decremento de “y” por cada aumento unitario de x.
____
Si P1 (x1 , y1) y P2 (x2 , y2) la pendiente del segmento P1P2 está dada por
Δy y 2 − y1
m= =
Δx x 2 − x1

____
Tenemos que m = tan θ , donde θ es el ángulo formado por P1P2 o su prolongación con la
dirección positiva del eje X.

Para un segmento vertical, la pendiente no está definida (m = ∞ ).


La pendiente podemos usarla para verificar si un conjunto de tres o más puntos son colineales o no.

EJEMPLO 8
Si A (– 4, 5), B (5, – 3) y C (– 1, k) son tres puntos colineales, determine el valor de k
SOLUCIÓN:
Puesto que los puntos son colineales debe cumplirse que mAB = mAC, luego
5 − ( −3) 5−k 8 5−k 7
= , = ∴k=
−4 − 5 −4 − ( −1) −9 −3 3

6.2. RECTAS EN EL PLANO

6.2.1. ECUACIÓN DE LA RECTA: PUNTO-PENDIENTE

Una recta podemos “definirla” como una “curva” de pendiente constante.

Si una recta L pasa por el punto P0 = (x0, y0) y tiene pendiente m, para cualquier otro punto
y− y0
P (x, y) que esté sobre la recta, debe cumplirse que m = . Por tanto una ecuación para esta
x− x0
recta está dada por y – y0 = m (x – x0)

EJEMPLO 9
1 1
i) Si m = y P0 (2, 4), la ecuación será y – 4 = (x – 2) ⇒ 3y – 12 = x – 2 ⇒ x – 3y + 10 = 0
3 3
ii) Si m = – 2, P0 (3, – 5), la ecuación será y + 5 = – 2 (x – 3) ⇒ 2x + y – 1 = 0

6
GEOMETRÍA ANALÍTICA

6.2.2. ECUACIÓN DE LA RECTA: PUNTO-PUNTO

Si la recta L pasa por los puntos P1 (x1, y1) y P2 (x2, y2), su ecuación está dada por

⎛ y − y1 ⎞ ⎛ y − y1 ⎞
y – y1 = ⎜⎜ 2 ⎟⎟ ( x − x 1 ) ó y – y2 = ⎜⎜ 2 ⎟⎟ ( x − x 2 )
⎝ x2 − x1 ⎠ ⎝ x2 − x1 ⎠
Esta expresión es válida para toda recta, excepto para rectas verticales.

◊ Es más cómodo hallar primero la pendiente y luego proceder como en el caso anterior.

EJEMPLO 10
Si L pasa por P1 (– 3, 2) y P2 (4, –2) su ecuación está dada por:
⎛ 2+2 ⎞ ⎛ 4⎞
y–2= ⎜ ⎟ ( x + 3) ⇒ y – 2 = ⎜ − ⎟ ( x + 3) ⇒ 7y – 14 = – 4 x – 12
⎝ − 3 − 4 ⎠ ⎝ 7⎠
⇒ 4x + 7y – 2 = 0

Podemos verificar que el segundo punto satisface esta ecuación:


4 (4) + 7 (– 2) – 2 = 16 – 14 – 2 = 0

6.2.3. ECUACIÓN PENDIENTE – INTERSECCIÓN CON EL EJE Y

Si una recta L está dada por y = a x + b, ésta corta a los ejes en los puntos (– b / a, 0) y (0 , b).
b− 0 b
Al encontrar la pendiente entre estos puntos tenemos m = = = a o sea
0 − (− b/a) b/a
m = a. Por tanto la ecuación podemos escribirla como y = m x + b

Luego, si la recta L corta al eje Y en el punto (0, b) y tiene pendiente m, su ecuación está dada por

y = m x + b.

El valor de b se conoce como la y-intersección u ordenada en el origen y representa el valor de la


ordenada donde la recta corta al eje Y.

EJEMPLO 11
2 2
Si m = y b = – 6 se tiene y = x + (– 6) o sea 5y = 2 x – 30 ⇒ 2x – 5y – 30 = 0
5 5

6 .2.4. ECUACIÓN SIMÉTRICA DE LA RECTA


Podemos verificar que si una recta L corta al eje X en el punto (a, 0) y al eje Y en el punto (0, b),
su ecuación está dada por
x y
+ =1
a b

Esta expresión es conocida como Ecuación Simétrica de la recta.


EJEMPLO 12
x y
Si una recta corta al eje X en (4, 0) y al eje Y en (0, 3), su ecuación es + = 1 o sea 3x + 4y – 12=0.
4 3

363

7
GEOMETRÍA ANALÍTICA

6.2.5. CASOS ESPECIALES

Recta vertical: x = constante.

Cuando x = constante, la recta es paralela al eje Y, lo que es


igual afirmar que es perpendicular al eje X. Y
El hecho que no aparezca la variable “y”, nos indica que y=k
esta variable puede tomar cualquier valor.
(0, k)
Recta horizontal: y = constante.
x=h
Es una recta paralela al eje X, o sea perpendicular al eje X Y.
De manera similar al caso anterior, el hecho de que no (h, 0)
aparezca la variable “x”, indica que esta variable puede
tomar cualquier valor.

6.2.6. ECUACIÓN GENERAL DE UNA RECTA

Toda ecuación de una recta puede escribirse en la forma Ax + By + C = 0, con A ≠ 0 ó B ≠ 0.

De manera recíproca, Ax + By + C = 0 con A ó B distintas de cero siempre representa una línea


A
recta, con pendiente m = − , con B ≠ 0.
B

Esta recta corta al eje X en el punto (– C / A, 0) y al eje Y en el punto (0, – C / B).

◊ Si A = 0, se tiene una recta paralela al eje X, es decir una recta horizontal (m = 0).

◊ Si B = 0, se tiene una recta vertical en cuyo caso decimos que m = ∞ .

La expresión Ax + By + C = 0 se conoce como la Ecuación General de una Recta.

6.3. POSICIONES RELATIVAS ENTRE PUNTOS Y RECTAS

6.3.1. RECTAS PARALELAS Y RECTAS PERPENDICULARES

Dos rectas son paralelas si y solo si tienen la misma pendiente

Sean L1 una recta con pendiente m1 y L2 una recta con pendiente m2, entonces
L1 ⎥⎥ L2 ⇔ m1 = m2
Y L1 L2

Dos rectas, no verticales ni horizontales (*), son perpendiculares si y solo si sus pendientes son
recíprocas negativas una de la otra, es decir si el producto de sus pendientes es – 1.
−1
L1 ⊥ L2 ⇔ m1 m2 = –1 m2 =
m1

364

8
GEOMETRÍA ANALÍTICA

(*) Esta relación no es válida para el caso de rectas verticales u horizontales, ya que en este caso
obtendríamos una expresión indefinida. Recordemos que si una recta es horizontal su pendiente es
cero, y si es vertical, la pendiente es infinita.

Si las rectas están dadas por L1: A1 x + B1 y + C1 = 0 y L2 : A2 x + B2 y + C2 = 0 , se tiene

A1 B1 C1
i) L1 ⎥⎥ L2 ⇔ = ≠ .
A 2 B2 C2
A 1 B1 C1
Si se cumple = = entonces L1 = L2 y decimos que las rectas son coincidentes.
A 2 B2 C2
A1 A 2
ii) L1 ⊥ L2 ⇔ ⋅ =−1 o sea A1 A2 + B1 B2 = 0
B1 B 2
Y Veamos como se obtiene esta relación:
L1
L2 De la figura deducimos que β = α + 90° o sea
90 – β = – α . Luego tan (90 – β) = tan (– α).

De la trigonometría tenemos:
α β tan (90 – β)= cot β y tan(– α) = – tanα . Luego
X cot β = – tanα ⇒ tanβ = – 1 / tan α
Por tanto m1 = – 1 / m2

EJEMPLO 13

Hallar la ecuación de la recta que pasa por (4, – 3) y es paralela a la recta con ecuación
5x – 3y + 8 = 0

SOLUCIÓN: Dado que las rectas son paralelas, tienen la misma pendiente.
A 5 5
La recta dada tiene pendiente m = − = − =
B −3 3
5
La ecuación buscada es y – (–3) = ( ) (x – 4) ⇒ 3y + 9 = 5x – 20 ⇒ 5x – 3 y – 29 = 0
3

EJEMPLO 14 Y
B (5, 4)
A (– 3,2)
Obtener las ecuaciones de las rectas que contienen las alturas del
triángulo cuyos vértices son A (– 3, 2), B (5, 4) y C (3, – 8)
X
SOLUCIÓN: Recordemos que en un triángulo la altura relativa a un
vértice, es perpendicular al lado opuesto, luego la pendiente de la
recta que la contiene es el recíproco negativo de la pendiente de C (3,–8)
dicho lado.
4−2 1 −1
◊ m AB = = por tanto la pendiente de la altura relativa al vértice C es m = =–4
5+3 4 1/ 4
la ecuación de hC será y + 8 = – 4(x – 3) o sea 4x + y – 4 = 0

2+8 5
◊ De manera similar m AC = = − , luego la pendiente de la altura relativa al vértice B es
−3−3 3
−1 3 3
m= = . La ecuación buscada es y – 4 = (x – 5) es decir 3x – 5y + 5 = 0
− 5/3 5 5

9
GEOMETRÍA ANALÍTICA

4+8
◊ Para la altura relativa al vértice A, se tiene mBC == 6 por tanto la pendiente para dicha
5−3
1 1
altura es m = − . La ecuación buscada es y – 2 = − (x + 3) o sea x + 6y – 9 = 0.
6 6

6.3.2. DISTANCIA DE UN PUNTO A UNA RECTA


La distancia desde un punto P0 (x0, y0) a la recta L: Ax + By + C = 0 se define como la longitud del
segmento perpendicular que une al punto P con la recta L. Está dada por:

Ax 0 + By 0 + C
d (P0, L) =
A 2 + B2

Veamos una forma de obtener esta fórmula:


En la figura, sea Q el punto sobre la recta L tal que
Y PQ ⊥ L. La distancia buscada es PQ.

Sea R el punto sobre la recta L tal que PR || al eje X.


T R P Tenemos que las coordenadas de R son de la forma
(x’, y0) ya que está a la misma altura que el punto P.
Q De la ecuación de la recta resulta:
X ⎛ By + C ⎞
O S A x’ + B y0 + C = 0 ∴ x’ = − ⎜ 0 ⎟
⎝ A ⎠
La distancia de P a R está dada por:
By0 + C Ax 0 + By0 + C
PR = | x0 – x’ | = | x0 + |=| | (1)
A A
Sean S y T los puntos donde la recta corta a los ejes X e Y respectivamente. Luego sus
coordenadas serán: S (– C / A, 0) y T (0, – C / B). La distancia hasta el origen será

C2 C2 C
TO = | C / B | y SO = | C / A |. (2) La distancia de T a S es TS = + = A 2 + B2
2 2 AB
A B
Como PR || al eje X se tiene Δ PQR ≅ Δ TOS (Teorema de semejanza AA)

Ax 0 + By 0 + C
PQ PR PQ A
Por tanto = (3). Sustituyendo (1) y (2) en (3) tenemos: = .
TO TS C C 2 2
A +B
B AB
Ax 0 + By 0 + C
Simplificando obtenemos PQ =
A 2 + B2

6.3.3. DISTANCIA ENTRE RECTAS PARALELAS


C1 − C 2
Si L1 : Ax + By + C1 = 0 y L2 : Ax + By + C2 = 0 son dos rectas
d (L1 , L2) =
paralelas, entonces A 2 + B2

NOTA: Para la aplicación de esta fórmula debe observarse que


los coeficientes de las variables deben coincidir en ambas
ecuaciones.

10
GEOMETRÍA ANALÍTICA

EJEMPLO 15

Encontrar la distancia desde el punto P0 (5, – 3) a la recta con ecuación 12 x – 5 y – 36 = 0

Ax 0 + By 0 + C
SOLUCIÓN: Tenemos d (P0, L) = . Introduciendo los valores dados. Resulta
A 2 + B2

12 (5) − 5 (−3) − 36 | 60 + 15 − 36 | 39
d (P0, L) = = = =3
122 + (−5)2 144 + 25 13

EJEMPLO 16
Hallar la distancia entre las rectas paralelas L1 : y = 2x – 5 y L2 : 2x – y + 3 = 0

SOLUCIÓN: Escribimos L1 en la forma de la ecuación general, para identificar correctamente los


valores de A, B y C.
Tenemos: L1: 2x – y – 5 = 0 y L2 : 2x – y + 3 = 0, luego A = 2, B = – 1 , C1 = – 5 y C2 = 3.
Introduciendo los valores en la fórmula, obtenemos
C1 − C 2 |− 5 − 3| 8 8 5
d (L1 , L2) = = = =
A 2 + B2 2 2 + (−1) 2 5 5

EJEMPLO 17
Hallar la distancia entre las rectas paralelas L1 : 4x – 3y + 5 = 0 y L2 : 8x – 6y + 7 = 0

Notemos que aunque las rectas sean paralelas, no coinciden los coeficientes. Para igualarlos tenemos
dos opciones, duplicar los coeficientes de la primera recta o bien dividir entre dos los coeficientes de la
segunda. Optando por la primera opción, resulta:
L1 : 8x – 6y + 10 = 0 y L2 : 8x – 6y + 7 = 0. Se tiene A = 8, B = – 6, C1 = 10 y C2 = 7.
Introduciendo los valores en la fórmula, obtenemos:
C1− C2 |10 − 7 | 3 3
d (L1 , L2) = = = =
2
A +B 2 2
8 + (−6) 2 64 + 36 10

EJEMPLO 18
Determine la longitud de la altura relativa al vértice A en el triángulo cuyos vértices son
A (– 3, 2), B (5, 4) y C (3, – 8).

SOLUCIÓN: La longitud buscada es la distancia desde el vértice A a la recta que pasa por B y C.
↔ 4+8
La ecuación de la recta BC tiene pendiente mBC = = 6 , luego resulta y – 4 = 6(x – 5) o sea
5−3
↔ | 6 (−3) − 2 − 26 | 46
6x – y – 26 = 0 . Por tanto d(A, BC ) = = ≈ 7.56
6 2 + (−1) 2 37

6.3.4. ÁNGULO FORMADO POR DOS RECTAS QUE SE CORTAN

tanA − tanB
De la trigonometría sabemos que tan (A – B) = , también sabemos que la pendiente
1 + tanA tanB
de una recta equivale a la tangente del ángulo formado por la recta y el semieje positivo X.

11
GEOMETRÍA ANALÍTICA

De la figura tenemos que θ = θ2 – θ1,


L2
Y
θ
L1 tanθ 2 − tanθ 1
luego tan θ =
1 + tanθ 2 ⋅ tanθ 1

θ2 m 2 − m1
θ1 o sea tan θ =
X 1 + m 2 ⋅ m1

Si L1 tiene pendiente m1 y L2 tiene pendiente m2, el ángulo θ que tiene lado inicial en L1 y lado
final en L2, está dado por:

m 2 − m1
tan θ =
1 + m 2 ⋅ m1

EJEMPLO 19
Hallar el ángulo entre las rectas L1: 3x – 2y + 5 = 0 y L2: 6x + 2y + 11 = 0
A 3 6
Solución: Tenemos m = − , luego m1 = y m2 = − = − 3
B 2 2
3 9
−3− −
m 2 − m1 2 = 2 = 9
Aplicando la fórmula anterior tan θ = =
1 + m 2 ⋅ m1 3 7 7
1 + (−3) ( ) −
2 2
9
Luego θ = tan– 1 ( ) ≈ 52.12º
7
6.3.5. INTERSECCIÓN CON LOS EJES COORDENADOS

Toda recta no paralela a los ejes coordenados corta a ambos ejes.


Si L está dada por Ax + By + C = 0, para hallar la intersección con el eje X, basta hacer y = 0.
En este caso se obtiene x = – C / A. Este valor se conoce como la abscisa en el origen o la
X-intersección. El punto de intersección es (– C / A, 0)
De manera similar para hallar el punto de intersección con el eje Y, hacemos x = 0 y obtenemos
y = – C / B. Este valor se conoce como la ordenada en el origen o la Y-intersección.

Al determinar las intersecciones con los ejes se facilita el bosquejo de la gráfica de la recta, puesto
que basta unir estos puntos y extender la prolongación según sea necesario en el problema en que
estemos trabajando.
Y
EJEMPLO 20
(0, 4)
La recta 3x – 2y + 8 = 0, corta al eje X en el punto (– 8/3, 0) y al eje Y en
el punto (0, 4)
(– 8/3, 0) X

12
GEOMETRÍA ANALÍTICA

6.3.6. PUNTO DE INTERSECCIÓN DE DOS RECTAS QUE SE CORTAN

Las coordenadas del punto de intersección de las rectas L1 : A1 x + B1 y + C1 = 0 y

⎧ A x + B 1y + C 1 = 0
L2: A2 x + B2 y + C2 = 0, es la solución del sistema de ecuaciones ⎨ 1
⎩A 2 x + B 2 y + C 2 = 0
EJEMPLO 21
Hallar el punto de intersección entre las rectas x + y =12 y x – 2y = 0.

SOLUCIÓN: Planteamos y resolvemos el sistema de ecuaciones formado por las dos rectas. La
solución corresponde al punto de intersección:
⎧ x + y = 12 (1)

⎩ x − 2y = 0 (2)
De (2) despejamos “x” y sustituimos en (1): Se tiene x = 2y (3), luego (2y)+y = 12 ⇒ 3y = 12 ∴y = 4.

Sustituyendo este valor en (3) se obtiene x = 2 (4) = 8.


Por tanto las rectas se cortan en el punto (x, y) = (8, 4)

EJEMPLO 22

La suma de las longitudes de los segmentos que una recta determina sobre los ejes coordenados es
5. Halle la ecuación de la recta sabiendo que pasa por (4, 3).

Y Y
Y
• P (4, 3) (0, b) • P (4, 3) • P (4, 3)

• •
X (a, 0) X X
(0, b) • (a, 0)
(a) ( b)

Al bosquejar una gráfica se aprecia que hay dos opciones:

Caso (a): b > 0 y a < 0 y , siendo a y b las intersecciones con los ejes X e Y respectivamente.

La suma de las longitudes de los segmentos es |a| + |b| = 5 o sea b – a = 5 (1)


ya que |a| = – a por ser a < 0 y |b| = b por ser b > 0.

x y
La ecuación de la recta es de la forma + = 1 , luego como pasa por (4, 3) tenemos
a b
4 3
+ = 1 lo que equivale a 4b + 3 a = a b (2)
a b
⎧ b−a = 5
Resolvemos (1) y (2): ⎨
⎩ 4b + 3a = ab

De (1) b = a + 5, sustituyendo en 2 obtenemos


2 ± 4 + 80
4(a + 5) + 3a = a (5 + a) ⇒ a2 – 2a – 20 = 0 a = = 1 ± 21
2
Como a < 0, se descarta 1 + 21 , resultando a = 1 – 21 ∴ b = 6 – 21

13
GEOMETRÍA ANALÍTICA

x y
Luego la ecuación buscada es: + =1
1 − 21 6 − 21

Caso (b): a > 0 y b < 0

Ahora de |a| + |b| = 5 resulta a – b = 5 o sea a = b + 5 (3)


Sustituyendo (3) en (2) se obtiene:
b2 – 2b – 15 = 0, cuyas raíces son b = – 3 ∨ b = 5.

Se descarta b = 5 porque b < 0. Como b = – 3, resulta a = 2 y la ecuación buscada es:


x y
− = 1 , lo que equivale a 3x – 2y – 6 = 0.
2 3

EJERCICIOS

1. Dado que A (8, – 3), encuentre las coordenadas del punto B tal que C (–10, 5) sea el punto medio
de AB .
2. Si A (6, 1) y B (– 4, – 3), verifique que P (5, – 11) está sobre la mediatriz de AB .

3. Halle la pendiente del segmento que une el par de puntos dados:


a) (3, 2) y (4, 5) b) (3, 5) y (4, 1) c) (1, 3) y (– 2, 0)
⎛1 5⎞ ⎛4 ⎞ 1
d) (4, – 3) y (– 2, – 3) e) ⎜ , − ⎟ y ⎜ , 2 ⎟ f) (2, − ) y (2, 5)
⎝3 3⎠ ⎝3 ⎠ 2
4. Halle el valor de x, de manera que la pendiente del segmento que une los puntos dados, sea igual
al valor indicado:
1 1
a) (3, x) y (4, – x); m = b) (4, 1) y (5, ); m = 2 c) (2, 3) y (3, x); m = 0
3 x
d) (5, x) y (6, 1); m = – 3 e) (2, |x|) y (1, x); m = 1

5. Demuestre que A, B y C son los vértices de un triángulo rectángulo i) usando la fórmula de


distancia , ii) usando pendiente y iii) calcule su área: a) A (6, 3), B (–1, – 4) y C (– 5, 0)
b) A (– 4, 5), B (5, – 3) y C (1, 7) c) A (2, – 2), B (– 8, 4) y C (5, 3)
6. Demuestre, usando pendientes, que los puntos (9, 2), (11, 6), (3, 5) y (1, 1) son vértices de un
paralelogramo.

7. Halle la distancia desde el punto medio del segmento que une A (– 2, –10) y B (4, 6) hasta el
punto medio del segmento que une C (3, 5) y D (–1, 3).

8. Hallar el perímetro del cuadrilátero cuyos vértices son (– 3, – 1), (0, 3), (3, 4) y (4, – 1)

9. Encuentre un punto P sobre el eje Y que equidiste de los puntos A (– 5, 2) y B (4, 3)

10. Los vértices de un triángulo son A (3, 8), B (2, – 1) y C ( 6, –1). Si D es el punto medio del lado BC,
calcular la longitud de la mediana AD.

11. Uno de los extremos de un segmento rectilíneo de longitud 5 es el punto (3, –2). Si la abscisa del
otro extremo es 6, hallar su ordenada. .

12. Uno de los extremos de un segmento es el punto (7, 8) y su punto medio es (4, 3). Hallar las
coordenadas del otro extremo.

13. Los vértices de un triángulo son A (–1, 2), B (1, – 4) y C (8, 3). Calcule las longitudes de sus
medianas. Determine las coordenadas del baricentro.

14
GEOMETRÍA ANALÍTICA

14. Halle las coordenadas de los puntos P1, P2 y P3 en el segmento que une A (2, 5) y B (4, 7),
que dividen al segmento en cuatro partes iguales.

15. Los vértices de un triángulo son los puntos A (8, 5), B (1, – 2) y C (– 3, 2). Determine la
pendiente de cada uno de sus lados. .

16. Los vértices de un triángulo son los puntos (2, – 2), (–1, 4) y (4, 5). Determine la pendiente de
cada uno de los lados.

17. Complete la siguiente tabla, determinando la pendiente del segmento P1P2 y la ecuación de la
recta que pasa por dichos puntos.

P1 P2 m Ecuación
a (2, – 1) (3, 5)
b (0, 5) (2, 9)
c (3, – 2) (4, 1)
d (5, 25) ( – 2, – 3)

18. Encuentre la ecuación de la recta que pasa por el punto dado y tiene la pendiente indicada:

P0 m Ecuación
a (2, 4) –1
b (3, 1) 2
c (4, – 2) –3
1
d (– 2, – 7) 3

19. Indique la pendiente de cada una de las siguientes rectas:


a) 2x – 5y – 7 = 0 b) 5x + y – 2 = 0 c) 3y = 1 – 4x d) 2x = 3 + 4y
x x y
e) x = 3 + 2y f) – 3y + 2x – 7 = 0 g) = – 4y + 2 h) − =1
2 3 6
20. Obtenga la ecuación de la recta que satisfaga las siguientes condiciones:

a) Pendiente 4, y pasa por el punto (– 3, 5) b) Pasa por los puntos (2, 3) y (4, 6)
c) Pendiente – 3 y su x-intersección es 2. d) Su x-intercepto es –2 y su y-intercepto es 7.
e) Pasa por (– 3, 5) y es paralela al eje X. f) Pasa por (6, – 4) y es paralela al eje Y.
g) Pasa por (– 5, 7) y es perpendicular al eje X. h) Pasa por (2, 3) y es perpendicular al eje Y.
i) Pasa por (– 6, 3) y tiene ángulo de inclinación de 45°
j) Pasa por el punto (1, 5) y tiene pendiente 2. k) Pendiente – 3 y su y-intercepto es – 2
l) Es paralela al eje X y pasa 5 unidades abajo del origen.
m) Pasa por los puntos (4, 2) y (– 5, 7) n) Pasa por los puntos (– 4, 6) y (1, – 18)
ñ) Pasa por (– 5, 7) y es perpendicular a la recta 5x + 4y = 20
o) Pasa por (1, – 2) y es paralela a la recta 3x – 4y + 8= 0
p) Pasa por (7, 2) y tiene la misma pendiente que el segmento que une los puntos (2, 4) y (3, – 1)
q) Pasa por (– 1, 3) y tiene la misma pendiente que la recta 2x – y + 3 = 0

21. Obtenga la ecuación de la mediatriz del segmento que une los puntos
a) A (– 2, 6) y B (3, –1) b) A (– 3, 2) y B (1, 6).

22. Hallar la mediatriz del segmento que los ejes coordenados determinan en la recta con ecuación
5x + 3y – 15 = 0.
23. Una recta de pendiente 3 pasa por el punto (3, 2). La abscisa de otro punto de la recta es 4.
Hallar su ordenada.

15
GEOMETRÍA ANALÍTICA

24. Una recta de pendiente – 2 pasa por el punto (2, 7) y por los puntos A y B. Si la ordenada de A
es 3 y la abscisa de B es 6 ¿Cuál es la abscisa de A y cuál la ordenada de B?

25. Encuentre el valor de A tal que el punto P (–1, 2) esté sobre la recta Ax + 2y – 7 = 0.

26. Los vértices de un triángulo son A (3, – 3), B (– 4, 1) y C (– 3, 3). Hallar la longitud y la ecuación
de la altura del vértice B sobre el lado AC y el área del triángulo.

27. Obtenga la ecuación de la recta que satisfaga las condiciones indicadas a continuación:
a) Pasa por (2, – 3) y es paralela a la recta con ecuación 3x + 2y – 2 = 0
b) Pasa por (1, 1) y es perpendicular a la recta con ecuación 2x – 4y + 5 = 0.
c) Pasa por (4, – 5) y es perpendicular a la recta que pasa por (1,1) y (3, 9).
d) Tiene pendiente – 3 y pasa por el punto de intersección de las rectas L1: 2x + y – 8 = 0
y L2 : 3x – 2y + 9 = 0
e) Es perpendicular a una recta que tiene pendiente – 4 y pasa por (5, – 3)

28. Hallar la ecuación de la recta cuya pendiente es – 4 y que pasa por el punto de intersección de las
rectas 2x + y – 8 = 0 y 3x – 2y + 9 = 0.

29. Una recta pasa por la intersección de las rectas 7x – 2y = 0 y 4x – y – 1 = 0 y es perpendicular


a la recta 3x + 8y – 19 = 0. Hallar su ecuación.
30. Hallar el valor de k para que la recta k x + (k – 1) y +15 = 0 sea paralela a L1: 8x + 6y +1 = 0.

31. Hallar el valor de k para que la recta k x + (k + 1) y – 18 = 0 sea perpendicular a la recta con
ecuación 3x – 2y + 5 = 0.

32. Hallar el valor de k tal que la recta k2 x + (k + 1) y + 3 = 0 sea perpendicular a L1: 2x – y – 11 = 0.


33. Determine el valor de k de manera que la recta 4x + 5y + k = 0 forme con los ejes coordenados
un triángulo rectángulo de área 20 u2.

34. Encuentre la distancia desde el punto (2, 3) hasta la recta 3x – 4 y + 10 = 0

35. Hallar la distancia del origen a la recta 12x – 5y + 9 = 0.

36. La distancia de la recta 4x – 3y + 1 = 0 al punto P es 4. Si la ordenada de P es 3, halle su abscisa.

37. Hallar la ecuación de la recta que pasa por el punto (3, 1) y es tal que la distancia de esta recta al
punto (– 1, 1) es igual a 2 2
38. La distancia de una recta al origen es 3. La recta pasa por el punto (3 5 , – 3). Hallar su ecuación.

39. Los vértices de un triángulo son A (4, – 4), B (– 3, 0) y C (– 2, 2). Encuentre la longitud de la
altura del vértice A sobre el lado BC y el área del triángulo.

40. Hallar el área del trapecio determinado por las rectas x – 2y = 0, x – 2 y + 6 = 0,


x + 3y – 20 = 0, 3x – y – 5 = 0.

41. Las ecuaciones de los lados de un cuadrilátero son 3x – 8y + 36 = 0, x + y – 10 = 0,


3x – 8y – 19 = 0 , x + y + 1 = 0. Demostrar que la figura es un paralelogramo. Hallar las
coordenadas de sus vértices y su área.

42. La suma de los segmentos que una recta determina sobre los ejes coordenados es 3. Halle la
ecuación de la recta, sabiendo que pasa por el punto (2, 2)
4
43. Una recta pasa por el punto P (2, ) y forma con los ejes coordenados un triángulo de
3
perímetro igual a 12. Hallar la ecuación de dicha recta.

16
GEOMETRÍA ANALÍTICA

44. Una recta pasa por el punto (24, – 5) y forma con los ejes coordenados un triángulo de área igual
a 30 u2. Determine su ecuación.

45. Determinar el valor de k para que la recta 4x + 5y + k = 0 forme con los ejes coordenados un
triángulo rectángulo de área igual a 10 u2.

46. La suma de los segmentos que una recta determina sobre los ejes coordenados es 5. Halle la
ecuación de la recta sabiendo que pasa por (4, 3)

47. Una recta pasa por el punto de intersección de las rectas 2x – 3y – 5 = 0 y x + 2y – 13 = 0 y el


segmento que determina sobre el eje x es igual al doble de su pendiente. Hallar la ecuación de dicha
recta.
48. Encuentre el ángulo formado por las rectas
a) 2x – y = 6 ∧ x + 2y = 0 b) 7x – 4y = 28 ∧ y + 8x = 0
c) 3x + 4y = 12 ∧ 5x – 12y = 10 d) 4x – 9y + 11 = 0 ∧ 3x + 2y – 7 = 0

49. Encuentre las medidas de los ángulos internos y la longitud de los lados del triángulo cuyos
vértices son A (5, 5), B (2, 8) y C (10, 10)
50. Determine las distancias entre las parejas de rectas paralelas
a) 2x – y = 0 ∧ 2x – y + 5 = 0 b) 3x + 4y = 12 ∧ 3x + 4y = 22c) 3x – 4y – 7 = 0 ∧ 6x – 8y + 15 = 0

SOLUCIONES
1. B (– 28, 13) 2. AP = BP = 145 3. a) 3 b) – 4 c) 1 d) 0 e) 11/3 f) ∞
1 1 1
4. a) – b) c) 3 d) 4 e) − 5. iii) a) 28 b) 29 c) 34 7. 6 8. 20.26 9. (0, – 2)
6 3 2
1 1 8 1
10. 9.055 11. 2 ó -6 12. (1, – 2) 13. 4 5 , 146 , 194 , G ( , )
2 2 3 3
5 11 7 13 3 1 7
14. ( , ), (3, 6) y ( , ) 15. – 1, 1, 16. – 2, , 17. a) 6x – y – 13 = 0 b) 2x – y + 5 = 0
2 2 2 2 11 5 2
1
c) 3x – y – 11 = 0 d) 4x – y + 5 = 0 e) x = f) 3x + y – 3 = 0 g) x + 3y – 6 = 0
2
18. a) x + y – 6 = 0 b) 2x – y – 5 = 0 c) 3x + y – 10 = 0 d) x – 3y – 19 = 0 e) 8x – 2y – 7 = 0
2 4 1 1 2 1
f) 2x – y + 1 = 0 g) 2x + y – 3 = 0 19. a) b) – 5 c) − d) e) f) g) − h) 2
5 3 2 2 3 8
20. a) 4x – y + 17 = 0 b) 3x – 2y = 0 c) 3x + y – 6 = 0 d) 7x – 2y + 14 = 0 e) y – 5 = 0 f) x – 6 = 0
g) x + 5 = 0 h) y – 3 = 0 i) x – y + 9 = 0 j) 2x – y + 3 = 0 k) 3x + y + 2 = 0 l) y + 5 = 0
m) 5x + 9y – 38 = 0 n) 24x + 5y + 66 = 0 ñ) 4x – 5y + 55 = 0 o) 3x – 4y – 11 = 0 p) 5x + y – 37 = 0
q) 2x – y + 5 = 0 21. a) 5x – 7y + 15 = 0 b) x – y + 5 = 0 22. 3x – 5y + 8 = 0 23. 5
24. xA = 4, yB = – 1 25. A = – 3 26. Área = 9 27. a) 3x + 2y = 0 b) 2x + y – 3 = 0
c) x + 4y + 16 = 0 d) 3x + y – 9 = 0 e) x – 4y – 17 = 0 28. 4x + y – 10 = 0 29. 8x – 3y + 5 = 0
1 4 9
30. k = 4 31. k = 2 32. 1, – 33. ± 20 2 34. 35. 36. x = – 3 o x = 7
2 5 13
18 5 54
37. x – y – 2 = 0 ó x + y – 4 = 0 38. y = – 3 ó 5 x + 2y – 9 = 0 3 9. ,9 40.
5 5
x
41. vértices: (1, – 2), (– 4, 3), (4, 6) y (9, 1); Área = 55 42. y = 2x – 2 ó y = +1
2
43. 4x + 3y – 12 = 0 ó 8x + 15y – 36 = 0 44. 5x + 12y – 60 = 0 ó 5x + 48y + 120 = 0 45. ± 20
x y
46. 3x – 2y – 6 = 0 ó + =1 47. x – 2y – 1 = 0 ó 3x – y – 18 = 0
1 − 21 6 − 21
48. a) 90º b) 36.9º c) 59.5º d) 80.3º 49. 90º, 59º, 31º ; 3 2 , 5 2 y 2 17
29 3 T
50. a) 5 b) 2 c) 51. F = x + 15 52. R = + 1000, 1000.1 ohms 53. 800 cal.
10 2 1000

17
GEOMETRÍA ANALÍTICA

6.4. CÓNICAS
INTRODUCCION
Las cónicas son figuras geométricas que se obtienen al cortar un cono circular recto de dos hojas
mediante un plano. Son conocidas como Circunferencia, Elipse, Parábola e Hipérbola.
La figura geométrica obtenida depende de la posición del plano respecto al eje de simetría del cono:
◊ Si el plano es perpendicular al eje de simetría y lo corta en un punto distinto del vértice se obtiene una
Circunferencia.
◊ Si inclinamos este plano, cortando sólo una rama del cono, se obtiene una Elipse, la cual se asemeja a
una circunferencia “achatada”.
◊ Cuando la inclinación del plano es paralela a una generatriz, sin contenerla, se obtiene una Parábola,
la cual es una figura “abierta”.
◊ Si continuamos inclinando el plano, de manera que corte las dos ramas del cono, se obtiene una
Hipérbola.
Estas figuras, además de provenir del corte de un cono, tienen otros rasgos en común que fueron
descubiertos hace muchos años. Para cada una de ellas si la representamos en un plano coordenado,
sus ecuaciones resultan de segundo grado.
Todas ellas, excepto la parábola, son figuras que tienen un centro. Excepto la circunferencia, tienen
asociados puntos y rectas llamados focos y directrices y se caracterizan por la razón entre la distancia de
cualquiera de sus puntos a un foco y la distancia de dicho punto a la directriz asociada. Esta razón se
llama “excentricidad” y se representa por “e”. Se tiene:
Circunferencia: e = 0, Elipse: 0< e<1
Parábola: e = 1, Hipérbola: e>1

Estas figuras las encontramos en muchas aplicaciones de las Matemática: estudio de órbitas planetarias,
Física atómica, Óptica, Acústica, etc. y de ahí la importancia de su estudio.

6.4.1. CIRCUNFERENCIA
DEFINICIÓN
Una circunferencia es el conjunto de puntos P situados en un plano que equidistan de un punto fijo C
llamado centro de la circunferencia. La distancia del punto P al centro C se llama radio.
Es decir la circunferencia es el conjunto de puntos P que satisfacen la expresión d (P, C) = r, donde C
es el centro de la circunferencia y r el radio.

ECUACIÓN DE LA CIRCUNFERENCIA CON CENTRO C (h, k) Y RADIO r


Al aplicar la fórmula de la distancia entre dos puntos, si
Y
P ( x, y) es un punto cualquiera de la circunferencia , se
obtiene : P(x,y)
d (P, C) = (x − h ) + ( y − k )
2 2
=r o sea
r

(x − h)2 + (y − k )2 = r 2 (1) (h, k)

Si el centro se encuentra en el origen, la ecuación se reduce a x2 + y2 = r2

Si desarrollamos la ecuación (1) se obtiene:

x2 – 2xh+ h2 + y2 – 2yk+ k2 = r2 o sea x2 + y2 – 2hx – 2ky + (h2+ k2 – r2 ) = 0

es decir una ecuación de la forma x2 + y2+ Dx + Ey + F = 0 (2)

18
GEOMETRÍA ANALÍTICA

Pero no toda ecuación de la forma (2) representa una circunferencia. Veamos qué debe cumplir para
representar una circunferencia.

Dada una ecuación de la forma (2) x2 + y2 + Dx + Ey +F = 0

reacomodando se tiene (x2 + Dx ) + (y2 + Ey )= –F

completando trinomios cuadrados perfectos en cada paréntesis:


D2 E2 D2 E2
(x2+ Dx + ) +(y2 + Ey + )= + –F
4 4 4 4
2 2 ⎛ 2 ⎞
⎛ D⎞ ⎛ E⎞ D E2
Factorizando ⎜ x + ⎟ + ⎜ y+ ⎟ = ⎜ + − F⎟
2⎠ 2⎠ ⎜ ⎟
⎝ ⎝ ⎝ 4 4 ⎠
Luego
D2 E2
i) si + – F > 0, la ecuación (2) representa una circunferencia con centro
4 4
⎛ −D −E⎞ D2 E2
⎜ , ⎟ y radio r = + −F
⎝ 2 2 ⎠ 4 4
D2 E2 ⎛ −D −E⎞
ii) si + – F = 0, se tendrá r = 0, es decir se reduce a un punto ⎜ , ⎟
4 4 ⎝ 2 2 ⎠
D2 E2
iii) si + – F < 0, no hay valores reales que satisfagan la ecuación.
4 4

EJEMPLO 1
Encontrar una ecuación de la circunferencia con centro en C (4, – 3) y radio r = 6

SOLUCIÓN: tenemos (x − h)2 + (y − k )2 = r 2 y (h, k) = (4, – 3), r = 6.


Sustituyendo los valores de h, k y r en la ecuación obtenemos: (x – 4)2 + [y – (–3)]2 = 62
⇒ (x – 4)2+ (y + 3)2 = 36 .
Desarrollando los binomios al cuadrado: x – 8x + 16 + y2 + 6y+ 9 – 36 = 0
2

Simplificando y reordenando: x2 + y2 – 8x + 6y – 11 = 0

EJEMPLO 2
Encontrar la ecuación de la circunferencia que pasa por P (3, – 1) y tiene centro en (1,2)

SOLUCIÓN: nos dan como información las coordenadas del centro (h, k) = (1, 2), falta determinar el
valor del radio. En este caso lo obtenemos al calcular la distancia desde el centro al punto P dado.
r = d (C, P) = (3 − 1) 2 + (−1 − 2) 2 = 2 2 + (−3) 2 = 13

luego al sustituir los datos en la ecuación (x − h)2 + (y − k )2 = r 2 se obtiene


2 2 2 2 2
(x – 1) + (y – 2) = ( 13 ) o sea x – 2x + 1 + y – 4y + 4 = 13.
Luego la ecuación buscada es x2 + y2 – 2x – 4y – 8 = 0

EJEMPLO 3
Los extremos de un diámetro de una circunferencia, tienen coordenadas (5,1) y (–1, –7).
Determine la ecuación de dicha circunferencia.

SOLUCIÓN:

19
GEOMETRÍA ANALÍTICA

El centro de una circunferencia se localiza en el punto medio de cualquiera de sus diámetros, luego al
aplicar la fórmula del punto medio obtenemos:
__ __
5 + (−1) 4 1 + (−7) −6
x = = =2 , y = = = −3 luego el centro es C (2, –3)
2 2 2 2
El radio es la mitad de la longitud del diámetro, luego al aplicar la fórmula para la distancia entre dos
puntos se obtiene:
d= [5 − (−1)]2 + [1 − (−7)]2 = 6 2 + 8 2 = 100 = 10 ∴ r = 5
(El radio también puede hallarse calculando la distancia desde el centro a uno de los extremos del
diámetro.)
La ecuación buscada es:
(x − h)2 + (y − k )2 = r 2
⇒ (x – 2)2 + (y + 3)2 = 52
(x2 – 4x + 4) + (y2 + 6y + 9) = 25 ⇒ x2 + y2 – 4x + 6y – 12 = 0

EJEMPLO 4
Obtener la ecuación de la circunferencia de radio r = 4, si su centro es el punto de intersección de las
rectas L1: 2x + y – 16 = 0 y L2 : 3x – 2y + 4 = 0

SOLUCIÓN:
Debemos encontrar primero las coordenadas del centro. Por la información dada, corresponden a la
solución del sistema de ecuaciones formado con las ecuaciones de las rectas dadas, o sea
⎧ 2x + y − 16 = 0 (1)
⎨ Al multiplicar por 2 la (1) y sumarla a la (2), se obtiene
⎩3 x − 2y + 4 = 0 ( 2)
4x + 2y − 32 = 0
3 x − 2y + 4 = 0
7x – 28 = 0 ∴ x = 4
Sustituyendo en la ecuación (1) y despejando y se obtiene y = 8. Luego el centro está dado por
C (4, 8)
La ecuación buscada es: (x – 4)2 + (y – 8)2 = 42 ⇒ x2 – 8x + 16 +y2 – 16y + 64 = 16
x2 + y2 – 8x – 16 y + 64 = 0

EJEMPLO 5

Encontrar la ecuación de la circunferencia que tiene centro en (9, 6) y es tangente a la recta con
ecuación 4x + 2y – 8 = 0

SOLUCIÓN:
Conocemos las coordenadas del centro, falta encontrar el radio y luego completar la ecuación
En vista de que la circunferencia es tangente a la recta dada, su radio es la distancia del centro a dicha
recta. Aplicando la fórmula para la distancia de un punto a una recta, obtenemos:
Ax + By+ C 4(9) + 2(6) − 8 40
r = d (C, L)= = = =4 5
A 2 + B2 42 + 22 20
La ecuación buscada es
(x – 9)2 + (y – 6)2 = (4 5 )2 ⇒ x2 –18x + 81 + y2 – 12y + 36 = 16 (5) = 80
x2 + y2 – 18x – 12 y + 37 = 0
EJEMPLO 6
Encontrar la ecuación de la circunferencia que pasa por los puntos A (6, 6), B (–1, 5) y C (7, –1)

SOLUCIÓN:

20
GEOMETRÍA ANALÍTICA

Primera forma: De la Geometría Euclideana sabemos que los puntos dados forman un triángulo inscrito
en la circunferencia, por tanto el centro se localiza en el circuncentro del triángulo o sea el punto donde
se intersecan las mediatrices del triángulo.

Recordemos que la mediatriz de un segmento es la recta que pasa perpendicular por el punto medio.
Luego una forma de hallar el centro consiste en resolver el sistema de ecuaciones formado por las
ecuaciones que representen las mediatrices (basta considerar dos de ellas) y para encontrar la ecuación
de una mediatriz necesitamos ubicar el punto medio y la pendiente de dicha recta.
Y
__
Encontremos primero la mediatriz del segmento AB A (6, 6)
B (– 1, 5)
6−5 1
*Pendiente: mAB = = , luego la pendiente de la mediatriz
6+1 7
−1 X
será m1 = = −7
1/ 7 C (7, – 1)
__ 6−1 5 6 + 5 11
*Punto medio de AB : x= = , y= =
2 2 2 2
*La ecuación buscada: y – 11 /2 = – 7 (x – 5/2) o sea y + 7x = 23 (1)
__
Busquemos ahora la mediatriz del segmento BC
5+1 −3 −1
*Pendiente: mBC = = , por tanto la pendiente de la mediatriz es m2 = = 4/3
− 1− 7 4 − 3/4
__ 5−1 4 −1 + 7 6
*Punto medio de BC : y = = = 2, x= = =3
2 2 2 2
*La ecuación buscada: y – 2 = (4/3) (x – 3) o sea 3y – 4x = – 6 (2)

⎧ y+ 7 x = 23
Resolvemos el sistema formado por (1) y (2) ⎨ .
⎩3 y − 4 x = − 6

Al aplicar cualquiera de los métodos se obtiene x = 3, y = 2, luego C (3,2)

El radio podemos determinarlo calculando la distancia desde el centro a cualquiera de los puntos dados,
digamos al punto A
r= (6 − 3) 2 + (6 − 2) 2 = 9 + 16 = 5
Por tanto la ecuación de la circunferencia es
(x –3 )2 + (y – 2)2 = 52 ⇒ x2 – 6x + 9 +y2 – 4y + 4 = 25 ⇒ x2 + y2 – 6x – 4y – 12 = 0
Segunda forma:
Otra manera de encontrar la ecuación pedida es considerar la ecuación general de una circunferencia
x2 + y2+ Dx + Ey + F = 0 , la cual debe satisfacerse con las coordenadas de todo punto que pertenezca a
la circunferencia. Observamos que tiene tres coeficientes desconocidos D, E y F, pero al conocer tres
puntos de la circunferencia podemos formar un sistema de tres ecuaciones con tres incógnitas y con él
encontrar los valores de los coeficientes y formar la ecuación buscada.

Con el punto A (6, 6): 6 2 + 62 + 6 D + 6 E + F = 0 ⇒ D + 6 E + F = – 72 (1)


Con el punto B (–1, 5): (– 1)2 + 52 – D + 5 E + F = 0 ⇒ – D + 5 E + F = – 26 (2)
Con el punto C (7, –1): 72 + (– 1)2 + 7 D – E + F = 0 ⇒ 7 D – E + F = – 50 (3)

Al resolver este sistema se obtiene D = – 6, E = – 4 y F = – 12. Al sustituir estos valores en la


ecuación general de la circunferencia se llega a la misma ecuación obtenida anteriormente:
x2 + y2 – 6x – 4y – 12 = 0

21
GEOMETRÍA ANALÍTICA

EJEMPLO 7
Encontrar la ecuación de la circunferencia que tiene su centro sobre el eje X y pasa por los puntos
(0, – 2) y (2, 4)

SOLUCIÓN: Dado que el centro está sobre el eje X, el valor de k = 0, luego el centro tiene coordenadas
(h, 0). Tenemos que hallar el valor de h y el radio r.
La distancia del centro a cualquier punto de la circunferencia es el radio, luego
r = (h− 2) 2 + (0 − 4) 2 = (h− 0) 2 + (0 + 2) 2 (1)

Eliminando los radicales y desarrollando cada expresión obtenemos


h2 – 4h + 4 +16 = h2 + 4 ⇒ – 4h + 16 = 0 ∴ h = 4. Luego el Centro es (4, 0)
Introduciendo el valor de h en la expresión (1) hallamos r = 20
2 2 2
La ecuación buscada es (x – 4) + y = ( 20 ) o sea x + y2 – 8x – 4 = 0
2

EJEMPLO 8

Obtener la longitud de la circunferencia y el área del círculo dada por x2 + y2 – 3x + 7y – 14 = 0

Solución: Recordemos de la Geometría que la longitud de la circunferencia está dada por L = 2π r y el


área del círculo por A = π r2, luego basta determinar el radio y aplicar dichas fórmulas.
D2 E2
El radio podemos hallarlo en forma directa aplicando la fórmula r = + − F , donde D, E y F son
4 4
los coeficientes en la ecuación general x2 + y2+ Dx + Ey +F = 0. En el caso particular de la ecuación dada
tenemos D = – 3, E = 7 y F = – 14, luego
(−3) 2 7 2 9 49 57
r= + − (−14) = + + 14 = = 5.3385
4 4 4 4 2

Otra forma de obtener el radio es llevar la ecuación a la forma: (x – h)2 + (y – k)2 = r2 e identificar aquí
el valor de r. Obviamente se obtiene el mismo valor.

Aplicando las respectivas fórmulas resulta: L = 2 π (5.3385) = 33.543 u y A = π (5.3385)2 = 89.53 u2

EJEMPLO 9
Encontrar una ecuación de la recta tangente a la circunferencia x2 + y2 – 4x+ 6y – 12 = 0
en el punto (5,1).
Y
SOLUCIÓN:
La pendiente de la ecuación buscada está P (5, 1)
−1
dada por mL = , donde C es el centro de la
m CP X
Circunferencia y P es el punto dado.
C (2, – 3) L

De la ecuación de la circunferencia deducimos las Coordenadas del centro al llevarla a la forma


(x − h)2 + (y− k )2 = r 2

22
GEOMETRÍA ANALÍTICA

(x2 – 4x ) + (y2 + 6y ) = 12
(x2 – 4x + 4 ) + (y2+ 6y + 9) = 12 + 4 + 9 ⇒ (x – 2)2 + (y + 3)2 = 25

luego el centro es C(2, – 3) y su radio r = 5.

y P − y C 1 − (−3) 4 −1 −3
Resulta mCP = = = y la pendiente de la recta es mL = =
xP − xC 5−2 3 4/3 4
La ecuación de la recta buscada es y – yo = m (x – xo); tomando (xo , yo) = (5 , 1) al sustituir los datos
obtenemos: y – 1 = (– 3/4) (x – 5)
4 (y – 1) = – 3 (x – 5) ⇒ 4y – 4 = – 3x +15 o sea 3x + 4y –19 = 0
.
EJEMPLO 10
Encontrar una ecuación de cada una de las dos rectas que tienen pendiente m = – 4/3 que son
tangentes a la circunferencia con ecuación x2 + y2 + 2x – 8y – 8 = 0
Y
SOLUCIÓN:
Dado que se conoce la pendiente de
P1
Las rectas buscadas, sólo falta encontrar las
coordenadas de los puntos de tangencia P1 y P2
Para ello encontramos primero la ecuación de la X
Recta que contiene el diámetro P1P2 y luego
buscamos los puntos donde esta recta corta a la
circunferencia. P2

Para hallar la ecuación de esta recta buscamos el centro de la circunferencia y consideramos que su
−1 3
pendiente es m = = por ser perpendicular a las rectas buscadas.
− 4/3 4
Al reacomodar la ecuación de la circunferencia se tiene:
(x2 + 2x ) + (y2 – 8y ) = 8 ⇒ (x2 +2x +1 ) + (y2 – 8y +16 ) = 8 + 1 +16 = 25
⇒ (x + 1)2 + (y – 4)2 = 52 (1)

Luego la circunferencia tiene centro (–1, 4) y la ecuación del diámetro que une los puntos de tangencia
3 3 19
es y – 4 = (x + 1) o sea y = x + (2)
4 4 4

Sustituyendo esta expresión en la ecuación de la circunferencia (1) se tiene:


3 19 3 3 2
(x + 1)2 + [( x + ) – 4]2 = 25 ⇒ (x + 1)2 +( x + ) = 25
4 4 4 4
9 25
(x + 1)2 + (x + 1)2 = 25 ⇒ (x + 1)2 = 25 ⇒ x + 1 = ± 4 ∴ x=3 ó x=–5
16 16
3 ⋅ (3) 19 3 ⋅ (−5) 19
para x = 3 en (2) obtenemos y = + = 7 y para x = – 5 se obtiene y = + =1
4 4 4 4
por tanto los puntos de tangencia de las rectas tangentes son (3, 7) y (– 5, 1) y las ecuaciones de las
rectas buscadas son:
L1 : Po (3, 7) y m = – 4/3 ⇒ y – 7 = (–4/3) (x – 3) o sea 4x + 3y – 33 = 0

L2 : Po (– 5, 1) y m = – 4/3 ⇒ y – 1 = (– 4/3) (x + 5) o sea 4x + 3y + 17 = 0

23
GEOMETRÍA ANALÍTICA

EJERCICIOS PROPUESTOS

1. Determine el centro y el radio de las siguientes circunferencias


a) x2 + y2 = 36 b) (x – 3)2 + (y – 5)2 = 144 c) (x + 5)2 + (y – 6)2 = 8
2 2 2 2
d) (x + 3) + (y + 8) = 81 e) (x + 4) + (y – 5) = 225

2. Determine cuales de las siguientes ecuaciones definen circunferencias. En caso afirmativo encuentre
el centro y el radio.
1
a) x2 + y2 + 6x + 8y = 0 b) x2 + y2 – 5 x – 4y + 10 = 0 c) x2 + y2 + 3x – 5y + =0
2
d) x2 + y2 – 4x – 6y + 12= 0 e) x2 + y2 + 6x + 2 = 0 f) x2 + y2 + 10x + 2y + 26 = 0
2 2 2 2
g) x + y + 10x – y + 29 = 0 h) 2x + 2y + 2x + 10y – 33 = 0
i) 5x2 + 5y2 – 6x – 2y + 17 = 0 j) x2 + y2 – 6x + 2y – 15 = 0

3. Encuentre la ecuación de la circunferencia que satisfaga las siguientes condiciones


a) C (3, – 5), r = 6 b) C (2, – 1), r = 8 c) C (– 6, 4), r = 3
d) C (2, – 1), pasa por (4, 0) e) C (5, – 2), pasa por (9, – 2) f) C (– 1, 2), pasa por (3, – 5)
g) C (3, – 1), pasa por (6, 3) h) C (3, – 2) y es tangente al eje X i) C (– 3, 5) y es tangente al eje Y
j) r = 2 y es tangente a ambos ejes en el I Cuadrante
k) r = 5 y es tangente a ambos ejes en el II Cuadrante
l) r = 4 y es tangente a ambos ejes en el III C m) r = 6 y es tangente a ambos ejes en el IV C
n) tiene centro en (1, 0) y es tangente a la recta 4x – y + 2 = 0
ñ) tiene como diámetro el segmento que une (5, – 3) con (– 1, – 7)
o) diámetro AB, con A (– 1, 3) y B (5, – 5) p) C (– 2, 3), tangente a x – 2y – 3 = 0
q) Pasa por (– 2, 3), (4, 1) y su centro está sobre la recta x – 2y + 2 = 0
r) Pasa por los puntos (1, – 1), (– 3, 5) y (0, 2) s) Pasa por (9, 8) y es tangente a ambos ejes

4. Hallar la ecuación de la circunferencia concéntrica con x2 + y2 + 6x + 2 = 0 y tangente a la recta


2x + y + 1 = 0.

5. Hallar las ecuaciones de las circunferencias que tienen su centro en la recta 4x – 5y + 1 = 0 y son
tangentes a las dos rectas 2x – 3y – 8 = 0 y 3x – 2y + 8 = 0.

6. Determine los puntos donde la circunferencia con ecuación x2 + y2 – 6 x – 7 = 0 corta al eje X

7. Determine donde se cortan la recta y = x – 1 y la circunferencia x2 + y2 – x – 3 = 0

8. Comprobar que la recta x – y – 2 = 0 es secante a la circunferencia x2 + y2 = 20 y hallar la longitud de


la cuerda correspondiente.

9. Hallar la ecuación de la recta tangente a la circunferencia x2 + y2 – 6x + 2y – 15 = 0 en el punto (6, 3)

10. Comprobar que los puntos (0, 1), (1, 0) y (3, 4) no son colineales. Hallar la ecuación de la
circunferencia que pasa por ellos.

11. Sea C la circunferencia (x – x0)2 + (y – y0)2 = r2 ( r > 0). Demostrar que esta ecuación equivale al
sistema de ecuaciones paramétricas S: x = x0 + r cos t, y = y0 + r sen t, donde t recorre el intervalo
[0, 2π].

12. Un punto se mueve de manera que su distancia al punto (2, 4) es el doble de su distancia al origen.
Encuentre la ecuación que describe su trayectoria.

13. Un punto se mueve de manera que el segmento que lo une con (2, 0), es siempre perpendicular al
segmento que lo une con (–2, 0). Encuentre la ecuación que describe su trayectoria.

24
GEOMETRÍA ANALÍTICA

SOLUCIONES
1. a) C (0, 0), r = 6 b) C (3, 5), r = 12 c) C (– 5, 6), r = 2 2 d) C (– 3, – 8), r = 9
5 1 3 5
e) C (– 4, 5), r = 15 2. a) C (–3, – 4), r = 5 b) C ( , 2), r = c) C ( − , ), r = 2 2
2 2 2 2
d) (2, 3), r = 1 e) (–3, 0), r = 7 f) No es circunferencia, P (– 5, – 1) g) No
⎛ 1 5⎞
h) ⎜ − ,− ⎟, r = 23 i) No. j) C (3, – 1), r = 5
⎝ 2 2⎠
3. a) x2 + y2 – 6x + 10y – 2 = 0 b) x2 + y2 – 4x + 2y – 59 = 0 c) x2 + y2 + 12x – 8y + 43 = 0
d) x2 + y2 – 4x + 2y = 0 e) x2 + y2 – 10x + 4y + 13 = 0 f) x2 + y2 + 2x – 4y – 60 = 0
g) x2 + y2 – 6x + 2y – 15 = 0 h) (x – 3)2 + (y + 2)2 = 4 i) (x + 3)2 + (y – 5)2 = 9
j) (x – 2)2 + (y – 2)2 = 4 k) (x + 5)2 + (y – 5)2 = 25 l) (x + 4)2 + (y + 4)2 = 16
m) (x – 6) + (y + 6) = 36 n) x + y – 2x – 19/17 = 0 ñ) x2 + y2 – 4x + 10y – 23 = 0
2 2 2 2

o) x2 + y2 – 4x + 2y – 20 = 0 p) 5x2 + 5y2 + 20x – 30y – 56 = 0


q) 5x2 + 5y2 – 8x – 14y – 39 = 0 r) x2 + y2 + 14x + 4y – 12 = 0 s) (x – 5)2 + (y – 5)2 = 25 ó
2 2
(x – 29) + (y – 29) = 841
25 81
4. x2 + y2 + 6x + 4 = 0 5. (x + 9 )2 + (y + 7 )2 = ∧ (x − 1)2 + (y − 1)2 =
13 13
1 3
6. (7, 0), (– 1, 0) 7. (2, 1), ( − , − ) 8. 6 2 9. 3x + 4y – 30 = 0
2 2
2 2
⎛ 2⎞ ⎛ 4⎞ 80
10. (x – 2)2 + (y – 2)2 = 5 12. ⎜ x + ⎟ + ⎜ y + ⎟ = 13. x2 + y2 = 4
⎝ 3⎠ ⎝ 3⎠ 9

6.4.2. PARÁBOLA

DEFINICIÓN: una parábola es el conjunto de puntos P situados en un plano que equidistan de un punto
fijo F, llamado foco, y una recta fija L, llamada directriz. Es decir los puntos que satisfacen la expresión
d (P, L) = d (P, F).

◊ A la línea perpendicular a la directriz que pasa por el foco


se le llama eje de la parábola. La parábola es simétrica Directriz
respecto a su eje. Lado Recto

◊ El punto medio entre la directriz y el foco se le llama vértice V


de la parábola.
F
◊ A la cuerda paralela a la directriz y que pasa por el foco se Eje
le llama lado recto
d (P, F)
◊ En general, para todas las cónicas, la razón se
d (P, L)
ECUACIONES
llama excentricidad y seDE por e. La parábola se
LA PARÁBOLA
denota
caracteriza porque siempre e = 1

ECUACIÓN DE LA PARÁBOLA CON VÉRTICE EN EL ORIGEN Y FOCO SOBRE EL EJE Y

Sea F (0, p) las coordenadas del foco. Dado que el vértice está en el origen, V (0, 0); el eje de simetría
es el eje Y, y la ecuación de la recta directriz es y = – p o sea y + p = 0.
Según la definición de la parábola, un punto P(x, y) está en la parábola, si y solo si, equidista del foco y
la directriz:
d (P, F) = d (P, L).

25
GEOMETRÍA ANALÍTICA

Sea Q (x, – p) el pie de la perpendicular que va de P a la directriz L. Luego


d (P, F) = x 2 + (y− p)2 ( distancia entre dos puntos)
d (P, L) = | y + p | (distancia de un punto a una recta

considerando que L : y + p = 0)
P(x, y)
2 2 F (0, p)
Luego x + (y− p) = | y + p |

Elevando al cuadrado ambos lados y simplificando V (0, 0)


Q (x, – p) y=–p
x2 + (y2– 2yp + p2) = y2 + 2yp + p2 ⇒ x2 – 2yp = 2yp ⇒ x2 = 4py
1 2
lo que también equivale a y= x
4p

TEOREMA: La ecuación de una parábola que tiene su foco en F (0, p) y su directriz es la recta
y = – p (y + p = 0) está dada por x2 = 4 p y.

* Si p > 0, la gráfica de la parábola se abre hacia arriba.


* Si p < 0, la gráfica de la parábola se abre hacia abajo.

ECUACIÓN DE LA PARÁBOLA CON VÉRTICE EN EL ORIGEN Y FOCO SOBRE EL EJE X

De manera similar, si intercambiamos los ejes, el foco será F (p, 0) y la directriz L: x + p = 0


Al aplicar la definición se obtiene la ecuación y2 = 4px

* Si p > 0, la gráfica de la parábola se abre hacia la derecha.


* Si p < 0, la gráfica de la parábola se abre hacia la izquierda.

En cada caso la longitud del lado recto, denotada por LR, es decir la cuerda que pasa por el foco
perpendicular al eje, está dada por LR = | 4p |.

Y
Y
LR LR D: x + p = 0

• • • •
(– p, 0) F (p, 0) X F(p, 0) (–p, 0) X
D: x + p = 0

p>0 p<0

26
GEOMETRÍA ANALÍTICA

TRASLACIÓN DE EJES
Y Y’
• P (x,y) ≡ P’ (x’,y’)
y’
y (h,k) ≡ (0,0)

x’ X’
k
h
x X

TEOREMA; Si (x, y) representa las coordenadas de un punto P respecto a un sistema dado de ejes XY
y P’ (x’, y’) representa a P después que los ejes son trasladados (paralelamente) a un nuevo origen con
coordenadas (h, k) con respecto al sistema dado de ejes, entonces
x ’ = x – h y y ’ = y – k.

En general , si se tiene la ecuación de una curva respecto a un sistema de coordenadas XY , entonces la


ecuación de la misma curva respecto al sistema trasladado X’Y’ se obtiene mediante la sustitución de x
por (x’ + h) y y por (y’ + k).

La gráfica en ambos sistemas corresponde al mismo conjunto de puntos, aunque la ecuación difiera.

6.3.4. ECUACIONES DE LAS PARÁBOLAS CON EJES PARALELOS A UNO DE LOS EJES
COORDENADOS

Si el vértice se “traslada” al punto (h, k), las ecuaciones toman la forma

(x – h)2 = 4p (y – k) si el eje de simetría es paralelo al eje Y


(y – k)2 = 4p (x – h) si el eje de simetría es paralelo al eje X

Es decir ecuaciones de la forma y = ax2 + bx + c y x = Ay2 + By + C

Un resumen de las características de las parábolas a partir de su ecuación se muestra a continuación:

Eje de La gráfica se abre


Ecuación Vértice Foco Directriz Simetría hacia...
Eje Y arriba si p>0
x2 = 4py (0,0) (0, p) y+p=0 ( x = 0) abajo si p<0
Eje X la derecha si p>0
y2 = 4px (0,0) (p, 0) x+p=0 ( y = 0) la izquierda si p<0
Paralelo al eje Y arriba si p>0
(x-h)2 = 4p(y-k) (h,k) (h, k+p) y=k-p x=h abajo si p<0
la derecha si p>0
(y-k)2 = 4p(x-h) (h,k) (h+p, k) x=h-p Paralelo al eje X la izquierda si p<0
y=k

En cada caso hemos de considerar los signos de h, k y p

La fórmula para el lado recto es válida en general LR = | 4p |

La expresión (x – h)2 = 4p (y – k) puede desarrollarse obteniéndose y = ax2 + bx +c

27
GEOMETRÍA ANALÍTICA

−b
La abscisa del vértice está dada por x = h = y la ordenada se obtiene al sustituir la x por este valor
2a
b2
en la ecuación, obteniéndose y = k = c – .
4a

De igual manera (y – k)2 = 4p(x – h) puede desarrollarse obteniéndose x = A y2 + B y + C.


−B
En este caso la ordenada del vértice está dada por y = k =
2A
EJEMPLOS
Dadas las siguientes parábolas determine su foco, directriz y longitud del lado recto. Bosqueje su gráfica.

1. x2 = 4y
Por la forma de la ecuación deducimos que corresponde a una Y
parábola con vértice en el origen y con eje de simetría , el eje Y.
Tenemos 4p = 4 ∴ p = 1, luego su foco es F (0,1).
La longitud del lado recto está dada por LR = | 4p | = 4
Su directriz está dada por la recta y = – p ∴ y = –1 • (4, 4)

2. y2 = 6x F(0, 1) • • (2, 1) X
De la ecuación deducimos que es una parábola con vértice en el
(0, 0)
origen, con el eje X como eje de simetría. Se tiene 4p = 6 ∴
p = 3/2, luego su foco es F (3/2, 0). Su lado recto mide LR = 6
Su directriz es x = – p o sea x = – 3/2

Y
(6, 6)

• 3
F( , 0) X Y
2

(6,– 6)
2 • X
3. y = – 8x
F (– 2, 0)
Similar a la anterior, 4p = – 8, luego p = – 2 y el foco es F (–2,0).
Su lado recto LR = | 4p | = 8
Su directriz es x = – p o sea x = – (– 2) = 2

4. 3x2 + 4y = 0
4 4 1 1
Esta ecuación equivale a x2 = − y, luego 4p = − ∴ p = − ; su foco F(0, p) = (0, − )
3 3 3 3
4 1
Su lado recto mide LR = | 4p | = y su directriz tiene ecuación y =
3 3
Y

X

F(0,– 1/3)

28
GEOMETRÍA ANALÍTICA

5. y2 – 12 = 12x
Tenemos y2 = 12x + 12 = 12 (x + 1) la cual es de la forma (y– k)2 = 4p(x– h). Identificamos
h = – 1, k = 0 y 4p = 12 ∴ p = 3.
Por tanto tenemos una parábola con vértice V (–1,0) , eje de Y
simetría paralelo al eje X.
Su foco es F(h + p, k) = (–1+3, 0) = (2,0). Su lado recto LR = | 4p | =
x=–4 X
12
Su directriz x = h – p = –1– 3 = – 4 V ( – 1, 0)• • F (2, 0)

6. y = x2 – 4x +2

La llevamos a la forma (x– h)2 = 4p (y– k). Para ello completamos un trinomio cuadrado perfecto con x.
y = (x2 – 4x ) + 2 = (x2 – 4x + 4 ) + 2 – 4 = ( x – 2)2 – 2 (x – 2)2 = y + 2
1
Luego h = 2, k = – 2, 4p = 1 ∴ p =
4
1 7
Vértice: V (h, k) = (2, –2) Foco: F (h, k + p) = (2, – 2 + ) = (2, − )
4 4
1 9
Lado recto: LR = | 4p| = 1 Directriz: y = k – p = – 2 – = −
4 4

7. y2 – 4y – 2x – 4 = 0

Separamos las variables y completamos el trinomio cuadrado perfecto para y.


y2 – 4 y = 2x + 4 ⇒ y2 – 4 y + 4 = 2x + 4 + 4 ⇒ (y – 2)2 = 2x + 8 = 2 (x + 4)
1
Identificamos h = – 4 , k = 2 , 4p = 2 ∴ p =
2
1 7
Luego Vértice : V (h, k) = (– 4, 2) Foco : F (h + p, k) = (– 4+ , 2) = ( − , 2)
2 2
1 9
Lado recto: LR = | 4p| = 2 Directriz: x = h – p = – 4 – = −
2 2

F (–3.5, 2)
V ( – 4, 2)• •

8. 4x2 + 40x + y + 106 = 0

De manera similar a la anterior, separamos las variables. Pero antes hay que extraer como factor común
el coeficiente de la x2. No hay que perder de vista de que el objetivo es transformar la ecuación en la
forma (x– h)2 = 4p (y– k).
4x2 + 40x = – y –106 ⇒ 4 ( x2 + 10x ) = – y – 106
1
4 (x2 + 10x + 25 ) = – y –106 + 100 ⇒ 4 (x + 5)2 = – y – 6 = – (y + 6) ∴ (x + 5)2 = – (y + 6)
4
Vemos que corresponde a una parábola con eje de simetría paralelo al eje Y, cóncava hacia abajo.
1 1
Identificamos h = – 5, k = – 6 4p = – ∴ p=– . Vértice: V (h, k) = (– 5, – 6)
4 16
1 97 1 95
Foco: F (h, k + p) = (– 5, – 6 – ) = (– 5, – ) Directriz: y = k – p = – 6 – (– )=–
16 16 16 16

29
GEOMETRÍA ANALÍTICA

9. Encuentre la longitud de la cuerda que determina la recta x – 2y + 3 = 0 en la parábola


con ecuación y2 = 4x.

SOLUCIÓN: Buscamos lo puntos de intersección de la recta con la parábola, es decir resolvemos el


⎧ x − 2y + 3 = 0 (1)
sistema de ecuaciones ⎨ 2
⎩ y = 4x (2)
De (1) tenemos x = 2y – 3. Sustituyendo en (2) y2 = 4 (2y – 3) = 8y – 12
y2 – 8y + 12 = 0 ⇒ (y – 2) (y – 6) = 0 ∴ y = 2 ∨ y = 6
Para y = 2 se obtiene x = 1 y para y = 6, x = 9, es decir los puntos de intersección son (1, 2) y
(9, 6). Aplicando la fórmula para la distancia entre dos puntos resulta:
d= (9 − 1) 2 + (6 − 2) 2 = 64 + 16 = 4 5

10. Hallar la longitud de la cuerda focal de la parábola y2 + 8x = 0 que es paralela a la recta


4x + 3y – 7 = 0.

SOLUCIÓN: Una cuerda focal es aquella que pasa por el foco. Necesitamos entonces determinar las
coordenadas del foco, hallar la ecuación de la recta que pasa por él, paralela a la recta dada, luego
encontrar los puntos donde esta recta corta a la parábola y finalmente hallar la distancia entre dichos
puntos
De la ecuación de la parábola deducimos que el foco está sobre el eje X.
Tenemos: y2 = – 8x ∴ p = – 2 , luego las coordenadas del foco son F (– 2, 0)
4
De la ecuación de la recta deducimos que su pendiente es m = − , luego la ecuación de la recta
3
4
paralela a ella que pasa por el foco es: y = − (x + 2 ) o sea 4x + 3y + 8 = 0
3
3
Despejamos x y la introducimos en la ecuación de la parábola: x = − y− 2
4
3
y = – 8 ( − y− 2 ) ⇒ y – 6y – 16 = 0 ⇒ (y – 8) (y + 2) = 0 ∴ y = 8 ∨ y = – 2
2 2
4
1 1
y=8⇒x=–8 ∧ y=–2 ⇒x= − ∴ (– 8, 8) y ( − , – 2) son los extremos de la cuerda.
2 2
1 625 25
Su longitud es d = (−8 + ) 2 + (8 + 2) 2 = =
2 4 2

EJERCICIOS

1. Encuentre el foco y la directriz de las siguientes parábolas. En cada caso bosqueje la gráfica
a) y2 = 12x b) x2 = 4y c) y2 = – 6x d) x2 = – 8y
2 2 2
e) x = 10y f) y = 4x g) y = – 6x h) 3x2 – 5y = 0
2 2 2
i) 9x – 2y = 0 j) 4y – x = 0 k) x + 16y = 0 l) x2 + 8y = 0

2. Encuentre el vértice, el foco, la directriz, la longitud del lado recto. Bosqueje la gráfica
a) (y – 3)2 + 4 = 2x b) (y – 4)2 = 4(x + 5) c) (x – 3)2 = 12(y + 4)
2 2
d) (x + 5) = 8 (y – 1) e) x – 4 = 8 (y – 3) f) x2 – 6x – 6y – 3 = 0
2 2
g) y – 4x – 4y – 12 = 0 h) x + 6x + 4y + 8 = 0 i) y2 + 6x + 10y + 19 = 0
2 2
j) 2y = 4y – 3x k) x – 24y – 6x + 9 = 0 l) x2 + 6x + 8y + 1 = 0

3. Encuentre la ecuación de la parábola que satisfaga las siguientes condiciones:

30
GEOMETRÍA ANALÍTICA

3
a) F (– 4, 4) y directriz x + 6 = 0 b) F (– 5, 3) y directriz y + 1 = 0 c) V (2, 3) y directriz x =
2
95 119
d) V (3, 4) y directriz x = e) F (3, – 1) y V (3, – 4) f) V (– 2, – 3) y F (– 2, – )
32 40
1
g) V (3, – 4) , pasa por (9, – 1) y eje paralelo al eje Y h). V (3, – 2) , pasa por (6, − ) y eje paralelo al
2
eje Y i) Pasa por los puntos (0, 0) , (8, – 4) y (3, 1) con eje de simetría paralelo al eje X

4. En cálculo se prueba que un rayo que emane del foco de una parábola al chocar con la parábola es
reflejado en una línea paralela al eje principal y viceversa, un rayo paralelo al eje principal al ser reflejado
pasa por el foco. Esta propiedad es aprovechada en las antenas parabólicas. Suponga que un rayo de
luz sale del foco de la parábola y2 = 8x y choca en el punto (2, 4) ¿Cuál es la ecuación del rayo
reflejado?
5. Bajo ciertas circunstancias, un cable que cuelga entre dos postes, puede ser aproximado por una
parábola. Asumiendo que un cable toma la forma de una parábola, encuentre su ecuación, si un punto a
10 pies horizontalmente desde el punto más bajo está un pie más arriba. (Sugerencia: tome el punto más
bajo en el origen.
6. El calor generado H (medido en watts) en una resistencia R (medida en ohms) de un circuito eléctrico
está dado por H = R i2 , donde i es la corriente ( medida en amperios) . Grafique H versus i si R = 6
ohms.
7. Durante un batazo, la posición vertical de una pelota está dada por y = 120 t – 16 t2, y su posición
horizontal por x = 60 t . Eliminando t en ambas ecuaciones, muestre que la trayectoria es una parábola.
Bosqueje la gráfica de esta parábola hasta que la pelota regresa al nivel del suelo. Si la barda del center
fielder está a 420 pies del home, y tiene una altura de 8 pies, diga si el batazo es jonrón.

8. Sea la curva y2 = 4px, con p ≠ 0. Se traza una recta perpendicular al eje X que corta a la curva en los
→ →
puntos P y Q de modo que PQ = 8 |p|. Probar que los vectores OP y OQ son ortogonales, siendo O el
origen del sistema.

SOLUCIONES
3 3
1. a) (3, 0), x = – 3 b) (0, 1), y = – 1 c) ( − , 0), x = d) (0, – 2), y = 2
2 2
1 1 1 1 1 1 5 5
e) ( , 0), x = − f) (0, ), y = − g) (0, − ), y = h) (0, ), y = –
40 40 16 16 24 24 12 12
9 9 1 1 1 1 5
i) ( , 0), x = – j) ( , 0) , x = – k) ( − , 0), x = l) (0, – 2), y = 2 2. a) V (2, 3), F ( , 3),
8 8 16 16 64 64 2
3
x= , LR = 2 b) V (– 5, 4), F (– 4, 4), x + 6 = 0, LR = 4 c) V (3, – 4), F (3, – 1), y + 7 = 0, LR = 12
2
129 127 1 1
d) V (– 5, 1), F (– 5, 3), y + 1 = 0, LR = 8 e) V (4, 3), F ( , 3), x = , LR = f) V (3, – 2), F (3, − ),
32 32 8 2
7 1 3 5
y = − , LR = 6 g) V (– 4, 2), F (– 3, 2), x + 5 = 0, LR = 4 h) V (– 3, ), F (– 3, − ), y = , LR = 4 i)
2 4 4 4
1 5 2 7 25 3
V (1, – 5), F ( − , – 5), x = , LR = 6 j) V ( , 1) F ( , 1), x = , LR =
2 2 3 24 24 2
k) V (3, 0), F (3, 6), y + 6 = 0, LR = 24 l) V (– 3, 1), F (– 3, – 1), y = 3, LR = 8

3. a) (y – 4)2 = 4 (x + 5) b) (x + 5)2 = 8 (y – 1) c) (y – 3)2 = 2 (x – 2)


d) x – 3 = 8 (y – 4)2 e) (x – 3)2 = 12 (y + 4) f) y + 3 = 10 (x + 2)2
g) (x – 3)2 = 12(y + 4) h) (x – 3)2 = 6 (y + 2) i) x = y2 + 2y 4. y = 4 5. x2 = 100y 7. a) y =
1
225 − (x – 225)2 b) Si es jonrón, al pasar la barda lleva una altura de 56 pies.
225

31
GEOMETRÍA ANALÍTICA

6.4.3. ELIPSE
DEFINICIÓN: Una Elipse es el conjunto de puntos P del plano tales que la suma de las distancias entre
P y dos puntos fijos F1 y F2, llamados focos, es constante.

d ( P , F1 ) + d (p , F2) = K (constante)

B2 (0, b)
LR

directriz directriz

V1 (– a, 0) F1 (– c, 0) C (0, 0) F2 (a, 0) V2 ( a, 0)
Eje menor

B1 (0,– b)

Eje mayor

ELEMENTOS DE LA ELIPSE

◊ El punto medio del segmento F1F2, se llama CENTRO de la Elipse

◊ El segmento que pasa por los focos y sus puntos extremos están en la Elipse, se llama EJE MAYOR

◊ El segmento que pasa por el centro y es perpendicular al eje mayor, y con sus extremos en la Elipse,
se llama EJE MENOR

◊ Los extremos de los ejes se llaman VÉRTICES


◊ El centro también es el punto medio de los ejes mayor y menor
◊ La cuerda que pasa por un foco, perpendicular al eje mayor se llama LADO RECTO

◊ Asociadas a cada elipse, hay dos rectas perpendiculares al eje mayor, llamadas DIRECTRICES, una
para cada foco.

En general para cada cónica, si L es una directriz, F es el foco asociado a dicha directriz, la razón
d ( P, F)
= e , se llama EXCENTRICIDAD. Una Elipse se caracteriza porque 0 < e < 1.
d ( P, L)
Es decir la distancia desde un punto de la Elipse al foco es menor que la distancia desde ese punto a la
directriz correspondiente.

32
GEOMETRÍA ANALÍTICA

ECUACIONES DE LA ELIPSE

ELIPSE “HORIZONTAL”
Si colocamos el centro de la Elipse en el origen y los focos sobre el eje X, con coordenadas ( ± c, 0) y
tomando , por conveniencia, d1 + d2 = 2a , tenemos

Si el punto P(x, y) está en la Elipse


d (P, F1) + d (P,F2) = 2a
Aplicando la fórmula para la distancia entre P (x, y)
dos puntos :
(x + c) 2 + y 2 + (x − c) 2 + y 2 = 2a b d1
d2
Después de eliminar los radicales
y simplificar se obtiene
V1 (– a, 0) F1 (– c, 0) F2 (c, 0) V2 (a, 0)
x2 y2
+ =1 , a > c c
a2 a2 − c2
Haciendo b 2 = a 2 − c 2 , podemos escribir
a
x2 y2
+ = 1 con a > b
a2 b2

x2 y2
La ecuación + = 1 representa entonces una Elipse con centro en el origen y focos (± c, 0) ,
a2 b2
donde c = a 2 − b 2 . Generalmente se le llama Elipse Horizontal
◊ Los vértices tienen coordenadas (± a, 0) que corresponden a los extremos del eje mayor y
(0, ± b) que corresponden a los extremos del eje menor.
◊ La longitud del eje mayor está dada por 2a. El valor de a es conocido como semieje mayor.
◊ La longitud del eje menor está dada por 2b. El valor de b es conocido como semieje menor.
◊ La distancia interfocal F1F2 está dada por 2c
c
◊ Se puede probar que la excentricidad está dada por e = y las ecuaciones de las directrices, para
a
a
este caso son x = ±
e

ELIPSE “VERTICAL”

De manera similar, si los focos se colocan sobre el eje Y, con coordenadas (0, ± c) resulta la ecuación
x2 y2
+ = 1 , con a > b. A esta Elipse se le llama Elipse Vertical.
b2 a2

En este caso los extremos del eje mayor son (0, ± a) y los extremos del eje menor son (± b, 0).
La longitud del eje mayor, la longitud del eje menor, y la distancia interfocal, también están dadas por
2a, 2b y 2c respectivamente.
a
En este caso puede probarse que las directrices están dadas por y = ± y la excentricidad sigue
e
c
siendo e = .
a
Si el centro se traslada al punto (h, k) las ecuaciones se transforman en

33
GEOMETRÍA ANALÍTICA

(x − h) 2 (y− k) 2
+ = 1 cuando el eje mayor es paralelo al eje X
a2 b2
(x − h) 2 (y− k) 2
+ = 1 cuando el eje mayor es paralelo al eje Y
b2 a2

Las coordenadas de los focos y vértices también resultan “transformadas”. El siguiente cuadro presenta
un resumen de cada situación. En todos los casos se cumple la relación b 2 = a 2 − c 2 , (a2 = b2 + c2),
2 b2 c
LR = y e= .
a a

Extremos Extremos
Ecuación Centro Focos del del Directrices
eje mayor eje menor
x2 y2 x= ±
a
+ =1 (0 , 0) (± c, 0) ( ± a, 0) (0, ± b)
a2 b2 e

x2 y2 y=±
a
+ =1 (0 , 0) (0, ± c) (0, ± a) (± b, 0) e
b2 a2
( x − h)2 ( y − k )2 x=h±
a
+ =1 (h, k) (h ± c, k) ( h ± a, k) (h, k ± b)
a 2
b 2 e
2
( x − h) ( y − k )2 y=k±
a
+ =1 (h, k) (h, k ± c) (h, k ± a) ( h ± b, k)
b 2
a 2 e

EJEMPLOS

1. Dada la cónica con ecuación 9x2– 36x + 25y2 + 150y + 36 = 0, determine las coordenadas de su
centro, vértices y focos
SOLUCIÓN:
Rescribimos la ecuación agrupando los términos en x y los términos en y. Luego factorizamos
asegurando coeficiente 1 para x2 y y2.
(9x2 – 36x ) + (25y2 + 150y) = – 36 ⇒ 9 (x2 – 4x ) + 25 (y2 + 6y ) = – 36

Completamos el trinomio cuadrado perfecto dentro de cada paréntesis. Recordemos que el término que
falta en este caso es el cuadrado de la mitad del coeficiente del término lineal.
Para no alterar la igualdad hemos de sumar al miembro de la derecha esta misma cantidad multiplicada
por el factor que anteriormente extrajimos de cada paréntesis.
9 (x2– 4x + 4) + 25 (y2+ 6y + 9) = – 36 + (9) (4)+ (25) (9) = – 36 + 36 + 225 = 225

Factorizamos las expresiones dentro de cada paréntesis


9(x – 2)2 +25 (y + 3)2 = 225

Dividimos cada término entre el valor obtenido a la izquierda


(x − 2) 2 (y+ 3) 2
+ =1
25 9
Reconocemos que es una Elipse e identificamos los valores de h, k, a y b.
Vemos que h = 2, k = – 3, luego el centro de la Elipse se localiza en C(2, – 3)
En la ecuación canónica de una Elipse, el mayor de los denominadores corresponde al valor de a2 y el
menor a b2. En este caso a2 = 25 ⇒ a = 5 ; b2 = 9 ⇒ b = 3.

34
GEOMETRÍA ANALÍTICA

Tenemos que c = a 2 − b 2 , luego c = 25 − 9 = 16 = 4


Identificamos la disposición del eje mayor: en este ejemplo, dado que el mayor denominador aparece
bajo el término en x, el eje mayor es horizontal.

Resulta : Extremos del eje mayor ( h± a, k) : (2 – 5, – 3) = ( – 3, – 3) y (2 + 5, –3) = (7, –3)


Extremos del eje menor (h, k± b) : (2, –3 –3) = (2, – 6) y (2, – 3 + 3) = (2, 0)
Focos (h± c, k) : (2 – 4, – 3) = ( –2, –3) y (2 + 4, –3) = (6, –3)
Y
B2 (2, 0) X

F1 (– 2, – 3) F2 (6, – 3)
• • • • •
V1(– 3, – 3) C (2, – 3) V2 (7, – 3)

2. Encuentre la ecuación de la Elipse que tiene un foco en F (4, – 3), y como directriz la recta
1
2x – y – 2 = 0 y excentricidad e =
2
SOLUCIÓN
PF
Tenemos e = ∴ PF = e ⋅ PD (1)
PD
| 2 x − y− 2 | | 2 x − y− 2 |
PF = (x − 4) 2 + ( y+ 3) 2 PD = =
22 + (−1)2 5

1 ( 2 x − y− 2 ) 2
Sustituyendo en (1) y elevando al cuadrado: ⋅
(x – 4)2 + (y + 3)2 = .
4 5
2 2
Desarrollando y simplificando se obtiene: 16x + 19y + 4xy – 152x + 116y + 496 = 0

3. Un arco de un paso a desnivel, tiene forma semielíptica con el eje mayor horizontal. La base del arco
mide 30 m. y la parte más alta está a 10 m. por encima de una carretera que pasa por abajo del puente.
Calcule la altura del arco sobre el punto del piso que está a 6 m. del centro.

SOLUCION
Si colocamos el sistema de coordenadas de
manera que el origen coincida con el centro de la
10 m h Elipse, vemos que le corresponde una ecuación
• x2 y2
6m de la forma + =1
a2 b2
30 m
De los datos deducimos que a = 15 y b = 10
x y2 2
Luego la ecuación que describe el arco es + = 1 . La altura buscada corresponde al valor
225 100
positivo de y cuando x = 6. Sustituyendo en la ecuación se obtiene:

36 y2 36
+ =1 ∴ y = 10 1 − = 9.16 m.
225 100 225

35
GEOMETRÍA ANALÍTICA

EJERCICIOS PROPUESTOS

1. Determine las coordenadas de los vértices y focos de las siguientes elipses. Encuentre la
excentricidad y la longitud del lado recto. Haga un bosquejo de la gráfica.
x 2 y2 x 2 y2 x2 y2 x 2 y2
a) + =1 b) + =1 c) + =1 d) + =1
9 4 36 25 100 144 81 49
x 2 y2
e) + =1 f) 4x2 + y2 = 1 g) 9x2 + 4y2 = 1 h) 25x2 + y2 = 25
8 5
x 2 y2 x 2 y2
i) 4x2 + 49y2 = 196 j). 4x2 + 25y2 = 36 k) + =1 l) + =1
16 7 8 9
2. Determine las coordenadas del centro y de los vértices y focos de las siguientes elipses. Encuentre la
excentricidad y la longitud del lado recto. Haga un bosquejo de la gráfica.

a) 6x2 + 9y2 – 24x – 54y + 51 = 0 b) x2 + 4y2 – 4x – 8y – 92 = 0


c) 2x2 + 3y2 – 8x + 24y + 50 = 0 d) 5x2 + 4y2 + 20x – 8y + 4 = 0
e) 9x2 + 4y2 – 8y – 32 = 0 f) 4x2 + 9y2 – 32x – 36y + 64 = 0
g) 4x2 + y2 – 40x – 10y + 109 = 0 h) x2 + 2y2 + 4y – 10 = 0
i) 4x2 + y2 = x + 5 j) 4x2 + 5y2 – 24x – 20y – 44 = 0

3. Determine la ecuación que satisfaga las condiciones indicadas para las siguientes Elipses con centro
en el origen:
16
a) V (0, 8) y F (0, − 2 7 ) b) F (0, 3) y longitud del eje menor 8 c) V (5, 0) y pasa por (3, )
5
2 1 6
d) F (3, 0) y longitud del eje mayor 12 e) F ( , 0) y pasa por ( , ) f) F (0, 2) y longitud del
2 2 4
eje mayor 8 g) Pasa por (2, 2) y (1, 4) h) Pasa por (2, – 2) y ( 6 , 1)

4. Determine la ecuación de la Elipse que satisfaga las condiciones siguientes:

a) F1 (–1, 2), F2 (–1, – 4) y longitud del eje mayor 10


b) V1 (1, 5), V2 (1, –1), F1 (1, 4) c) C (4, – 2), V (9, – 2) F (0, – 2)
5 3
d) V ( ± , 0) , F ( , 0) e) C (1, 2), F (1, 7), pasa por (4, 6)
2 2
f) V1 (– 5, 9), V2 (– 5, 1), LR = 3 g) C (3, 1), V (3, – 2), LR = 16/3
h) La distancia interfocal es 6, su excentricidad es e = 3/4, su centro C (3, – 2), eje menor horizontal.
i) Vértices (0, ± 6) y pasa por (3, 2)
j) C (1, 2), la distancia entre sus directrices es 5 y la distancia entre sus focos es 4, eje mayor paralelo
al eje X. k ) Centro en el origen, F (6, 0) y corta al eje Y en (0, – 3)

5. Calcular el área y el perímetro del rombo cuyos vértices son los focos y los extremos del eje menor de
la Elipse 16x2 + 25y2 = 400.

6. Dada la Elipse x2+ 4y2 = 20 y la recta y = – x + b, encontrar los valores de b de manera que : a) la
recta corte a la elipse en un punto b) la recta corte a la elipse en dos puntos c) la recta no corte a la
elipse.

7. Un arco de puente tiene forma semielíptica con el eje mayor horizontal. La base del arco mide 80 m y
su altura máxima es de 30 m. Hallar la altura del arco en un punto situado a 15 m del centro.

8. Encuentre el lugar geométrico de los puntos P (x, y) tales que la razón de la distancia de P al punto

36
GEOMETRÍA ANALÍTICA

6
(3, 0) a su distancia a la recta x = 5 es igual a .
3

SOLUCIONES
1. a) V (± 3, 0), B (0, ± 2), F (± 5 , 0) b) V (± 6, 0), B (0, ± 5), F (± 11 , 0)
c) V (0, ± 12), B (± 10, 0), F (0, ± 44 ) d) V (± 9, 0), B (0, ± 7), F (± 4 2 , 0)
1 3
e) V (± 2 2 , 0), B (0, ± 5 ), F (± 3 , 0) f) V (0, ± 1), B (± , 0), F (0, ± )
2 2
1 1 5
g) V (0, ± ), B ( ± ,0), F (0, ± ) h) V (0, ± 5), B (± 1, 0), F (0, ± 2 6 )
2 3 6
6 3 21
i) V (± 7, 0), B (0, ± 2), F (± 3 5 , 0) j) V (± 3, 0), B (0, ± ), F (± , 0)
5 5
k) V (± 4, 0), B (0, ± 7 ), F ( ± 3, 0) l) V (0, ± 3), B (± 2 2 , 0), F (0, ± 1)

2. a) C (2, 3), V (2 ± 3, 3), B (2, 3 ± 6 ), F (2 ± 3 , 3), e = 3 /3, LR = 4


b) C (2, 1), V (2 ± 10, 1), B (2, 1 ± 5), F (2 ± 5 3 , 1), e = 3 /2, LR = 5
c) C (2, – 4), V (2 ± 3 ,– 4 ), B (2, – 4 ± 2 ), F (2 ± 1, – 4), e = 3 /3, LR = 4 3 /3
d) C (– 2, 1), V (– 2, 1 ± 5 ), B (– 2 ± 2, 1), F (– 2, 1± 1), e = 5 /5, LR = 8 5 /5
e) C (0, 1), V (0, 1± 3), B (± 2, 1), F (0, 1 ± 5),e= 5 /3, LR = 8/3
f) C (4, 2), V (4 ± 3, 2), B (4, 2 ± 2), F (4 ± 5 , 2), e = 5 /3, LR = 8/3
g) C (5, 5), V (5, 5 ± 4), B (5 ± 2, 5), F (5, 5 ± 2 3 ), e = 3 /2, LR = 2
h) C (0, – 1), V (± 2 3 , – 1), B (0, – 1 ± 6 ), F (± 6 , – 1), e = 2 /2, LR = 2 3
i) C (1/8, 0), V (1/8, ± 9/4), B (1/8 ± 9/8, 0), F (1/8, ± 9 3 /8), e = 3 /2, LR = 9/8
j) C ( 3, 2), V ( 3 ± 5, 2), B ( 3, 2 ± 2 5 ), F (3 ± 5 , 2), e = 5 /5, LR = 4

x 2 y2 x 2 y2 x 2 y2 x 2 y2
3. a) + = 1 b) + = 1 c) + = 1 d) + =1
36 64 16 25 25 16 36 27

e) x2 + 2y2 = 1 f) 16x2 + 12y2 = 192 g) 4x2 + y2 = 20 h) 3x2 + 2y2 = 20

(x + 1) 2 (y+ 1) 2 (x − 1)2 (y− 2)2 (x − 4)2 (y+ 2)2


4. a) + = 1 b) + = 1 c) + =1
16 25 5 9 25 9
(x − 1)2 (y− 2)2
d) 16x2 + 25y2 = 100 e) + = 1 f) 8x2 + 3y2 + 80x – 30y + 227 = 0
15 40
8 x 2 y2
g) 9 (x – 3)2 + 8 (y – 1)2 = 72 h) 16 (x – 3)2 + 7 (y + 2)2 = 112 i) + =1
81 36
j) (x – 1 )2 + 5(y – 2)2 = 5 k) x2 + 5y2 = 45

5. P = 20 u., A = 24 u2 6. a) b = ± 5 b) |b| < 5 c) |b| > 5 7. 27.81 m.


8. (x + 1)2 + 3y2 = 24

37
GEOMETRÍA ANALÍTICA

6.4.4. HIPÉRBOLA
DEFINICIÓN Una Hipérbola es el conjunto de puntos P del plano, tales que el valor absoluto de la
diferencia de las distancias entre P y dos puntos fijos F1 y F2 , llamados focos, es constante.
| d(P, F1) – d(P. F2)| = k (constante)

Y
Asíntota
Asíntota

Vértice
Vértice X
Foco Foco
x2 y2
− =1
a2 b2

ELEMENTOS DE UNA HIPERBOLA

◊ El punto medio del segmento que une los focos F1F2 se llama el centro de la hipérbola.
◊ La recta que pasa por los focos, se llama eje transverso o eje principal de la hipérbola
◊ Los puntos donde el eje transverso corta a la hipérbola se llaman vértices.
◊ La distancia entre los vértices se conoce como la longitud del eje transverso.
◊ La recta perpendicular al eje transverso, que pasa por el centro se llama eje conjugado.
◊ Asociadas a cada foco hay dos rectas perpendiculares al eje transverso llamadas directrices
d (P, F)
◊ La razón = e es la excentricidad. Para una hipérbola e > 1 . Lo que significa que la distancia
d (P, L)
de cualquier punto de la hipérbola a un foco es mayor que la distancia de dicho punto a la directriz
asociada a ese foco.

ECUACIONES DE LA HIPÉRBOLA

Si colocamos el centro de la hipérbola en el origen y los focos sobre el eje X con coordenadas ( ± c, 0).
Tomando por conveniencia | d1 – d2 | = 2a , tenemos :
Si el punto P (x, y) está en la hipérbola ,
| (x + c )2 + y 2 − (x − c )2 + y 2 | = 2a

luego (x + c )2 + y 2 − (x − c )2 + y 2 = ± 2a ⇒ (x + c )2 + y 2 = (x − c )2 + y 2 ± 2a
Elevando al cuadrado se obtiene (x + c)2 + y2 = [ (x – c)2 + y2] ± 4a (x − c )2 + y 2 + 4a2

Simplificando (x + c )2 + y 2 = (x − c )2 + y 2 ± 2a

Elevando al cuadrado se obtiene (x + c)2 + y2 = [ (x – c)2 + y2] ± 4a (x − c )2 + y 2 + 4a2

Simplificando 4xc = ± 4a (x − c )2 + y 2 + 4a2

Reacomodando y dividiendo entre 4: (xc – a2 ) = ± 4a (x − c )2 + y 2


Elevando al cuadrado de nuevo cada lado: x2c2 – 2xca2 + a4 = a2 (x2– 2xc + c2 + y2)
x2 y2
simplificando ( c2– a2) x2 – a2y2 = a2 ( c2 – a2) ⇒ − =1
a2 c2 − a2

38
GEOMETRÍA ANALÍTICA

x2 y2
Haciendo b2 = c2 – a2, podemos escribir − =1
a2 b2
x2 y2
Por tanto la ecuación − = 1 corresponde a una hipérbola con centro en el origen y focos
a2 b2
( ± c, 0) donde c = a2 + b2

De manera similar si los focos se colocan sobre el eje Y, con coordenadas (0, ± c) resulta la ecuación
2
y x2
2
− 2
=1 donde c = a2 + b2
a b
x2 y2 x2
Si (x, y) está en la hipérbola − = 1 , se tiene que ≥ 1 o sea x2 ≥ a2, lo que significa que esta
a2 b2 a2
hipérbola no tiene puntos en el intervalo x ∈ (– a, a) y por tanto no corta al eje Y.
y2 x2 y2
De manera similar si (x, y) está en la hipérbola
2
− 2
= 1 se tiene 2
≥ 1 , y2 ≥ a2, lo que significa
a b a
que esta hipérbola no tiene puntos para y ∈ (– a, a) y por tanto no corta al eje X.

Y Y

X X

x2 y2 y2 x2
− =1 − =1
a2 b2 a2 b2

SIGNIFICADO DE b. ASÍNTOTAS
x2 y2 bx a2
Al considerar la ecuación − = 1 , al despejar y se obtiene y = ± 1−
a2 b2 a x2
a2
Cuando | x | aumenta, la cantidad 1− se acerca a 1. Luego para | x | muy grande,
x2
bx bx
y≈± , es decir la gráfica de la hipérbola se acerca a los puntos de las rectas, y = ± ,
a a
Estas rectas reciben el nombre de asintotas de la hipérbola.

El segmento de recta con extremos en (0, ± b) se le llama eje conjugado, y a los puntos
(0, ± b) extremos del eje conjugado.

39
GEOMETRÍA ANALÍTICA

y2 x2 ax
Para la hipérbola − = 1, las ecuaciones de las asintotas resultan y = ±
. Observe que se
2 2 b
a b
parecen a las asíntotas del caso anterior, pero no son iguales. En general una forma de obtenerlas es
sustituir el 1 que aparece en el lado derecho por 0 y resolver para y.

Si el centro se traslada al punto (h, k) , las ecuaciones se transforman en


(x − h)2 (y− k )2
− = 1 cuando el eje principal es paralelo al eje X
a2 b2
(y− k )2 (x − h)2
− = 1 , cuando el eje principal es paralelo al eje Y.
a2 b2
De igual manera las coordenadas de los vértices, focos, etc. se “trasladan”.
El siguiente cuadro presenta un resumen de cada situación. En todos los casos se cumple la relación
a2 + b2 = c2.
Nota: observe que esta relación se parece, pero es diferente a la que resultó para la Elipse. En el caso de
la hipérbola c > a y c > b, entre a y b no hay relación definida, pueden ser iguales o distintas. En todos
2 b2 c
los casos se tiene LR = y e= .
a a

Extremos
Ecuación Centro Focos Vértices Del eje Directrices Asintotas
conjugado
x2 y2 a bx
− =1 (0, 0) (± c, 0) (± a, 0) (0, ± b) x= ± y= ±
e a
a2 b2
y2 x2 ax
− =1 (0, ± c) (0, ± a) (± b, 0) y= ±
(0, 0) a b
a2 b2 y= ±
e
(x − h)2 − (y− k )2 =1 y=k ±
b
( x − h)
(h, k) (h ± c, k) (h± a, k) (h, k ± b) a a
a2 b2 x=h ±
e

(y− k )2 (x − h)2 y=k ±


a
( x − h)
− =1 (h, k) (h, k ± c) (h, k ± a) (h ± b, k) a b
a2 b2 y=k ±
e

EJEMPLOS
Para las siguientes hipérbolas, determine las coordenadas del vértice, los extremos del eje conjugado,
focos; las longitudes del eje transverso, del eje conjugado, lado recto y la distancia interfocal. Encuentre
la excentricidad y determine las ecuaciones de las
directrices y de las asíntotas.
Y
1. 9x2 – 4y2 = 36 B(0, 3)
SOLUCIÓN: Primero la expresamos en la forma canónica F(3.6, 0)
F’( – 3.6, 0)
x2 y2
− = 1 . Para ello dividimos cada lado entre 36, X
a2 b2 V’( – 2, 0) V(2, 0)

9 x 2 4 y 2 36 x 2 y2
obteniendo − = ⇒ − =1 .
36 36 36 4 9 B’(0, –3)
2 2
Identificamos que a = 4 y b = 9
Resulta a = 2 y b = 3 . Calculamos el valor de c:

40
GEOMETRÍA ANALÍTICA

c = a 2 + b2 = 22 + 32 = 13 ≈ 3.6
Dado que en la forma canónica aparece el término con la variable x positivo, nos indica que el eje
transverso es el eje x. También podemos guiarnos con la tabla anterior
Tenemos: Vértices ( ± 2, 0) Extremos del eje conjugado: (0, ± 3) Focos : (± 3.6, 0)
Longitud eje transverso: 2a = 4 Longitud eje conjugado: 2b = 6
Distancia interfocal: 2c ≈ 7.2 Lado recto : LR = 2 (32 )/ 2 = 9
Excentricidad: e ≈ 3.6 / 2 = 1.8 Directrices x = ± a/e = ± 2 / 1.8 ≈ ± 1.11...
3x
Asíntotas: y = ±
2
2. 4y2 – x2 = 16
y2 x 2
SOLUCIÓN: Dividiendo entre 16 se obtiene − =1
4 16
Identificamos que el eje transversal es el eje Y y que a2 = 4, b2 = 16 o sea a = 2 y b = 4
Calculamos c, obteniendo c = 4 + 16 = 20 = 2 5 ≈ 4.47
Vértices: (0, ± 2) Extremos del eje conjugado: (± 4, 0) Focos : (0,± 2 5 )
Longitud del eje transverso: 2a = 2(2) = 4 Longitud del eje conjugado: 2b = 2(4) = 8
2 b2 2(4 2 )
Distancia interfocal: 2c = 2 (2 5 )= 4 5 ≈ 8.944 Lado recto LR =
= = 16
a 2
c 2 5 ax 2x x
Excentricidad: e = = = 5 Asíntotas y = ± =± =±
a 2 b 4 2
a 2 2 5
Directrices: y = ± = ± =±
e 5 5

3. 4x2 – 5y2 – 32x –50y – 41 = 0

SOLUCIÓN: Rescribimos la ecuación agrupando los términos en “x” y los términos en “y”. Luego
factorizamos para asegurar que los coeficientes de x2 y y2 sean igual a 1. Completamos trinomios
cuadrados perfectos y después de simplificar obtenemos la forma canónica.

(4x2 – 32x ) + (– 5y2 – 50y) = 41 ⇒ 4 (x2 – 8x ) – 5 (y2 + 10x ) = 41


4 (x2 – 8x +16 ) – 5 (y2 + 10x + 25 ) = 41 + 4 (16) – 5 (25) = – 20
4 (x – 4 )2 – 5 (y + 5)2 = – 20 ⇒ 5 (y + 5)2 – 4 (x – 4 )2 = 20
5 (y+ 5) 2 4 (x − 4) 2 (y+ 5) 2 (x − 4) 2
− =1 ⇒ − =1
20 20 4 5
Tenemos h = 4 , k = – 5 , a = 2 , b = 5 , c = 3
Centro: (h, k) = (4, – 5) Vértices: (h, k ± a) ∴ V1 = (4, –7) , V2 = (4, – 3)
Extremos del eje conjugado: (h ± b, k) B1 = (4 – 5 , – 5) B2 = (4+ 5 , – 5)
Focos (h, k ± c) F1 = (4, – 8) F2 = (4, – 2)
Longitud eje transverso: 2a = 4, Longitud eje conjugado 2b = 2 5
2b2 2 ( 5 )2
Distancia interfocal: 2c = 6 Lado recto: LR = = =5
a 2
4. Los extremos del eje conjugado de una hipérbola son (0, ± 3) y la longitud de su eje transverso es 8.
Hallar su ecuación y su excentricidad.

SOLUCIÓN:
En vista de que los extremos del eje conjugado están ubicados sobre el eje Y, a igual distancia del
origen, el centro de la hipérbola es el origen y su eje transverso está sobre el eje X, luego le

41
GEOMETRÍA ANALÍTICA

x2y2
corresponde una ecuación de la forma − = 1 . Como la longitud del eje transverso es 2a = 8,
a2 b2
tenemos a = 4. De la información sobre las coordenadas de los extremos del eje conjugado deducimos
que b = 3. Por tanto el valor de c es c = 4 2 + 32 = 5. La excentricidad está dada por e = c / a, luego
e = 5 / 4 = 1.25.
x2 y2 x 2 y2
La ecuación de la hipérbola es − = 1 o sea − =1
42 32 16 9
x 2 y2
5. Encuentre el ángulo agudo que determinan las asíntotas de la hipérbola − =1
9 16
SOLUCIÓN:
Para hallar las asíntotas, sustituimos el 1 de la ecuación por 0 y simplificamos. En el ejemplo se obtiene:
4 4 4
y = ± x . Luego las pendientes son m1 = y m2 = − .
3 3 3
4 ⎛ 4⎞ 8
− ⎜− ⎟
m1− m2 3 ⎝ 3⎠ 24 ⎛ 24 ⎞
Tenemos tan θ = = = 3 =− ∴θ = tan– 1 ⎜ − ⎟ = – 73.74º
1 + m1 m2 ⎛ 4 ⎞⎛ 4 ⎞ 7 7 ⎝ 7 ⎠
1 + ⎜ ⎟⎜ − ⎟ −
⎝ 3 ⎠⎝ 3 ⎠ 9
El resultado aparece negativo por el orden en que se tomaron las asíntotas; al invertir el orden resulta la
medida del ángulo buscado: θ = 73.74º.

EJERCICIOS
1. Para las siguientes hipérbolas, determine las coordenadas del centro, vértices, los extremos del eje
conjugado, focos; las longitudes del eje transverso, del eje conjugado, lado recto y la distancia interfocal.
Encuentre la excentricidad y determine las ecuaciones de las directrices y de las asíntotas
y2 x 2 (x − 2)2 (y+ 2)2
a) x2 – y2 = 9 b) − = 1 c) 4x2 – 5y2 = 7 d) 16x2 – 9y2 = 144 e) − =1
16 20 4 8
(y+ 5)2 (x − 4)2
f) − = 1 g) 4x2 – 9y2 + 32x + 36y + 64 = 0 h) x2 – y2 + 6x + 10y – 4 = 0
4 5

2. Determine la ecuación de la hipérbola que satisfaga las siguientes condiciones


a) Focos (0, ± 4), vértices (0, ± 1) b) Vértices (±3, 0), asíntotas y = ± x

c) Los extremos del eje conjugado son (0, ± 3) y su lado recto mide 6.

d) Tiene centro en el origen, su eje conjugado está en el eje X, la longitud de cada lado recto es 2/3 y
pasa por A (– 1, 2)

e) Pasa por (6, 2), tiene su centro en el origen, su eje transverso está sobre el eje X y una de sus
asíntotas es la recta 2x – 5y = 0.

f) El centro es el punto (2, -2) y uno de sus vértices es el punto (0, -2). La longitud del lado recto es 8.

g) V (± 2, 0); la longitud del eje conjugado es 6

h) Los focos de una hipérbola son los puntos (4, – 2) y (4, – 8). La longitud de su eje transverso es 4.
i) Pasa por (4, 6), eje transverso paralelo al eje X, asíntotas 2x + y – 3 = 0 2x – y – 1 = 0

j) Tiene los mismos focos que la Elipse 3y2 + 4x2 = 48, y su excentricidad es una unidad mayor que la
excentricidad de la Elipse.
k) Sus vértices son (– 3, 2) y (– 3, – 2). Su eje conjugado mide 6 u.

42
GEOMETRÍA ANALÍTICA

l) Centro en el origen, pasa por (3, – 1), eje transverso paralelo al eje X, una asíntota es 2x + 3 2 y = 0

3. Hallar el ángulo agudo de intersección de las asíntotas de la hipérbola


a) 4x2 – 9y2 + 32x + 36y + 64 = 0 b) 9x2 – y2 – 36x – 2y + 44 = 0

4. Determine donde corta la recta 2x – 9y + 12 = 0 a las asíntotas de 4x2 – 9y2 = 11

5. Hallar la distancia desde uno de los focos a cualquiera de las asíntotas de 16x2 – 9y2 = 144

6. Encuentre la ecuación de una hipérbola que tiene los mismos focos que la Elipse 4x2 +3y2 = 48 y su
excentricidad es una unidad mayor que la excentricidad de la Elipse.

7. Los focos de una hipérbola coinciden con los focos de la elipse 9x2 + 25y2 = 225. Hallar su ecuación si
su excentricidad es 2.

8. Calcular el área del triángulo formado por las asíntotas de la hipérbola 9x2 – 4y2 = 36 y la recta
9x + 2y – 24 = 0
y2 x2
9. Probar que la hipérbola − = 1 queda definida por el sistema de ecuaciones paramétricas
a2 b2
x = a sec θ , y = b tan θ, donde θ recorre el intervalo [0, 2π].
SOLUCIONES
1. a) V (±3, 0), B (0, ±3), F (±3 2 , 0), LR = 6, e = 2 , D: x = ±3 2 /2, A: y = ± x
b) V (0, ±4), B (±2 5 , 0), F (0, ±6), LR = 10, e = 1.5, D: y = ±8/3, A: y = (±2 5 /5)x
c) V (0, ± 7 /2), B (± 35 /5, 0), F (0, ±3 35 /10), LR = 4 7 /5, e = 3 5 /5, D: y = ± 35 /6
A: y = ±(4/3)x d) V (±3, 0), B (0, ±4), F (±5, 0), LR = 32/3, e = 5/3, D: x = ± 9/5, A: y = ±(4/3)x
e) C (2, – 2), V (2 ± 2, – 2), B (2, – 2 ± 2 2 ), F (2 ± 2 3 , – 2), LR = 8, e = 3 , D: x = 2 ± 2 3 /3,
A: y + 2 = ± 2 (x – 2) f) C (4, – 5), V (4, – 5 ± 2), B (4 ± 5 ,– 5), F (4, – 5 ± 3), LR = 5, e = 1.5, D: y = – 5 ± 4/3,
A: y + 5 = ± (2 5 /5) (x – 4 ) g) C (– 4, 2), V (– 4, 2 ± 2), B (– 4 ± 3, 2), F (– 4, 2 ± 13 ), LR = 9, e = 13 /2,
D: y = 2 ±4 13 /13, A: y – 2 = ± (2/3) (x + 4) h) C (– 3, 5), V (– 3, 5 ± 2 3 ), B (– 3 ± 2 3 , 5), F (– 3, 5 ± 2 6 ),
LR = 2 , D: y = 5 ± 6 , A: y – 5 = ± (x + 3) 2. a) 15y2 – x2 =25 b) 4x2 – y2 = 36 c) x2 – y2 = 9
(x − 2)2 (y+ 2)2 (y+ 5)2 (x − 4)2
d) y2 – 3x2 = 1 e) 4x2 – 25y2 = 44 f) − = 1 g) 9x2 – 4y2 = 36 h) − =1
4 8 4 5
y2 (x + 3)2
i) 4x2 – y2 – 8x + 2y – 8 = 0 j) 45y2 – 36x2 = 80 k) − = 1 l) 2x2 – 9y2 = 9 3. a) 67.38º b)
4 9
3
36º 52’ 4. (3, 2), ( − , 1) 5. 4 6. 45y2 – 36x2 = 80 7. 3x2 – y2 = 12
2
(REPASO) SELECCIÓN MÚLTIPLE

1. Una circunferencia de radio 3 tiene centro en (3,2). La ecuación de la circunferencia está dada por
A. x2 + y2 + 6x + 4y + 4 = 0 B. x2 + y2 – 6x – 4y = 9 C. x2 + y2 + 6x + 4y = 9
2 2 2 2
D. (x + 3) + (y + 2) – 9 = 0 E. x + y – 6x – 4y + 4 = 0
2 445
2. El centro de la circunferencia cuya ecuación está dada por x2 – x + y2 + y − = 0 es
3 36
−1 1 1 1 1 1 1 1 2
A. ( , ) B. ( ,− ) C. ( − ,− ) D. ( , ) E. ( – 1, )
2 2 2 3 2 3 2 3 3

3. La gráfica que representa al conjunto {(x,y) / x2 – 4y2 – 2x + 8y –2 = 0} es una


A. Circunferencia B. Parábola C. Elipse D. Hipérbola E. Recta

43
GEOMETRÍA ANALÍTICA

4. La gráfica que representa al conjunto {(x, y) / x2 + 3y2 = 36 } es una


A. Circunferencia B. Parábola C. Elipse D. Hipérbola E. Recta

5. Dada la parábola con ecuación y2 – 2y + x = 0, las coordenadas de su vértice son


A. (1, 1) B. (– 2, 1) C. (2, 1) D. (1, – 1) E. (2, – 1)

6. La ecuación de la parábola con vértice en el punto (3, 2) y foco en el punto (5, 2) es


A. (y – 2)2 = 8 (x – 3) B. (x – 3)2 = 5 (y – 2) C. (y – 2)2 = 8 (x – 5)
2 2
D. (x – 3) = 8 (y – 2) E. (y – 2) = 5 (x – 3)

7. Dada la Elipse con ecuación 9x2 + 4y2 – 18x + 16y – 11 = 0, entonces las coordenadas de su centro
son
A. (2, 1) B. (1, – 2) C. (1, 2) D. (– 2, 1) E. (9, 4)

8. La ecuación de la Elipse con centro en (0, 2), con eje mayor paralelo al eje Y, de longitud 6 y con eje
menor de longitud 2, está dada por
x 2 (y− 2) 2 (y− 2) 2 x 2 x 2 (y− 2) 2 x 2 (y− 2) 2
A. + = 1 B. + = 1 C. + = 1 D. + =1
9 4 36 4 1 9 4 36
9. Dada la Elipse con ecuación 9x2 + 25y2 – 36x – 189 = 0, entonces el centro de la Elipse está dado
por el punto
A. (0, – 2) B. (0, 2) C. (– 2, 0) D. (2, 0) E (0, 0)

10. Los vértices de la Elipse con ecuación 9x2 + 25y2 – 36x – 189 = 0 están dados por
A. (–2, 0) y (6, 0) B. (–3, 0) y (7, 0) C. (–2, 0) y (7, 0) D. (–3, 0) y (6, 0)

11. Los puntos de intersección de la circunferencia con ecuación x2 + y2 = 1 y la Elipse con ecuación
x2 + 4y2 = 4 son

A. (0, 0) y (1, –1) B. (1, 0) y (–1, 0) C. (–1, 0) y (0, 1) D. (0, –1) y (0,1)

12. La ecuación de la Elipse con centro en (– 4, – 2), con eje mayor paralelo al eje X y de longitud 10, y
eje menor de longitud 6 está dada por
(x − 4) 2 (y− 2) 2 (x + 4) 2 (y+ 2) 2
A. + =1 B. + =1
25 36 25 9
(y+ 4) 2 (y+ 2) 2 (x − 4) 2 (y− 2) 2
C. + =1 D. + =1
100 36 25 9

13. La ecuación de la hipérbola con centro en el origen, un vértice en el punto (0, 3), un foco en el punto
(0, 5) está dada por
y2 x 2 x 2 (y− 3)2 y2 x 2 y2 x 2
A. − =1 B. − =1 C. − =1 D. − =1 E.
9 16 16 25 16 9 9 25
2 2
14. Dada la hipérbola con ecuación 9(x – 1) – 16 (y + 2) = 144, entonces las coordenadas del centro
son
A. (–2, 1) B. (1, 2) C. (1, – 2) D (1, 2)

15. En la hipérbola del ejercicio anterior, los focos están dados por
A. F1 (– 4, – 2) y F2 (6, – 2) B. F1 (– 4, 2) y F2 (6, – 2)
C. F1 (4, 2) y F2 (6, – 2) D. F1 (– 4, – 2) y F2 (6, 2)

16. La representación gráfica de la ecuación x2 – 26 = y (y + 2) es una


A. Recta B. Circunferencia C. Elipse D. Parábola E. Hipérbola

44
GEOMETRÍA ANALÍTICA

17. La ecuación del círculo, cuyo centro es el mismo que el del círculo cuya ecuación es :
x 2 + y 2 + 8 x − 4 y = 0 y el radio es el mismo del círculo cuya ecuación es: x 2 + y 2 − 10 x + 2y = 10
está dada por:
A. (x + 4 )2 + (y − 2)2 = 9 B. (x − 4 )2 + (y − 2)2 = 25 C. (x + 4 )2 + (y + 2)2 = 25
D. (x + 4 )2 + (y + 2)2 = 36 E. (x + 4 )2 + (y − 2)2 = 36

18. El lado de un rombo es igual a 10. A través de dos vértices opuestos pasa una elipse, cuyos focos
coinciden con los otros dos vértices del rombo. La ecuación de la elipse, tomando por ejes de
coordenadas las diagonales del rombo, si las coordenadas de un foco son (8,0) es:
x2 y2 x2 y2 x2 y2 x2 y2 x2 y2
A. + = 1 B. + = 1 C. + =1 D. + =1 E. + =1
64 36 100 36 36 100 100 64 64 100
19. La ecuación del círculo, cuyo centro es el mismo que el del círculo cuya ecuación es
2x 2 + 2y 2 − 10 x + 6 y = 3 , y el radio es el mismo del círculo cuya ecuación es 2x 2 + 2y 2 = 35 está
dada por:
2 2
⎛ 5⎞ ⎛ 3⎞ 35 35
A. ⎜ x + ⎟ + ⎜ y− ⎟ = B. (x − 5 )2 + (y− 3 )2 = 35 C. (x − 5 )2 + (y+ 3 )2 =
⎝ 2 ⎠ ⎝ 2⎠ 2 2
2 2 2 2
⎛ 5⎞ ⎛ 3⎞ 35 ⎛ 5⎞ ⎛ 3⎞
D. ⎜ x − ⎟ + ⎜ y+ ⎟ = E. ⎜ x − ⎟ + ⎜ y+ ⎟ = 35
⎝ 2 ⎠ ⎝ 2⎠ 2 ⎝ 2 ⎠ ⎝ 2⎠

20. Se da la elipse 15 x 2 + 25 y 2 − 375 = 0 . A través del foco está trazada una perpendicular a su eje
mayor. La distancia desde los puntos de intersección de esta perpendicular con la elipse hasta el foco
es:
A. 3 B. 4 C. 5 D. 6 E. 8

45
CRISTIANA, SOCIALISTA, SOLIDARIA !
MINISTERIO DE EDUCACIÒN
CENTRO CIVICO MODULO “L” PLANTA ALTA
Dirección General de Educación Secundaria
Correo: guillem@mined.gob.ni
DIBUJO TÉCNICO

A. Introducción
Desde sus orígenes, el hombre ha tratado de comunicarse mediante grafismos o
dibujos. Las primeras representaciones que conocemos son las pinturas rupestres, en
ellas no solo se intentaba representar la realidad que le rodeaba, animales, astros, al
propio ser humano, etc., sino también sensaciones, como la alegría de las danzas, o
la tensión de las cacerías.
A lo largo de la historia, este ansia de comunicarse mediante dibujos ha evolucionado,
dando lugar por un lado al dibujo artístico y por otro al dibujo técnico. Mientras el
primero intenta comunicar ideas y sensaciones, basándose en la sugerencia y
estimulando la imaginación del espectador, el dibujo técnico, tiene como fin, la representación de los objetos lo
más exactamente posible, en forma y dimensiones.
Hoy en día, se está produciendo una confluencia entre los objetivos del dibujo
artístico y técnico. Esto es consecuencia de la utilización de los ordenadores en
el dibujo técnico, con ellos se obtienen recreaciones virtuales en 3D, que
representan los objetos en verdadera magnitud y forma, a la vez que, las
perspectivas adoptadas y los efectos de iluminación y ambientales, conllevan
un alto grado de sugerencia para el espectador mezclándose en la frontera, lo
real y lo imaginario, lo técnico y lo artístico

B. Dibujo definición
Un dibujo es la representación gráfica sobre una superficie, generalmente plana, por medio de líneas o sombras,
de objetos reales o imaginarios o de formas abstractas.

C. Ramas del dibujo.


Según su objetivo se divide en dos formas:
1. Dibujo artístico que se realiza libremente y con finalidad estética.
2. Dibujo técnico que se realiza con otros medios auxiliares, siguiendo normas y fines prácticos.

Dibujo artístico
Es la representación de un objeto por medio de líneas que limitan sus formas y contornos.
Dado que no es posible presentar en una sola imagen todos los aspectos visibles de un
objeto, el arte del dibujo o Dibujo artístico se sustenta de la sugerencia, estimulando la
imaginación del espectador para aportar lo que falta en la representación.
D. Concepto de dibujo técnico.

CRISTIANA, SOCIALISTA, SOLIDARIA !


MINISTERIO DE EDUCACIÒN
CENTRO CIVICO MODULO “L” PLANTA ALTA
Dirección General de Educación Secundaria
Correo: guillem@mined.gob.ni
El dibujo técnico es la representación gráfica de un objeto o una idea práctica. Esta representación se guía por
normas fijas y preestablecidas para poder describir de forma exacta y clara, dimensiones, formas, características y
la construcción de lo que se quiere reproducir.

E. Tipos de dibujo técnico.


Con el desarrollo industrial y los avances tecnológicos el dibujo ha aumentado su campo de acción. Los principales
son:
Dibujo arquitectónico: El dibujo arquitectónico abarca una gama de representaciones gráficas con el se elaboran
los planos para la construcción de viviendas, edificios para diversas aplicaciones, carreteras y puentes entre otros.
Dibujo mecánico: El dibujo mecánico se emplea en la representación de piezas o partes de equipos o máquinas
para aplicaciones, industriales o trasporte, tales como vehículos, embarcaciones, grúas, motos, aviones, y
máquinas con aplicaciones industriales (rotativas para periódicos, fábricas para el procesamiento de alimentos,
etc.).
Dibujo eléctrico: Este tipo de dibujo se encarga de la representación gráfica de instalaciones eléctricas en la
industria, vivienda o en cualquier estructura arquitectónica que requiera de electricidad ó, máquinas de trasporte
tales como barcos, aviones o automóviles. Utilizando cierta simbología normalizada que representan a los
componentes y sus conexiones.
Dibujo electrónico: Se representa los circuitos que dan funcionamiento preciso a diversos aparatos que en la
actualidad constituyen un adelanto tecnológico como las computadoras, amplificadores, transmisores, relojes,
televisores, radios y otros.
Dibujo geológico: El dibujo geológico se emplea en geografía y en geología, en él se representan las diversas
capas de la tierra empleando una simbología y da a conocer los minerales contenidos en cada capa. Se usa
mucho en minería y en exploraciones de yacimientos petrolíferos.
Dibujo topográfico: El dibujo topográfico nos representa gráficamente las características de una determinada
extensión de terreno, mediante signos convencionalmente establecidos. Nos muestra los accidentes naturales y
artificiales, cotas o medidas, curvas horizontales o curvas de nivel.
Dibujo urbanístico: Este tipo de dibujo se emplea en la organización de ciudades: en la ubicación de centros
urbanos, zonas industriales, bulevares, calles, avenidas, jardines, autopistas, zonas recreativas entre otros. Se
dibujan anteproyectos, proyectos, planos de conjunto, planos de pormenor.
Dibujo técnico de instalaciones sanitarias: Tiene por finalidad representar el posicionamiento de cada una de
las piezas sanitarias: ducha, lavamanos, retrete, etc. Incluyendo la ubicación de las tuberías internas o externas.
Importancia del dibujo técnico como elemento de comunicación.
En el lenguaje la comunicación se realiza por medio de signos que representan las palabras.
El dibujo técnico emplea signos gráficos, regido por normas internacionales convirtiéndolo en un lenguaje que tiene
carácter universal con el cual nos podemos comunicar con otras personas sin importar el idioma.
Para que un dibujo técnico represente un elemento de comunicación completo y eficiente, debe ser claro, preciso y
constar de todos sus datos; todo esto depende de la experiencia del dibujante en la expresión gráfica que realice,
bien sea un croquis, una perspectiva o un plano.

CRISTIANA, SOCIALISTA, SOLIDARIA !


MINISTERIO DE EDUCACIÒN
CENTRO CIVICO MODULO “L” PLANTA ALTA
Dirección General de Educación Secundaria
Correo: guillem@mined.gob.ni
F. Características del dibujo técnico.
El dibujo técnico posee 3 características que deben ser respetadas a la hora de realizar un trabajo:
• Grafico
• Universal
• Preciso

CRISTIANA, SOCIALISTA, SOLIDARIA !


MINISTERIO DE EDUCACIÒN
CENTRO CIVICO MODULO “L” PLANTA ALTA
Dirección General de Educación Secundaria
Correo: guillem@mined.gob.ni
Es fundamental que todas las personas, diseñadores o técnicos, sigan unas normas claras en la representación de
las piezas. A nivel internacional, las normas ISO son las encargadas de marcar las directrices precisas.
En dibujo técnico, las normas de aplicación se refieren a los sistemas de representación, presentaciones (líneas,
formatos, rotulación, etc.), representación de los elementos de las piezas (cortes, secciones, vistas, etc.), etc.

G. Instrumentos empleados en el dibujo técnico


La realización de un dibujo técnico exige cálculo, medición, líneas bien trazadas, precisión, en fin, una serie de
condiciones que hacen necesario el uso de buenos instrumentos, buenos materiales, y sumado a esto, el
conocimiento teórico que unido a la práctica hacen sobresalir a un dibujante.

Mesa de dibujo.
Es un instrumento de dibujo sobre el que se fija el papel para realizar el dibujo. Por lo
general se construye de madera o plástico liso y de bordes planos y rectos lo cual permite
el desplazamiento de la regla T.
El tamaño depende del formato que se vaya a utilizar. Para el formato escolar es
suficiente un tamaño de 40 centímetros de altura por 60 centímetros de anchura

La regla T.
La regla T recibe ese nombre por su semejanza con la letra T. Posee dos brazos
perpendiculares entre sí. El brazo transversal es más corto. Se fabrican de madera o
plástico.
Se emplea para trazar líneas paralelas horizontales en forma rápida y precisa. También
sirve como punto de apoyo a las escuadras y para alinear el formato y proceder a su
fijación.

La regla graduada.
Es un instrumento para medir y trazar líneas rectas, su forma es rectangular, plana y tiene
en sus bordes grabaciones de decímetros, centímetros y milímetros.
Por lo general son de madera o plástico. Aunque son preferibles las de plástico
transparente para ver las líneas que se van trazando.
Sus longitudes varían de acuerdo al uso y oscilan de 10 a 60 centímetros Las más
usuales son las de 30 centímetros.
Las escuadras.
Las escuadras se emplean para medir y trazar líneas horizontales, verticales, inclinadas, y
combinada con la regla T se trazan líneas paralelas, perpendiculares y oblicuas. Pueden
llevar graduados centímetros y milímetros.
Las escuadras que se usan en dibujo técnico son dos:
- La de 45º que tiene forma de triángulo isósceles con ángulo de 90º y los otros dos de
45º.
- La escuadra de 60º llamada también cartabón que tiene forma de triángulo escaleno,
cuyos ángulos miden 90º, 30º y 60º.

CRISTIANA, SOCIALISTA, SOLIDARIA !


MINISTERIO DE EDUCACIÒN
CENTRO CIVICO MODULO “L” PLANTA ALTA
Dirección General de Educación Secundaria
Correo: guillem@mined.gob.ni
El compás.
Es un instrumento de precisión que se emplea para trazar arcos, circunferencias y
transportar medidas.
Está compuesto por dos brazos articulados en su parte superior donde está ubicada una
pieza cilíndrica llamada mango por donde se toma y maneja con los dedos índice y pulgar.
Uno de los brazos tiene una aguja de acero graduable mediante un tornillo de presión y
una tuerca en forma de rueda. El otro brazo posee un dispositivo que permite la
colocación de portaminas u otros accesorios.
El transportador de ángulos.
Es un instrumento utilizado para medir o transportar ángulos. Son hechos de plástico y
hay de dos tipos: en forma de semicírculo dividido en 180º y en forma de círculo completo
de 360º.
Los números están dispuestos en doble graduación para que se puedan leer de derecha a
izquierda y de izquierda a derecha, según donde esté la abertura del ángulo.

Lápices.
Están formados por una mina de grafito y una envoltura de madera de sección redonda o
hexagonal. Para dibujar son mejores los hexagonales porque facilitan la sujeción entre los
dedos y evitan que se ruede al dejarlos sobre la mesa de dibujo.
Grados de dureza de la mina.
La mina de los lápices posee varios grados desde el más duro hasta el más blando. Con
los de mina dura se trazan líneas finas de color gris y las más blandas líneas gruesas y de
color negro.
Están clasificados por letras y números. La H viene de la palabra hard que significa duro,
la F significa firme y la B de black que significa negro.
Los más duros son: 4H, 3H, 2H y H. Los intermedios son: HB y F. Los más blandos son:
B, 2B, 3B y 4B.
Otro método común utiliza una escala numérica para señalar la graduación de un lápiz.
Fue creado por Conté y adoptado inicialmente en los Estados Unidos por Thoreau en el
siglo XIX. La tabla siguiente muestra las equivalencias aproximadas entre los dos
sistemas:
Tono U.S.A. Mundial
#1 = B
#2 = HB
#2½ * = F
#3 = H
#4 = 2H

Portaminas o lapiceros.

CRISTIANA, SOCIALISTA, SOLIDARIA !


MINISTERIO DE EDUCACIÒN
CENTRO CIVICO MODULO “L” PLANTA ALTA
Dirección General de Educación Secundaria
Correo: guillem@mined.gob.ni
los portaminas son de metal o plástico y aloja en su interior la mina o minas que se
deslizan mediante un resorte hacia afuera, que han de servir para escribir o trazar. Las
minas son de distinta dureza. Aventaja a los lápices por el afilado de la mina y su
resguardo.

Goma de borrar.
Las gomas de borrar se emplean para hacer desaparecer trazos incorrectos, errores,
manchas o trazos sobrantes. Por lo general son blandas, flexibles y de tonos claros para
evitar manchas en el papel.
Antes de borrar debe asegurarse de que está limpia y, si hemos de borrar partes
pequeñas, trazos sobrantes o líneas cercanas, debemos usar la plantilla auxiliar del
borrado de acero laminado.
El papel.
El papel es una lámina fina hecha de unas pastas de materiales distintos como trapos,
madera, cáñamo, algodón y celulosa de vegetales. Es utilizado en todo el mundo para
escribir, imprimir, pintar, dibujar y otros.
Existen de diferentes tipos, tonos y texturas. Pero en el dibujo técnico se utilizan dos
clases: el papel opaco y el papel traslúcido.
El papel opaco no es transparente, tiene varios tonos, desde el blanco al blanco
amarillento. La cara donde se dibuja es lisa y brillante.
El papel traslúcido es transparente. Es utilizado para dibujos o copias de planos a lápiz o
tinta.

H. Manejo de instrumentos y construcciones geométricas básicas


En la ejecución de la solución de problemas geométricos en geometría descriptiva se emplean los siguientes tipos
de líneas:
1. Líneas auxiliares y de construcción: Son empleadas como guías y en los pasos necesarios para realizar
determinadas construcciones geométricas, por lo tanto, no forman parte del resultado final. Su trazo debe ser
fino, ya que debido a su abundancia podrían generar confusión en la lectura del dibujo. Sin embargo, deben
ser perceptibles con el fin de hacer posible la comprensión del procedimiento seguido en la resolución del
problema, por esta misma razón es importante que una vez concluido el trabajo no se borren. Se recomienda
el uso del lápiz 4H para su construcción.
2. Líneas visibles definitivas: corresponden a las líneas visibles de la solución definitiva a un determinado
problema geométrico, por lo que deben resaltar entre
todas las demás líneas. Su trazo debe ser grueso pero no

CRISTIANA, SOCIALISTA, SOLIDARIA !


MINISTERIO DE EDUCACIÒN
CENTRO CIVICO MODULO “L” PLANTA ALTA
Dirección General de Educación Secundaria
Correo: guillem@mined.gob.ni
demasiado para evitar suciedad en la superficie de trabajo. Se recomienda el uso del lápiz F o HB.
3. Líneas invisibles definitivas: son líneas de trazos cortos empleadas en el dibujo de segmentos y aristas
invisibles de la solución a un determinado problema geométrico. Para su construcción se recomienda el uso
del lápiz H.
4. Líneas de eje: se aplican en el trazado de ejes de sólidos radiales, como prismas, pirámides, conos y cilindros.
Tienen el mismo espesor de las líneas invisibles por lo que para su trazado se emplea el lápiz H.
5. Líneas de verdadero tamaño: corresponden a segmentos de recta y figuras planas en proyecciones auxiliares
cuyas dimensiones son iguales a las del elemento original. Deben ser trazadas empleando un lápiz 2H.
I. Construcciones geométricas básicas
I.1 Ejercicios de Trazado Básico
A continuación se presentan algunos de los problemas de trazado elemental más comunes en el dibujo técnico,
acompañados de la solución correspondiente.

1. Trazar una perpendicular a un segmento AB por su extremo


A
Se traza una circunferencia con centro en A y radio cualquiera; esta
circunferencia corta al segmento AB en el punto 1 y a su prolongación
en el punto 2. Luego, haciendo centro en 1 y luego en 2, se trazan
arcos de radio igual a una distancia mayor que el segmento A1. El
corte de tales arcos es el punto 3. Finalmente, la recta definida por los
puntos A y 3 es la perpendicular buscada.

2. Trazar una perpendicular a un segmento AB por un punto cualquiera P


Se traza un arco de centro en P y radio cualquiera, tal que dicho arco corte a la recta
definida por AB en los puntos 1 y 2. Luego, con centro en 1 y en 2 y radio P1 se trazan
dos arcos que se cortan en el punto 3; la línea recta que pasa por los puntos P y 3 es la
perpendicular buscada.

CRISTIANA, SOCIALISTA, SOLIDARIA !


MINISTERIO DE EDUCACIÒN
CENTRO CIVICO MODULO “L” PLANTA ALTA
Dirección General de Educación Secundaria
Correo: guillem@mined.gob.ni
3. Trazar la mediatriz de un segmento AB y determinar su punto medio M
La mediatriz de un segmento es una recta perpendicular a aquél que lo divide en dos
partes iguales.
M12BAFig. A.6
Se comienza trazando arcos con centro en A y B de radio igual a cualquier distancia
mayor que la mitad de AB. Los cortes de tales arcos son los puntos 1 y 2. La línea recta
que pasa por los puntos 1 y 2 es la mediatriz del segmento AB; el corte entre la mediatriz
de AB y el propio segmento AB es su punto medio M.

4. Determinar la bisectriz del ángulo formado por las rectas “a” y “b”
Haciendo centro en el vértice O del ángulo, se traza un arco de radio cualquiera que corta
a las rectas “a” y “b” en los puntos 1 y 2, respectivamente. Luego, con el mismo radio, se
trazan otros dos arcos cuyos centros son 1 y 2; el corte entre tales arcos define al punto 3.
La línea recta que pasa por el vértice del ángulo y por el punto 3 es la bisectriz pedida.

5. Dividir el ángulo recto en tres partes iguales


Haciendo centro en el vértice O del ángulo, se traza un arco de radio cualquiera que
corta a los lados del ángulo recto en los puntos 1 y 2. Manteniendo la misma abertura
del compás, se hace centro en forma sucesiva en los puntos 1 y 2 y se trazan arcos
que cortan al arco 12 en los puntos 3 y 4. Luego las líneas rectas O3 y O4 dividen al
ángulo recto en tres partes iguales.

6. Dividir un segmento de recta AB en n partes iguales


Sea un segmento de recta AB, el cual se quiere dividir en ocho (8) partes iguales. En general, el segmento AB tiene una
longitud tal que al ser dividida por n da como resultado un número no entero (muchas veces irracional) por lo que es
aconsejable realizar la división aplicando la aritmética.
El procedimiento gráfico consiste en trazar por uno de los extremos de AB (A en la figura) una línea recta de cualquier
longitud divisible por n (ocho en el ejemplo). Luego de dividir esta línea en n partes iguales, se traza otra línea recta que
pase por el otro extremo del segmento AB (B en la figura) y por la última marca hecha en la línea cualquiera. Seguidamente
se trazan paralelas a la línea así definida por las demás marcas (numeradas en la figura) obteniéndose en el segmento AB
una serie de puntos que corresponden a su división.

CRISTIANA, SOCIALISTA, SOLIDARIA !


MINISTERIO DE EDUCACIÒN
CENTRO CIVICO MODULO “L” PLANTA ALTA
Dirección General de Educación Secundaria
Correo: guillem@mined.gob.ni
J. Construcción de Polígonos Regulares
1. Construir un triángulo equilátero conocidos su centro O y uno de sus vértices A
Con centro en O y radio OA se traza una circunferencia. Luego, se busca el punto K, corte entre la prolongación de OA y la
circunferencia trazada. Con centro en K y radio KO se dibuja un arco, que, al cortar a la circunferencia, define los vértices B
y C del triángulo pedido.

2. Construir un triángulo equilátero conocido uno de sus lados AB


Con centro en A y radio AB se traza un arco. Luego, con centro en B e igual radio se dibuja un segundo arco. Los puntos de
corte entre los arcos señalados definen las dos posibles soluciones para el vértice C del triángulo buscado.

3. Construir un triángulo equilátero conocida su altura AM


Se comienza dividiendo el segmento AM en tres partes iguales. El punto del segmento ubicado a una tercera parte de la
longitud de la altura desde M es el centro O del polígono. Con centro en O y radio OA se traza una circunferencia. Luego,
se traza una perpendicular a AM por el punto M. Finalmente, los puntos de corte entre dicha perpendicular y la
circunferencia son los vértices B y C del triángulo.

CRISTIANA, SOCIALISTA, SOLIDARIA !


MINISTERIO DE EDUCACIÒN
CENTRO CIVICO MODULO “L” PLANTA ALTA
Dirección General de Educación Secundaria
Correo: guillem@mined.gob.ni
4. Construir un triángulo equilátero conocidos el punto medio M del lado AB y el punto medio N del lado BC.
Con radio MN y haciendo centro sucesivamente en M y en N, se trazan dos arcos
que, al cortarse entre sí, definen las dos posibles soluciones para el vértice B.
Luego, se prolongan los segmentos BM y BN copiando la distancia BM (o BN). De
esta manera se obtienen los vértices A y C del polígono.

5. Construir un cuadrado conocidos el centro O y uno de sus vértices A


En primer lugar se dibuja una circunferencia de centro en O y de radio OA. Luego, se prolonga el segmento OA hasta cortar
a la circunferencia, lo cual da como resultado el vértice C. Finalmente, se dibuja por O una perpendicular a OA que corta a
la circunferencia en los vértices B y D del polígono.

6. Construir un cuadrado conocido uno de sus lados AB


Haciendo centro en A (o en B) se traza un arco de radio AB. Por B se levanta una perpendicular a AB que corta al arco
trazado en el vértice D. Luego, con idéntico radio (AB) se hace centro sucesivamente en B y D y se trazan arcos que se
cortan en el punto C.

CRISTIANA, SOCIALISTA, SOLIDARIA !


MINISTERIO DE EDUCACIÒN
CENTRO CIVICO MODULO “L” PLANTA ALTA
Dirección General de Educación Secundaria
Correo: guillem@mined.gob.ni
7. Construir un cuadrado conocida una de sus diagonales AC.
Se determina en primer lugar la mediatriz de AC. El corte de dicha mediatriz con el segmento AC es el centro del cuadrado
O. Haciendo luego centro en O y con radio OA u OC, se traza una circunferencia que corta a la mediatriz de AC en los
puntos B y D, vértices del polígono.
8. Construir un cuadrado conocidos M, punto medio del lado AB, y N, punto medio del lado BC
En primer lugar, se determina la mediatriz y el punto medio K del segmento MN. Luego, con centro en K y radio KM o KN,
se traza una circunferencia que, al cortar la mediatriz de MN, define las dos posibles soluciones para el punto B. Luego, se
prolongan los segmentos BM y BN copiando la distancia BM (o BN). De esta manera se obtienen los vértices A y C del
polígono. Por último, se dibujan dos arcos de radio AB y centros en A y C, respectivamente; el corte de tales arcos resulta
en el vértice D del cuadrado.

9. Construir un pentágono regular conocido el centro O y el vértice A


Se comienza dibujando una circunferencia de centro en O y radio OA. Luego, se traza el diámetro perpendicular a OA y se
dibuja un arco con radio OA y centro en cualquiera de los extremos del mencionado diámetro. La unión de los cortes entre
el arco y la circunferencia define al punto M. Seguidamente, haciendo centro en M y con radio MA se dibuja un arco que
corta al diámetro en el punto K; la distancia AK corresponde al tamaño de los lados del pentágono.
10. Construir un pentágono regular conocido el lado AB
Se comienza trazando por B una perpendicular a AB y dibujando un arco de centro en B y radio BA, el cual corta a la
perpendicular en el punto 1. Luego, con centro en M, punto medio de AB, y radio M1 se dibuja un arco que corta a la
prolongación de AB en el punto 2. En seguida se dibuja un arco de centro en A y radio A2; el corte entre este arco y el

CRISTIANA, SOCIALISTA, SOLIDARIA !


MINISTERIO DE EDUCACIÒN
CENTRO CIVICO MODULO “L” PLANTA ALTA
Dirección General de Educación Secundaria
Correo: guillem@mined.gob.ni
primero que se realizó define al vértice C del pentágono. A continuación se levanta por M una perpendicular a AB y se
dibuja un arco con centro en C y radio AB; el corte entre estos elementos define a D.
Finalmente, manteniendo una abertura de compás igual a AB, se trazan arcos haciendo centro en D y A sucesivamente; el
corte entre ellos resulta ser el vértice E.

11. Construir un pentágono regular conocidos los vértices A y C


En primer lugar se levanta por A una perpendicular al segmento AC y se traza un arco con centro en A y radio AC, el cual
corta a la perpendicular en el punto 1. Seguidamente se determina el punto medio M del segmento A1 y se construye una
circunferencia con centro en M y radio M1; el corte entre esta circunferencia y la línea recta definida por M y C resulta en el
punto 2.
La distancia C2 es igual al tamaño de los lados del pentágono, por lo tanto, el corte entre dos arcos de radio C2 y con
centros en A y C define al vértice B. Por último, se procede como en el ejemplo anterior.
12. Construir un pentágono regular conocido su centro y una recta “m” sobre la cual se encuentra el lado AB
Se traza una perpendicular a la recta “m” por el punto O que corta la corta en R (punto medio de AB). Luego, se construye
una circunferencia con centro en O y radio OR (inscrita en el pentágono); el corte entre la prolongación de OR y la
circunferencia es el punto D. A continuación se construye el pentágono A’B’C’D’E’ con centro en O y vértice en D’. Luego,
se trazan las líneas definidas por OA’ y OB’; el corte entre éstas y la recta “m” son los vértices A y B del pentágono
buscado. Por último se procede como en los ejemplos 9 y 10 o aplicando semejanza de figuras planas, pues el pentágono
ABCDE es semejante al pentágono ’B’C’D’E’ y tienen igual centro..

CRISTIANA, SOCIALISTA, SOLIDARIA !


MINISTERIO DE EDUCACIÒN
CENTRO CIVICO MODULO “L” PLANTA ALTA
Dirección General de Educación Secundaria
Correo: guillem@mined.gob.ni
13. Construir un hexágono regular conocidos el centro O y uno de sus vértices A
Primero se traza una circunferencia con centro en O y radio OA; el radio de esta circunferencia es igual ala longitud de los
lados del hexágono. Luego, con el mismo radio OA y centro en A se dibujan arcos que cortan a la circunferencia en B y F.
Seguidamente, con centro en B y F se trazan otros arcos – con radio OA – que generan los vértices C y E. Finalmente, si se
prolonga el radio OA, se obtiene sobre la circunferencia el vértice D.
14. Construir un hexágono regular conocido uno de sus lados AB
Se comienza dibujando arcos con centro en A y B de radio AB; el punto de corte entre ellos es el centro del hexágono O.
Luego se procede como en el ejemplo anterior.

15. Construir un hexágono regular conocidos M, punto medio de AB y N, punto


medio del lado CD
En primer lugar, se construye un par de arcos de radio MN haciendo centro en los puntos My N de manera sucesiva; los
puntos comunes a ambos arcos son las dos posibles soluciones para el punto P, punto medio del lado EF (en la figura se
ha dibujado la solución de la izquierda). Luego, hallando mediatrices, se determina el centro O del triángulo equilátero MNP,
el cual es también centro del hexágono, y se traza la circunferencia circunscrita al triángulo MNP que es también la
circunferencia inscrita en el hexágono.
Seguidamente, se construyen por cada uno de los vértices del triángulo MNP líneas paralelas al correspondiente lado

CRISTIANA, SOCIALISTA, SOLIDARIA !


MINISTERIO DE EDUCACIÒN
CENTRO CIVICO MODULO “L” PLANTA ALTA
Dirección General de Educación Secundaria
Correo: guillem@mined.gob.ni
opuesto; sobre tales líneas se sitúan tres de los lados del hexágono buscado. Los puntos de corte (Q, R y S) entre las
mediatrices de los lados del triángulo MNP y la circunferencia inscrita en el hexágono representan los puntos medios de los
lados BC, DE y FA, de manera que por cada uno de ellos se trazan perpendiculares a la mediatriz que lo ha generado,
dando lugar a las rectas sobre las que se hallan los lados BC, DE y FA. Finalmente, los cortes entre rectas adyacentes
resultan en los vértices del hexágono pedido

16. Construir un hexágono regular conocido el centro y una recta “m” que contiene
al lado AB
Se comienza trazando líneas que pasen por O y formen 60º con la recta “m”; los puntos de corte entre dichas líneas y la
propia recta “m” son los vértices A y B del polígono. Luego se procede como en el ejemplo Nº 13

K. CONSTRUCCIONES DE POLÍGONOS REGULARES DADA LA CIRCUNFERENCIA CIRCUNSCRITA


La construcción de polígonos inscritos en una circunferencia dada, se basan en la división de dicha circunferencia
en un número partes iguales. En ocasiones, el trazado pasa por la obtención de la cuerda correspondiente a cada
uno de esos arcos, es decir el lado del polígono, y otras ocasiones pasa por la obtención del ángulo central del
polígono correspondiente.

Cuando en una construcción obtenemos el lado del polígono, y hemos de llevarlo sucesivas veces a lo largo de la
circunferencia, se aconseja no llevar todos los lados sucesivamente en un solo sentido de la circunferencia, sino,
que partiendo de un vértice se lleve la mitad de los lados en una dirección y la otra mitad en sentido contrario, con
objeto de minimizar los errores de construcción, inherentes al instrumental o al procedimiento.

CRISTIANA, SOCIALISTA, SOLIDARIA !


MINISTERIO DE EDUCACIÒN
CENTRO CIVICO MODULO “L” PLANTA ALTA
Dirección General de Educación Secundaria
Correo: guillem@mined.gob.ni
TRIÁNGULO, HEXÁGONO Y DODECÁGONO (construcción exacta)
Comenzaremos trazando dos diámetros perpendiculares entre sí, que nos
determinarán, sobre la circunferencia dada, los puntos A-B y 1-4
respectivamente.

A continuación, con centro en 1 y 4 trazaremos dos arcos, de radio igual al de la


circunferencia dada, que nos determinarán, sobre ella, los puntos 2, 6, 3 y 5. Por
último con centro en B trazaremos un arco del mismo radio, que nos determinará
el punto C sobre la circunferencia dada.

Uniendo los puntos 2, 4 y 6, obtendremos el triángulo inscrito. Uniendo los punto


1, 2, 3, 4, 5 y 6, obtendremos el hexágono inscrito. Y uniendo los puntos 3 y C,
obtendremos el lado del dodecágono inscrito; para su total construcción solo
tendríamos que llevar este lado, 12 veces sobre la circunferencia.

De los tres polígonos, solo el dodecágono admite la construcción de estrellados, concretamente del estrellado de 5. El
hexágono admite la construcción de un falso estrellado, formado por dos triángulos girados entre sí 60º.

CUADRADO Y OCTÓGONO (construcción exacta)

Comenzaremos trazando dos diámetros perpendiculares entre sí, que nos determinarán, sobre la circunferencia dada, los
puntos 1-5 y 3-7 respectivamente.

A continuación, trazaremos las bisectrices de los cuatro ángulos de 90º,


formados por la diagonales trazadas, dichas bisectrices nos determinarán
sobre la circunferencia los puntos 2, 4, 6 y 8.

Uniendo los puntos 1, 3, 5 y 7, obtendremos el cuadrado inscrito. Y uniendo


los puntos 1, 2, 3, 4, 5, 6, 7 y 8, obtendremos el octógono inscrito.

El cuadrado no admite estrellados. El octógono sí, concretamente el


estrellado de 3. El octógono también admite la construcción de un falso
estrellado, compuesto por dos cuadrados girados entre sí 45º.

NOTA: De esta construcción podemos deducir, la forma de construir un


polígono de doble número de lados que uno dado. Solo tendremos que
trazar las bisectrices de los ángulos centrales del polígono dado, y estas
nos determinarán, sobre la circunferencia circunscrita, los vértices necesarios para la construcción

CRISTIANA, SOCIALISTA, SOLIDARIA !


MINISTERIO DE EDUCACIÒN
CENTRO CIVICO MODULO “L” PLANTA ALTA
Dirección General de Educación Secundaria
Correo: guillem@mined.gob.ni
PENTÁGONO Y DECÁGONO (construcción exacta

Comenzaremos trazando dos diámetros perpendiculares entre sí, que nos


determinarán sobre la circunferencia dada los puntos A- B y 1-C
respectivamente. Con el mismo radio de la circunferencia dada trazaremos un
arco de centro en A, que nos determinará los puntos D y E sobre la
circunferencia, uniendo dichos puntos obtendremos el punto F, punto medio del
radio A-O

Con centro en F trazaremos un arco de radio F-1, que determinará el punto


G sobre la diagonal A-B. La distancia 1-G es el lado de pentágono inscrito,
mientras que la distancia O-G es el lado del decágono inscrito.

Para la construcción del pentágono y el decágono, solo resta llevar dichos


lados, 5 y 10 veces respectivamente, a lo largo de la circunferencia.

El pentágono tiene estrellado de 2. El decágono tiene estrellado de 3, y un


falso estrellado, formado por dos pentágonos estrellados girados entre sí 36º.

HEPTÁGONO (construcción aproximada)

Comenzaremos trazando una diagonal de la circunferencia dada, que nos


determinará sobre ella puntos A y B.

A continuación, con centro en A, trazaremos el arco de radio A-O, que nos


determinará, sobre la circunferencia, los puntos 1 y C, uniendo dichos puntos
obtendremos el punto D, punto medio del radio A-O. En 1-D habremos
obtenido el lado del heptágono inscrito.

Solo resta llevar dicho lado, 7 veces sobre la circunferencia, para obtener
el heptágono buscado. Como se indicaba al principio de este tema, partiendo
del punto 1, se ha llevado dicho lado, tres veces en cada sentido de la
circunferencia, para minimizar los errores de construcción.

El heptágono tiene estrellado de 3 y de 2.

NOTA: Como puede apreciarse en la construcción, el lado del heptágono inscrito en una circunferencia, es igual a la
mitad del lado del triángulo inscrito.

ENEÁGONO (construcción aproximada)

CRISTIANA, SOCIALISTA, SOLIDARIA !


MINISTERIO DE EDUCACIÒN
CENTRO CIVICO MODULO “L” PLANTA ALTA
Dirección General de Educación Secundaria
Correo: guillem@mined.gob.ni
Comenzaremos trazando dos diámetros perpendiculares, que nos determinarán,
sobre la circunferencia dada, los puntos A-B y 1-C respectivamente.

Con centro en A, trazaremos un arco de radio A-O, que nos determinará, sobre
la circunferencia dada, el punto D. Con centro en B y radio B-D, trazaremos un
arco de circunferencia, que nos determinará el punto E, sobre la prolongación de la
diagonal 1-C. Por último con centro en E y radio E-B=E-A, trazaremos un arco de
circunferencia que nos determinará el punto F sobre la diagonal C-1. En 1-F
habremos obtenido el lado del eneágono inscrito en la circunferencia.

Procediendo como en el caso del heptágono, llevaremos dicho lado, 9 veces


sobre la circunferencia, para obtener el heptágono buscado.

El eneágono tiene estrellado de 4 y de 2. También presenta un falso estrellado,


formado por 3 triángulos girados entre sí 40º

DECÁGONO (construcción exacta)

Comenzaremos trazando dos diámetros perpendiculares, que nos determinarán,


sobre la circunferencia dada, los puntos A-B y 1-6 respectivamente.

Con centro A, y radio A-O, trazaremos un arco que nos determinará los
puntos C y D sobre la circunferencia, uniendo dichos puntos, obtendremos el
punto E, punto medio del radio A-O. A continuación trazaremos la circunferencia
de centro en E y radio E-O. Trazamos la recta 1-E, la cual intercepta a la
circunferencia anterior en el punto F, siendo la distancia 1-F, el lado del
decágono inscrito.

Procediendo con en el caso del heptágono, llevaremos dicho lado, 10 veces


sobre la circunferencia, para obtener el decágono buscado.

El decágono como se indicó anteriormente presenta estrellado de 3, y un falso estrellado, formado por dos pentágonos
estrellados, girados entre sí 36º.

CRISTIANA, SOCIALISTA, SOLIDARIA !


MINISTERIO DE EDUCACIÒN
CENTRO CIVICO MODULO “L” PLANTA ALTA
Dirección General de Educación Secundaria
Correo: guillem@mined.gob.ni
Solucionario de la Guía de Autoestudio 2014
Geometría Euclidiana y
Geometría Analítica

Para Docentes de Educación Secundaria

g
h

r
UNIDAD DE GEOMETRÍA EUCLIDIANA

1. En la …gura, el ]COB = 120o y el ]COD mide la mitad del ángulo BOA. Entonces, la medida del ]BOA es:

Solución:

Sea m\BOA = x, luego m\COD = x2 . Se tiene

x
+ 120o + x = 180o
2
3x
= 60o
2
x = 40o

2. Si dos planos diferentes se intersecan, su intersección es:

Solución:

Por uno de los axiomas de la Geometría Euclidiana si dos planos diferentes se intersecan, su intersección es una
única recta.
!?m
3. . En la …gura, m !, m
!?m!¿cuál de las siguientes expresiones es siempre verdadera?
1 4 2 3

Solución:

Con la información dada las parejas de rectas perpendiculares están “libres”, luego pueden ser giradas y

2
seguirían satisfaciendo los datos dados. Por tanto no puede a…rmarse ni A, ni B, ni C, ni D.


m1


m2
↔ ↔
m4 m3

4. R; S y T son tres puntos colineales como se muestran en la …gura. Si ST = 4x + 4 y RS es la mitad de ST ,


entonces la longitud de RT es:

Solución:

Dado que los puntos son colineales, se tiene

RT = RS + ST
1
= ST + ST
2
3
= ST
2
3
= (4x + 4)
2
= 6x + 6

5. A partir de la información indicada en la …gura, el valor de Y es:

Solución:

Sean A, B y C los puntos indicados en la …gura, y sean m\BAC = , m\ACB = . Se tiene = 50o , por ser
opuesto por el vértice con el ángulo que mide 50o y = 180o 130o = 50o . El ángulo que mide y o es un ángulo
exterior con respecto al 4ABC, luego su medida equivale a la suma de los ángulos internos no adyacentes, es
decir y = + = 50o + 50o = 100o

3
6. En la …gura, si AB k CD, el valor de X es:

Solución:

Dado que las rectas son paralelas, xo = m\F CE, por ser ángulos correspondientes. A su vez este ángulo por
ser externo al 4ECD, es la suma de las medidas de los ángulos CED y EDC.Se tiene m\CED = 90o y
m\EDC = 180o 140o = 40o , luego x = 90o + 40o = 130o :

7. A partir de la información brindada en la …gura, el valor de Z resulta:

Solución:

Las marcas en el ángulo A, indican que AD es bisectriz de dicho ángulo, luego x = 40o y m\A = 80o . Al
considerar el 4ABC, se tiene z = 180o 80o 70o = 30o .

8. En la …gura, AD ? AC; EB k DC,entonces el valor de Y es:

Solución:

Dado que EBkDC, se tiene x = 180o 130o = 50o por ser ángulos internos al mismo lado, entre paralelas y como
AD?AC el 4ADC es triangulo rectángulo y por tanto “y”es el complemento de “x”, luego y = 90o 50o = 40o .

4
9. En la …gura el valor de X es

Solución:

Se tiene m\ABC = 180o 140o = 40o , x = 115o 40o = 75o ya que el \ACD es externo al 4ABC.

10. En la …gura el valor de X es:

Solución:

Se tiene m\EDB = 180o 150o = 30o , \ABC = \DBE por ser opuestos por el vértice y por el teorema de
semejanza AA, 4ABC 4DBE, luego m\BAC = xo = m\EDB = 30o .

11. A B C D; E y F son puntos medios de AB y CD respectivamente; Si AC = 10 y BD = 12, entonces


EF =?

Solución:

Sean AE = EB = x, CF = F D = y (E y F son puntos medios de AB y CD respectivamente).Se tiene:

BC = AC AB = 10 2x (1)

y también
BC = BD CD = 12 2y (2)

5
Igualando (1) y (2): 10 2x = 12 2y: Al simpli…car se obtiene:

y x=1 (3)

Por otro lado se tiene

EF = EB + BC + CF = x + (10 2x) + y = 10 x + y = 10 + (y x)

Al introducir (3) resulta


EF = 10 + 1 = 11

o o
12. En la …gura + = 255o , entonces ¿m\A =?

Solución:

En el 4ABC, tenemos que

m\A = 180o m\ABC m\ACB = 180o (m\ABC m\ACB) (1)

Se tiene que m\ABC = 180o y m\ACB = 180o , luego m\ABC + m\ACB = 360o ( + ); Como
o o o
+ = 255, resulta m\ABC + m\ACB = 360 255 = 105 ;

Sustituyendo en (1) obtenemos m\A = 180o 105o = 75o

13. ¿Para qué valor de x, los segmentos ABy CD son paralelos?

Solución:

Como el ángulo a la izquierda de C es congruente con el ángulo a la derecha, también mide 25o . Luego
m\ACD = 180o 2 (25o ) = 130o .

Como el 4AP C, es recto en P , m\P AC = 90o 25o = 65o .

6
Para que AB y CD sean paralelos, el ángulo CAB debe ser el suplemento del ángulo ACD ya que serían
ángulos internos al mismo lado entre paralelas o sea m\CAB = 180o 130o = 50o .

Se tiene entonces

x + 50 + 65 = 180
x = 180 50 65
x = 65

14. Si AB k CD, ¿cuál es el valor de X?

Solución:

Trazamos EF , paralela a las rectasAB y CD, luego m\AEF = 180o 120o = 60o y m\F EC = 180o xo .
Además se tiene m\AEF + m\F EC = m\AEC = 90o , luego 60o + (180o xo ) = 90o . Al despejar x, resulta
xo = 150o .

15. Si la medida de un] es tres veces la medida de su suplemento, ¿cuál es la medida de dicho ]?

Solución:

Sean la medida del ángulo buscado y la medida de su suplemento, luego + = 180o ) = 180o .

El ejercicio indica que = 3 , luego

= 3(180o )
= 540o 3
o
4 = 540
= 135o

16. . Dos veces la medida de un ] es 30 menos que cinco veces la medida de su complemento, ¿cuál es la medida
de dicho ángulo?

Solución:

Sean la medida del ángulo buscado y la medida de su complemento, luego

+ = 90o
= 90o

7
Al interpretar la información del ejercicio se tiene

2 = 5 30o
2 = 5 (90o ) 30o
2 = 450o 5 30o
7 = 420o
= 60o

!ym
17. En la …gura las rectas m ! son paralelas. Entonces el valor de x es:
1 2

Solución:

!ym
Sean A, B, C y D los puntos indicados en la …gura. Al trazar por B una paralela a m ! , se forman ángulos
1 2

alternos –internos entre paralelas, y por tanto congruentes con los ángulos indicados inicialmente, luego

x + 60 = 110
x = 110 60
x = 50

Otra Forma:

!. Se tiene m\ADB = xo , por ser alterno –interno


Al prolongar CB, sea D el punto donde corta a la recta m1

con el ángulo que se forma en C. El ángulo ABC es externo al 4ABD, luego 60 + x = 110 ) x = 50.
!ym
18. En la …gura las rectas m ! son paralelas. Entonces el valor de x es:
1 2

8
Solución:

Como las rectas son paralelas se tiene:

(3x + 10) + (x 6) = 84
4x + 4 = 84
4x = 80
x = 20

19. Si m\P = 90o ; \1 = \2; \3 = \4, entonces m\R es

Solución:

Sean y las medidas de los ángulos indicados en la …gura. Se tiene + = 90o . Como \SQR = \2 y
\1 = \2, se tiene
2 m\SQR = 180o (1)

Similarmente se obtiene que


2 m\QSR = 180o (2)

Al sumar (1) y (2) resulta

2 m\SQR + 2 m\QSR = (180o ) + (180o )


o
= 360 ( + )

Como + = 90o ,

2(m\SQR + m\QSR) = 360o 90o


2(m\SQR + m\QSR) = 270o
m\SQR + m\QSR = 135o

Luego m\R = 180o (m\SQR + m\QSR) = 180o 135o = 45o

20. En una recta se toman los puntos A; B y C, de manera que B es punto medio de . Se toma otro punto O, tal
AO OC
que B O C. Encuentre el valor numérico de:
OB

Solución:

9
Se tiene AB = BC = x, por ser B punto medio de AC. Sea OB = y, luego OC = x y, AO = x + y. Al
sustituir estos valores en la expresión dada se tiene: AO OC = (x y) (x + y) = 2y, luego

AO OC 2y
= =2
OB y

Nota: en ejercicios de este tipo no se admite asignar valores arbitrarios, ya que se estaría resolviendo para
valores especí…cos. El planteamiento es general. Cuando se a…rma que B O C, está indicando que O es un
punto cualquiera que se encuentra entre B y C, y el valor numérico encontrado es valido para cualquier punto
O que esté entre B y C.

21. Un poste cercano a un árbol mide 2m y su sombra en un momento dado mide 1:8m, entonces si la sombra del
árbol en ese momento mide 11m, la altura del árbol es:

Solución:

Dado que los rayos del sol prácticamente caen paralelos y que el poste y el tronco del árbol son perpendiculares
al piso, el árbol y su sombra y la línea que une sus extremos forman un triángulo semejante al formado por el
poste su sombra y la línea que une sus extremos, tenemos
h 2
=
11 1:8
11:2
h =
1:8
h = 12:22

22. Una varilla clavada en el piso y cercana a un árbol mide 3m y su sombra mide 1:5m, entonces si el árbol mide
36m, su sombra mide.

Solución:

El problema es similar al anterior, en este caso se tiene


x 1:5
=
36 3
36 1:5
x =
3
x = 18

23. El perímetro de un triángulo rectángulo isósceles con hipotenusa igual a 10 redondeado a dos decimales es

Solución:

p
En un triángulo rectángulo isósceles, la hipotenusa mide 2x, siendo x la longitud de sus catetos, luego
p
2x = 10
10
x = p
2
p
x = 5 2

10
Su perímetro será
p
P = 10 + 2 5 2
p
= 10 + 10 2
= 24:14

24. En el triángulo rectángulo de la …gura, los valores de x y y, respectivamente son

Solución:

Por el teorema de la altura se tiene

4x = 82 = 64
x = 16

y la hipotenusa mide 4 + x = 20. Por el teorema de los catetos se tiene

y2 = 4 20 = 80
p
y = 80
p
y = 4 5
y 8:94

25. Un método para encontrar la altura de un edi…cio es colocar un espejo en el suelo y después situarse de manera
que la parte más alta del edi…cio pueda verse en el espejo ¿qué altura tiene un edi…cio si una persona cuyos
ojos están a 1:5m del piso observa la parte superior del edi…cio cuando el espejo está a 120 m del edi…cio y la
persona está a6m del espejo?

Solución:

Dado que las leyes de la óptica indican que en un espejo plano, el ángulo de incidencia es igual al ángulo de
re‡exión, se forman dos triángulos rectángulos semejantes, luego

h 1:5
=
120 6
h = 30 m

26. La altura respecto a la hipotenusa de un triángulo rectángulo mide 10m y los segmentos que determina sobre
la hipotenusa son entre sí como 7 es a 14. Entonces la longitud del cateto menor es

11
Solución:

Sean m y n los segmentos determinados por la altura sobre la hipotenusa, con m < n, luego
m 7
=
n 14
n = 2m

Por el teorema de la altura

m n = 102
m 2m = 100
m2 = 50
p
m = 5 2
p
n = 10 2
p
La hipotenusa mide c = m + n = 15 2

Por el teorema de los catetos

a2 = m (m + n)
p p
a2 = 5 2 15 2
a2 =
150
p p
a = 150 = 5 6
a 12:25

27. El perímetro de un rectángulo es 85m y su diagonal mide M . Por lo tanto los lados del rectángulo miden:

Solución:

Sean a y b los lados del rectángulo. Se tiene

P = 2 (a + b) = 85
a+b = 42:5 (1)

Además
a2 + b2 = 32:52 = 1056:25 (2)

Despejando b de (1), e introduciendo en (2)


2
a2 + (42:5 a) = 1056:25
2
2a 85a + 750 = 0
a = 12:5 _ a = 30

12
Al sustituir en (1) se obtiene b = 30 _ b = 12:5

28. El perímetro de un triángulo mide50 y sus lados son proporcionales a 4; 6 y 8. Entonces su lado mayor mide.

Solución:

a b c
Sean a, b y c las longitudes de los lados, con a < b < c, luego P = a + b + c = 50 y = =
4 6 8

Por las propiedades de las proporciones


a b c a+b+c 50
= = = =
4 6 8 4+6+8 18
8 50 200
c = =
18 9
p p
29. En un triángulo rectángulo, un lado mide 2 106, otro 5 15. Si el lado desconocido es el menor, ¿cuánto
mide?

Solución:

p p p
Como 2 106 > 5 15, la hipotenusa de este triángulo es 2 106, luego el cateto menor es
r
p 2 p 2 p p
a= 2 106 5 15 = 424 375 = 49 = 7

30. El área del triángulo de la …gura, redondeada al entero más cercano, mide:

Solución:

p
Aplicamos la fórmula de Herón: A = s (s a) (s b) (s c), donde s es el semiperímetro. Se tiene s =
6+7+9
= 11, luego
2
p p p
A = 11 (11 6) (11 7) (11 9) = 11 5 4 2 = 440 20:97

31. ¿Cuál es el área del triángulo de la …gura?

Solución:

13
Dado que es un triángulo rectángulo su área es la mitad del producto de sus catetos. El cateto desconocido
mide
p p p
b= 102 62 = 100 36 = 64 = 8
1
Por tanto A = 2 (6) (8) = 24

32. Si un rectángulo de 3mde ancho y 10mde largo tiene la misma área que un triángulo rectángulo isósceles,
entonces la longitud de cada cateto del triángulo es

Solución:

El área de un triángulo rectángulo isósceles está dada por A = 12 x2 , donde x es la longitud de sus catetos, luego
tenemos que el área del rectángulo es 30, por tanto
1 2
x = 30
2 p
x = 60
p
x = 2 15

33. El área de un trapecio isósceles de bases 22m y 10m y cuyos lados congruentes miden 10 es

Solución:

Por ser un trapecio isósceles, al proyectar la base menor sobre la base mayor, la base mayor queda dividida en
tres segmentos de 6, 10 y 6 metros. Aplicando el teorema de Pitágoras, se tiene
p p
h = 102 62 = 64 = 8
(B+b) h
Aplicando la fórmula para el área de un trapecio: A = 2 resulta
(22 + 10) 8
A= = 128 m2
2
34. La siguiente …gura consta de siete cuadrados congruentes. El área total de esta …gura es 63cm2 . Entonces el
perímetro de la …gura es:

14
Solución:

Observamos que el perímetro está formado por 16 veces el lado de cada cuadrado. Como hay siete cuadrados
congruentes, cada uno tiene un área de
63
x2 = =9
7
x = 3

Por tanto el perímetro de la …gura es P = 16 3 = 48cm.

35. Si ACEG es un cuadrado y el área del cuadrilátero BDF H mide 162 ¿cuánto mide AC? (las marcas iguales
representan partes congruentes).

Solución:

La …gura indica que B, D, F y H son puntos medios de los lados del cuadrado ACEG, luego su área es el
p
doble del área del cuadrado BDF H, es decir [ACEG] = 2 162 = 324 luego AC = 324 = 18

36. Se tiene un trapecio ABCD donde es la base menor. BC = 10cm y CD = 20cm. Las medidas de los ángulos
A; B y C son 30 ; 150 y 120 respectivamente, entonces AD =?

Solución:

Sean B0 y C0 las proyecciones de B y C sobre la base mayor y sean AB0 = x, C0D = y. Por ser BC paralela a
AD,
m\D = 180 m\C = 180o 120o = 60o
p
El 4CC0D es un triángulo 30 – 60, luego h = CC0 = 10 3 y y = C0D = 10. También el 4AB0B resulta ser
p p p
un triángulo 30 –60, con su cateto menor BB0 = h = 10 3, luego AB0 = 10 3 3 = 30.

Tenemos entonces que la base mayor mide

AD = x + 10 + y = 30 + 10 + 10 = 50

15
37. Si las medianas en un triángulo rectángulo, trazadas a partir de los vértices de los ángulos agudos miden 5cm
p
y 40cm, entonces la medida de la hipotenusa del triángulo rectángulo es.

Solución:

p
Sean M y N los puntos medios de BC y AB respectivamente. Sean AM = 5 y CN = 40, BC = a, AB = c,
a c
luego BM = y N B = .
2 2

Sea la hipotenusa AC = b: Aplicando el teorema de Pitágoras en los 4ABM y 4BCN


a2
AM 2 = AB 2 + BM 2 = c2 + = 25 (1)
4
c2
CN 2 = N B 2 + BC 2 = a2 + = 40 (2)
4
Al sumar (1) y (2) resulta
5c2 5a2
+ = 65
4 4
4
a2 + c2 = (65) = 52 = b2
5
p p
b = 52 = 2 13

38. En la …gura, los cuadrados ABCD y EF GH son congruentes. AB = 10cm y G es el centro del cuadrado
ABCD. Entonces el área total cubierta por el polígono AHEF BCDA es.

Solución:

Dado que los cuadrados son congruentes sus áreas son iguales y como el lado AB mide 10, cada uno tiene un
área de 100cm2 , pero ellos comparten el 4ABG de manera que para el área total del polígono a la suma de
las áreas de los cuadrados debemos restarle el área de este triángulo para que sea considerada solo una vez.

Dado que G es el centro del cuadrado ABCD, el área del triángulo es la cuarta parte del área del cuadrado o
sea 25cm2 . Luego el área buscada es

A = [ABCD] + [EF GH] [ABG] = 100 + 100 25 = 175cm2

16
39. ABCD es un cuadrado, el 4ABE es isósceles, CF = F B. Entonces, la medida del ángulo EF B es igual a.

Solución:

Como el 4ABE es isósceles, AE = BE y por ser ABCD un cuadrado, E es el punto medio de DC y por
tanto EC = CF , ya que por ser CF = F B, F es punto medio de BC. Luego el 4ECF resulta ser triangulo
rectángulo isósceles y como consecuencia m\CF E = 45o . El ángulo buscado es el suplemento del \CF E,
luego
m\EF B = 180o m\CF E = 180o 45o = 135o

40. En la …gura, ABCF es un paralelogramo. B; C y D son colineales. Si AB = 18; AD = 30 y F E = 12.


¿Cuánto mide AE?

Solución:

Se tiene que CF = AB = 18, ya que ABCF es un paralelogramo. CE = CF F E = 18 12 = 6.

Por otro lado BD y AF son paralelas, luego \F AE = \CDE, ya que son alternos internos entre paralelas y
\F EA = \CED, ya que son opuestos por el vértice. Como consecuencia se tiene 4F EA 4CED.

Sea AE = x, luego ED = AD AE = 30 x. Por la semejanza anterior,


CE ED
=
FE EA
6 30 x
=
12 x
6x = 360 12x
18x = 360
x = 20

41. En un trapecio isósceles, la diferencia de las bases es de 10m. La altura mide 12m. y el perímetro 76m.
Entonces su área es:

17
Solución:

Como B b = 10, al proyectar la base menor sobre la base mayor se forman tres segmentos de longitudes 5, b
y 5 como se muestra en la …gura. Luego como la altura es 12, en los extremos del trapecio se forman triángulos
rectángulos de catetos 5 y 12. Aplicando el teorema de Pitágoras hallamos que la hipotenusa mide 13 lo cual
corresponde a la longitud de los lados no paralelos del trapecio. Considerando que el perímetro mide 76 m. se
tiene:

2b + 2(13) + 2(5) = 76
b = 20

Al considerar que B b = 10, resulta B = 30. Aplicando la fórmula para el área de un trapecio, el área buscada
resulta
(B + b) h
A =
2
(20 + 30) 12
=
2
= 300 m2

42. En la …gura ABCD es un cuadrado de lado 1cm y CE = 2cm, entonces el área del triángulo ADF en cm2 es
igual a

Solución:

Dado que ABCD es un cuadrado AD y CE son paralelas, resultando que 4ADF 4ECF por el teorema de
semejanza AA, ya que \DAF = \CEF por ser alternos internos entre paralelas y \DF A = \CF E por ser
opuestos por el vértice.

De la semejanza resulta que


AD DF
=
CE CF
1 DF
=
2 CF
CF = 2DF (1)

Como CD = CF + F D = 1, resulta DF = 13 . Por tanto el área buscada resulta


1 1 1 1
[ADF ] = AD DF = 1 =
2 2 3 6

18
43. Sea ABC un triángulo isósceles con AB = BC = 10 y AC = 16. Sea BD la mediana trazada sobre el lado AC
y sea G el baricentro. Entonces el área del triángulo ADG es

Solución:

Por ser BD mediana, D es punto medio de AC, o sea AD = DC = 8. Ya que 4ABC es isósceles, BD además
de mediana también es altura, luego m\ADB = 90o . Aplicando el teorema de Pitágoras hallamos que
p
BD = 102 82 = 6

Como G es el baricentro, BG = 2 GD y como BD = BG + GD = 6, resulta GD = 2. Por tanto el 4ADG


resulta ser un triángulo rectángulo con catetos de longitudes 8 y 2, por tanto su área es
1 1
[ADG] = AD DG = 8 2=8
2 2

44. Sea ABC un triángulo isósceles con AB = AC = 17cm y P un punto cualquiera del lado BC, diferente de los
puntos extremos. Por P se trazan una paralela a AC que corta a AB en Q y una paralela a AB que corta a
AC en R. El perímetro del cuadrilátero AQP R es.

Solución:

Dado que QP kAC y RP kAB, AQP R es un paralelogramo y de ahí AQ = RP y AR = QP . Del paralelismo de


los segmentos señalados anteriormente también resulta que los 4QBP y 4RP C son semejantes con el 4ABC
y por tanto también son isósceles y de ahí QB = QP y RP = RC. Por tanto el perímetro del cuadrilátero
AQPR, resulta

P = AQ + QP + AR + RP
= (AQ + QB) + (AR + RC)
= AB + AC
= 17 + 17
= 34
63

19
45. De acuerdo a la información que se proporciona en la …gura, el segmento de mayor longitud es.

Solución:

Dado que la suma de los ángulos internos de un triángulo suman 180o , en el 4ABD, resulta que el ángulo
ABD mide 180o 70o 60o = 50o y en el 4BDC, m\BDC = 180o 55o 60o = 65o .

Una de las propiedades de los triángulos indica que el lado mayor se opone al ángulo mayor y viceversa. Al
comparar las medidas de los ángulos del 4ABD, resulta que el mayor mide 70o y su lado opuesto es BD, luego
BD es mayor que AB y AD. Pero al considerar el 4BDC, su ángulo mayor es 65o y el lado que se le opone
es BC y por tanto BC > BD. Luego el lado mayor de la …gura resulta BC.

46. En la …gura ABCD es un cuadrado de lado 1; 4CM N es equilátero. El área de 4CM N es igual a.

Solución:
p
3 2
El área de un triángulo equilátero está dada por 4 x , donde x es la longitud de su lado, luego debemos
encontrar primero cuanto mide cada lado del triángulo equilátero CM N .

Como ABCD es un cuadrado y CM = CN = x, se tiene que 4CDM = 4CBN y de ahí M D = N B y como


AD = AB, resulta AM = AN y por tanto el 4M AN es rectángulo isósceles, luego

MN = x
p
x = 2AN
x
AN = p
2
p
x 2 x
Como AB = 1, resulta N B = 1 AN = 1 p
2
= p
2
. El 4CBN es un triángulo rectángulo luego al aplicar
el teorema de Pitágoras resulta
p !2
2 2 2 2 x
CN = CB + N B = 1 + p = x2
2

20
p p
Al desarrollar, simpli…car y resolver la ecuación resultante se obtiene x = 6 2. Por tanto el área buscada
es p
3 p p 2
[CM N ] = 6 2 0:4641
4
47. La siguiente …gura muestra dos cuadrados de lado 1cm, donde AEF G se ha obtenido de ABCD al girar este
cuadrado 45 sobre el vértice A. Entonces el área sombreada es.

Solución:

Al girar 45o , la recta diagonal AC se convierte en la recta AB la cual equivale a la recta diagonal AF , por
tanto A, B, F son colineales. Además se tiene m\BF H = 45o y por tanto F BH es un triángulo rectángulo
isósceles con F B = BH.

p
Luego [AGHB] = [AGF ] [F BH]. Por ser AEF G un cuadrado de lado 1, su diagonal mide 2 y como
p
AB = 1, BF = BH = 2 1. Como [AGF ] tiene como área la mitad de la área del cuadrado, que tiene lado
de longitud 1, resulta
1 1 p 2 1 1 p p
[AGHB] = 2 1 = 2 2 2+1 = 2 1
2 2 2 2

48. Los ángulos agudos de un triángulo rectángulo, que también es isósceles, miden

Solución:

Por ser triángulo rectángulo isósceles tiene un ángulo de 90o y los otros dos ángulos congruentes, y dado que
la suma de los ángulos internos de un triángulo suman 180o , cada uno de ellos mide 45o .

49. En la …gura ABCD es un cuadrilátero con AD kBC . La diagonal AC es perpendicular al lado CD .m\BAC =
p
30 ; AC = 4 3 y AB = BC. Entonces el área de ABCD es igual a.

21
Solución:

p
Como AB = BC, el 4ABC es isósceles con m\ABC = 120o y m\BCA = 30o y su base AC = 4 3. Al trazar
una perpendicular desde B a AC, sea E el pie de la perpendicular.

Por ser isósceles, BE también es mediana es decir E es punto medio de AC, luego se forman dos triángulos 30
p
–60 con las hipotenusas AB = BC y catetos mayor AE = EC = AC 2 = 2 3: Luego como el cateto mayor en
un triángulo 30 –60, es veces el cateto menor, en este caso se tiene BE = 2.

Como ADkBC el \BAD es el suplemento del \ABC, resulta m\BAD = 180o 120o = 60o y como m\BAC =
p
30o , se tiene m\CAD = 30o y de ahí también el 4ADC es un triángulo 30 –60 con cateto mayor AC = 4 3.
p
Como en todo triangulo 30 –60, el cateto mayor es 3 el cateto menor, se tiene CD = 4. Finalmente tenemos

[ABCD] = [ABC] + [ACD]


1 1
= AC BE + AC CD
2 2
1 p 1 p
= 4 3 2+ 4 3 4
2p 2
= 12 3

50. Se tiene un trapecio ABCD donde BC es la base menor. BC = 10cm y CD = 20cm. Las medidas de los
ángulos A; B y C son 30 ; 150 y 120 respectivamente, entonces el área del trapecio mide.

Solución:

Sean B0 y C0 las proyecciones de B y C sobre la base mayor y sean AB0 = x, C0D = y. Por ser BC paralela a
AD, m\D = 180 m\C = 180o 120o = 60o .

p
El 4CC0D es un triángulo 30 – 60, luego h = CC0 = 10 3 y y = C0D = 10. También el 4AB0B resulta ser
p p p
un triángulo 30 –60, con su cateto menor BB0 = h = 10 3, luego AB0 = 10 3 3 = 30.

Tenemos entonces que la base mayor mide

AD = x + 10 + y = 30 + 10 + 10 = 50

22
Luego el área del trapecio resulta
p
(50 + 10) 10 3 p
[ABCD] = = 300 3
2

51. En la …gura, m\BAC = ; m\BP C = m y \BQC = 90 : Entonces la medida de \BHC es.

Solución:

Como m\BP C = m\BQC = 90 , también m\AP H = m\AQH = 90 . APHQ es un cuadrilátero convexo y


en todo cuadrilátero convexo la suma de sus ángulos internos es 360o , luego m\BHC + + 90o + 90o = 360o
o
y de ahí m\BHC = 180 .

52. Si las medianas en un triángulo rectángulo, trazadas a partir de los vértices de los ángulos agudos miden 5cm
p
y 20cm, entonces la medida en cm de la hipotenusa del triángulo rectángulo es.

Solución:

p
Sean M y N los puntos medios de BC y AB respectivamente. Sean AM = 20 y CN = 5, BC = a, AB = c,
a c
luego BM = y N B = .
2 2

Sea la hipotenusa AC = b. Aplicando el teorema de Pitágoras en los 4ABM y BCN

a2
AM 2 = AB 2 + BM 2 = c2 + = 20 (1)
4
c2
CN 2 = N B 2 + BC 2 = a2 + = 25 (2)
4

Al sumar (1) y (2) resulta

5c2 5a2
+ = 45
4 4
4
a2 + c2 = (45) = 36 = b2
5
b = 6

23
53. En la …gura, los dos cuadrados tienen el mismo centro. La razón entre el lado del cuadrado menor y el lado del
2
cuadrado mayor es . Entonces la razón entre el área sombreada y el área del cuadrado mayor es.
5

Solución:

b 2
Sean “b”la longitud del lado del cuadrado menor y “a”la longitud del lado del cuadrado mayor, luego = .
a 5

Por la simetría de la …gura se deduce que el área sombreada, es decir el trapecio ABF E, representa la cuarta
parte de la diferencia entre los dos cuadrados, luego
1
[ABF E] = ([ABCD] [EF GH])
4
2
[ABCD] = a y [EF GH] = b2

1 2
y de ahí [ABF E] = a b2 . La razón buscada será
4
1
[ABF E] 4 a2 b2 1 a2 b2 1 b2
= = = 1
[ABCD] a2 4 a2 4 a2

b 2
Como = , resulta
a 5
[ABF E] 1 4 21
= 1 =
[ABCD] 4 25 100

54. En la …gura, AB = AC = 4, BD = DC = 3 y m\BAC = 60 , entonces la longitud del segmento AD es

Solución:

Al unir B con C, obtenemos un triángulo equilátero, ya que AB = AC y m\BAC = 60 . Se tiene que


4ABD = 4ACE, ya que sus tres pares de lados son congruentes, de ahí resulta m\BAD = m\CAD y por
tanto AD es bisectriz del \BAC.

24
Al prolongar AD, sea E el punto donde corta a BC. Luego como el 4ABC es equilátero, AE además de
bisectriz es mediatriz y por tanto AE ? BC y BE = EC = 2. Resulta entonces que el 4BED es rectángulo,
p p
con hipotenusa BD = 3 y un cateto, BE = 2. Por el Teorema de Pitágoras, DE = 32 22 = 5:

p
Por otro lado AE es una altura en un triángulo equilátero de lado 4 y por tanto AE = 2 3. Finalmente
p p
obtenemos que AD = AE DE = 2 3 5:

55. En la …gura el cuadrilátero ACDE es un trapecio tal que ED = 15cm , AC = 24 cm y la altura es 12cm.
Sabiendo que B es el punto medio del lado AC, el área del cuadrilátero OBCD es.

Solución:

AB 12 4
Como EDkAC, resulta que 4ABO 4DEO, con razón de semejanza DE = = .
15 5

Sean a y b las alturas de los triángulos ABO y DEO respectivamente, indicadas en la …gura. Dado que los
a 4
elementos homólogos en triángulos semejantes están en la misma razón de semejanza, se tiene = .
b 5

16 20
Como a + b = 12 (la altura del trapecio), al considerar la razón anterior resulta a = ,b= . Al analizar la
3 3
…gura vemos que [OBCD] = [ACDE] [ABE] [DEO]. Tenemos que el área del trapecio ACDE resulta

24 + 15
[ACDE] = 12 = 234
2

El 4ABE, tiene base 12 y altura 12, luego su área es


1
[ABE] = 12 12 = 72
2

Para el 4DEO, resulta


1 20
[DEO] = 15 = 50
2 3
) [OBCD] = 234 72 50 = 112

56. En la …gura, ABCD es un cuadrado de lado 6cm y CE = DE = 5cm, entonces la longitud de es.

25
Solución:

Sean F y G los puntos medios de CD y BA respectivamente. Luego CF = F D = BG = GA = 3 y F G = 6:

Como CE = DE, el 4CED es isósceles y por tanto E, F , G son colineales y EF ? CD y EG ? AB. El 4CF E
p
es rectángulo en F , luego por el Teorema de Pitágoras, EF = 52 32 = 4. EG = EF + F G = 4 + 6 = 10.

p p
El 4F GA también es rectángulo con EG = 10 y GA = 4, luego EA = 102 + 32 = 109:

57. En la …gura, a partir de la información dada, ¿cuál es el valor de x?

Solución:

Se tiene \A = \E , por dato, y \ACB = \ECD, por ser opuestos por el vértice, luego 4ABC 4EDC,
por el teorema de semejanza AA. Entonces:
CD BC
=
CE AC
x 66
=
10 132
x = 5

58. ABCD es un paralelogramo. P es un punto de la diagonal AC. Trazamos por P paralelas a los lados del
paralelogramo. Estas paralelas intersecan a los lados del paralelogramo en los puntos indicados en la …gura.
Sabiendo que el área de ABCD es 40cm2 , entonces el área del cuadrilátero RQM N es igual a.

Solución:

Dado que RM kADkBC y N QkABkDC, resulta que los cuadriláteros AN P R, P QBR, DM P N y M CQP son
paralelogramos y los segmentos N R, RQ, QM y M N son diagonales de esos paralelogramos. Es sabido que
una diagonal divide a un paralelogramo en dos triángulos congruentes, con áreas igual a la mitad del área del
paralelogramo. Por tanto [RQM N ] = [ABCD] = 20:

26
59. En el triángulo rectángulo ABC¿cuál es la longitud del segmentoBC?

Solución:

Basta aplicar el teorema del cateto:

3x = 62
x = 12

60. Sea ABCD un cuadrado. Por el vértice A se traza un segmento que corta a la prolongación del ladoBC en E,
al lado DC en F y a la diagonal BD en G. Si AG = 3 y GF = 1 ¿cuál es la longitud de F E?

Solución:

Sea x la longitud de cada lado del cuadrado. Desde G tracemos una perpendicular a AD y sea H el pie de esta
perpendicular. Luego 4AGH 4AF D. Como AG = 3 y GF = 1, resulta AF = 4:

De la semejanza se tiene
AG AH
=
AF AD
3 AH
=
4 x
3
AH = x
4
3 1
y HD = AD AH = x x = x:
4 4
Como G está sobre la diagonal, HG = HD = 14 x. De la misma semejanza se tiene
AG HG
=
AF DF
x
3 4
=
4 DF
1
DF = x
3

27
1
Luego F C = DC DF = x 3x = 23 x. Como ABCD es un cuadrado, ADkBC y por tanto ADkCE. De ahí
resulta que 4ADF 4ECF . De esta semejanza se tiene
FE FC
=
FA FD
2
FE 3x
= 1
4 3x
FE = 8

61. En la …gura de abajo si la medida de los arcos AD y BC son 140o y 80 respectivamente, entonces el valor de
es.

Solución:

d + mCD
mAC d
Tenemos que = . Dado que
2
d + mBC
mAB d + mCD
d + mAD
d = 360o y
d + mAD
mBC d = 80o + 140o = 220o

d + mCD
mAC d
d + mCD
d = 140o , luego 140o
Entonces mAC = = 2 = 70o
2
62. El triángulo ABC está inscrito en un semicírculo de diámetro AB. Si AC = 8 y CD = 6, el área de la región
sombreada tiene un valor de.

Solución:

El área sombreada es la diferencia entre el área del semicírculo y el área del triángulo. El 4ABC es rectángulo
en C, por estar inscrito en un semicírculo.

p
Luego por el Teorema de Pitágoras, AB = 82 + 62 = 10, entonces r = 5. Por tanto el área del semicírculo es
1 2 1 25
r = 52 =
2 2 2

El área del triángulo está dada por


1 1
AC BC = 8 6 = 24
2 2
El área buscada es
25
A= 24 15:27
2

28
63. El triángulo ABC está inscrito en un semicírculo de diámetro AB. Si AC = 8 y CD = 4:8, el área de la región
sombreada tiene un valor de

Solución:

p
Por el Teorema de Pitágoras, AD = 82 4:82 = 6:4. Como el 4ABC es rectángulo en C, se tiene por el
teorema del cateto . Y de nuevo por el Teorema de Pitágoras resulta BC = 6. Dado que estos valores coinciden
con los datos del ejercicio anterior, el área resulta la misma.

64. La circunferencia de la …gura tiene radio 2 y el arco XY Z tiene longitud . ¿Cuánto mide la cuerda XZ?

Solución:

Sea la medida del ángulo central XOZ. La longitud de un arco está dada por s = r , con el ángulo medido
s o
en radianes. Tenemos s = y r = 2, luego = r = 2, es decir 90 .

p
Luego XOZ es un triángulo rectángulo isósceles con XZ como hipotenusa y por tanto XZ = 2 2

65. En la …gura el área del círculo mayor es 1 m2 . El círculo menor es tangente internamente al círculo mayor y
también es tangente a los lados del ángulo inscrito que mide 60 . Entonces el área del círculo menor es

29
Solución:

Desde el vértice del ángulo inscrito, trazamos un diámetro. Sean O y O0 los centros de los círculos, mayor y
menor respectivamente. Sea B el otro extremo del diámetro trazado, C el punto donde uno de los lados (el
arriba) del ángulo corta a la circunferencia. Y sea D el punto de tangencia de este lado del ángulo con el círculo
menor.

Sean R y r los radios de los círculos, mayor y menor respectivamente.Tenemos que AO0 biseca al ángulo inscrito,
luego m\O0AD = m\BAC = 30o .

Como AB es un diámetro del circulo mayor, m\ACB = 90o resultando que m\ABC = 60o , y el 4ABC es
30 –60.
Dado que AB = 2R, se obtiene que BC = R, ya que BC es el cateto menor y AB la hipotenusa del 4ABC.
Como AC es tangente al círculo menor en D, AD?DO0, es decir m\ADO0 = 90o y de ahí m\AO0D = 60o .

Luego también el 4AO0D es un triángulo 30 –60 y su cateto menor O0D = r, mide la mitad de su hipotenusa,
AO0. Se tiene O0B = r, por ser radio del circulo menor y de ahí AO0 = AB O0B = 2R r. Luego

AO0 = 2 O0D
2R r = 2r
2
r = R
3

Como el área del círculo mayor es R2 = 1 el área del círculo menor es


2
2R
r2 =
3
4
= R2
9
4
=
9

66. En la …gura C es el centro de la circunferencia de radio r y T P es un segmento tangente en T , de longitud 2r,


entonces P C mide

Solución:

30
Como T P es tangente a la circunferencia en T , m\P T C = 90o . Luego aplicando el teorema de Pitágoras
resulta q
2
p p
PC = (2r) + r2 = 5r2 = r 5

67. Los extremos de la …gura son semicírculos, ¿Cuál es el área de la región sombreada?

Solución:

Como el área sombreada únicamente son los extremos y estos son semicírculos, al unirlos se forma un circulo
de diámetro 8, es decir de radio 4, luego

A = r2 = 42 = 16

68. En la …gura AC es un diámetro. Si m\AB = 50 , entonces m\BAC =?

Solución:

d = 50 , se tiene m\BCA = 1 mAB


Dado que mAB d = 25 , por ser ángulo inscrito que subtiende dicho arco;
2
m\ABC = 90o , por estar inscrito en una semicircunferencia ( es diámetro). Luego la medida del ángulo
buscado es:

m\BAC = 180 m\BCA m\ABC


= 180o 25o 90o
= 65o

69. En la …gura, los círculos son tangentes y tienen radio igual a 10. Si se unen los centros de los círculos se forma
un cuadrado. ¿Cuál es el área de la región sombreada?

31
Solución:

El área de la región sombreada es la diferencia entre el área del cuadrado formado y las áreas de los cuatro
sectores circulares que se forman. Dado que la distancia entre los centros de dos círculos tangentes exterior-
mente, es la suma de las longitudes de los radios, resulta que el cuadrado formado tiene lado de longitud 20 y
por tanto el área del cuadrado es 400.

Cada sector formado tiene un ángulo central de 90o , luego entre los cuatro forman un circulo de radio 10, cuyas
áreas suman entonces r2 = 102 = 100 : Por tanto el área buscada es:

A = 400 100

70. En la …gura, la medida del arco AB es 30 , y la medida del \BP A es 35 . Las medidas del arco CD y el
ángulo DAC (en grados) son respectivamente.

Solución:

Dado que \BP A es un ángulo exterior, formado por dos secantes, su medida es la semidiferencia de los las
medidas de los arcos que intercepta. Es decir

d
mCD d
mAB
m\BP A =
2

d resultando
De esta expresión despejamos mAB,

d
mCD = d
2 m\BP A + mAB
= 2 35o + 30o
= 100o

Por otro lado se tiene que el \DAC es un ángulo inscrito que subtiende el arco DC, luego
1 d 1
m\DAC = mCD = 100o = 50o
2 2

71. La expresión (p + q)p = (r + s)r, se cumple en la situación representada por

32
Solución:

Al recordar las relaciones métricas en una circunferencia, vemos que los productos de esta forma surgen cuando
se tienen dos secantes que se cortan (también aparecen cuando hay semejanzas de triángulos) o una secante y
una tangente que se cortan. A partir de estas relaciones tenemos:

En la …gura a), la relación es r2 = s (s + p) :

En la …gura b), la relación es r(r + s) = p(p + q), la cual es la misma expresión dada. La respuesta es ésta.
Para estar más seguros vemos que resulta en las otras.

En la …gura c), la relación es r s = p q y en la …gura d), r2 = (p + q), que son diferentes a la dada.

Solo b) satisface y por tanto es la respuesta.

72. En la …gura se dan tres semicircunferencias mutuamente tangentes.CD y DA son diámetros de las circunfer-
encias menores. El punto B está en la semicircunferencia mayor. BD ? BC . Si BD = 2; entonces el área
sombreada es igual a.

Solución:

El área de la región sombreada es la diferencia entre el área del semicírculo exterior menos las áreas de los
semicírculos interiores. Sean r1 , r2 , R los radios del semicírculo menor, del semicírculo mediano y del semicírculo
exterior respectivamente. Luego CD = 2r1 , DA = 2r2 y CA = 2R. Como CA = CD + DA, se tiene

2R = 2r1 + 2r2
R = r 1 + r2 (1)

Al unir B con A y con C, se forma un triángulo rectángulo, con CA como hipotenusa y BD como altura
relativa a la hipotenusa. Por el teorema de la altura,

BD2 = CD DA
2
2 = 2r1 2r2
r1 r 2 = 1 (2)

1
Como el área de un semicírculo está dada por 2 r2 , el área buscada es

1
A= R2 r12 r22
2

33
Al considerar (1)
1 h 2
i
A = (r1 + r2 ) r12 r22
2
1
= r12 + 2r1 r2 + r22 r12 r22
2
1
= 2r1 r2
2
= r1 r 2

Al considerar (2)
A = r1 r 2 = 1

73. Las medidas de los arcos AB y AC se indican en la …gura. La medida del \BAC es.

Solución:

El \BAC es un ángulo inscrito en una circunferencia, por tanto su medida es la mitad de la medida del arco
que subtiende, en este caso el arco BC. Tenemos que

d = 360o
mBC d
mAB d = 360o
mAC 110o 130o = 120o

d = 60o
Luego m\BAC = 12 mBC

74. En la …gura, BC une los centros de los círculos tangentes. AB ? BC; BC = 8 y AC = 10, entonces la longitud
de la circunferencia pequeña es igual a

Solución:

Sean R y r los radios de las circunferencias grande y pequeña respectivamente. Como las circunferencias son
tangentes exteriormente, R + r = BC = 8. Dado que el 4ABC es rectángulo en B, tenemos R = AB =
p
102 82 = 6 y r = 2. Luego la longitud de la circunferencia pequeña resulta C = 2 r = 4 .

34
75. La …gura representa un hexágono regular, ¿cuál es el valor de x?

Solución:

Todo hexágono regular puede dividirse en seis triángulos equiláteros congruentes. En la …gura se indica que x
p
equivale al doble de la altura de cada triangulo: x = 2h. Como el lado de cada triangulo mide 6 3, las alturas
p p
miden h = 23 6 3 = 9
) x = 2h = 18

76. La …gura representa un círculo inscrito en un cuadrado que a su vez está inscrito en otro cuadrado. B es punto
medio de AC ¿Cuál es el área de la región sombreada?

Solución:

Si llamamos A1 al área del cuadrado mayor,A2 al área del cuadrado menor y A3 al área del círculo, el área
de la región sombreada resulta A = A1 A2 + A3 : El lado del cuadrado mayor mide 0:4, luego su área es
A1 = 0:16. Como B es punto medio AB = 0:2. Los triángulos que se forman en cada esquina del cuadrado
mayor, son rectángulos isósceles, y sus hipotenusas forman los lados del cuadrado menor, por tanto, el lado del
p p 2
cuadrado menor resulta 0:2 2 y su área es A2 = 0:2 2 = 0:08.

Como el circulo está inscrito en el cuadrado menor, su diámetro es el lado de dicho cuadrado, y su radio es la
p p 2
mitad o sea r = 0:1 2, su área A3 = r2 = 0:1 2 = 0:0628. Luego

A = 0:16 0:08 + 0:0628 = 0:1428

77. Los segmentos AC y BD se cortan en P y son tangentes a las circunferencias en los puntos A, C, B y D.

35
Solución:

Dado que P B y P C son segmentos tangentes a la circunferencia de la izquierda, desde un mismo punto, son
congruentes, luego P C = P B = 19. Como AC = AP + P C,

AP = AC P C = 31 19 = 12

78. Seis triángulos equiláteros de 1cm. de lado se unen para formar un hexágono como se muestra en la …gura. Se
circunscribe un círculo alrededor del hexágono ¿cuál es el área de la región sombreada?

Solución:

Tenemos que el área de la región sombreada es el área del circulo menos el área del hexágono. El radio del
circulo es la longitud del lado de los triángulos, es decir r = 1, luego su área es r2 = .

p
3 2
El área de cada triángulo equilátero es 4 x , donde x es el lado del triángulo, y como el lado mide 1, se reduce
p p
3 3
a 4 . Como hay seis triángulos, el área del hexágono es 6 4 . Por tanto el área de la región sombreada es
p !
3
A= cm2
2

79. Un triángulo ABC está inscrito en una circunferencia como se muestra en la …gura. Se tiene m\A = 50o y
0 0 0
m\C = 60o . Se trazan tangentes por A; B y Cde manera que se forma el triángulo circunscrito A ; B ; C .
0
Entonces la medida del ángulo A es:

Solución:

36
Como BA0 y CA0 son tangentes a la circunferencia, los \A0BC y \A0CB son ángulos semiinscritos que
subtienden el arco BC y el ángulo A es un ángulo inscrito que subtiende el mismo arco. Por tanto estos
ángulos son congruentes, es decir

m\A0BC = m\A0CB = m\A = 50o

Luego al considerar el 4A0BC, se tiene

m\A0 = 180 m\A0BC m\A0CB


= 180o 50o 50o
= 80o

80. El triángulo ABC es equilátero y sus lados AC y BC son tangentes a la circunferencia con centro en O y radio
p
3. El área del cuadrilátero AOBC es

Solución:

Se tiene OC?AB, ya que los triángulos OAB y ABC son isósceles. También 4OAC = 4OBC, ya que sus tres
lados son congruentes. Como además el 4ABC es equilátero, m\ACO = 30o , luego el 4OAC es un triángulo
p
30 –60 y de ahí resulta que OC = 2 3 y AC = 3.

Tenemos entonces
1 p p
[AOBC] = 2 [OAC] = 2 3 3=3 3
2
81. Si un ángulo central de 30 en una circunferencia intercepta un arco de 6m de longitud, entonces el radio de la
circunferencia mide.

Solución:

Se tiene s = r , donde s es la longitud del arco, r el radio de la circunferencia y es el ángulo central


o
correspondiente, medido en radianes. Como = 30 equivale a =6 radianes, tenemos

6 = r
6
36
r =

37
82. En la …gura se tiene una circunferencia de radio 1 y un hexágono regular de lado 1. Si O es el centro de la
circunferencia, entonces el área de la región sombreada es.

Solución:

En vista que el hexágono tiene lado 1 y la circunferencia tiene radio 1, el centro del hexágono es un punto de
la circunferencia. La región sombreada puede descomponerse en dos triángulos que tienen la misma base y la
misma altura que los triángulos que forman el hexágono. Luego el área buscada es
p p
3 2 3
A=2 1 = 0:866
4 2

83. Los arcosAB y BC son semicírculos cuyos centros están sobre un diámetro del círculo que se muestra en la
…gura.Si BC = 2AB, entonces la razón entre el área de la región sombreada y el área de la región no sombreada
es:

Solución:

Sean r1 el radio del semicírculo mayor,r2 el radio del semicírculo mediano y r3 el radio del semicírculo menor.
Se tiene
AB
r3 = 2 BC = 2AB AC = AB + BC
2r2 = 2AB 2r1 = 3AB
r2 = AB r1 = 32 AB

Luego las áreas de estos semicírculos son:

1 1 3 2 9
Semicírculo mayor: 2 r12 = 2 2 AB = 8 AB 2

1 1
Semicírculo mediano: 2 r22 = 2 AB 2

1 1 AB 2 1
Semicírculo menor: 2 r32 = 2 2 = 8 AB 2

38
El área sombreada está dada por: área del semicírculo mayor menos el área del semicírculo mediano más el
área del semicírculo menor o sea

9 1 1 3
Área sombreada = 8 AB 2 2 AB 2 + 8 AB 2 = 4 AB 2

La razón buscada resulta


3
Área sombreada AB 2 1
= 4 =
Área no sombreada 3 2
AB 2
2
84. Una moneda circular de radio 1, está sobre una mesa. Si ponemos cuatro monedas más grandes de igual tamaño
alrededor de ella, ¿cuál es el radio de las monedas grandes que permite que cada una sea tangente a las dos
adyacentes y a la de radio 1?

Solución:

Sea R el radio de las monedas grandes. Como estas monedas son tangentes a las monedas adyacentes y a la
vez son tangentes a la moneda pequeña, al unir los centros de las monedas grandes se forma un cuadrado de
lado 2R.

Al trazar una diagonal, esta debe pasar por el centro de la moneda pequeña, la cual tiene diámetro 2, luego la
longitud de la diagonal resulta 2R + 2.

p
Por tanto, dado que en todo cuadrado de lado x, su diagonal mide 2x, se cumple en este caso que
p
2R + 2 = 2 (2R)
p
2 2 2 R = 2
1
R = p
2 1
p
Al racionalizar el denominador obtenemos R = 2+1

85. En la siguiente …gura ABC y AEB son semicírculos, F es el punto medio del diámetro AC; B es punto medio
del arco AC y AF = 1. ¿Cuál es el área de la región sombreada?

39
Solución:

El área de la región sombreada resulta de la diferencia entre el semicírculo AEB y el segmento circular deter-
minado por la cuerda AB en el semicírculo ABC.

Como F es el punto medio del diámetro AC, B es punto medio del arco AC, resulta BF ?AC, luego el 4ABF
p
es un triángulo rectángulo isósceles de cateto 1 y por tanto AB = 2. AB es diámetro del semicírculo AEB,
p
2
luego su radio es 2 y el área de este semicírculo resulta
p !2
1 2
A1 = =
2 2 4

El área del segmento circular, está dada por la diferencia entre el área del sector circular que lo contiene y el
área del triángulo determinado por la cuerda y los radios extremos.

En este caso el sector circular correspondiente tiene ángulo central de 90o y radio 1, por tanto su área es la
1
cuarta parte del área de un círculo de radio 1 o sea y el triángulo correspondiente tiene base 1 y altura 1,
4
1 1 1
luego su área es 2.
El área del segmento circular resulta A2 =
4 2
Finalmente el área buscada es
1 1
A = A1 A2 = =
4 4 2 2

86. Si el radio de un círculo aumenta en unidades, ¿cuánto aumenta su perímetro?

Solución:

Sean L y L0 los perímetros del círculo original y el círculo con el radio aumentado, respectivamente. Luego
2
L = 2 r y L0 = 2 (r + ) = 2 r + 2 . El aumento es la diferencia

2 2
4 = L0 L= 2 r+2 2 r=2

87. Dos semicírculos de radio 3 están inscritos en un semicírculo de radio 6 como se muestra en la …gura. Un círculo
de radio r es tangente a los tres semicírculos. ¿Cuánto vale r ?

Solución:

40
Cuando se tienen círculos tangentes exteriormente, la distancia entre los centros es la suma de los radios, y
cuando son tangentes interiormente, la distancia entre los centros es la diferencia entre los radios. Además en
ambos casos los centros y el punto de tangencia están alineados.

Sean A, B, C y D los centros de los semicírculos y del círculo interior como se muestra en la …gura. Se tiene
AB = AD = 3 + r, CA = 6 r, BC = CD = 3. Como 4ABD es isósceles y C es punto medio de BD,
AC?BC, luego el 4ABC es rectángulo en C y por tanto sus lados cumplen con el teorema de Pitágoras. Luego

2 2
(3 + r) = (6 r) + 32
9 + 6r + r2 = 36 12r + r2 + 9
18r = 36
r = 2

88. En la …gura los círculos adyacentes son tangentes y tienen radio 1. ¿Cuánto vale el área de la región sombreada?

Solución:

Al considerar el círculo central y dos círculos externos contiguos, vemos que encierran la sexta parte del área
buscada. Vemos también que esta fracción corresponde al área de un triángulo equilátero de lado 2 menos tres
sectores circulares de radio 1 y de 60o cada uno, que juntos forman un semicírculo de radio 1.

41
Luego "p #
3 2 1 p
A=6 2 12 = (6 3 3 )u2
4 2

89. En la …gura, m\BCA = 90o ; BA = 5y AC = 3: ¿Cuál es el área del círculo con centro en O?

Solución:

Como el 4ABCes rectángulo en C, aplicamos el Teorema de Pitágoras para hallar BC


p p
BC = AB 2 AC 2 = 52 32 = 4

Como BC es diámetro del círculo, se tiene r = 2 y su área resulta A = r2 = 4

90. El lado mayor del rectángulo de la …gura mide 20. La curva trazada en su interior está formada por cinco
semicircunferencias ¿cuál es la longitud de la curva?

Solución:

Se observa que la curva está formada por 5 semicircunferencias, cuyos diámetros suman 20, luego cada diámetro
mide 20 5 = 4 y los respectivos radios la mitad o sea 2 unidades. Luego
1
L=5 2 r=5 r=5 2 = 10
2

91. La …gura muestra dos segmentos perpendiculares tangentes a ambas circunferencias, las cuales son tangentes
entre sí. Si el radio de la circunferencia pequeña mide 1, entonces el radio de la circunferencia más grande mide

Solución:

42
Sea r el radio de la circunferencia buscado. Sean A y C los centros de las circunferencias, pequeño y grande
respectivamente. Desde A y C trazamos perpendiculares a los segmentos perpendiculares iniciales, formando
el cuadrado rotulado en la …gura como ABCD.

Sean E, F , G y H los puntos donde estas perpendiculares cortan a los segmentos perpendiculares, como se
indica en la …gura.

Tenemos que AE = AG = DH = BF = 1, el radio de la circunferencia pequeña. Como CH = BG = CF = r,


tenemos que CD = CB = BA = DA = r 1, luego por esto y la perpendicularidad anterior ABCD es un
cuadrado.

Como las circunferencias son tangentes exteriormente, la distancia entre sus centros es la suma de sus radios,
es decir AC = r + 1. Luego el 4ABC es un triángulo isósceles, rectángulo en B, con AB = BC = r 1y
p p p
AC = r + 1. Luego AC = 2AB, es decir r + 1 = 2(r 1). Al despejar r, se obtiene r = 3 + 2 2

92. Tres círculos de radio 1, con sus centros colineales son tangentes como se muestra en la …gura. ¿Cuál es el área
de la región sombreada?

Solución:

Rotulemos los puntos extremos de la región sombreada, como se muestra en la …gura, vemos que se forma un
rectángulo. En los extremos de la región se tienen dos semicírculos, que juntos forman un circulo. Luego la
región sombreada es la diferencia entre las áreas del rectángulo y los dos círculos que se forman.

Dado que el radio de los círculos es 1, AD = 2 y AB = 4

Luego
A=2 4 2 12 = 8 2

43
93. La …gura muestra un hexágono regular inscrito en un círculo. Si el área del círculo es 1; ¿cuánto mide el área
del triángulo ABC?

Solución:

Se observa que los triángulos ABC y ABO tienen la misma área, ya que tienen la misma base y la misma
altura. Por ser un hexágono regular el 4ABO es un triángulo equilátero de lado igual al radio del círculo.

Como el área del circulo es 1, se tiene

r2 = 1
1
r = p

Luego el área del triángulo es p p


2
3 1 3
[ABC] = p =
4 4

94. ¿Qué polígono regular tiene la misma cantidad de diagonales que de lados?

Solución:

n(n 3)
Como el número de diagonales en un polígono está dado por D = 2 , donde n es el número de lados del
polígono. Luego

n (n 3)
= n
2
n2 3n = 2n
n2 5n = 0
n (n 5) = 0
n = 0_n=5

Se descarta n = 0, por carecer de sentido. Por tanto el polígono buscado es un pentágono.

95. Sean O el centro de una circunferencia de radio r y ED = r. Si m\DEC = k (m\BOA), entonces el valor
de k es:

44
Solución:

Trazamos el radio OD y vemos que el 4ODE es isósceles ya que OD = ED = r, luego \DEC = \DOC.
Como el \DEC es un ángulo exterior con sus lados secantes a la circunferencia, su medida está dada por

d
mAB d
mCD
m\DEC = (1)
2

d y m\DOC = mCD
Como los ángulos BOA y DOC, sus medidas están dadas por m\BOA = mAB d =
m\DEC. Se tiene
d = DC
m\DEC = k(m\BOA) = k mAB d

Sustituyendo en (1):

d
mAB d
k mAB
d
k mAB =
2
d
2k mAB d
= mAB d
k mAB
3k = 1
1
k =
3

96. Si se aumenta el radio de un círculo en un 100%, ¿en qué porcentaje aumenta su área?

Solución:

2
Si el radio original es r, el circulo con el radio aumentado, tiene radio 2r. Se tiene A1 = r2 y A2 = (2r) =
4 r2 . El aumento está dado por

4A = A2 A1 = 4 r2 r2 = 3 r2

Porcentaje de aumento:
4A 3 r2
100% = 100% = 300%
A1 r2
97. Se tienen tres círculos concéntricos de radios 1; 2 y 3 respectivamente. ¿Cuál es la razón entre el área de la
región cuadriculada y el área de la región oscura?

Solución:

El circulo pequeño tiene área , ya que su radio es 1

El círculo mediano tiene área 4 , ya que su radio es 2

45
El círculo grande tiene área 9 , ya que su radio es 3

Área de la región oscura = área del circulo grande –área del circulo mediano = 9 4 =5

Área de la región cuadriculada = área del circulo mediano –área del circulo pequeño = 4 =3

Área de la región cuadriculada 3 3


= =
Área de la región oscura 5 5

98. El segmento AB es diámetro de una circunferencia de radio 1 y lado del triángulo equilátero ABC. Si la
circunferencia corta a AC y BC en los puntos D y E respectivamente, entonces la longitud AE es:

Solución:

Como m\AEB
p = 90o , AE es una altura del triángulo equilátero ABC. Como p el radio es 1, AB = 2, luego
3 p 3
AE = 2 = 3: (En todo triángulo equilátero de lado x, la altura mide x)
2 2
99. En una circunferencia se tienen dos cuerdas paralelas de longitudes 10 y 14 que distan 6 entre sí. Entonces la
longitud de la cuerda paralela a ambas y que equidista de ellas mide:

Solución:

Sean CD = 10 y AB = 14, las cuerdas dadas. Como la distancia entre ellas es 6, la cuerda paralela equidistante
de ellas está a 3 unidades de cada una. Sea EF la cuerda buscada. Inicialmente no sabemos la posición de
las cuerdas con respecto a un diámetro paralelo a ellas. Comencemos asumiendo que están al mismo lado del
diámetro paralelo, como se muestra en la …gura

Al trazar desde el centro una perpendicular a las cuerdas, esta pasa por el punto medio de cada cuerda. Sean
P , Q, R los puntos medios de las cuerdas, como se muestra en la …gura.

46
Se tiene P B = 7, RD = 5. Sea QF = y, la longitud de la cuerda buscada es EF = 2y. Supongamos que
la cuerda AB está a x unidades del centro. Se forman tres triángulos rectángulos, todos ellos con hipotenusa
igual al radio de la circunferencia.

Al aplicar el teorema de Pitágoras en cada uno ellos se forma el siguiente sistema de ecuaciones

r2 = x2 + 12x + 36 + 25 r2 = x2 + 6x + 9 + y 2 r2 = x2 + 49
r2 = x2 + 12x + 61

Restando la tercera ecuación de la primera

12x = 12
x = 1

El valor negativo de x, nos indica que las cuerdas están en lados opuestos del diámetro paralelo a las cuerdas.
Sustituyendo el valor de x en la tercera ecuación, obtenemos r2 = 50. Sustituyendo el valor de x y r2 en la
segunda ecuación obtenemos 50 = 1 6 + 9 + y2
p p p
) y= 46yEF = 2y = 2 46 = 184

100. Un triángulo equilátero y un hexágono regular están inscritos en el mismo círculo. Si se divide el área del
hexágono entre el área del triángulo se obtiene:

Solución:

Al observar el gra…co fácilmente se deduce que el área del hexágono es el doble del área del triángulo. Esto
puede veri…carse considerando que el lado de un triángulo equilátero inscrito en un círculo de radio r, está dada
p
por 3r y por tanto su área es p p
3 p 2 3 3 2
A1 = 3r = r
4 4
También se tiene que el lado de un hexágono inscrito es igual al radio de la circunferencia, luego su área es
p
6 3 2
A2 = r
4

Luego p
6 3 2
A2 r
= p4 =2
A1 3 3 2
r
4

47
101. En el prisma recto de la …gura, las bases son triángulos equiláteros, con perímetros de 30cm. Si la altura del
prisma es 10cm. ¿Cuál es el área total de la super…cie del prisma?

Solución:

Como las bases son triángulos equiláteros de perímetro 30 cm, sus lados miden 10 cm y por tanto tienen una
área de p
3 p
Ab = 102 = 25 3
4
Su área lateral es AL = P h = 30 10 = 300. El área total está dada por
p p
AT = 2 Ab + AL = 2 25 3 + 300 = 50 3 + 300

102. Tres vértices de un cubo, de los cuales no hay dos que estén en la misma arista, se unen para formar un
triángulo. Si la arista del cubo tiene longitud 1. ¿Cuál es el área del triángulo formado?

Solución:

En la …gura se muestra un triángulo que satisface el enunciado. Vemos que sus lados son diagonales de las caras
p
y como las aristas de los cubos tienen longitud 1, estas diagonales miden 2. Como el área de un triángulo
p
3 2
equilátero de lado x está dada por 4 x , en este caso tenemos
p p
3 p 2 3
A= 2 =
4 2

103. La …gura representa un cubo. La intersección del plano ABG y el plano BCE es la recta

Solución:

48
Al bosquejar los planos indicados vemos que comparten los puntos B y F , y dado que la intersección de dos
!
planos diferentes es una única recta, la intersección es la recta BF .

104. De un cubo de 5” de arista se forma un cilindro circular recto de 3” de diámetro, entonces el volumen de la
parte sobrante del cubo, en pulgadas cúbicas, es aproximadamente

Solución:

El volumen de la parte sobrante es la diferencia entre el volumen del cubo y el volumen del cilindro, luego
2
3 45
V = 53 (5) = 125 125 35:34 89:67 90
2 4

105. La altura de un prisma rectangular es un tercio de su longitud y el ancho es la mitad de su longitud. Si la


diagonal del prisma mide 30cm, su volumen es

Solución:

Sean z la altura, y la longitud y x el ancho del prisma. Se tiene z = y3 , x = y2 . La diagonal está dada por
p
d = x2 + y 2 + z 2 = 30
x2 + y 2 + z 2 = 900

Al expresar en términos de “y” esta ecuación, se obtiene


y2 y2
+ y2 + = 900
4 9
1 1
+1+ y2 = 900
4 9
49 2
y = 900
36
180
y =
7

49
El volumen está dado por

V = xyz
y y
= y
2 3
y3
=
6
2
1 180
=
6 7
2833:8 cm3

106. Al introducir un trozo de metal en un tanque rectangular con agua, de dimensiones 50cm 37cm, el nivel del
agua subió 1cm. ¿Cuál es el volumen del trozo de metal?

Solución:

El volumen del trozo de metal es equivalente al volumen que incrementó el tanque, lo cual equivale al volumen
de un paralelepípedo de dimensiones 1 50 37, es decir 1850 cc:

107. ¿Cuál es el número máximo de diagonales que pueden trazarse sobre las caras de un cubo de manera que no
hayan dos diagonales que tengan un punto en común?

Solución:

Trazamos inicialmente sobre una de las caras una diagonal, digamos AD, ninguna otra puede involucrar estos
puntos para satisfacer la condición. Con los puntos restantes trazamos otra diagonal, digamos BE. Nos quedan
cuatro vértices en este caso los vértices C, F , G y H, con los cuales solo podemos trazar dos diagonales más.
En total cuatro diagonales. Cualquier otra variante conduce a la misma cantidad.

50
c
108. En la …gura se muestra un paralelepípedo rectangular. Si a = 2b y b = , ¿Cuál es el volumen en términos de
2
c?

Solución:

Como a = 2b y b = 2c , entonces a = c. Por ser un paralelepípedo rectangular,

c c3
V = abc = c c=
2 2

109. El área de la base de una pirámide es 45 y el área de una sección transversal es 20. Si la altura de la pirámide
es 6 ¿a qué distancia de la sección transversal está el vértice?

Solución:

Sea A0 = 20, el área de la sección transversal y A = 45, el área de la base de la pirámide. Sea h la distancia
desde la sección transversal al vértice. Se tiene
2
A0 h 20 4
= = =
A 6 45 9
h 2
=
6 3
h = 4

110. El área de la base de una pirámide es 45 y el área de una sección transversal es 20. Si la altura de la pirámide
es 6 ¿cuál es la razón entre los volúmenes de la pirámide mayor y la menor?

Solución:

Los datos forman parte del ejercicio anterior, de manera que ya sabemos que la distancia desde la sección
transversal al vértice es h = 4. Luego, si V es el volumen de la pirámide mayor y V 0 el de la pirámide menor,
se tiene
3 3
V 6 3 27
= = =
V0 4 2 8

51
111. La base de una pirámide es un triángulo equilátero cuyo perímetro es 12. Si la altura es 10; el volumen de la
pirámide es

Solución:

Como la base de la pirámide es un triángulo equilátero cuyo perímetro es 12, su lado mide 4, luego
p
3 2 p
Ab = 4 =4 3
4

y como h = 10, el volumen de la pirámide es


p
1 1 p 40 3
V = Ab h = 4 3 10 =
3 3 3

112. La …gura muestra dos esferas tangentes que descansan sobre una mesa plana. Si los radios de las esferas son
8cm. y 16cm respectivamente, entonces la distancia en cm. entre los puntos de contacto de las esferas con la
mesa es:

Solución:

Al considerar los centros de las esferas y los puntos de tangencia con la mesa, se forma un trapecio, como el
que se muestra en la …gura, en el cual OO0 = 24 ya que las esferas son tangentes. OA = 16 y O0B = 8

Si trazamos una paralela a la mes desde O0, sea D el punto donde corta al radio OA. Tenemos que AB = P O0,
P A = O0B = 8, luego OP = OA P A = 8. Como OA?AB, también se tiene OP ?P O0. Luego por el teorema
de Pitágoras se tiene
p p
P O0 = AB = 242 82 = 16 2

113. En una pirámide cuadrada, en la que el lado de la base mide 8cm y la altura mide 20cm, se traza una sección
paralela a la base a 14cm de ésta. Entonces el área de dicha sección es

52
Solución:

Sea x la longitud de la arista de la sección, se tiene entonces


x 6
=
8 20
48
x =
20
x = 2:4

2
Como es un cuadrado, su área es A = x2 = (2:4) = 5:76

114. Los diámetros de dos cilindros circulares rectos concéntricos son 12 y 6 pulgadas respectivamente y la generatriz
común es de 20 pulgadas, entonces el volumen del espacio que queda entre ambos cilindros es.

Solución:

El volumen buscado es la diferencia entre los volúmenes de los cilindros. Como los diámetros son 12 y 6, los
radios son 6 y 3, luego,
V = R2 r2 h= 62 32 20 = 540

115. El volumen de una cisterna cilíndrica es 1200m3 y su altura es igual al diámetro, por lo tanto su área total es

Solución:

Como el diámetro es igual a la altura se tiene r = h2 . Luego

V = r2 h
2
h
V = h
2
h3
1200 =
r4
3 4800
h =
h 11:5176

53
El área total está dada por AT = 2 AB + AL :

2
h h2
El área de la base es AB = r2 = =
2 4

h
El area lateral es AL = 2 rh = 2 2 h= h2

h2 3 2
Luego AT = 2 AB + AL = 2 + h2 = h
4 2

3 2
Al sustituir el valor de h obtenemos AT = h 625:13
2
116. Un cono de revolución tiene 13cm. de generatriz y el radio de la base es de 5 cm. Se corta por un plano paralelo
a la base que corta a la generatriz en un punto distante 5:2cm. del vértice. Entonces el volumen del tronco de
cono formado es

Solución:

Sea r el radio de la base menor del cono truncado. Sea y la altura del cono menor y H la altura del cono
truncado. Al considerar la altura del cono, los radios de las bases y la generatriz obtenemos los triángulos
rectángulos que se muestran en la …gura.

p p
La altura del cono original es h = g2 r2 = 132 52 = 12: Como los triángulos formados son semejantes
se tiene
y r 5:2
= =
12 5 13
24
y = , r=2
5

24 36
Como H + h = 12, se tiene H = 12 = : El volumen buscado es
5 5
36
V = H R2 + r2 + Rr = (25 + 4 + 10) 294:05
3 3 5

117. Dado un cono circular recto con radio 3m y generatriz 5m, entonces su área lateral es

Solución:

El área lateral está dada por AL = rg = 3 5 = 15 .

98

54
118. El área lateral de un tronco de cono que se forma cuando se corta un cono recto de 6cm. de radio y 8cm de
altura, por medio de un plano paralelo a la base del cono y que lo corta a una altura de 4:5cm es

Solución:

Se tiene AL = g (R + r), donde g es la generatriz del tronco de cono. La generatriz del cono está dada por
p p
g0 = h2 + r2 = 82 + 62 = 10

Como los triángulos que se forman con la altura, la generatriz y los radios son semejantes, se tiene
r 3:5
=
6 8
r = 2:625

Por el teorema de Thales,


g 10
=
4:5 8
g = 5:625

Luego AL = g (R + r) = 5:625 (6 + 2:625) 152:42

119. Dos esferas de metal de radios 2a y 3a se funden juntos para hacer una esfera mayor. El radio de la nueva
esfera es

Solución:

El volumen de la nueva esfera es la suma de los volúmenes de las esferas dadas. La esfera de radio 2 a tiene un
volumen
4 3 4 3 32 3
V1 = r = (2a) = a
3 3 3
La esfera de radio 3 a tiene un volumen

4 3 4 3 108 3
V2 = r = (3a) = a
3 3 3

El volumen de la nueva esfera es

32 3 108 3 140 3
V = V 1 + V2 = a + a = a
3 3 3
140 3 4 3
a = r
3 3p
3
r = a

55
120. Un cono tiene una altura igual al doble de su radio. Una esfera tiene un radio igual al radio de la base del
cono. La razón entre el volumen del cono y el volumen de la esfera es

Solución:

1 2 1 2 2 3
Volumen del cono Vc = r h= r 2r = r
3 3 3
4 3
Volumen de la esfera VE = r
3

Luego
2 3
Vc r 1
= 3 =
VE 4 3 2
r
3
121. Un cono tiene una altura igual al triple de su radio. Una esfera tiene un radio igual al radio de la base del
cono. La razón entre el volumen del cono y el volumen de la esfera es

Solución:

1 2 1 2
Volumen del cono Vc = r h= r 3r = r3
3 3
4 3
Volumen de la esfera VE = r
3

Luego
Vc r3 3
= =
VE 4 3 4
r
3
122. La altura de un cono es 5cm. Un plano a 2cm del vértice es paralelo a la base del cono. Si el volumen del cono
más pequeño es 24cm3, el volumen del cono más grande es

Solución:

56
V1 h 3
Se tiene que los volúmenes de cono semejantes son proporcionales al cubo de su razón de semejanza V2 = H :
5 3
Luego el volumen del cono más grande es V = 2 24 = 375
40
123. Un cubo está inscrito en una esfera. Si el área de la super…cie total del cubo es m2 , entonces el área de la
super…cie de la esfera es

Solución:

Tenemos que la diagonal del cubo es el diámetro de la esfera. Si x es la longitud de la arista del cubo, su área
total es
40
6x2 =
20
x2 =
3
p p
3
En un cubo la diagonal es d = 3x, luego el radio de la esfera es r = 2 x.

El área de la super…cie de la esfera es S = 4 r2 , al sustituir los valores encontrados resulta


3 2 20
S = 4 r2 = 4 x =3 = 20
4 3

124. La base de una pirámide hexagonal tiene un área de 26m2 . Si el volumen de dicha pirámide es 78m3 , entonces
su altura mide

Solución:

Tenemos que el volumen de una pirámide está dado por


1
V = AB h
3
3V
h =
AB

3V 3:78
Al sustituir los datos se obtiene h = = =9
AB 26

57
125. Si el cono de la …gura tiene un volumen de ,C es el vértice, un diámetro y m\ACB = 120 ; entonces el diámetro
de la base, en centímetros, es.

Solución:

Al considerar el triángulo formado por el diámetro AB y el vértice, tenemos que m\CAB = m\CBA = 30o .
Luego la altura del cono es h = pr . El volumen es
3

1 2 1 2 r r3
V = r h= r p = p
3 3 3 3 3

Se tiene entonces
p
r3 1000 3
p =
3 3 9
r3 = 1000
r = 10

y por tanto el diámetro es d = 20.

126. El área de la super…cie total de un cubo es 12m2 . Entonces la longitud de su diagonal es

Solución:

p
El área de la super…cie total de un cubo de lado x, está dada por AT = 6x2 y su diagonal por d = 3x. Luego

6x2 =
12
p
x = 2

p p p p
Yd= 3x = 3 2= 6

127. Si la generatriz de un cono mide 25my el diámetro de su base es 8m; su volumen mide

Solución:

p
El volumen está dado por V = 13 r2 h y la altura es h = g2 r2 . Tenemos que g = 25 y d = 8 o sea r = 4,
p p p
luego h = 252 42 = 609, por tanto V = 13 42 609 413:48.

58
128. En una esfera de radio 2, se tiene inscrito un cilindro de manera que el diámetro del cilindro es igual al radio
de la esfera. Entonces el área lateral del cilindro es

Solución:

Sean A, B y C los puntos marcados en la …gura. Tenemos que AB es diámetro del cilindro y BC diámetro de
la esfera, luego AB = 2 y BC = 4. La altura del cilindro es AC, el cual al aplicar el Teorema de Pitágoras
resulta
p p p p
AC = BC 2 AB 2 = 42 22 = 12 = 2 3

El área lateral de un cilindro está dada por AL = 2 rh. Como el diámetro del cilindro mide 2, su radio mide
p p
1. Luego AL = 2 1 2 3 = 4 3 .

59
MINISTERIO DE EDUCACION
CONSEJO NACIONAL DE UNIVERSIDADES
UNAN-MANAGUA
UNI
UNAN-LEON

Solucionario de Guía de Estudio de Matemática

Agosto, 2014

1
UNIDAD DE GEOMETRÍA ANALÍTICA

1. El triángulo de vértices A( 5; 1), B(2; 3) y C(3; 2) es:

Solución:

Basta con encontrar la distancia entre los vértices del triángulo y luego comparar los resultados. Recordemos
que la distancia entre dos puntos cuyas coordenadas son A(x1 ; y1 ) y
B(x2 ; y2 ) está dada por la fórmula

q
2 2
d (A; B) = (x1 x2 ) + (y1 y2 )

Luego q q
2 2 2 2 p p
d (A; B) = ( 5 2) + ( 1 3) = ( 7) + ( 4) = 49 + 16 = 65 = 8:0623

q q p p
2 2 2 2
d (B; C) = (2 3) + (3 ( 2)) = ( 1) + (5) = 1 + 25 = 26 = 5:099

q q p p
2 2 2 2
d (A; C) = ( 5 3) + ( 1 ( 2)) = ( 8) + (1) = 64 + 1 = 65 = 8:0623

de donde claramente se ve que d (A; B) = d (A; C) y así podemos decir que dicho triángulo es isósceles.

2. El perímetro P y el área A del cuadrilátero cuyos vértices son A ( 3; 1), B (0; 3), C (4; 3) y D (4; 1) son:

Solución :

Sean los puntos A ( 3; 1), B (0; 3), C (4; 0), D (4; 1) ; entonces su perimetro es

P = d (A; B) + d (B; C) + d (C; D) + d (A; D)

de aqui
q q q q
2 2 2 2 2 2 2 2
P = (0 + 3) + (3 + 1) + (4 0) + (3 3) + (4 0) + ( 1 3) + (4 + 3) + ( 1 + 1)
p p p p
= 25 + 16 + 16 + 49
= 20u

Para el área se tiene un trapecio recto de bases b1 = BC; b2 = AD y h = CD; entonces


p p p
b1 = 16 = 4; b2 = 49 = 7; h = 16 = 4

de donde, su área es
(b1 + b2 ) h
A =
2
(4 + 7) 4
=
2
= 22u2

2
3. Los vértices de un triángulo son A (3; 8), B (2; 1), C (6; 1). La longitud de la mediana trazada al lado BC
es:

Solución :

Sea E el punto medio de BC; entonces


6+2 1 1
x= = 4; y = = 1 ! E (x; y) = E (4; 1)
2 2

La mediana es la recta que pasa por A y E y su longitud es AE; entonces


q p p
2 2
AE = (4 3) + ( 1 8) = 1 + 81 = 82

4. Los vértices de un cuadrado son ( 1; 3), (3; 1), ( 1; 1) y (3; 3). La longitud de sus diagonales es:

Solución:

Nótese que los puntos con coordenadas A( 1; 3) y B(3; 1) son vértices no consecutivos del cuadrilátero, al
igual que C( 1; 1), D(3; 3). Por lo cual, el problema se resume a
encontrar la distancia entre cualesquiera de las parejas de vértices no consecutivos, no importando con cual
trabajar ya que las diagonales de un cuadrado son congruentes.

Luego q q q
2 2 2 2 2
p
d (A; B) = ( 1 3) + (3 ( 1)) = ( 4) + (4) = 2 (4) = 4 2
p
En consecuencia la longitud de las diagonales del cuadrado es 4 2.

5. Dos vértices opuestos de un cuadrado son (5; 1) y ( 1; 3). El área del cuadrado es:

Solución:

Encontremos la longitud de la diagonal que se forma a partir de dichos puntos, es decir, la distancia entre los
vértices opuestos A(5; 1) y B( 1; 3). Así
q q p p p
2 2 2 2
d (A; B) = (5 ( 1)) + (1 3) = (6) + ( 2) = 36 + 4 = 40 = 2 10:

D2
Recordemos que una de las fórmulas que podemos emplear para calcular el área de un cuadrado es A = .
2
En nuestro caso tenemos p 2
2 10 4(10)
A= = = 2(10) = 20:
2 2
Por lo tanto, el área de dicho cuadrado es 20 unidades cuadradas.

6. Uno de los extremos de un segmento rectilíneo de longitud 5 es el punto A(3; 2). Si la abcisa del otro extremo
es 6, su ordenada es:

Solución :

3
Segun datos del problema, se tiene que

A (3; 2) ; B (6; y) ; D (A; B) = 5

entonces
q
2 2
5 = (6 3) + (y + 2)
25 = 9 + y 2 + 4y + 4
y 2 + 4y 12 = 0

al resolver dicha ecuación, se obtienen que las soluciones son

y= 6; y = 2

7. Sea un segmento cuyos extremos son los puntos A( 2; 3) y B(6; 3). Los puntos de trisección del segmento
son:

Solución :

Para P se tiene
1
AP AB 1
r= = 3 =
PB 2 2
AB
3
entonces
1
xA + rxB 2+(6) 2
X = = 2 =
1+r 1 3
1+
2
1
yA + ryB 3 + ( 3)
Y = = 2 =1
1+r 1
1+
2

de aqui,
2
P ;1
3
para Q se tiene
2
AQ AB
r= = 3 =2
QB 1
AB
3
entonces
xA + rxB 2 + 2 (6) 10
X = = =
1+r 1+2 3

yA + ryB 3 + 2 ( 3)
Y = = = 1
1+r 1+2

de aqui
10
Q ; 1
3

4
8. Uno de los extremos de un segmento es el punto (7; 8) y su punto medio es (4; 3). El otro extremo es:

Solución :

x1 + x2 y1 + y2
Utilizando la fórmula del punto medio x = ; y= se tiene
2 2
7+x
4 = =) x = 1
2
8+y
3 = =) y = 2
2

por tanto, B = (1; 2) es el extremo buscado.

9. Una recta de pendiente 3 pasa por el punto (3; 2). Si la abcisa de otro punto de la recta es 4, su ordenada es:

Solución :

y2 y1
Utilizando la fórmula de la pendiente m = se tiene
x2 x1
y 2
3 =
4 3
y = 5

CA 3
10. Dados los puntos A(3; 2) y B(5; 4) halla un punto C, alineado con A y B, de manera que se obtenga = .
CB 2

Solución:

El problema pide encontrar las coordenadas del punto C. Utilizando la ecuación de la abscisa y ordenada de
x1 + rx2
un punto que divide a un segmento en una razón dada x = y
1+r
y1 + ry2
y= y sustituyendo los valores correspondientes tenemos
1+r
3 15 21
3 + (5) 3+ 21
x= 2 = 2 = 2 =
3 5 5 5
1+
2 2 2

3
2 + (4) 2+6 8 16
y= 2 = = =
3 5 5 5
1+
2 2 2
21 16
En consecuencia las coordenadas del punto C son ;
5 5
11. Dado el segmento de extremos P1 (3; 2) y P2 ( 4; 1), encuentre las coordenadas del punto P que lo divide en
la razón 2.
Solución:

Consideremos los puntos P1 (3; 2) y P2 ( 4; 1) extremos del segmento que es dividido por un punto P en una
x1 + rx2 y1 + ry2
razón r = 2. Al sustituir los valores correspondientes en las fórmulas x = y y= resulta
1+r 1+r
que las coordenadas de P son

5
3 + ( 2) ( 4) 3+8
x= = = 11
1 + ( 2) 1

2 + ( 2) (1) 2 2
y= = =4
1 + ( 2) 1

Por lo tanto, P tiene por ( 11; 4).

12. Las medianas de un triángulo el baricentro B(x; y) es tal que las distancias de este punto al vértice M (2; 4) y
MB
al punto medio N (1; 1) del lado opuesto están en la relación = 2. Las coordenadas de B son:
MN

Solución :

Recordemos la mediana es el segmento de recta que une el vértice de un triángulo con el punto medio del lado
opuesto. El punto de intersección de las medianas se llama Baricentro.

x1 + rx2 y1 + ry2
Utilizando las fórmulas x = y y = y considerando al vértice M (2; 4) y al punto medio
1+r 1+r
N (1; 1) del lado opuesto como extremos de la mediana M N resulta que las coordenadas del baricentro B son

2 + (2) (1) 2+2 4


x= = =
1+2 3 3

4 + (2) ( 1) 4 2 2
y= = =
1+2 3 3
4 2
Es decir, B ; .
3 3
1
13. Las coordenadas del punto que divide al segmento con extremos A( 1; 4) y B( 5; 8) en la razón son:
3

Solución:

x1 + rx2 y1 + ry2
Usando las fórmulas x = y y= y sustituyendo los valores correspondientes tenemos
1+r 1+r
1 5 2
1+ ( 5) 1+
3 3 = 3 =1
x= =
1 2 2
1+
3 3 3

1 8 20
4+ ( 8) 4+
y=
3
= 3 = 3 = 20 = 10
1 2 2 2
1+
3 3 3

Es decir, las coordenadas del punto son (1; 10).


1
14. Las coordenadas del punto que divide al segmento con extremos A(3; 2) y B( 1; 1) en la razon son:
2

Solución:

6
x1 + rx2 y1 + ry2
Similarmente al ejercicio anterior, al hacer uso de las fórmulas x = y y= y sustituir valores
1+r 1+r
resulta
1 1 5
3+ ( 1) 3
x=
2
= 2 = 2 =5
1 3 3 3
1+
2 2 2
1 1 3
2+ ( 1) 2
2 2 = 2 =1
y= =
1 3 3
1+
2 2 2
5
Es decir, las coordenadas del punto son ;1 .
3
15. Encontrar el punto medio del segmento cuyos extremos son A(5; 4); B( 3; 8).

Solución:

x1 + x2 y 1 + y2
Las coordenadas del punto medio de un segmento están dadas por las fórmulas x = y y= .
2 2
De este modo al sustituir valores tenemos
5 + ( 3) 5 3 2
x= = = =1
2 2 2

4+8 12
y= = =6
2 2
En consecuencia, las coordenadas del punto medio son (1; 6).

16. El punto medio de un segmento es (2; 2). Si uno de sus extremos es ( 2; 3), el otro es:

Solución:

Consideremos el punto con coordenadas (2; 2) como el punto medio del segmento con extremos ( 2; 3) y (a; b) :
El problema pide encontrar los valores de las coordenadas del extremo desconocido. Usando las fórmulas de
las coordenadas del punto medio tenemos
2+a
2= )4= 2+a)a=4+2=6
2

3+b
2= )4=3+b)b=4 3=1
2
Por lo tanto, las coordenadas del otro extremo son (6; 1).

17. Encuentre dos puntos equidistantes de (2; 1), los tres sobre la misma línea, si la abscisa de uno de ellos es x = 6
y la ordenada del otro es y = 1

Solución:

Como los puntos son colineales y equidistantes del punto con coordenadas (2; 1) esto quiere decir que dicho
punto es el punto medio del segmento cuyos extremos son (6; y1 ) y (x2 ; 1) dado que sabemos la abscisa de uno

7
x1 + x2
de ellos es 6 y la ordenada del otro es 1. Usando las fórmulas de las coordenadas del punto medio x =
2
y1 + y2
y y= tenemos
2
6 + x2
2= ) 4 = 6 + x2 ) x2 = 4 6 = 2
2

y1 + ( 1)
1= ) 2 = y1 + ( 1) ) y1 = 2 + 1 = 3
2
Por tanto, los puntos que equidistan de (2; 1) son (6; 3) y ( 2; 1)

18. Una recta l1 pasa por los puntos A(3; 2) y B( 4; 6) y otra recta l2 pasa por los puntos C( 7; 1) y el punto
D(x; 6). Sabiendo que l1 es perpendicular a l2 , el valor de x es:

Solución :

Aplicando la fórmula de la pendiente para ambas rectas


6 2 8 8
m1 = = =
4 3 7 7
6 1 7
m2 = =
x 7 x+7

Como l1 ? l2 entonces m1 m2 = 1; de aqui


8 7
= 1
7 x+7
x = 1

19. Dados los vértices de un triángulo A(2; 0), B(1; 3) y C(2; 5), el otro extremo de la mediana correspondiente
a B es:

Solución:

Basta encontrar el punto medio del segmento cuyos extremos son A(2; 0) y C(2; 5). Haciendo uso de las
fórmulas para las coordenadas del punto medio tenemos

2+2 4
x= = =2
2 2
0 + ( 5) 5
y= =
2 2
5
Es decir, el otro extremo de la mediana correspondiente al vértice B es 2;
2
20. La mediatriz del segmento determinado por los puntos A( 2; 3) y B(4; 1) pasa por el punto

Solución:

Recordemos que mediatriz de un segmento es la recta perpendicular que pasa por su punto medio. Entonces
el ejercicio se reduce a encontrar el punto medio del segmento determinado por A( 2; 3) y B(4; 1): Usando las
fórmulas para encontrar las coordenadas del punto medio tenemos
2+4 2
x= = =1
2 2

8
3+1 4
y= = =2
2 2
Por lo tanto, la mediatriz de dicho segmento pasa por el punto (1; 2).

21. Determine la pendiente de la recta que pasa por los puntos ( 4; 1) y (5; 2).

Solución:

La ecuación de la pendiente de una recta que pasa por dos puntos es


y2 y1
m=
x2 y2

Usando esta ecuación y sustituyendo los valoresde las coordenadas de los puntos tenemos
2 ( 1) 2+1 3 1
m= = = =
5 ( 4) 5+4 9 3

1
Es decir, la pendiente de la recta es
3
22. Determine la pendiente de la recta que pasa por los puntos ( 3; 3) y (4; 4)

Solución:

Usando la ecuación de la pendiente de una recta que pasa por dos puntos y sustituyendo los valores correspon-
dientes tenemos
4 3 7 7
m= = = = 1
4 ( 3) 4+3 7
Es decir, la pendiente de la recta es 1.

23. Determine la pendiente de la recta que pasa por los puntos ( 5; 2) y ( 5; 4)

Solución:

Por la forma de las coordenadas de los dos puntos puede notarse que dicha recta es paralela al eje y y por lo
tanto no existe la pendiente. Además se puede comprobar al usar la ecuación de la pendiente.

24. La pendiente de la recta que pasa por los puntos (x; 3) y ( 2; 6) es 3, entonces el valor de x es:

Solución:

Al sustituir valores en la ecuación de la pendiente de una recta tenemos


6 ( 3) 6+3 9
3= = =
2 x 2 x 2 x

de lo cual se sigue
3( 2 x) = 9

luego por distributividad resulta


6 3x = 9

9
es decir,
3x = 9 + 6 = 15

obteniendo así que


15
x= = 5
3
Por tanto, el valor de x es 5.

25. La pendiente de la recta que pasa por los puntos ( 3; 4) y (1; y) es cero, entonces el valor de la ordenada es:

Solución:

Como la pendiente de la recta es cero, entonces estamos hablando de una recta horizontal y por tanto y = 4.
Valor que se puede obtener también sustituyendo los datos dados en la ecuación de la pendiente.

26. Una recta de pendiente 2 pasa por el punto A( 1; 4). Hallar su ecuación en la forma simétrica.

Solución:

Usando la ecuación punto pendiente y y0 = m(x x0 ) y sustituyendo valores

y 4 = 2(x ( 1))
y 4 = 2x 2
2x + y = 2
y
x+ = 1
2

27. Hallar la ecuación de la recta cuya pendiente es 4 y que pasa por el punto de intersección de las rectas
2x + y 8 = 0 y 3x 2y + 9 = 0.

Solución:

Encontremos el punto de intersección de las rectas 2x + y 8 = 0 y 3x 2y + 9 = 0 lo cual es equivalente a


resolver el sistema (
2x + y 8=0
3x 2y + 9 = 0

cuya solución es x = 1, y y = 6. Por lo tanto, el punto de intersección de las rectas es (1; 6). Luego, usando la
ecuación punto pendiente

y 6 = 4(x 1)
y 6 = 4x + 4
4x + y 10 = 0

28. Hallar la ecuación de la mediatriz del segmento cuyos extremos son P1 ( 3; 2) y P2 (1; 6).

Solución:

10
Encontrando la pendiente de la recta que contiene al segmento resulta
6 2
m1 = =1
1 ( 3)

La mediatriz del segmento tendrá por pendiente

m2 = ( 1) (1) = 1

Determinemos las coordenadas del punto medio


3+1
x = = 1
2
2+6
y = =4
2

Por lo cual la recta que pasa por el punto ( 1; 4) con pendiente m = 1, tiene por ecuación

y 4 = 1(x ( 1))
= x 1
x+y 3 = 0

29. Una recta pasa por el punto A(7; 8) y es paralela a la recta que pasa por los puntos C( 2; 2) y D(3; 4). Hallar
su ecuación

Solución:

Como las rectas son paralelas tienen la misma pendiente. Determinemos la pendiente de la recta que pasa por
los puntos C( 2; 2) y D(3; 4) como sigue
4 2 6
m= =
3 ( 2) 5

Usando la ecuación punto pendiente resulta


6
y 8 = (x 7)
5
5y 40 = 6x + 42
6x + 5y 82 = 0

30. Hallar el valor de k para que la recta k 2 x + (k + 1)y + 3 = 0 sea perpendicular a la recta 3x 2y 11 = 0.

Solución:

Recordemos que para que dos rectas sean perpendiculares debe veri…carse que el producto de sus pendientes
sea 1. La pendiente de la recta 3x 2y 11 = 0 es
3 3
m= =
2 2

lo cual implica que la pendiente de la otra es


2
m0 =
3

11
es decir,

k2 2
=
k+1 3
3k 2 = 2k + 2
3k 2 2k 2 = 0

cuyas raices son p


1 7
k=
2
31. Sean las rectas paralelas 3x 4y + 8 = 0 y 6x 8y + 9 = 0. La distancia entre ellas es:

Solución :

La distancia entre las dos rectas paralelas está dada por

jc1 c2 j
d= p
a2 + b2

Para aplicar la fórmula, los coe…cientes de ambas ecuaciones deben ser iguales, por lo cual multiplicando por 2
en 3x 4y + 8 = 0 se tiene la ecuación equivalente.

j9 16j j 1j 7
d= p =p =
36 + 64 100 10

32. Hallar los ángulos interiores del triángulo cuyos vértices son los puntos ( 2; 1) ; (3; 4) y (5; 2).

Solución:

Recordemos que el ángulo entre dos rectas está dado por la fórmula
m2 m1
tan =
1 + m1 m2

donde m1 y m2 representan las pendientes de las rectas involucradas respectivamente.

Encontremos la pendiente de las rectas que pasa por los puntos:


( 2; 1) y (3; 4)
4 1 3
m1 = =
3 ( 2) 5

(3; 4) y (5; 2)
2 4
m2 = = 3
5 3
( 2; 1) y (5; 2)
2 1 3
m3 = =
5 ( 2) 7

son respectivamente la pendiente de las rectas l1 , l2 y l3 .

12
De esta manera el ángulo entre l1 y l2 es
3
3 5 9
tan = 3 =
1+ 5 ( 3) 2
9 1
= tan
2
= 77 28 1600
0

Por otro lado el ángulo entre l1 y l3 es


3 3
5 7 18
tan = 3 3 =
1+ 7 5
13
1 18
= tan
13
= 54 90 4400

Del mismo modo el ángulo entre l2 y l3 es


3
7 ( 3) 9
tan = 3 =
1+ 7 ( 3) 8
1 9
= tan
8
= 48 21 5900
0

33. Dos rectas se cortan formando un ángulo de 45 . La recta inicial pasa por los puntos ( 2; 1) y (9; 7) y la
recta …nal pasa por los puntos (3; 9) y A cuya abscisa es 2. Hallar la ordenada de A.

Solución:

Al despejar m2 de la fórmula
m2 m1
tan 45 =
1 + m1 m2
se tiene
tan 45 + m1
m2 =
1 (tan 45 ) m1
pero según los datos del problema
7 1 6
m1 = =
9 ( 2) 11
y
y 9 y 9 9 y
m2 = = =
2 3 5 5
de donde,

y = 9 5m2
tan 45 + m1
y = 9 5
1 (tan 45 ) m1
6
1+ 11
= 9 5 6
1 11
= 9 17
= 8

Usando el hecho que tan 45 = 1:

13
34. Una recta l1 pasa por los puntos (3; 2) y ( 4; 6) y la otra recta pasa por el punto ( 7; 1) y el punto A
cuya ordenada es 6. Hallar la abscisa del punto A, sabiendo que l1 es perpendicular a l2 .

Solución:

Recordemos que como l1 es perpendicular a l2 entonces el producto de sus pendientes es 1. Encontremos las
pendientes de l1 y l2 como sigue

6 2 8
m1 = =
4 3 7
por lo cual
7
m2 =
8
De modo que
7 7
=
8 x+7
7 (x + 7) = 56
7x = 7
x = 1

35. Encuentre la pendiente y el ángulo de inclinación de una recta paralela a la recta que pasa por los puntos
(1; 2) y (3; 8).

Solución:

La pendiente de la recta que pasa por los puntos (1; 2) y (3; 8) es


8 ( 2)
m1 = =5
3 1

pero dicha pendiente es la misma para ambas rectas ya que son paralelas, es decir, m2 = 5. Así el ángulo de
inclinación buscado ; 0 < < 90 .

Ahora bien

tan = 5
1
= tan (5)
= 78 41 2400
0

36. Hallar los ángulos agudos del triángulo rectángulo cuyos vértices son A (2; 5), B (8; 1) y C ( 2; 1).

Solución:

Consideremos

l1 la recta que pasa por ( 2; 1) y (8; 1)


l2 la recta que pasa por ( 2; 1) y (2; 5)
l3 la recta que pasa por (8; 1) y (2; 5)

14
por lo tanto sus pendientes son respectivamente
1 1 1
m1 = =
8 ( 2) 5
5 1
m2 = =1
2 ( 2)
5 ( 1)
m1 = = 1
2 8

Luego
1
1 5 3
tan A = 1 =
1+1 5
2
13
A = tan
2
A = 56 18 3500
0

1
5 ( 1) 2
tan C = 1 =
1+ 5 ( 1) 3
1 2
A = tan
3
A = 33 410 2400

37. La ecuación de una circunferencia es x2 + y 2 = 50. El punto medio de una cuerda de esta circunferencia es el
punto ( 2; 4). La ecuación de la cuerda es:

Solución :

El radio pasa por el puntoP ( 2; 4) y (0:0) : La ecuación del radio es y = mx + b, la pendiente está dada por

y2 y1 4 0
m= = = 2
x2 x1 2 0
aplicando la fórmula punto pendiente, obtenemos la ecuación del radio

y y0 = m (x x0 )
y = 2x

1
La ecuación de la cuerda es perpendicular a la ecuación del radio por lo que su pendiente es m =
2
Además la ecuación de la cuerda pasa por el punto medio P ( 2; 4) ; es

y y0
= m (x x0 )
1
y 4 = (x + 2)
2
x 2y + 10 = 0

38. Un espejo parabólico tiene una profundidad de 12cm en el centro y un diámetro en la parte superior de 32cm.
¿Cuál es la distancia del vértice al foco?

15
Solución :

Los ejes coordenados se eligen de modo que la parábola tenga su vértice en el origen y su eje a lo largo del eje
Y y se abre hacia arriba. La ecuación es x2 = 4py

Como el punto (16; 12) está en la parabola, sus coordenadas satisfacen la ecuación dada.
2
(16) = 4p (12)
16
p =
3

donde p es la distancia en centrimetro del centro al foco.

39. La ecuación de la hipérbola de centro en el origen, eje focal 12 y pasa por el punto (8; 14) es:

Solución:

El eje mayor tiene por ecuación


2a = 12 a=6
x2 y2
La grá…ca pasa por el punto (8; 14) y la ecuación de una hiperbola está dada por = 1; sustituyendo el
a2 b2
punto dado, se tiene
64 196
=1
36 b2
de donde
b2 = 252

La ecuación buscada es
x2 y2
=1
36 252
40. En una elipse, los radios focales son los segmentos que unen los focos con un punto cualquiera de ella. Las
ecuaciones de las rectas que contienen los radios focales correspondientes al punto (2; 3) de la elipse 3x2 +4y = 48
son:

Solución :

La ecuación de la elipse es 3x2 + 4y 2 = 48 la cual puede ser escrita por

x2 y2
+ =1
16 12

de donde

a2 = 16; b2 = 12
c2 = a2 b2 = 16 12 = 4
c = 2

Encontramos la pendiente de la recta que pasa por ( 2; 0) y (2; 3)


3 0 3
m1 = =
2+2 4

16
Con la ecuación punto pendiente encontramos la ecuación

3x 4y + 6 = 0

La ecuación de la recta que pasa por (2; 3) y ( 2; 0) por ser una paralela al eje y tiene por ecuación x = 2 ó
x 2 = 0.

41. La ecuación de la circunferencia que pasa por el origen y tiene su centro en el punto común a las rectas
x + 3y 6=0 y x 2y 1 = 0 es:

Solución :

La ecuación de la circunferencia de centro (h; k) está dada por

2 2
(x h) + (y k) = r2

necesitamos resolver el sistema de ecuación de las rectas dadas para encontrar el centro de la misma, es decir

x + 3y 6=0
x 2y 1=0

cuya solución es el punto (3; 1) : Como la circunferencia pasa por el punto (0; 0) entonces podemos obtener el
radio por

2 2
(x 3) + (y 1) = r2
2 2
( 3) + ( 1) = r2
10 = r2

2 2
la ecuación es (x 3) + (y 1) = 10; lo cual puede escribirse por

x2 6x + y 2 2y = 0

4
42. La ecuación de una hipérbola con centro en el origen, longitud del eje transverso 8; excentricidad y con focos
3
sobre el eje X es:

Solución :

La longitud del eje transverso está dada por

2a = 8
a = 4

la excentricidad
c 4
e = =
a 3
16
c =
3
151

17
la ecuación buscada tiene la forma
x2 y2
=1
a2 b2
pero c2 = a2 + b2 entonces

b2 = c2 a2
2
16 2
b2 = (4)
3
112
b2 =
9

de aqui, la ecuación buscada es

x2 y2
112 = 1
16 9
7x2 9y 2
= 112

3
43. El …lamento de una lámpara de ‡ash está a de pulgadas del vértice del re‡ector parabólico y se encuentra
8
en su foco. La ecuación del re‡ector, suponiendo que está dirigido hacia la derecha y su vértice en el origen es:

Solución :

La ecuación solicitada tiene la forma

y2 = 4px
2 3
y = 4 x
8
3x 2y 2 = 0

44. Una parábola cuyo foco es F (0; 6) y la ecuación de la directriz es y = 6, tiene por ecuación:

Solución :

Los datos del problema permiten deducir que la ecuación de la parabola corresponde a la ecuación

x2 = 4py

entonces p = 6 y sustituyendo en la ecuación


x2 = 24y
5
45. Si la excentricidad de una cónica es e = , entonces se trata de una:
2

Solución :

La cónica corresponde a una hipérbola, ya que e > 1

18
46. La ecuación de la circunferencia con centro en el origen y que pasa por ( 3; 4) es

Solución:

Dado que ( 3; 4) es un punto de la circunferencia entonces satisface la relación

x2 + y 2 = r2
2
( 3) + 42 = r2
25 = r2

Por lo tanto, la ecuación pedida es


x2 + y 2 = 25

47. De los siguientes puntos el único que se encuentra sobre la circunferencia x2 + y 2 = 1 es

Solución:

Para encontrar que puntos pertenecen a la circunferencia unitaria basta sustituir las coordenadas de estos en
la ecuación x2 + y 2 = 1. Veamos
p 2
2
2 + ( 1) = 2 + 1 = 3 6= 1
p !2 2
3 1 3 1
+ = + =1
2 2 4 4
2 2
(1) + (1) = 1 + 1 = 2 6= 1
2 2
( 1) + ( 1) = 1 + 1 = 2 6= 1
2 2
(2) + (1) = 4 + 1 = 5 6= 1
p !
3 1
Por lo tanto el único punto perteneciente a la circunferencia es ;
2 2
p p
48. Si los extremos de un diámetro es una circunferencia con centro en el origen son 5; 2 y 5; 2 , la
ecuación de dicha circunferencia es

Solución:

p p
Determinemos la longitud del diámetro entre los puntos 5; 2 y 5; 2 como sigue
r r
p p 2 2
p 2
2
5 5 + ( 2 2) = 2 5 + ( 4)
p
= 36
= 6

pero D = 2r, es decir,


r=3

Por lo cual, la ecuación buscada es


x2 + y 2 = 9

19
49. Si (2; 2) es el punto medio de una cuerda en la circunferencia x2 + y 2 = 16 , la ecuación de dicha cuerda es

Solución:

Como (2; 2) es el punto medio de una cuerda en la circunferencia x2 + y 2 = 16, entonces el radio pasa por los
puntos (2; 2) y (0; 0). Pero la ecuación del radio es de la forma y = mx, relación que cumplen los puntos
anteriores, es decir,

2m = 2
m = 1

La cuerda es perpendicular al radio, ya que si una cuerda pasa por el centro de una circunferencia y biseca a
otra cuerda, que no sea el diámetro, entonces es perpendicular a la cuerda, de modo que tiene por pendiente

m0 = 1

Usando la ecuación punto - pendiente tenemos que la ecuación de dicha cuerda es

y 2 = 1 (x 2)
y 2 = x+2
x+y 4 = 0

50. La ecuación de la circunferencia con centro en el origen y que pasa por el punto de intersección de las rectas
3x + 3y = 15 y 2x + 2y = 22 es

Solución:

Encontremos el punto de intersección de dichas rectas, lo que es equivalente a resolver


(
3x + 3y = 15
2x + 2y = 22

cuyas solución es el par (2; 3). Ahora bien

22 + 32 = r2
4+9 = r2
13 = r2

Por lo tanto, la ecuación es


x2 + y 2 = 13
p
51. La ecuación de una elipse con focos en 5; 0 y longitud del eje mayor igual a 6 es:

Solución :

20
Como la longitud del eje mayor es 6 entonces

2a = 6
a = 3

p
ademas c = 5 lo cual indica que la ecuación tiene sus focos en el eje X y por tanto la ecuación tiene la
forma
x2 y2
+ =1
a2 b2
El valor de b2 lo podemos obtener de la ecuación

c2 = a2 b2

de donde
b2 = 4

por tanto, la ecuación es


x2 y2
+ =1
9 4
la cual puede ser escrita por
4x2 + 9y 2 = 36

52. La ecuación de una parábola que tiene su foco en el punto F (2; 0) y su directriz es la recta de ecuación x = 2
es:

Solución :

La ecuación de una parábola con foco (2; 0) es de la forma

y 2 = 4px

entonces

y2 = 4 (2) x
2
y = 8x

53. Hallar el centro y radio de la circunferencia que pasa por los puntos A(0; 6), B(4; 2) y C(9; 3)

Solución :

2 2
La ecuación de la circunferencia es (x h) + (y k) = r2 ; al sustituir cada punto en dicha ecuación, se forma
las ecuaciones
2 2
( h) + (6 k) = r2
2 2
(4 h) + ( 2 k) = r2
2 2
(9 h) + (3 k) = r2

21
al desarrollar

h2 + 36 12k + k 2 = r2
16 8h + h2 + 4 + 4k 2 = r2
81 18h + h2 + 9 6k + k 2 = r2

al igualar las ecuaciones (1; 2) y (2; 3) se tiene

2k h=2
k+h=7

cuya solución es h = 4; k = 3; que al sustituir en la ecuación (1) se tiene que r = 5; por lo tanto la ecuación es
2 2
(x 4) + (y 3) = 25

54. Dada la parábola que tiene por ecuación x2 = 6y, encontrar las coordenadas del foco y la ecuación de la
directriz:

Solución :

La coordenada del foco y la directriz es:

4p = 6
3
p =
2

3
Las coordenadas del foco es 0; y la ecuación de la directriz viene dada por
2
y = p
3
y =
2
1 2
55. Las coordenadas del foco y la ecuación de la directriz de la parábola x = y es
4

Solución:

1 2
La parábola x = y puede ser vista también de la forma y 2 = 4x y tiene eje focal sobre el eje de las x,
4
de donde

4p = 4
p = 1

Por lo tanto, las coordenadas del foco son ( 1; 0) y la ecuación de la recta directriz es x = 1.
p
56. La ecuación de la parábola con vértice en el origen y foco 2; 0 es

Solución:

p p
Dado que el foco tiene por coordenadas 2; 0 , entonces p = 2 y la ecuación solicitada es y 2 =
p p
4 2 x = 4 2x

22
57. El foco y la directriz de la parábola 2y x2 = 0 son:

Solución:

La parabóla que tiene por ecuación 2y x2 = 0 es una parábola cuyo eje focal esta sobre el eje y. Vista de
1
otra forma la ecuación anterior es x2 = 2y, de donde 4p = 2, es decir, p = . Por lo tanto, las coordenadas
2
1 1
del foco son 0; la directriz es la recta y =
2 2
58.. La ecuación de la parábola cuyo foco es (4; 0) y directriz x= 4 es

Solución:

Como el foco de la parábola es (4; 0) y directriz x = 4, entonces p = 4. Por lo tanto, la ecuación buscada
2
es y = 4 (4) x = 16x.

59.. La ecuación de la parábola cuyo eje de simetría es el eje y, vértice en el origen y que pasa por ( 2; 2) es

Solución:

Como la parábola tiene eje de simetría al eje y y pasa por el punto ( 2; 2), entonces estamos tratando con
una parábola vertical y por ende dichos puntos cumplen la relación

x2 = 4py

en particular tenemos

2
( 2) = 4p ( 2)
4 = 8p
1
p =
2

En consecuencia, la ecuación de la parábola es x2 = 2y

60. Si la longitud del eje mayor es 16 y la distancia focal es 8, entonces la ecuación de la elipse con eje focal en el
eje y es

Solución:

Sabemos que la longitud del eje mayor de una elipse está dado por 2a y la distancia focal por 2c, con los datos
del problema obtenemos

2a = 16
a = 8

2c = 8
c = 4

23
Pero a2 = b2 + c2 , es decir,

b2 = 82 42
= 64 16
= 48

Por lo tanto, la ecuación de la elipse es


x2 y2
+ =1
48 64
4
61.. Si la excentricidad es y la distancia focal es 16, la ecuación de la elipse con eje focal en el eje x es
5

Solución:

Como la distancia focal es 16, entonces

2c = 16
c = 8

pero al sustituir los valores de e y c en


c
e=
a
tenemos
4 8
=
5 a
4a = 40
a = 10

Ahora encontremos

b2 = 100 64
= 36

Finalmente, la ecuación de la elipse es


x2 y2
+ =1
100 36
62. La excentricidad de la elipse 2x2 + 4y 2 = 8 es

Solución:

La ecuación de la elipse 2x2 + 4y 2 = 8 en su forma canónica es


x2 y2
+ =1
4 2
De manera que
p
c = 4 2
p
= 2

Por lo tanto, la excentricidad es p


2
e=
2

24
63. El único punto que pertenece a la elipse con eje mayor 20 y eje menor 10 es

Solución:

Similarmente al ejercicio 60 tenemos

2a = 20
a = 10

2b = 10
b = 5

Por lo cual, la ecuación de la elipse es


x2 y2
+ =1
100 25
Veamos
p !2
5 3
75
2 2
( 5) 25
+ = + 4
100 25 100 25
1 3
= +
4 4
= 1

p !2
5 3
75
2 2
(5) 25
+ = + 4
100 25 100 25
1 3
= +
4 4
= 1

p !2
2 3
2 2
(5) 25 3
+ = +
100 25 100 25
1 3
= +
4 25
37
= 6= 1
100

p 2 75
2 5 3
(5) 2 25
+ = + 4
100 25 100 25
1 3
= +
4 4
= 1

De lo cual se ve que tres de los puntos dados satisfacen la ecuación, es decir, son puntos de la circunferencia.

25
p
64. La ecuación de la elipse que pasa por 3; 2 3 , con vértice correspondiente al eje menor (0; 4) es

Solución:

Como uno de los vértice correspondiente al eje menor es (0; 4), entonces a = 4 y estamos tratando con una
p
elipse vertical. Así el punto 3; 2 3 debe cumplir la relación

x2 y2
+ =1
b2 a2

es decir,
p 2
32 2 3
+ = 1
b2 42
2
b = 36

Por lo tanto, la ecuación es


x2 y2
+ =1
36 16
65. Los focos de la hipérbola 4x2 9y 2 = 36 son

Solución:

La hipérbola 4x2 9y 2 = 36 puede ser vista como


x2 y2
=1
9 4

de donde a2 = 9 y b2 = 4, y por lo tanto

c2 = 9+4
p
c = 13
p
En consecuencia, las coordenadas de los focos son 13; 0 .

66. Las asíntotas de la hipérbola 25y 2 16x2 = 400, son

Solución:

Como la hipérbola tiene por ecuación 25y 2 16x2 = 400, entonces a2 = 16 y b2 = 25, es decir, a = 4 y
b= 5. Lo cual implica que las directrices son las rectas
4
y= x
5
3
67. La ecuación de la hipérbola con asíntotas y = x, es
2

Solución:

3
Como las asíntotas son las rectas y = x entonces a = 2 y b = 3. Por lo tanto, la ecuación de la hipérbola
2
es
x2 y2
=1
4 9

26
68. Las coordenadas de los vértices de una hipérbola son ( 1; 0) y sus focos ( 2; 0). Entonces su ecuación es

Solución:

Según los datos del problema a = 1 y c = 2, por lo cual

b2 = 4 1
= 3

La ecuación de la hipérbola es
x2 y2
=1
1 3
69. La excentricidad de la hipérbola y 2 4x2 = 4 es

Solución:

De la ecuación de la hipérbola se tiene que a2 = 4 y b2 = 1, así que

c2 = 5
p
c = 5

Por lo tanto, la excentricidad es p


5
e=
2
70. El foco y la directriz de una parábola cuya ecuación es y 2 = 36x son respectivamente:

Solución :

La parábola está sobre el eje X; también

4p = 36
p = 9

el foco está dado por (9; 0) y la ecuación de la directriz por x = 9

27

Potrebbero piacerti anche